All Evidence MCQs (complete) Flashcards

1
Q

Which of the following questions is least likely to be objectionable under Rule 611?

A. Prosecutor asks the defendant on cross-examination: “While you were hiding behind the dumpster, isn’t it true that you took the gun and threw it in the dumpster and then took off your hat and stuffed it down your shirt?”

B. Defense counsel asks an eyewitness: “Isn’t it true that you and the defendant have never liked each other?”

C. Prosecutor asks a defense witness: “So it sounds like you have lied about everything that matters in this case. Are you the type of person who lies all the time?

D. Plaintiff’s counsel asks his client on direct examination: “You saw the defendant’s car halfway into the intersection when you entered the intersection, is that correct?”

A

B. Defense counsel asks an eyewitness: “Isn’t it true that you and the defendant have never liked each other?”

This is a perfectly acceptable impeachment question, asking about bias against the defendant. A leading question is appropriate and permissible on cross-examination.

How well did you know this?
1
Not at all
2
3
4
5
Perfectly
2
Q

Bernard was on trial on ten counts of conspiracy to sell narcotics and for arranging the murder of an undercover police officer. The chief investigator in the case, Detective Mums, has been investigating the case for six months and has helped the prosecutor prepare and coordinate the seventeen witnesses necessary to prove their case. The prosecutor tells the court that the detective is essential to her presentation. Detective Mums also plans on testifying in the case herself. Bernard intends to testify in his own defense. He also intends on calling his brother as a character witness. Before the trial begins, the judge issues a ruling under Rule 615 to sequester the witnesses. Who of the following is required to leave the courtroom?

A. Bernard.

B. Detective Mums.

C. Bernard’s brother.

D. The widow of the murdered undercover police officer, who wishes to watch the entire trial even though she will not be called as

A

C. Bernard’s brother.

Rule 615 requires most witnesses to be excluded from the courtroom, and there is no exception for character witnesses or family members of parties.

How well did you know this?
1
Not at all
2
3
4
5
Perfectly
3
Q

Which of the following is permissible?

A. In a jury trial, the judge asks questions of one of the witnesses after the direct and cross-examination.

B. In a bench trial, the trial judge interviews one of the witnesses outside of court over the weekend in order to gather more information about the case.

C. In a jury trial, a juror visits the scene of the crime over the weekend when the court is not in session.

D. In a bench trial, the judge admits hearsay evidence even though it does not meet any exception. The judge explains that she will consider the evidence but give it very little weight.

A

A. In a jury trial, the judge asks questions of one of the witnesses after the direct and cross-examination.

Rule 614(b) allows a judge to ask questions of a witness.

How well did you know this?
1
Not at all
2
3
4
5
Perfectly
4
Q

A leading question is one that suggest to the witness the answer desired by the examiner. Rule 611(c) of the Federal Rules of Evidence generally prohibits the use of leading questions on direct examination. There are, however, some exceptions to that rule. Which of the following would be a permissible instance of using leading questions on direct examination?

A. When the direct examination is conducted during a videotaped deposition of a witness.

B. When counsel is attempting to rehabilitate the testimony of the witness on re-direct examination.

C. When the witness being examined on direct is an adverse party or a hostile witness.

D. When both counsel stipulate that they’ll not object to any leading questions during trial.

A

C. When the witness being examined on direct is an adverse party or a hostile witness.

Under Rule 611(c)(2), Mode . . . of Examining Witnesses . . .; Leading Questions, when the witness is hostile or adverse, leading questions as if on cross-examination may be used.

How well did you know this?
1
Not at all
2
3
4
5
Perfectly
5
Q

At trial, a party wishes to prove the following two facts
(i) Drinking alcohol can result in a state of intoxication;
(ii) Individuals who are between the ages of eighteen and twenty drink alcoholic beverages at a greater rate than any other age group.
What are the permissible ways to get these facts into evidence?

A. The trial court should take judicial notice of (i), but it should require the party to prove (ii) using expert evidence.

B. The trial court should take judicial notice of both facts.

C. The trial court should use expert testimony for (i) and (ii).

D. There is no way to prove these facts at trial.

A

A. The trial court should take judicial notice of (i), but it should require the party to prove (ii) using expert evidence.

Rule 201 states that a court should take judicial notice of any fact that is not subject to reasonable dispute because it (1) is generally known within the trial court’s territorial jurisdiction; or (2) can be accurately and readily determined from sources whose accuracy cannot reasonably be questioned. The fact that drinking alcohol can result in a state of intoxication is a fact that is generally known. It can also be readily determined from a source whose accuracy cannot reasonably be questioned, such as a medical treatise. However, the proportion of people in a certain age group that drink alcohol is a matter that may be open to dispute, and there is no unimpeachable source that could provide that information. Therefore, the state would have to prove this fact using an expert witness.

How well did you know this?
1
Not at all
2
3
4
5
Perfectly
6
Q

Barry is on trial for possession of heroin with intent to sell. The prosecutor proves that Barry was caught in possession of sixty small baggies of heroin. The prosecutor asks the judge to take judicial notice of the fact that anyone with sixty bags of heroin intends to sell the heroin because that amount is far more than anyone would personally use. Should the judge take judicial notice of this fact?

A. No, because a judge may not take judicial notice of a fact in a criminal case.

B. Yes, if the judge has sufficient personal experience with individuals using or buying heroin so that she knows herself that sixty bags must mean that the defendant intended to sell the heroin.

C. Yes, if the judge determines that this fact is beyond reasonable dispute and generally known to individuals in that area.

D. Yes, if the judge has tried so many heroin cases that she knows herself that sixty bags must mean that the defendan

A

C. Yes, if the judge determines that this fact is beyond reasonable dispute and generally known to individuals in that area.

Rule 201(b) states that a judge should take judicial notice of a fact if it is not subject to reasonable dispute and (1) is generally known or (2) can be accurately or readily determined from an unimpeachable source.

How well did you know this?
1
Not at all
2
3
4
5
Perfectly
7
Q

Which of the following facts would a court be most likely to take as judicially noticed?

A. The fact that an individual who is carrying various credit cards that each have a different name on them has an intent to commit credit card fraud.

B. The fact that a witness had been previously convicted of a crime.

C. The fact that Barack Obama was re-elected President of the United States in 2012.

D. The fact that drinking alcohol is

A

C. The fact that Barack Obama was re-elected President of the United States in 2012.

A fact should be judicially noticed only if it is (1) beyond dispute and (2) it is generally known OR can be accurately verified by an unimpeachable source. The fact of Obama’s reelection in 2012 is both beyond dispute and is generally known (and can also be verified by any number of unimpeachable sources).

How well did you know this?
1
Not at all
2
3
4
5
Perfectly
8
Q

In U.S. v. Rollin, Rollin is being charged with robbery of a federal post office. At the trial, the Government asks the court to take judicial notice of the fact that at Metro City’s latitude, the sun remained in the sky at 5:30 pm EST on June 21, 2004, the date of the robbery. The trial judge takes judicial notice of this fact. Therefore, the effect of the judicial notice of the fact is that:

A. The Government’s burden of production of evidence for the fact judicially noticed is now satisfied.

B. The fact judicially noticed is established beyond a reasonable doubt.

C. The fact judicially noticed is conclusively established.

D. The burden of persuasion has now shifted to the defendant Rollin to prove beyond a reasonable doubt a fact contrary to the Government’s contention.

A

A. The Government’s burden of production of evidence for the fact judicially noticed is now satisfied.

When a court takes judicial notice, then the party that requested judicial notice has met its burden of production of evidence for the fact judicially noticed.

How well did you know this?
1
Not at all
2
3
4
5
Perfectly
9
Q

In a personal injury action, plaintiff requests the court to take judicial notice of a 1910 city ordinance prohibiting the riding of bicycles in the park after dark. Defendant, while riding his bicycle, hit plaintiff at 11 p.m.

A. This is a legislative fact not covered by the Federal Rules of Evidence.

B. This law may be judicially noticed even if it is not commonly known.

C. This is an adjudicative fact not noticeable because it is not a commonly known fact.

D. This is an adjudicative fact but is not judicially noticeable.

A

A. This is a legislative fact not covered by the Federal Rules of Evidence.

Ascertainment of the governing law involves judicial notice of a legislative fact; only adjudication facts—the what, when, where, how, why, who aspects of the litigation—are encompassed by Rule 201.

How well did you know this?
1
Not at all
2
3
4
5
Perfectly
10
Q

A state court is LEAST likely to take judicial notice of which of the following?

A. In Australian law, there is no private action for environmental issues.

B. Main Street, upon which the courthouse is situated, runs north and south.

C. The blood type that occurs with greatest frequency in the population is O-positive

D. The sun rose at 6:52 a.m. on Friday, December 12, of last year.

A

A. In Australian law, there is no private action for environmental issues.

The state court is least likely to take judicial notice of the Australian law. Most state courts will not take judicial notice of the law of a foreign country. Note that foreign law is a legislative fact and thus would not be covered by Federal Rule 201, which covers only adjudicative facts, even if the case were in federal court.

How well did you know this?
1
Not at all
2
3
4
5
Perfectly
11
Q

Which of the following statements regarding judicial notice is most accurate?

A. Once a court takes judicial notice of a fact in any proceeding, the jury is required to accept that fact as conclusive.

B. If a court on its own authority takes judicial notice of a fact, a party is not entitled to a hearing as to the propriety of the action.

C. Judicial notice of a fact may be taken for the first time during the appellate stages of litigation.

D. A court may take judicial notice of a fact only when requested by one of the parties.

A

C. Judicial notice of a fact may be taken for the first time during the appellate stages of litigation.

Judicial notice may be taken at any stage of the proceedings, including during appellate review.

How well did you know this?
1
Not at all
2
3
4
5
Perfectly
12
Q

In a civil case, a plaintiff must demonstrate that gold is denser than silver. After consulting a textbook on metallurgy, the judge took judicial notice of the fact at the plaintiff’s request.
What is the effect of the judge’s ruling?

A. While the ruling is evidence that the jury might consider on the issue, they are not required to accept it as conclusive.

B. It shifts the burden of production on the issue to the defendant.

C. It shifts the burden of persuasion on the issue to the defendant.

D. The fact is conclusively established.

A

D. The fact is conclusively established.

The fact that gold is denser than silver is an appropriate subject for judicial notice because it is a fact capable of accurate and ready determination by resorting to sources that have unquestionable accuracy (i.e., the metallurgy textbook).

How well did you know this?
1
Not at all
2
3
4
5
Perfectly
13
Q

Dana is on trial for child abuse. The prosecution’s theory is that on October 5th, Dana got angry at her six-year-old son for having stained the carpet in her bedroom, and so she struck him multiple times, causing significant bruising to his face and body. Which of the following evidence (if any) should be excluded as IRRELEVANT to the prosecutor’s case?’

A. On October 12th, Dana’s son told his teacher at school that a week before his mother had hit him and made him cry.

B. One year ago, Dana’s son was removed from her by the state for six months because Dana hit him in the face with a coffee mug and broke his jaw.

C. All of the above facts are relevant to the case.

D. Dana did not send her son to school for five days after October 5th.

A

C. All of the above facts are relevant to the case.

All of the above facts are relevant because they have a tendency to make a material fact (that Dana did indeed strike her son) more probable than it would be without the evidence. Some of this evidence is barred as propensity evidence or by the hearsay rule, but the question asks about relevance, not admissibility.

How well did you know this?
1
Not at all
2
3
4
5
Perfectly
14
Q

The defendant is being tried for murder in the bludgeoning death of his brother. The defendant denies any involvement in the crime. He callsa witness to the stand, who testifies that, in his opinion, the defendant is a nonviolent, peaceable man.

Which of the following, if offered by the prosecution, would most likely be admissible?

(A) A neighbor’s testimony that the witness has beaten his wife on several occasions.
(B) A police officer’s testimony that the defendant has a general reputation in the community as a violent person.
(C) A neighbor’s testimony that the defendant has a reputation for being untruthful.
(D) Evidence that the defendant has a conviction for aggravated battery.

A

(B) A police officer’s testimony that the defendant has a general reputation in the community as a violent person.

Criminal = Reputation, Opinion ONLY
1) Did D open the door? YES
2) Is the character trait relevant? YES (violence)

The defendant put his character of nonviolence at issue by having his witness testify to the defendant’s nonviolent nature, which is relevant to whether he committed the crime charged.

Note: The D’s reputation for truthfulness is not relevant to whether he has committed the crime for which
he is charged. (And because the defendant has not placed his credibility in issue by taking the stand as a witness, his reputation for truthfulness cannot be offered for impeachment purposes.)

How well did you know this?
1
Not at all
2
3
4
5
Perfectly
15
Q

A lawyer represented a sole proprietor in a federal income tax refund case in U.S. district court in which the sole proprietor was claiming certain additional business expense deductions based on sales expenses that had inadvertently been left off of his tax return for the year.

What is most likely protected from admission into evidence under the attorney-client

A) An employment contract between the sole proprietor and a salesperson drafted by the lawyer.
B) Sales records created by the sole proprietor, and provided to the lawyer to enable the lawyer to prepare for the litigation.
C) A letter sent to the lawyer by the sole proprietor detailing business expenses.
D) The sole proprietor’s tax return for the tax year in question, which was prepared by the lawyer.

A

A letter sent to the lawyer by the sole proprietor detailing business expenses.

The letter is a communication by a client to his attorney about a matter under litigation.

Note: The letter protected by the attorney-client privilege, the information contained in the letter, i.e., the sole proprietor’s business expenses, is not protected by this privilege

Note: B is incorrect because, even though the records were provided by the client to the lawyer and pertained to the legal matter at hand, the records were not prepared in order to obtain legal advice.

How well did you know this?
1
Not at all
2
3
4
5
Perfectly
16
Q

A telecommunications company retained a lawyer to represent it in a commercial dispute in which it was the plaintiff. The fee agreement provided that the lawyer would bill the company on an hourly basis but would not collect anything except costs “unless and until the telecommunications company received a recovery or settlement in the action.”

Due to disagreements with the lawyer’s strategy for the trial, the telecommunications company discharged the lawyer and hired a new law firm on a contingency basis to represent it in the case. The company was ultimately awarded a large judgment.

The original lawyer sued the company for her uncollected fees. The company denied owing the lawyer anything, asserting that no money was due yet because the judgment had not actually been collected. The lawyer wants to introduce evidence that the judgment has been collected by showing disbursements from the new law firm’s trust account to the telecommunications company.

Is this evidence admissible?

A) No, because the evidence is protected by the attorney-client privilege.
B) No, because the evidence is irrelevant.
C) Yes, because the evidence does not constitute a confidential communication covered by the attorney-client privilege.
D) Yes, because the attorney-client privilege does not apply to disputes between the client and the attorney.

A

C) Yes, because the evidence does not constitute a confidential communication covered by the attorney-client privilege.

The attorney-client privilege prevents anyone from testifying about confidential communications made to an attorney for the purpose of obtaining legal services.

BUT the payments made out of the trust account were not communications made for the purpose of obtaining representation, and are therefore not covered by this privilege.

Note: Although there is an exception to attorney-client privilege when there is a dispute between the attorney and the client, the dispute at issue is between the company and the original lawyer, not the company and the law firm that made this disbursement. Moreover, the payments made out of the trust account were not communications made for the purpose of obtaining representation, and are therefore not covered by this privilege.

How well did you know this?
1
Not at all
2
3
4
5
Perfectly
17
Q

A governmental inspector of a construction project was struck and injured by a truck. The inspector brought a negligence action against the construction firm, alleging that the driver of the truck was an employee of the firm. At trial, the head of the construction firm testified that the truck driver was an independent contractor. The inspector’s lawyer, in cross-examining the head of the construction firm, sought to ask whether the construction firm had purchased liability insurance for the truck.

Can the judge permit this question over the objection of the lawyer for the construction firm?

A) No, because evidence of liability insurance is inadmissible in a negligence action.
B) No, because whether the construction firm had liability insurance can only be proved by extrinsic evidence.
C) Yes, because evidence of liability insurance is subject to disclosure during discovery under the Federal Rules of Civil Procedure.
D) Yes, because the evidence tends to show that the truck driver was an employee of the construction firm.

A

D) Yes, because the evidence tends to show that the truck driver was an employee of the construction firm.

Other purpose: proving control

How well did you know this?
1
Not at all
2
3
4
5
Perfectly
18
Q

A plaintiff sued a defendant for injuries he suffered while shaving with a razor manufactured by the defendant. The plaintiff alleged that the razor was defectively designed. The plaintiff’s expert witness testified that the manufacturer should have used certain safeguards in the razor’s design that would have made the razor safer. In his testimony, the defendant did not deny that the safeguards urged by the plaintiff’s expert were feasible, but he argued that they were unnecessary and that the razor was not defectively designed. The plaintiff seeks to cross-examine the defendant about a safety modification the defendant has made to the razor since the plaintiff’s injury.

For which of the following substantive purposes may evidence of this modification be used?

A) Both to prove that the razor was defectively designed and to prove that the safeguards described by the expert were feasible.
B) Only to prove that the razor was defectively designed.
C) Only to prove that the safeguards described by the expert were feasible.
D) Neither to prove that the razor was defectively designed nor to prove that the safeguards described by the expert were feasible.

A

D) Neither to prove that the razor was defectively designed nor to prove that the safeguards described by the expert were feasible.

Plaintiff may not introduce evidence of remedial measure – feasibility unless the feasibility of such measures is disputed

How well did you know this?
1
Not at all
2
3
4
5
Perfectly
19
Q

A woman took out a loan from a large bank in order to start a new business. After she missed several payments, the bank sued the woman to collect the outstanding balance. In pre-trial settlement negotiations, the woman explained that she was unable to make the payments because her business was struggling. She noted that she should have listened to her boyfriend, who believed that debt would be the downfall of the country and that people should do whatever necessary to destroy big banks.

The parties eventually reached a compromise whereby the bank would extend the time to repay the loan, but would retain the right to sue under the original terms if the woman missed any payments. When the woman failed to make the payments, the bank properly reinstituted its suit against the woman.

At trial, the woman claimed that the loan contract was invalid because she was heavily medicated at the time she made the contract, and was thus incompetent. The woman has called her boyfriend to the stand to testify to these facts. The bank seeks to introduce the woman’s statements about her boyfriend made during the earlier settlement negotiations.

Are the woman’s statements about her boyfriend likely to be admitted?

A) No, because the statements were made during settlement negotiations.
B) No, because the statements are inadmissible hearsay.
C) Yes, because the statements prove bias or prejudice of a witness.
D) because the negotiations at issue resulted in a settlement agreement.

A

C) Yes, because the statements prove bias or prejudice of a witness.

Statements made during settlement negotiations are inadmissible to prove or disprove the validity or amount of a disputed claim. Such statements may be admitted for other purposes, however, such as to prove the bias or prejudice of a witness. In this case, the bank intends to introduce the woman’s statements about her boyfriend to show his bias against banks. Accordingly, they are admissible even though they were made during a settlement negotiation.

How well did you know this?
1
Not at all
2
3
4
5
Perfectly
20
Q

A customer sued a home improvement store for damages due to an alleged back injury that occurred when an employee driving a forklift backed into the unsuspecting customer who was facing the opposite direction. The store asserted that the forklift accident had not caused the customer’s injury. At trial, the customer seeks to introduce an affidavit of a physician, who has since died, that she examined the customer the day after the incident and concluded that the customer had suffered a back injury within the past 36 hours. Is this affidavit admissible?

A) No, because of the physician-patient privilege.
B) No, because it is inadmissible hearsay.
C) Yes, because it is recorded former testimony.
D) Yes, because it is a statement regarding a present physical condition.

A

B) No, because it is inadmissible hearsay.

Out of court statement? Yes
Used for the truth of the matter asserted? Yes

Is it hearsay (exemptions)? No
Exceptions? No

Note: * statement was not made by the customer about his own physical condition but by the physician – medical treatment exception does not apply

How well did you know this?
1
Not at all
2
3
4
5
Perfectly
21
Q

In a murder trial, the prosecutor planned to call an eyewitness to the stand to testify that he saw the defendant kill the victim. However, the witness recently suffered a severe head injury that seriously affected his memory. The witness can no longer remember witnessing the murder. Prior to the witness’s injury, he testified to what he saw before the grand jury.

The prosecutor would like to introduce the witness’s grand jury testimony as substantive evidence that the defendant committed the murder. The defendant objects to the introduction of the evidence. Should the court admit the witness’s grand jury testimony into evidence?

A) Yes, because the witness is unavailable to testify.
B) if used to refresh the witness’s recollection.
C) No, because the witness does not meet the “unavailability” standard.
D) No, because the former testimony exception does not apply to these facts.

A

D) No, because the former testimony exception does not apply to these facts.

Although the witness is “unavailable” for the purposes of the hearsay rules (as will be discussed below), and there is a “former testimony” exception to the hearsay rule, the former testimony exception does not apply to grand jury testimony.

Note that this WOULD be OK for prior inconsistent statement, but that’s not happening here.

How well did you know this?
1
Not at all
2
3
4
5
Perfectly
22
Q

A defendant was sued in civil court for assault. The defendant, as his first witness in his case in chief, called a friend to testify that, on the day before the day in question, the defendant had told her that he was leaving town that afternoon to drive across the country.

Is this testimony admissible to show that the defendant was not in town when the assault allegedly occurred?

A) Yes, because the defendant is available to testify.
B) Yes, because it is a declaration of the defendant’s present mental state.
C) No, because it is hearsay.
D) No, because the witness is biased.

A

B) Yes, because it is a declaration of the defendant’s present mental state.

A statement of present intent, motive, or plan can be used to prove conduct in conformity with that state of mind. Consequently, the defendant’s statement to his friend may be introduced into evidence to show that the defendant was not in town when the assault allegedly occurred.

How well did you know this?
1
Not at all
2
3
4
5
Perfectly
23
Q

A woman and her sister walked into the woman’s house. The woman went into her bedroom to say hello to her husband, while the sister waited in the other room. As soon as the woman walked into the bedroom, her sister heard her exclaim, “Where did you get all that money? There must be several thousand dollars on this dresser!” The woman came out of her bedroom shortly thereafter, and told her sister that the sister should leave immediately. That night, the woman called her sister and told her that her husband had robbed a bank. The husband was later arrested for bank robbery. At trial, the woman refused to testify against her husband, and the prosecution called the sister as a witness. The sister testified that the woman had said there was several thousand dollars on the dresser, and that the woman called her to tell her that the husband had robbed a bank. The husband objected to both pieces of testimony.

How should the judge rule regarding the sister’s testimony?

A) The judge should admit both pieces of testimony.
B) The judge should admit the testimony regarding the woman’s statement about the money on the dresser only.
C) The judge should admit the testimony regarding the woman’s statement about her husband robbing a bank only.
D) The judge should exclude both pieces of testimony.

A

B) The judge should admit the testimony regarding the woman’s statement about the money on the dresser only.

Excited Utterance/Present Sense Impression: woman made the statement immediately upon noticing the money on the dresser.

No exception for the later phone call.

How well did you know this?
1
Not at all
2
3
4
5
Perfectly
24
Q

A defendant was on trial for having committed a murder in 1995. Taking the stand, the defendant denied being present in the city where the murder occurred at the time of the killing. The prosecution sought to admit into evidence a copy of the local newspaper published the day after the killing. The newspaper contained an article in which the defendant was quoted as stating that he had heard shots on the day of the murder from inside his apartment. The defendant objected to the introduction of this evidence. Should the judge admit the newspaper article into evidence?

A) Yes, both to impeach the defendant’s credibility and to prove that he was in the city on the day of the murder.
B) Yes, but only for the purposes of impeaching the defendant’s credibility.
C) No, because the article is hearsay not within any exception.
D) No, because the article was not properly authenticated.

A

A) Yes, both to impeach the defendant’s credibility and to prove that he was in the city on the day of the murder.

Is it hearsay? No, inconsistent statement and opposing party statement

Newspaper exception: Ancient document
quote exception

How well did you know this?
1
Not at all
2
3
4
5
Perfectly
25
Q

In the prosecution of a defendant for murder, the state seeks to qualify a forensic analyst as an expert in order to have her testify as to her professional opinion of the crime scene. The defense has objected on the ground of inadequate qualifications. The prosecution now seeks to introduce a letter written by the editor-in-chief of a well-respected academic journal of forensic science, stating that the forensic analyst has published a number of well-reviewed papers on the subject of crime-scene analysis and is generally acknowledged in her field as very qualified.

On the issue of the forensic analyst’s qualifications, may the judge consider the editor’s letter?

A) Yes, because the letter is not hearsay.
B) Yes, because the judge may consider the letter without regard for the hearsay rule.
C) No, because the letter is hearsay not within any exception.
D) No, because it is the role of the jury to determine the credibility of the evidence of the forensic analyst’s qualifications.

A

B) Yes, because the judge may consider the letter without regard for the hearsay rule.

Whether a witness is qualified to offer expert opinion testimony is a preliminary question for the court—not the jury. The court is not bound by the rules of evidence when determining such questions.

How well did you know this?
1
Not at all
2
3
4
5
Perfectly
26
Q

A jurisdiction defines receiving stolen property as (i) receiving control of stolen property, (ii) with the knowledge that the property is stolen, and (iii) with the intent to permanently deprive the owner of the property. A defendant, charged with receiving stolen property after the police found a stolen television in his home, denied that he knew it was stolen. On cross-examination, the prosecutor asked the defendant, “Didn’t you also previously buy a stolen stereo from the same man who sold you this television?” The defendant’s attorney immediately objected.

What is the strongest basis for the defense attorney’s objection?

A) The probative value of the prosecutor’s question is substantially outweighed by the danger of unfair prejudice.
B) The prosecutor’s question was irrelevant because it does not establish an element that the prosecutor must prove.
C) The relevance of the prosecutor’s question depends upon whether the defendant knew the stereo was stolen, and the prosecutor has not offered sufficient proof to support that finding.
D) The risk of unfair prejudice is not substantially outweighed by the probative value of the prosecutor’s question.

A

A) The probative value of the prosecutor’s question is substantially outweighed by the danger of unfair prejudice.

A court may exclude relevant evidence when its probative value is substantially outweighed by a danger of unfair prejudice, confusing the issues, misleading the jury, undue delay, wasting time, or needless cumulation of evidence.

Note: An FRE 403 objection requires that the court decide if the proffered evidence’s probative value is substantially outweighed by one of the above-listed dangers—not vice versa.

How well did you know this?
1
Not at all
2
3
4
5
Perfectly
27
Q

In his criminal trial for battery, a defendant sought to introduce evidence of his peaceful character. He had met the leader of a local animal rights group once during a recent animal rights demonstration in front of the mayor’s office, during which the defendant succeeded in calming down an angry group of protestors. The defendant planned to ask the group leader to testify about this incident to the jury. The prosecution objected to the introduction of this evidence.

How should the court rule on the objection?

A) Sustain the objection, because this testimony constitutes an inappropriate use of character evidence.
B) Sustain the objection, because the leader of the animal rights group only met the defendant once.
C) Overrule the objection, because the defendant may introduce evidence of his good character if relevant to the crime charged.
D) Overrule the objection, because specific acts are admissible in criminal cases if introduced by the defendant.

A

A) Sustain the objection, because this testimony constitutes an inappropriate use of character evidence.

A criminal defendant may introduce evidence that his/her character is inconsistent with the crime charged. But the defendant may only do so through reputation or opinion testimony—not specific acts of conduct.

How well did you know this?
1
Not at all
2
3
4
5
Perfectly
28
Q

A politician on trial for the misdemeanor assault and battery of a reporter asserts that the reporter started the altercation by shouting questions in his face and shoving him. At trial, the politician did not take the stand and did not introduce evidence of his own character, but he did call the reporter’s neighbor to testify that the reporter has a reputation among neighbors for violent outbursts. After the neighbor testified, the prosecution moved to introduce testimony by a community leader that the politician has a reputation for violence in the community as evidence that the politician started the altercation.

Is the prosecution’s evidence regarding the politician’s reputation admissible?

A) No, because the politician did not “open the door” to the introduction of evidence of his bad character by introducing evidence of his own good character.
B) No, because the politician’s character is not at issue, as he did not testify.
C) Yes, because the politician “opened the door” to the evidence of his bad character for violence by introducing evidence of the reporter’s character for violence.
D) Yes, because violence is an essential element of battery.

A

C) Yes, because the politician “opened the door” to the evidence of his bad character for violence by introducing evidence of the reporter’s character for violence.

A criminal defendant opens the door for the prosecution to introduce evidence of the defendant’s bad character by introducing:
(1) evidence of his/her own good character for a trait pertinent to the charged crime or
(2) evidence of the alleged victim’s bad character.

How well did you know this?
1
Not at all
2
3
4
5
Perfectly
29
Q

A defendant was charged with battery following a bar fight with his neighbor. At trial, the defendant asserted that he did not initiate the altercation, but instead acted in self-defense. In addition to testifying about the event in question, he sought to testify that the preceding night, he and a coworker had gone out for a drink at the same bar, and that the evening had passed peacefully. Prior to his own testimony, the defendant sought to introduce testimony of a lifelong acquaintance of the neighbor that, in the opinion of the acquaintance, the neighbor had a violent streak. After testifying, the defendant sought to introduce testimony of the pastor of the church that the defendant regularly attended that the defendant had a reputation among the members of the church as a nonviolent person. Following testimony introduced by the prosecution that impeached the defendant’s truthfulness, the defendant sought to introduce testimony of his employer that, in his opinion, the defendant was a truthful individual.

Which of the proffered testimony is most likely to be successfully challenged by the prosecution?

A) The testimony of the lifelong acquaintance of the neighbor regarding the neighbor’s violent streak.
B) The testimony of the defendant regarding his peaceful behavior on the night before the bar fight.
C) The testimony of the defendant’s pastor as to the defendant’s reputation as a nonviolent person.
D) The testimony of the defendant’s employer that the defendant was a truthful individual.

A

B) The testimony of the defendant regarding his peaceful behavior on the night before the bar fight.

A criminal defendant may introduce evidence that his character is inconsistent with the crime charged, but only through reputation or opinion testimony—not specific instances of conduct.

How well did you know this?
1
Not at all
2
3
4
5
Perfectly
30
Q

A defendant was charged with burglary. One of the key pieces of evidence in the case was a note left by the burglar that read, “It’s just 2 easy.” At trial, the defendant testified in his defense, asserting that he did not commit the crime. On cross-examination, the prosecutor, having a proper factual basis, asked the defendant if he had been convicted of felony burglary five years ago after having left a note at the crime scene that read, “It’s just 2 easy.” The defendant’s attorney, having received proper notice from the prosecutor regarding use of the prior conviction, objected to the prosecutor’s question as seeking to elicit improper criminal-propensity evidence. The court, after determining that the probative value of this evidence and its prejudicial effects were equal, overruled the objection and instructed the defendant to answer the question.

Has the court acted properly?

A) No, because evidence of the defendant’s prior conviction constitutes improper criminal-propensity evidence.
B) No, because the court did not find that the probative value of the conviction outweighed its prejudicial effects.
C) Yes, because the defendant may be impeached by a prior conviction of burglary within the last 10 years.
D) Yes, because the prior conviction helps establish the defendant as the perpetrator of the burglary for which he is on trial.

A

D) Yes, because the prior conviction helps establish the defendant as the perpetrator of the burglary for which he is on trial.

Evidence of a criminal defendant’s prior crimes or bad acts may be admissible for relevant, noncharacter purposes (i.e., MIMIC evidence). However, this and other relevant evidence may be excluded if its probative value is substantially outweighed by the danger of unfair prejudice.

How well did you know this?
1
Not at all
2
3
4
5
Perfectly
31
Q

A defendant was charged with aggravated assault arising from an altercation following a car accident. The defendant called a witness who testified that, in his opinion, the defendant was a nonviolent person. On cross-examination, the prosecutor asked the witness whether he was aware that the defendant had been involved in a bar fight during the past year. Although the defendant was actually involved in such a fight, the prosecutor herself was not aware of the incident, as the defendant had not been arrested or charged. However, the prosecutor had witnessed the defendant’s short temper during her interactions with the defendant and knew that he was a drinker. As it happened, the witness was aware of the defendant’s involvement in the fight.

Should the court require the witness to answer the prosecutor’s question?

A) Yes, because the witness’s knowledge of defendant’s past behavior goes to the witness’s credibility.
B) Yes, as a specific instance of the defendant’s conduct.
C) No, because the defendant was not arrested or charged in connection with the bar fight.
D) No, because the prosecution did not know the defendant had been involved in a bar fight.

A

D) No, because the prosecution did not know the defendant had been involved in a bar fight.

A criminal defendant may call a witness to testify that the defendant’s character is inconsistent with the charged crime.

The prosecution may then
(1) ask the witness about a specific act committed by the defendant or
(2) call another witness to provide reputation or opinion testimony on the defendant’s corresponding bad-character trait.

Here, the prosecutor asked the defendant’s character witness about a bar fight (specific act) the defendant had within the past year. This question would affect the witness’s credibility because it raises a doubt about the witness’s opinion that the defendant was a nonviolent person. But since the prosecution did not know the defendant had been involved in a bar fight, she asked the question on a hunch. Therefore, the question was not asked in good faith, and the court should not require the witness to answer it

How well did you know this?
1
Not at all
2
3
4
5
Perfectly
32
Q

A defendant was charged with assault. Upon learning that the defendant intended to testify in his own defense, the government gave the defense proper notice of its intent to introduce as impeachment evidence the defendant’s conviction for embezzlement nine years prior. The defense filed a motion to exclude all evidence of the defendant’s conviction, arguing that it would prejudice the defendant. In a pretrial hearing, the judge noted that the conviction would likely have little prejudicial effect.

Is the judge likely to grant the defendant’s motion?

A) No, because the conviction relates to a crime involving dishonesty and occurred within the last 10 years.
B) No, because the probative value of such a conviction outweighs its prejudicial effect.
C) Yes, because a prior conviction may not be used to impeach a defendant who testifies in his own defense.
D) Yes, because the conviction for embezzlement is not probative in determining whether the defendant committed an assault.

A

A) No, because the conviction relates to a crime involving dishonesty and occurred within the last 10 years.

Any witness can be impeached with evidence of a prior conviction for a crime involving dishonesty (e.g., embezzlement) if the conviction occurred within the previous 10 years.

Note: The embezzlement conviction is not probative in determining whether the defendant committed an assault. But the conviction may still be used to impeach the defendant’s character for truthfulness

How well did you know this?
1
Not at all
2
3
4
5
Perfectly
33
Q

A defendant is on trial for bank robbery. In seeking to prove that the defendant was the robber, the prosecution introduced a handwritten note given by the robber to the bank teller on her first day of work. The teller testified that the note presented to her on the witness stand was the note that she had received from the robber. The prosecution also seeks to have the teller testify as a lay witness that the handwriting on the note is that of the defendant, who was a bank customer, based on her comparison of the note with 10 customer signature cards, including the defendant’s, presented to her by the prosecutor after the robbery.

Is the teller’s testimony that the handwriting on the note matches that on the defendant’s customer signature card admissible?

A) No, because a lay witness may not testify as to whether a document is in a person’s handwriting.
B) No, because the teller’s familiarity with the defendant’s handwriting arose from the actions of the prosecutor.
C) Yes, because the process was not unduly suggestive since the prosecutor presented the teller with 10 customer signature cards.
D) Yes, because a lay witness may testify as to whether a document is in a person’s handwriting.

A

B) No, because the teller’s familiarity with the defendant’s handwriting arose from the actions of the prosecutor.

A lay witness with personal knowledge of a claimed author’s handwriting may testify as to whether a document is in that person’s handwriting. However, the lay witness must not have become familiar with the handwriting for the purpose of the current litigation.

How well did you know this?
1
Not at all
2
3
4
5
Perfectly
34
Q

A defendant was charged with fraud in connection with the sale of nutritional supplements. The prosecution alleged that the defendant verbally represented himself as a physician to convince elderly individuals to sign contracts authorizing the defendant to charge their credit cards monthly fees for deliveries of these supplements. In fact, the defendant had no medical training. The contracts did not identify the defendant as a physician, but the prosecution intended to introduce witness testimony that the defendant verbally represented himself as a physician. The sole issue in dispute is whether the defendant made such representations.

At trial, the prosecution introduced a photocopy of a contract between the defendant and one of the alleged victims in order to lay a foundation that the alleged victim bought supplements from the defendant. The defendant did not deny that the alleged victim bought supplements from him, but objected to the introduction of the contract on the ground that the prosecution was required to introduce an original contract under the best evidence rule. The court sustained the defendant’s objection.

Did the court err in making its ruling?

A) Yes, because the best evidence rule allows for the introduction of an original or duplicate document.
B) Yes, because the best evidence rule is not implicated in this case.
C) No, because the best evidence rule applies to a document that has a legal effect, such as a contract.
D) No, because a photocopy is not admissible when the absence of the original is not explained.

A

B) Yes, because the best evidence rule is not implicated in this case.

*Duplicates are admissible unless original’s authenticity is questioned or it would be unfair to admit them.

The best evidence rule requires that the original document or a reliable duplicate be produced to prove the contents of a writing. But this rule applies only when a witness is relying on the document when testifying or the contents of the document are at issue.

How well did you know this?
1
Not at all
2
3
4
5
Perfectly
35
Q

A boat owner initiated a products liability action against the manufacturer of the boat’s engine. The owner alleged that the engine manufacturer failed to warn the owner about the proper operation of a switch on the engine and that improper operation of the switch caused the owner’s injuries. The boat owner offered evidence that the manufacturer had begun including a written warning about the switch for all boats manufactured beginning in the year after the owner’s boat was manufactured. The owner had owned his boat for five years prior to his injury.

Is this evidence admissible?

A) No, because evidence of the manufacturer’s warning is inadmissible as a remedial measure.
B) No, because the remedial-measures exclusion is limited to negligence cases.
C) Yes, because the manufacturer began providing the warning before the boat owner’s accident.
D) Yes, because evidence of the need for a warning is not subject to the remedial-measures exclusion.

A

C) Yes, because the manufacturer began providing the warning before the boat owner’s accident.

Evidence of a remedial measure is inadmissible if it was undertaken by the defendant after the plaintiff was injured. A remedial measure undertaken before the plaintiff was injured is not subject to exclusion.

How well did you know this?
1
Not at all
2
3
4
5
Perfectly
36
Q

Two officers, a veteran and a novice, brought an experienced drug-sniffing dog on patrol one evening. They properly pulled over a driver on a bridge for erratic driving, and the veteran waited in the car to let the novice handle the stop. As the novice officer questioned the driver through the driver’s window, the dog circled the car and began barking and pawing desperately at the passenger-side door. The novice officer noticed that the driver had a package sitting on his passenger seat. When asked about the package, the driver responded that it was just meat from a butcher shop. As the novice officer stepped away from the window to write a ticket, the driver grabbed the package and threw it out of the passenger window and over the railing of the bridge. The dog immediately stopped barking. The veteran officer saw the whole event.

Assuming that the parties stipulate that the veteran is an expert in the training and reactions of drug-sniffing dogs, is the veteran officer’s testimony describing the dog’s reactions to the package admissible in a trial of the driver for possession of illegal drugs?

A) No, because the dog’s behavior was assertive conduct, making it hearsay not within any exception.
B) No, because the dog’s presence at the stop resulted in an improper search.
C) Yes, as evidence that the package contained illegal drugs.
D) Yes, because evidence generated by a machine or animal falls under an exception to the hearsay rule.

A

C) Yes, as evidence that the package contained illegal drugs.

The rule against hearsay bars the admission of an out-of-court statement made by a person—not a machine or animal—that is offered to prove the truth of the matter asserted therein.

Here, the prosecutor seeks to introduce the veteran officer’s testimony describing the dog’s reactions to the package. Although the dog’s behavior was assertive conduct, the veteran officer’s testimony does not implicate the hearsay rule because it relates to evidence generated by an animal

How well did you know this?
1
Not at all
2
3
4
5
Perfectly
37
Q

A witness to an armed robbery identified a suspect in a proper police lineup that was not attended by the suspect’s attorney. Charges were brought against the suspect, but the witness, a tourist from out of the country, had returned to her home country before the trial began. At trial, the prosecutor seeks to introduce the witness’s prior statement of identification into evidence. The defendant objects to the introduction of the evidence.

Should the court allow the prior statement of identification into evidence?

A) Yes, because it is admissible as nonhearsay.
B) Yes, because the witness is unavailable, so the statement of identification falls under a hearsay exception.
C) No, because the defendant’s attorney was not present at the identification.
D) No, because the witness is unavailable.

A

D) No, because the witness is unavailable.

A declarant’s prior statement that identifies a person as someone the declarant perceived earlier is nonhearsay if the declarant testifies and is subject to cross-examination about the statement.

Here, the witness’s statement identifies the defendant as someone the witness perceived earlier. But since the witness returned to her home country before trial, she is unavailable to testify and is not subject to cross-examination about her statement.

How well did you know this?
1
Not at all
2
3
4
5
Perfectly
38
Q

In a criminal trial for arson, a prosecution witness testifies under oath that she saw the defendant set fire to the victim’s home. The defendant’s attorney does not cross-examine the witness but seeks to introduce testimony that the witness gave at a deposition several months before the trial. At the deposition, the witness testified under oath that she did not see the defendant set fire to the victim’s home.

Should the court admit the deposition testimony?

A) The court should admit the witness’s deposition testimony for impeachment purposes but not as substantive evidence.
B) The court should admit the witness’s deposition testimony for impeachment purposes and as substantive evidence.
C) The court should not admit the witness’s deposition testimony because the defendant’s attorney did not allow the witness the chance to explain her inconsistent testimony.
D) The court should not admit the witness’s deposition testimony because it is hearsay.

A

B) The court should admit the witness’s deposition testimony for impeachment purposes and as substantive evidence.

A prior inconsistent statement is admissible nonhearsay if
(1) it was given under penalty of perjury at a trial, hearing, deposition, or other proceeding and
(2) the declarant testifies and is subject to cross-examination.

It can also be introduced extrinsically for impeachment purposes if the witness has the opportunity to explain or deny, and the adverse party can examine the witness about the statement.

Here, the witness’s prior inconsistent statement—that she did not see the defendant set fire to the victim’s home—was given under penalty of perjury at a deposition. And since the witness testified and was subject to cross-examination at trial, her deposition testimony is admissible nonhearsay and can be used as substantive evidence

How well did you know this?
1
Not at all
2
3
4
5
Perfectly
39
Q

A defendant is acquitted of murder. Subsequently, the family members of the victim bring a wrongful death action against the defendant. The defendant seeks to introduce a properly authenticated, certified copy of the final judgment to show that the defendant did not wrongfully kill the victim. The victim’s family members object to the introduction of the judgment.

May the defendant introduce the copy of the final judgment from his criminal case?

A) No, because a judgment in a criminal case is inadmissible in a subsequent civil action.
B) No, because the judgment is inadmissible hearsay.
C) Yes, because the copy of the judgment satisfies the original document rule.
D) Yes, because the level of proof in a civil action is less than that in the murder case.

A

B) No, because the judgment is inadmissible hearsay.

Hearsay: One exception exists for judgments of conviction. However, no such exception exists for judgments of acquittal

How well did you know this?
1
Not at all
2
3
4
5
Perfectly
40
Q

A defendant was charged with and tried for a crime. During the presentation of its case-in-chief, the prosecution introduced an inculpatory statement made by the defendant in an email regarding the commission of the crime. The defendant requested the immediate introduction of a subsequent related email sent by the defendant that contained an exculpatory statement. The defendant established that fairness requires that the two statements be considered at the same time.

Should the court honor the defendant’s request?

A) No, because the exculpatory statement was not made at the same time as the inculpatory statement.
B) No, because the statement the defendant seeks to introduce is exculpatory.
C) Yes, as a matter of judicial notice.
D) Yes, because fairness requires that the two statements be considered at the same time.

A

D) Yes, because fairness requires that the two statements be considered at the same time.

Rule of completeness = a party may introduce any part of a previously admitted writing or recorded statement, or any other writing or recorded statement, that in fairness should be considered at the same time

Note: The rule of completeness does not require that the responding writing or recorded statement be part of or made at the same time as the previously admitted statement.

How well did you know this?
1
Not at all
2
3
4
5
Perfectly
41
Q

A plaintiff sued a defendant under a disabilities discrimination statute, alleging that the defendant refused to hire the plaintiff because of her physical disability. The defendant has asserted that he refused to employ the plaintiff because he reasonably believed that she would be unable to perform the job. The defendant sought to testify that the plaintiff’s former employer advised him not to hire the plaintiff because she was unable to work productively for more than three hours each day.

Is the defendant’s testimony admissible?

A) No, because the defendant’s opinion of the plaintiff’s abilities is not based on personal knowledge.
B) No, because the former employer’s statement is hearsay not within any exception.
C) Yes, as evidence of the defendant’s reason for refusing to hire the plaintiff.
D) Yes, as evidence that the plaintiff would be unable to work longer than three hours each day.

A

C) Yes, as evidence of the defendant’s reason for refusing to hire the plaintiff.

Non-hearsay purpose: effect on the listener. Even if the former employer’s statements were not true, they had an effect on the defendant.

How well did you know this?
1
Not at all
2
3
4
5
Perfectly
42
Q

A plaintiff sued a defendant for libel after the defendant published an article on his website calling the plaintiff an adulterer. At trial, the defendant’s attorney called the plaintiff’s wife, who testified that on two separate occasions, the wife had found the plaintiff in a hotel with another woman. The plaintiff objected to the wife’s testimony.

Should the court sustain the plaintiff’s objection?

A) No, because character may be proven by specific instances of conduct in civil cases.
B) No, because the plaintiff’s infidelity is directly at issue in the trial.
C) Yes, because the wife’s testimony is barred by spousal privilege.
D) Yes, because the wife’s testimony is improper character evidence.

A

B) No, because the plaintiff’s infidelity is directly at issue in the trial.

Character evidence is admissible if a person’s character is an essential element of a civil claim, criminal charge, or asserted defense.

How well did you know this?
1
Not at all
2
3
4
5
Perfectly
43
Q

While driving through her neighborhood, a woman was involved in a car accident with her neighbor. The neighbor alleged that the woman failed to stop at a stop sign. The neighbor brought an action for negligence against the woman. At trial, the woman intends to call a witness to testify that the woman stops at the stop sign each time she encounters it. The witness is the woman’s coworker and frequently carpools with the woman to work, driving on a route with the same stop sign. However, the witness was not present for the accident with the neighbor.

Should the court admit the witness’s testimony?

A) No, because the testimony is improper character evidence.
B) No, because the witness did not observe the accident.
C) Yes, because it can be used to prove that the woman stopped at the stop sign on the day of the accident.
D) Yes, because the woman’s character for careful driving is an essential element of her defense.

A

C) Yes, because it can be used to prove that the woman stopped at the stop sign on the day of the accident.

Evidence of a person’s habit is admissible to prove that the person acted in accordance with that habit on a particular occasion.

Note: A witness who is familiar with the person—and his/her habit—may testify to prove the existence of that habit The witness need not have been present at the event in question to do so

How well did you know this?
1
Not at all
2
3
4
5
Perfectly
44
Q

The driver of a car was involved in an accident with a bicyclist. The bicyclist died as a result of his injuries, and the state has brought a vehicular manslaughter case against the driver, alleging that the driver caused the accident when he failed to stop at a red light. At trial, the driver calls his friend, who was not present at the scene of the accident, as a witness. The friend plans to testify that he knows that the driver is a safe driver, and that the driver could not have failed to stop at a red light on the day of the accident. The prosecution objects, seeking to exclude the friend’s testimony.

Which of the following is the best basis for the prosecution’s objection?

A) The friend is biased in favor of the driver.
B) The friend is not an expert on the issue of traffic accidents.
C) The friend lacks personal knowledge of the accident.
D) The friend’s testimony offers character evidence in a criminal case.

A

D) The friend’s testimony offers character evidence in a criminal case.

A lay witness may testify to any relevant matter of which he/she has personal knowledge.

This requires that the witness
(1) perceived the matter firsthand and
(2) have a present recollection of that observation

Note that the friend is likely biased in favor of the driver BUT OC can impeach the witness for bias at a later time, if the court allows the friend to testify

How well did you know this?
1
Not at all
2
3
4
5
Perfectly
45
Q

In a civil assault suit between a plaintiff and a defendant, a witness testified that the defendant had been with her on the night of the alleged assault, more than 200 miles away from where the assault was alleged to have occurred. To challenge the witness’s credibility, the plaintiff’s attorney sought to present evidence of the witness’s juvenile conviction for voluntary manslaughter five years ago. The defendant objected to the admission of this evidence.

Should the judge admit the evidence?

A) No.
B) No, but only if the judge concludes that the probative value of this evidence is outweighed by its prejudicial effect to the witness.
C) Yes, but only if the judge concludes that the probative value of this evidence outweighs its prejudicial effect to the witness.
D) Yes.

A

A) No.

Evidence of a juvenile conviction is never admissible in a civil case to attack a witness’s character for truthfulness.

How well did you know this?
1
Not at all
2
3
4
5
Perfectly
46
Q

A pedestrian was struck by a school bus while crossing a major intersection. The pedestrian brought suit against the bus company for negligence. At trial, the pedestrian testified that he always waits for the pedestrian-crossing signal before crossing major intersections. On cross-examination, the bus company asked the pedestrian about a ticket he had received for jaywalking at a major intersection three months before the accident in question.

Is the bus company’s question likely permissible?

A) No, because the probative value of the evidence is substantially outweighed by the danger of unfair prejudice.
B) No, because the receipt of a ticket for jaywalking is not probative of the truthfulness of the recipient of the ticket.
C) Yes, because the evidence contradicts the pedestrian’s testimony that he always waits for the pedestrian-crossing signal.
D) Yes, because the prior ticket demonstrates that the pedestrian did not wait for the crossing signal when the accident occurred.

A

C) Yes, because the evidence contradicts the pedestrian’s testimony that he always waits for the pedestrian-crossing signal.

A witness may be impeached by evidence that directly contradicts the witness’s testimony on a material issue. Both intrinsic and extrinsic evidence may be used for this purpose.

Note: The receipt of a ticket for jaywalking does not, in and of itself, relate to the truthfulness of the recipient. But in this instance, the ticket calls into question the pedestrian’s truthfulness as to his statement that he always waits for the pedestrian-crossing signal.

How well did you know this?
1
Not at all
2
3
4
5
Perfectly
47
Q

During the filming of an action movie, a stuntman was injured when he jumped out of a window and fell onto a negligently installed safety net. A cameraman, who had been hired to shoot behind-the-scenes footage, taped the stuntman falling onto the safety net and the net collapsing beneath him, causing him serious injury.

The stuntman has sued the movie producer for negligence and seeks to enter a digital copy of the cameraman’s video footage into evidence at trial. The stuntman also plans to call an expert witness to testify that, based upon the video, the stuntman’s jump out of the window followed all safety protocols and that the safety net could only collapse if it was not installed properly. The cameraman has already testified that, based on his personal knowledge, the video that the stuntman seeks to admit is an accurate digital copy of the video the cameraman recorded of the accident.

Assuming there is no genuine question as to the authenticity of the original video recording, is the digital copy of the video admissible?

A) No, because the digital copy violates the best evidence rule.
B) No, because the digital copy is not a reliable form of evidence as it can be easily manipulated.
C) Yes, because the cameraman has properly authenticated the digital copy.
D) Yes, because the expert witness can guide the jury through the video and explain how and why the stuntman was injured.

A

C) Yes, because the cameraman has properly authenticated the digital copy.

Here, there is no question about the authenticity of the original recording and no indication that circumstances make it unfair to admit a copy. Therefore, admission of the copy does not violate the best evidence rule.

Note: The expert witness can likely guide the jury through the video and explain how and why the stuntman was injured. But this has no bearing on the admissibility of the recording.

How well did you know this?
1
Not at all
2
3
4
5
Perfectly
48
Q

A defendant was charged with theft of merchandise from a store. On the witness stand, the defendant admitted to taking the merchandise on the day in question but contended that she lacked the intent to do so. A rebuttal witness testified that she was standing outside the store after purchasing something, and she saw the defendant outside the store furtively removing the merchandise from her coat. When asked about her recollection of the date, the witness testified that she knew that it was the day in question because that date was on her receipt. The defendant objected, asserting that the prosecution must produce the receipt.

How is the court likely to rule on this objection?

A) Overrule the objection, because the date is a collateral issue.
B) Overrule the objection, because the date is irrelevant.
C) Sustain the objection, because the receipt is the most reliable evidence of the date.
D) Sustain the objection, because the witness’s knowledge of the date is based on the receipt.

A

A) Overrule the objection, because the date is a collateral issue.

Under the best evidence rule, an original or reliable duplicate is not required, and other evidence of content is admissible, when the contents go toward a collateral issue.

Note: Here, the witness relied on the date on the receipt when testifying about the date of the theft. But the date of the theft is collateral (i.e., undisputed) because the defendant has already admitted to taking the merchandise on the day in question. This means that the prosecution need not produce the receipt (or a reliable duplicate) for the witness to testify about it.

How well did you know this?
1
Not at all
2
3
4
5
Perfectly
49
Q

A plaintiff brought a defamation action against his friend, an internet blogger, for the publication of defamatory accusations against the plaintiff that the friend allegedly published on his internet blog. The plaintiff testified that because he had always been a daily reader of the friend’s writing, he read the defamatory remarks on June 10, the same day they were published. When the plaintiff’s attorney asked the plaintiff how he remembered the date, the plaintiff answered, “When I called his house to demand that he take down the post, his girlfriend answered the phone and said that he was out seeing a movie that had come out that day.” The friend’s attorney objected and moved to strike the testimony.

Should the court strike the plaintiff’s testimony about the girlfriend’s statement on the phone?

A) No, because the court may take judicial notice of the movie release date.
B) No, because the statement is not being offered for its truth.
C) Yes, because a court may refuse to admit evidence related to a collateral issue.
D) Yes, because the plaintiff has not established that the girlfriend is unavailable to testify.

A

B) No, because the statement is not being offered for its truth.

An out-of-court statement implicates the rule against hearsay only when it is offered to prove the truth of the matter asserted therein. Therefore, a statement offered for some other purpose is not barred by this rule.

Here, the plaintiff is offering the girlfriend’s statement to show how he was able to recall the date on which the accusations were published—not to prove that the friend in fact went to the movies on that day. Since the statement is not being offered for its truth, it is not barred by the hearsay rule.

How well did you know this?
1
Not at all
2
3
4
5
Perfectly
50
Q

A defendant is on trial for embezzling $50,000 from his former employer. The prosecution wishes to offer into evidence an anonymous letter, in its entirety, that was received by the defendant’s former supervisor. The supervisor testified that the letter was written in the defendant’s handwriting, which the supervisor knew from their years of working together. The letter reads, “I am consumed by guilt for what I have done. Here is half the money I took from you, and if you promise not to prosecute, I will send you the rest later this year. If you accept this arrangement, please post a personal ad in the local paper using the phrase ‘All is forgiven.’” The note was accompanied by $25,000 in cash. The defense objects to the admission of the letter.

Is the letter admissible?

A) No, because public policy calls for the exclusion of statements made in a negotiation to settle a claim.
B) No, because the letter has not been properly authenticated.
C) Yes, because the letter contains statements by an opposing party to the current litigation.
D) Yes, because statements of the declarant’s present intent, motive, or plan are excepted from hearsay.

A

C) Yes, because the letter contains statements by an opposing party to the current litigation.

Note: The prosecution authenticated the letter when it had the supervisor, a lay witness with personal knowledge of the defendant’s handwriting, testify that the letter was written by the defendant.

How well did you know this?
1
Not at all
2
3
4
5
Perfectly
51
Q

A woman sued her neighbor for conversion, alleging that the neighbor and his nephew took the woman’s vintage automobile from her garage and sold it on the black market. Prior to trial, the nephew was in a car accident in which his leg was badly broken. The nephew was rushed to the hospital for emergency surgery. On his way into surgery, the nephew told a nurse that he and the neighbor had taken and sold the woman’s automobile. The nephew later died in surgery due to unforeseen complications.

Which of the following hearsay exceptions will allow admission of the nurse’s testimony as to the nephew’s statement?

A) Dying declaration.
B) Excited utterance.
C) Statement against interest.
D) Statement of then-existing state of mind.

A

C) Statement against interest.

The statement against interest exception applies to hearsay statements
(1) that are contrary to an unavailable declarant’s proprietary or pecuniary interest;
(2) tend to invalidate the declarant’s claim against someone else, or
(3) expose the declarant to civil or criminal liability.

Note: The “dying declaration” exception applies to statements that (1) were made while an unavailable declarant believed his/her death was imminent and (2) concerned the circumstances of that impending death. But here, there is no indication that the nephew believed his death was imminent, and his statement did not pertain to the circumstances of his death.

How well did you know this?
1
Not at all
2
3
4
5
Perfectly
52
Q

A plaintiff filed suit against a supermarket for injuries that he sustained when he slipped on a piece of lettuce in the supermarket’s produce aisle. A supermarket employee who witnessed the plaintiff’s fall immediately prepared a written summary of the events that had occurred in order to alert the supermarket’s management. At trial, the supermarket’s lawyer called the employee as a witness. The employee testified that she could not recall the events, even after looking through her written summary while on the witness stand. The supermarket’s lawyer then asked her to read her summary to the jury. The plaintiff objected to the testimony.

May the court admit the testimony at this time over the plaintiff’s objection?

A) No, because it is hearsay not within any exception.
B) No, because such testimony may only be offered into evidence by an adverse party.
C) Yes, because the employee is on the witness stand and can be cross-examined.
D) Yes, because the employee is unable to remember the actual events even after referencing her summary.

A

D) Yes, because the employee is unable to remember the actual events even after referencing her summary.

Recorded recollection: allows a record to be read into evidence if it (
1) concerns a matter that a witness once knew but cannot recall at trial; (2) was made or adopted by the witness when the matter was fresh in his/her mind, and
(3) accurately reflects the witness’s personal knowledge at the time it was made.

Note: The employee is on the witness stand and can be cross-examined. But this is not a basis to admit a witness’s hearsay statement. Instead, the statement must fall under a hearsay exclusion or exception—e.g., the recorded recollection exception

How well did you know this?
1
Not at all
2
3
4
5
Perfectly
53
Q

A defendant is on trial for burglary. The jurisdiction has a statute that a charge of burglary may be elevated to aggravated burglary if the crime is committed with a firearm. An eyewitness, who is the burglary victim’s neighbor, called the police on the night in question and reported that she saw the defendant climb out of the victim’s window and that he was carrying a gun. The eyewitness wrote a detailed description of the incident and the weapon in her diary soon after she called the police.

During the trial, the prosecutor put the eyewitness on the stand, but she could no longer recall whether the defendant was holding anything. The prosecutor asked her to read the diary entry to herself to see if it refreshed her memory. When the eyewitness admitted that it did not, the prosecutor sought to have the eyewitness read the diary entry to the jury and to introduce the diary entry as an exhibit. The defendant objects to both.

How should the court rule?

A) The court should sustain both objections.
B) The court should sustain the objection as to the eyewitness’s reading of the diary entry but overrule the objection as to entering it as an exhibit.
C) The court should overrule the objection as to the eyewitness reading the diary entry to the jury but sustain the objection as to entering it as an exhibit.
D) The court should overrule both objections.

A

C) The court should overrule the objection as to the eyewitness reading the diary entry to the jury but sustain the objection as to entering it as an exhibit.

Recorded recollection: only an adverse party may introduce the record

Here, the eyewitness’s memory was not refreshed by having her read the diary entry to herself. And since the diary entry satisfies the criteria for admission under the recorded recollections hearsay exception, the court should allow the eyewitness to read the diary entry into evidence

How well did you know this?
1
Not at all
2
3
4
5
Perfectly
54
Q

A defendant is on trial for armed robbery and felony murder. The prosecution seeks to admit testimony by the first witness to arrive at the scene of the crime. The witness discovered the victim just before he died of a gunshot wound, and the victim identified the defendant as his assailant only moments before he died. The defense asserts that the victim was too delirious from blood loss to know that he was dying and hopes to present a statement from the victim’s widow to support this assertion. However, both parties agree that the widow’s statement is privileged under federal law.

In what manner should the court determine whether the victim’s statement is a dying declaration?

A) Allow the prosecution to admit the testimony only if the judge determines that the witness is credible.
B) Allow only the unprivileged evidence from both sides at the jury trial so the jury may decide whether the victim believed he was dying.
C) Consider all of the evidence from both sides outside the presence of the jury.
D) Consider only the unprivileged evidence from both sides outside the presence of the jury.

A

D) Consider only the unprivileged evidence from both sides outside the presence of the jury.

The court must decide preliminary questions of fact related to whether evidence is admissible, a privilege exists, or a witness is qualified.

Any hearing on these matters must be conducted outside the jury’s presence if
(1) the matter involves the admissibility of a confession;
(2) a defendant in a criminal case is a witness and so requests: OR
(3) justice so requires.

The court may only consider unprivileged evidence when making this determination, so the widow’s privileged statement may not be considered. And since the victim’s statement identifying the defendant as the assailant would clearly prejudice the defendant, the hearing should be conducted outside the jury’s presence.

Note: the court must decide the preliminary question of whether the victim knew that he was dying before ruling on the admissibility of the victim’s statement as a dying declaration.

How well did you know this?
1
Not at all
2
3
4
5
Perfectly
55
Q

A woman is on trial for a burglary that took place at about 6:00 p.m. on November 1. A surveillance video from a local gas station shows that the woman visited the gas station at 7:00 p.m. on November 1. The woman alleges as an alibi that she was actually at the gas station at 6:00 p.m. She argues that the gas station failed to change the time display on its camera to reflect the end of Daylight Savings Time on the morning of November 1. The judge, on his own initiative and after first giving the prosecutor the opportunity to object, took judicial notice of the fact that Daylight Savings Time did end in the year in question on November 1. The judge instructed the jury that it may or may not accept any judicially noticed fact as conclusive.

Were the judge’s actions with regard to judicial notice of this fact proper?

A) No, because the court should have instructed the jury that it is required to accept the noticed fact as conclusive.
B) No, because the fact is not one that is generally known within the territorial jurisdiction of the trial court.
C) Yes, because the court gave the prosecution an opportunity to be heard on the propriety of taking judicial notice before doing so.
D) Yes, because the court may take judicial notice on its own initiative.

A

D) Yes, because the court may take judicial notice on its own initiative.

A court may take judicial notice of any adjudicative fact that is not subject to reasonable dispute because it
(1) is generally known within the territorial jurisdiction of the trial court or
(2) can be accurately and readily determined from sources whose accuracy cannot reasonably be questioned.

But this fact can be accurately and readily determined from sources whose accuracy cannot be reasonably questioned (e.g., calendar, almanac), so the court may take judicial notice of this fact on its own initiative.

Note: a judge is not required to provide this opportunity before taking judicial notice of an adjudicative fact.

How well did you know this?
1
Not at all
2
3
4
5
Perfectly
56
Q

A female and a male candidate were both competing for a junior coaching position with a college football team. Although the female candidate was more qualified and experienced, the head coach chose the much less competent male candidate for the coaching position. The female candidate subsequently sued the head coach for employment discrimination. At trial, the female candidate’s lawyer sought to introduce testimony by a football player on the team claiming that the head coach had told him that “women don’t belong on the football field unless they are wearing a cheerleading uniform.”

Is the head coach’s statement admissible?

A) No, because the probative value of the statement is substantially outweighed by its prejudicial effect.
B) No, because the statement is hearsay that does not fall within an exception.
C) Yes, because it is relevant to the claim of employment discrimination against the head coach.
D) Yes, because the statement falls within the state of mind exception to the hearsay rule.

A

C) Yes, because it is relevant to the claim of employment discrimination against the head coach.

Non-hearsay: illustrate the declarant’s world view or belief system.

Note: The then-existing state of mind exception to the hearsay rule allows statements of a declarant’s then-existing mental state—e.g., motive, intent, or plan—to be admitted as substantive proof that the declarant later acted in accordance with that mental state. But here, the coach’s statement is not hearsay, so no exception is needed. Additionally, the statement is not one of motive, intent, or plan.

How well did you know this?
1
Not at all
2
3
4
5
Perfectly
57
Q

A defendant is on trial for the crime of menacing due to allegedly making threatening phone calls to a woman living in his apartment building. The prosecution called a female witness who lived in the defendant’s prior apartment building to testify that she also received a number of unidentified but identical threatening phone calls while the defendant lived in her building. The defense objected to the testimony on the ground of relevance. The prosecution responded by explaining that it plans to introduce further evidence establishing that the calls received by this witness were made by the defendant.

Is the witness’s testimony admissible?

A) No, because calls by an unidentified caller are not relevant to the case.
B) No, because the witness’s statement cannot be admitted prior to the production of evidence establishing that the defendant made the calls to the witness.
C) Yes, on the condition that evidence is introduced later that would permit the jury to reasonably find by a preponderance of the evidence that the caller was the defendant.
D) Yes, on the condition that the court finds by a preponderance of the evidence that the caller was the defendant.

A

C) Yes, on the condition that evidence is introduced later that would permit the jury to reasonably find by a preponderance of the evidence that the caller was the defendant.

Conditional relevance: When the relevance of evidence depends on whether a fact exists, proof must be introduced to allow the court to determine whether the jury could reasonably find the conditional fact by a preponderance of the evidence. However, the court may admit the proposed evidence on the condition that such proof be introduced later.

How well did you know this?
1
Not at all
2
3
4
5
Perfectly
58
Q

A plaintiff manufacturer brought an action for breach of contract against a defendant retailer for the amount due under a contract for a shipment of widgets. The contract, which was admitted into evidence, indicated that the plaintiff was entitled to payment of the contract price once the widgets were delivered to a commercial carrier. The plaintiff called a witness who has worked in the plaintiff’s shipping warehouse for many years. The witness testified that the shipment of widgets ordered by the defendant was delivered to the commercial carrier with instructions to deliver the widgets to the defendant’s shipping address. The receipt for this delivery to the commercial carrier was entered into evidence.

When the defendant’s attorney asked the witness how he knew the defendant’s shipping address, the witness stated, “We keep all of our customers’ addresses in our shipping records, but I know that one by heart because they have been a regular customer for years. They order a lot of widgets, and they always call to make sure I used the right address.” The defendant’s attorney then objected to the witness’s testimony about the defendant’s shipping address and asked that the testimony be stricken from the record on the ground that the shipping records had never been shown to the defense or offered as evidence.

Should the court sustain the defendant’s objection to the witness’s testimony?

A) No, because the witness can testify about the plaintiff’s regularly conducted business with the defendant.
B) No, because the witness had personal knowledge of the shipping address used to send the shipment of widgets.
C) Yes, because the defense has not been given the opportunity to examine the plaintiff’s shipping records.
D) Yes, because the plaintiff must offer its original shipping records to establish the defendant’s shipping address.

A

B) No, because the witness had personal knowledge of the shipping address used to send the shipment of widgets.

A witness may testify to any relevant fact about which he/she has personal knowledge—i.e., firsthand experience or observation—even when other evidence may contain the same information.

Note: The mere fact that the witness can testify about the plaintiff’s regularly conducted business with the defendant is not sufficient for the witness to testify about the defendant’s shipping address. Instead, the witness can testify about the shipping address because he has personal knowledge of it

How well did you know this?
1
Not at all
2
3
4
5
Perfectly
59
Q

A car struck a truck at an intersection. The driver of the truck sued the driver of the car, claiming that the car driver ran a red light. At trial, the only witness to the accident testified that he clearly remembered that the car’s traffic light had been red and that the car ran the light. However, in the investigating officer’s report, which was made hours after the accident, the witness is quoted saying, “I saw the whole thing. The car had the green light.” The car driver did not cross-examine the witness, and the witness was dismissed and left the jurisdiction.

After the truck driver presented his case, the car driver moved to introduce the witness’s statement from the investigating officer’s report solely to impeach the witness’s testimony. The truck driver objected.

How should the court rule?

A) Overrule the objection, and admit the statement as impeachment evidence only.
B) Overrule the objection, and admit the statement as substantive evidence that the car driver did not run the red light.
C) Sustain the objection, because extrinsic evidence may not be used to impeach a witness under these circumstances.
D) Sustain the objection, because the statement is inadmissible hearsay.

A

C) Sustain the objection, because extrinsic evidence may not be used to impeach a witness under these circumstances.

Note: Extrinsic evidence of a witness’s prior inconsistent statement is admissible only if the impeached witness has the opportunity to explain or deny—and the adverse party can examine the witness about—the statement (or if justice so requires).

How well did you know this?
1
Not at all
2
3
4
5
Perfectly
60
Q

The defendant in a civil fraud case plans to testify regarding the sale of land at issue in the case. The plaintiff seeks, for the sole purpose of impeaching the defendant’s character for truthfulness, to introduce evidence of the defendant’s recent conviction for felony assault. The defendant has filed a motion to exclude evidence of the conviction, and the judge has scheduled a hearing on the defendant’s motion.

Which of the following accurately describes each party’s burden regarding introduction of the conviction?

A) The defendant must show that the probative value of the conviction is outweighed by the prejudicial effect in order to prevail.
B) The defendant must show that the probative value of the conviction is substantially outweighed by the prejudicial effect in order to prevail.
C) The plaintiff must show that the probative value of the conviction outweighs its prejudicial effect in order to prevail.
D) The plaintiff must show that the probative value of the conviction substantially outweighs its prejudicial effect in order to prevail.

A

B) The defendant must show that the probative value of the conviction is substantially outweighed by the prejudicial effect in order to prevail.

Convictions for felonies not involving dishonesty that are no more than 10 years old are admissible against a civil witness unless the party opposing the introduction of the conviction shows that its probative value is substantially outweighed by its prejudicial effect.

Note: The burden is on the defendant—as the party opposing the admission of the conviction—to show that the probative value of the felony assault is substantially outweighed by its prejudicial effect.

How well did you know this?
1
Not at all
2
3
4
5
Perfectly
61
Q

A landowner sued a defendant for trespass and destruction of property. The defendant called a witness to testify that at the time of the alleged trespass, the defendant was with the witness miles away from the landowner’s property. On cross-examination, the landowner asked the witness if he was the defendant’s best friend.

Is this question proper?

A) No, because it exceeds the scope of the defendant’s direct examination of the witness.
B) No, because it is not relevant to the issue of whether the defendant trespassed on and destroyed the landowner’s property.
C) Yes, because character evidence is admissible in a civil action to prove conformity.
D) Yes, because it impeaches the witness’s testimony.

A

D) Yes, because it impeaches the witness’s testimony.

Impeachment = bias

How well did you know this?
1
Not at all
2
3
4
5
Perfectly
62
Q

A man sued his neighbor for assault. The man testified that on the three nights that his neighbor had threatened to hurt him, his wife had recorded the events in her diary. However, the man could not remember the dates of the events or the reasons that the neighbor made the threats. The man’s attorney permitted the man to examine his wife’s diary while on the stand. After a brief examination of the diary, the man then testified to the exact dates of the threats and the reasons that the neighbor made the threats. The neighbor seeks to introduce the relevant portions of the diary into evidence to prove that the man was lying.

Can the diary be entered into evidence?

A) No, because it can only be used to refresh the man’s recollection while testifying.
B) No, because it is hearsay not subject to an exception.
C) Yes, because a writing used to refresh a witness’s recollection may always be admitted for substantive purposes.
D) Yes, because the neighbor is entitled to enter into evidence any portion of the diary relevant to the assault claim to impeach the man.

A

D) Yes, because the neighbor is entitled to enter into evidence any portion of the diary relevant to the assault claim to impeach the man.

Once a witness has used a writing to refresh his/her recollection, the adverse party is entitled to (
1) have the writing produced for inspection,
(2) cross-examine the witness about the writing, and
(3) introduce into evidence any portion of the writing that relates to the witness’s testimony.

Here, the man examined his wife’s diary on the stand for the purpose of refreshing his recollection. As a result, the neighbor is entitled to introduce any relevant portions of the diary into evidence to prove that the man was lying—i.e., to impeach the man. Accordingly, the diary can be entered into evidence.

How well did you know this?
1
Not at all
2
3
4
5
Perfectly
63
Q

A plaintiff hopes to introduce an original x-ray image of his broken arm at trial to help prove the injury element of his negligence claim against the defendant. The plaintiff called a doctor to lay the foundation in order to enter the image into evidence. The doctor testified that the image was an accurate depiction of the plaintiff’s arm taken the day after the alleged injury. The doctor also testified that he had taken the x-ray image himself.

If the above testimony is the only testimony presented about the x-ray image, is the image admissible?

A) No, because the best evidence rule bars the image’s admission.
B) No, because the image has not been properly authenticated.
C) Yes, because the doctor authenticated the image.
D) Yes, because the image is self-authenticating.

A

B) No, because the image has not been properly authenticated.

Here, the doctor merely testified that the x-ray image was an accurate depiction of the plaintiff’s arm taken the day after the alleged injury and that he had taken the image. But since the x-ray image is a physical representation of something that cannot otherwise be seen, this testimony was insufficient to properly authenticate the image

(1) the process for creating the evidence was accurate, (2) the machine that produced the evidence was working properly, and
(3) the operator of the machine was qualified to operate it.

How well did you know this?
1
Not at all
2
3
4
5
Perfectly
64
Q

A client sued a lawyer for malpractice for recording the wrong property deed with the county register of deeds. At trial, the attorney representing the client seeks to introduce into evidence a certified copy of the deed that was recorded with the register, who is the proper officer with whom to record deeds pursuant to state law. The defendant-lawyer objected to the introduction of the evidence.

Should the judge admit the evidence?

A) No, as it is hearsay not within any exception.
B) No, because the original should be presented since the contents are at issue.
C) Yes, if a representative of the register of deeds testifies to lay a foundation for admissibility.
D) Yes.

A

D) Yes.

Copies of documents recorded and filed in a public office as authorized by law are self-authenticating if certified as correct.

The best evidence rule generally requires that an original document be produced when the contents of the document are at issue. This rule is not implicated here because the contents of the deed are not at issue. What is at issue is whether the defendant-lawyer filed the correct deed with the register

How well did you know this?
1
Not at all
2
3
4
5
Perfectly
65
Q

An animal rights activist is on trial for the burglary of a pharmaceutical lab. The prosecution’s theory of the case is that the activist broke into the lab using a maintenance access tunnel connecting the lab to a neighboring building. The tunnel was difficult to locate by anyone unfamiliar with its existence. The prosecution hopes to call an intern who works for the architect of the lab to testify that the activist visited the architect’s office on several occasions to interview the architect, who is now out of the country, for a story. The prosecution further hopes to have the intern testify that the activist had ample time to inspect a three-dimensional model of the lab that sat in the architect’s office and clearly depicted the access tunnel. The model has since been recycled as part of the architect’s regular business practices and cannot be retrieved to be entered as evidence. The intern has no knowledge of the lab or access tunnel independent of the model. Although the intern was included on the prosecution’s witness list, the prosecution did not provide advance notice of the intern’s testimony.

Can the intern properly testify as to the appearance of the model?

A) No, because the prosecution did not establish that no other reproduction of the model exists.
B) No, because the prosecution did not give advance notice of this oral testimony.
C) Yes, because the best evidence rule does not apply.
D) Yes, because the model was not destroyed in bad faith.

A

C) Yes, because the best evidence rule does not apply.

Real or physical evidence is not subject to best evidence rule, only recordings, writings, or photographs

How well did you know this?
1
Not at all
2
3
4
5
Perfectly
66
Q

In a probate proceeding, the beneficiary of a purported holographic will sought to introduce a photocopy of that will as evidence of the contents of the missing original holographic will. An heir of the testator opposed the introduction of this photocopy into evidence. Contending that the testator was not of sound mind when the original will was written, the heir demanded that the beneficiary produce the original will. The applicable jurisdiction recognizes the validity of a holographic will and has adopted a dead man’s statute.

Is the photocopy of the will admissible without an explanation of the unavailability of the original?

A) No, because the will is a document that has legal effect.
B) No, because there is a question as to the authenticity of the will.
C) Yes, because the dead man’s statute permits the introduction of the photocopy.
D) Yes, because the photocopy is a duplicate of the original.

A

D) Yes, because the photocopy is a duplicate of the original.

Best evidence rule: a duplicate is admissible to the same extent as the original unless (1) a genuine question is raised about the original’s authenticity or (2) the circumstances make it unfair to admit the duplicate.

How well did you know this?
1
Not at all
2
3
4
5
Perfectly
67
Q

A defendant was charged with the murder of her own child. Two months prior to trial, the prosecutor interviewed a family friend who had known the defendant her whole life. During the interview, the prosecutor asked if the defendant had ever committed violent acts against animals as a child. The friend responded with a nod of his head up and down. He then stated, “I often saw bruises on [the defendant’s] child but figured they were from playing.”

During the defendant’s trial, the prosecution sought to introduce into evidence the statement from the friend, as well as the fact that the friend nodded in response to the prosecutor’s question, to demonstrate that the defendant had a violent history against animals and that the defendant’s child often had bruises. The friend died just prior to the start of the trial. The defendant has objected to this evidence on hearsay grounds.

How should the court rule on the admissibility of the aforementioned evidence?

A) Admit the evidence of the nod only as nonhearsay.
B) Admit the evidence of the nod and the statement as nonhearsay.
C) Admit the evidence of the nod and the statement as hearsay falling under an exception.
D) Refuse the admission of both the nod and the statement.

A

D) Refuse the admission of both the nod and the statement.

Here, the prosecution seeks to admit the family friend’s nonverbal affirmative head nod as substantive proof that the defendant committed violent acts against animals as a child. Therefore, the head nod is hearsay and is inadmissible unless excluded or excepted from the hearsay rule. The friend is unavailable as a witness since he died prior to trial, which broadens the possible hearsay exceptions that could apply to his assertion. But since no exclusion or exception applies, the court should refuse its admission

How well did you know this?
1
Not at all
2
3
4
5
Perfectly
68
Q

The owner of an electronics store brought a civil suit for the value of stolen electronics against one of his former employees, who had previously been convicted in a criminal court for the theft of the same goods. During the civil trial, the plaintiff-owner called a witness whom he hoped would testify that she saw the defendant in possession of the stolen goods the day after the electronics store was robbed. The witness, however, testified that she did not see the defendant in possession of the goods and that she was actually out of town the day after the robbery. The plaintiff seeks to introduce the witness’s testimony from the criminal case, in which she testified that she saw the defendant in possession of the goods the next day. The defendant objects to the introduction of the testimony.

Should the court allow the testimony into evidence?

A) Yes, for impeachment only.
B) Yes, as substantive evidence only.
C) Yes, both for impeachment and as substantive evidence.
D) No, not for any reason.

A

C) Yes, both for impeachment and as substantive evidence.

Here, the witness’s prior inconsistent statement was given under penalty of perjury at the defendant’s criminal trial.

And since the witness testified and was subject to cross-examination at the civil trial, her prior inconsistent testimony is admissible non-hearsay and can be used as substantive evidence

How well did you know this?
1
Not at all
2
3
4
5
Perfectly
69
Q

A man witnessed a hit-and-run accident in which a pickup truck struck and killed a child. The next day, the witness gave police a signed, handwritten statement with a description of the truck, including the make and model, as well as a description of the driver of the truck. After several months, the authorities identified and charged a man who fit the witness’s description, although the truck was never located. In the interim, however, the witness had suffered a brain injury that left his memory spotty.

At the man’s trial, the prosecutor called the witness to testify about the accident but the witness had difficulty remembering key details. The prosecutor first showed the witness a photo of a vehicle of the same make and model as the truck in an attempt to refresh the witness’s memory. The defense objected but was overruled by the judge. When this failed to refresh the witness’s memory, the prosecutor showed the witness his earlier handwritten and signed statement. Again, the defense objected and was overruled. The witness testified that he still could not remember the details of the accident, but that he recognized his handwriting on the statement and that the statement accurately reflected what he witnessed at the time. The prosecutor then moved to introduce the photo and statement into evidence as exhibits, and the defense again objected.

How should the judge rule?

A) The judge should admit both the photo and the statement into evidence as exhibits.
B) The judge should admit the photo, but not the statement, into evidence as an exhibit.
C) The judge should admit the statement, but not the photo, into evidence as an exhibit.
D) The judge should not admit the photo or the statement into evidence as exhibits.

A

D) The judge should not admit the photo or the statement into evidence as exhibits.

A witness’s memory can be refreshed by allowing the witness to review any item before or while testifying. The item need not be admissible to be used for this purpose. But if a party seeks to admit the item, then the item must satisfy all evidentiary rules—e.g., relevance, authentication, hearsay.

Here, the prosecutor seeks to admit a photograph of a vehicle of the same make and model as the truck allegedly involved in the hit and run. To be admissible, the photograph must be relevant—i.e., have any tendency to make a material fact more or less probable. But since the photo merely depicts a truck similar to the man’s alleged truck, it is not relevant. Therefore, the court should not admit the photo

How well did you know this?
1
Not at all
2
3
4
5
Perfectly
70
Q

A defendant, his cousin, and a friend planned to rob a convenience store. The friend watched the defendant and his cousin put on ski masks and enter the convenience store. Moments later, the friend heard a gunshot and watched as the defendant fled the convenience store. The cousin stumbled out of the convenience store, pointed at his bleeding foot, and told the friend, “He shot me! The moron dropped his gun and shot me. I’m going to kill that idiot!” The cousin ran after the defendant. The next day, the cousin was found dead from a bullet wound to the chest. Police later charged the defendant with murder.

At trial, the defendant claimed self-defense and seeks to introduce the cousin’s statement to the friend as evidence that the cousin was the initial aggressor. The prosecution objects.

What is the defendant’s best argument for the court to find that the statement is admissible?

A) The declarant is unavailable as a witness.
B) The statement indicated the cousin’s present intent.
C) The statement was a dying declaration.
D) The statement was made by a coconspirator.

A

B) The statement indicated the cousin’s present intent.

Statements of the declarant’s then-existing state of mind—e.g., statements of motive, present intent, or plan—are excepted from hearsay. This is true regardless of whether the declarant is available as a witness

How well did you know this?
1
Not at all
2
3
4
5
Perfectly
71
Q

In a civil trial for professional malpractice, the plaintiff sought to show that the defendant, an engineer, had designed the plaintiff’s flour mill with inadequate power. The plaintiff called an expert witness who based his testimony solely on his own professional experience but also asserted, when asked, that the book Smith on Milling Systems was a reliable treatise in the field and consistent with his views. On cross-examination, the defendant asked the witness whether he and Smith were ever wrong. The witness answered, “Nobody’s perfect.” The defendant asked no further questions. The defendant later called his own expert witness and asked, “Do you accept the Smith book as reliable?” The witness said, “It once was, but it is now badly out of date.” The plaintiff requested that the jury be allowed to examine the book and judge for itself the book’s reliability.

Should the court allow the jury to examine the book?

(A) No, because the jury may consider only passages read to it by counsel or by a witness.
(B) No, because the plaintiff’s expert did not rely on the treatise in his testimony but on his own experience.
(C) Yes, because an expert has testified that the treatise is reliable.
(D) Yes, because the jury is the judge of the weight and credibility to be accorded both written and oral evidence.

A

(A) No, because the jury may consider only passages read to it by counsel or by a witness.

Learned treatise rule: if the court finds a publication to be a reliable authority, then “statements” from it may be read into evidence, but the publication may not be received as an exhibit.

Note: There is a concern that if juries were allowed unrestricted access to the whole publication, they might rely on parts of the publication that are not germane to the case

How well did you know this?
1
Not at all
2
3
4
5
Perfectly
72
Q

At a defendant’s trial for a gang-related murder, the prosecution introduced, as former testimony, a statement by a gang member who testified against the defendant at a preliminary hearing and has now invoked his privilege against self- incrimination.

If the defendant now seeks to impeach the credibility of the gang member, which of the following is the court most likely to admit?

(A) Evidence that the gang member had three misdemeanor convictions for assault.
(B) Testimony by a psychologist that persons with the gang member’s background have a tendency to fabricate.
(C) Testimony by a witness that, at the time the gang member testified, the gang member was challenging the defendant’s leadership role in the gang.
(D) Testimony by a witness that the gang member is a cocaine dealer.

A

C) Testimony by a witness that, at the time the gang member testified, the gang member was challenging the defendant’s leadership role in the gang.

This is evidence of “bias.” It shows that the gang member had a motive to implicate the defendant falsely, because by doing so he would remove the defendant from the position that he wanted to have,

Note: Expert testimony on credibility is usually found inadmissible because credibility issues are for the jury, not for the imprimatur of an expert.

How well did you know this?
1
Not at all
2
3
4
5
Perfectly
73
Q

A defendant was charged with aggravated assault. At trial, the victim testified that the defendant beat her savagely, but she was not asked about anything said during the incident. The prosecutor then called a witness to testify that when the beating stopped, the victim screamed: “I’m dying—don’t let [the defendant] get away with it!”

Is the testimony of the witness concerning the victim’s statement admissible?

A) No, because it is hearsay not within any exception.
(B) No, because the victim was not asked about the statement.
(C) Yes, as a statement under belief of imminent death, even though the victim did not die.
(D) Yes, as an excited utterance.

A

(D) Yes, as an excited utterance.

In this case, the assault was a startling event, and the victim made the statement immediately after the beating, trying to identify the perpetrator

Note: NOT a dying declaration for two reasons
1) the declarant has to be unavailable, as the dying declaration is one of the “unavailability-dependent” exceptions of Rule 804. Here, the victim testified and so obviously is not unavailable.
2) dying declaration is admissible only in homicide prosecutions and civil cases.

How well did you know this?
1
Not at all
2
3
4
5
Perfectly
74
Q

A woman who is a computer expert decided to dedicate herself to exposing persons who trafficked in child pornography. She posted a number of sexually oriented photographs on her website. The file for each photograph contained an embedded Trojan horse program (a program that would allow the woman to enter the computer of anyone who downloaded the photograph). A man downloaded one of those photographs onto his personal computer. Using the embedded program, the woman entered the man’s computer and found a file containing a pornographic photograph of a child. She copied the file and turned it over to a federal law enforcement agency. A federal agent told her that a successful prosecution would require more than one photograph and offered her a monetary reward for additional photographs leading to the man’s conviction. The woman entered the man’s computer again, and this time she found hundreds of child pornography photographs, which she turned over to the federal agency.

The man was charged with multiple counts of violating federal child pornography statutes. He has moved to suppress the photographs that the woman discovered on his computer. The motion is based on both the Fourth Amendment and a federal statute forbidding interception of electronic communication without permission. The parties have stipulated that the woman’s conduct in downloading photographs from the man’s computer violated the interception statute.

How should the court rule on the defendant’s motion to suppress?

A) Deny the motion as to all photographs.
(B) Grant the motion as to all photographs, because the woman acted without probable cause.
(C) Grant the motion as to all photographs, because the woman violated the federal interception statute.
(D) Grant the motion only as to the second set of photographs.

A

(D) Grant the motion only as to the second set of photographs.

Because authorities encouraged and offered to reward the woman for the second computer search, the woman was acting as a government agent with regard to that search, which violated the Fourth Amendment because it was conducted without a warrant.

How well did you know this?
1
Not at all
2
3
4
5
Perfectly
75
Q

In a personal injury case, the plaintiff sued a retail store for injuries she sustained from a fall in the store. The plaintiff alleged that the store had negligently allowed its entryway to become slippery from snow tracked in from the sidewalk. Before the lawsuit was filed, when the plaintiff first threatened to sue, the store’s manager said, “I know that there was slush on that marble entryway, but I think your four-inch-high heels were the real cause of your fall. So let’s agree that we’ll pay your medical bills, and you release us from any claims you might have.” The plaintiff refused the offer. At trial, the plaintiff seeks to testify to the manager’s statement that “there was slush on that marble entryway.”

Is the statement about the slush in the entryway
admissible?

(A) No, because it is a statement made in the course of compromise negotiations.
(B) No, because the manager denied that the slippery condition was the cause of the plaintiff’s fall.
(C) Yes, as a statement by an agent about a matter within the scope of his authority.
(D) Yes, because the rule excluding offers of compromise does not protect statements of fact made during compromise negotiations

A

(A) No, because it is a statement made in the course of compromise negotiations.

Note: protects not only offers of compromise, but also conduct or statements made in the course of compromise negotiations. The rationale is to allow the parties and counsel to speak freely during settlement negotiations, without having to worry that their statements will be used against them at trial.

How well did you know this?
1
Not at all
2
3
4
5
Perfectly
76
Q

At a civil trial for slander, the plaintiff showed that the defendant had called the plaintiff a thief. In defense, the defendant called a witness to testify, “I have been the plaintiff’s neighbor for many years, and people in our community generally have said that he is a thief.”
Is the testimony concerning the plaintiff’s reputation in the community admissible?

(A) No, because character is an essential element of the defense, and proof must be made by specific instances of conduct.
(B) Yes, to prove that the plaintiff is a thief, and to reduce or refute the damages claimed.
(C) Yes, to prove that the plaintiff is a thief, but not on the issue of damages.
(D) Yes, to reduce or refute the damages claimed, but not to prove that the plaintiff is a thief.

A

B) Yes, to prove that the plaintiff is a thief, and to reduce or refute the damages claimed.

In slander cases, where the defendant makes a statement that the plaintiff has an unsavory character, the plaintiff’s character is considered “in issue” (i.e., an essential element of the claim or defense under the substantive law) in two respects:

1) the plaintiff’s actual character will determine whether the defendant was incorrect in his assessment, and thus liable for slander, because truth is a defense.

2) the plaintiff will allege that he has been damaged by the statement, which is another way of saying that his true character has been besmirched; but if the plaintiff actually has a bad reputation anyway, then damages are limited.

How well did you know this?
1
Not at all
2
3
4
5
Perfectly
77
Q

The director of a company that produces organic products was being sued by the minority shareholders of the company for allegedly breaching his duty of care and unwisely investing corporate funds into an organic clothing venture. During the trial, a witness for the minority shareholders testified that the director had been accused of battery three years ago, although he was never charged or arrested for the crime. The director’s lawyer then attempted to call a witness to testify that the witness heard the individual who had accused the director of battery verbally retract his accusation. The lawyer for the minority shareholders objected to the introduction of this evidence. The court overruled the objection and allowed the testimony.

Did the court err by overruling this objection?

A) No, because the admission of additional evidence refuting the battery accusation was necessary to remove any unfair prejudice.
B) No, because a court can always allow additional evidence to rebut previously admitted inadmissible evidence.
C) Yes, because the evidence of the retraction of the battery accusation was not relevant to the lawsuit.
D) Yes, because the individual’s retraction of the battery accusation constitutes hearsay.

A

A) No, because the admission of additional evidence refuting the battery accusation was necessary to remove any unfair prejudice.

Under the doctrine of curative admission, when inadmissible evidence is improperly admitted against a party, the court may permit that party to introduce additional inadmissible evidence for the purpose of rebuttal. This is meant to remedy the prejudicial effect caused by the previously admitted evidence.

Note: Here, the testimony that the director was once accused of battery was improper because it is not relevant to whether he breached his duty of care to the minority shareholders.

The director then offered testimony from a witness who heard the alleged battery victim recant his accusation. Although that evidence is hearsay—and also not relevant to the lawsuit—it was necessary to remove the unfair prejudice caused by the initial improper evidence

How well did you know this?
1
Not at all
2
3
4
5
Perfectly
78
Q

A man is on trial for the misdemeanor crime of public lewd conduct for allegedly streaking through a gym in the early evening of May 5. This crime is not classified as a sexual offense in the jurisdiction. At trial, a trainer at the gym identified the man as the streaker and testified that, as the man ran through the gym, he cheered for a professional football team that had recently made the playoffs. The man, however, testified that he was nowhere near the gym on the day in question. In rebuttal, the prosecution seeks to call the manager of another local gym to testify that on the afternoon of May 4, the man had streaked through her gym while cheering for the same football team. The defense has objected to the manager’s testimony.

Should the manager’s testimony be admitted?

A) No, because a prior bad act is not admissible to show that the man possessed a character trait in accord with which he acted on May 5.
B) No, because character may generally be proven only by reputation or opinion evidence, not specific acts.
C) Yes, to demonstrate the man’s propensity for streaking through gyms.
D) Yes, to identify the man as the person who streaked through the gym on May 5.

A

D) Yes, to identify the man as the person who streaked through the gym on May 5.

MIMIC: Evidence of a criminal defendant’s prior crimes or bad acts is inadmissible to show the defendant’s propensity to commit the charged crime. But it may be admissible for other purposes, like his identity

How well did you know this?
1
Not at all
2
3
4
5
Perfectly
79
Q

A defendant was charged with possession of prescription drugs without a prescription. At trial, the defendant testified that he received the drugs from a friend and believed they were an acceptable over-the-counter medication. The prosecutor intends to ask the defendant about his previous three misdemeanor convictions in the last five years for possession of the same type of prescription drugs without a prescription.

Should the court allow the prosecutor to ask about the defendant’s previous convictions?

A) No, because the convictions are inadmissible character evidence.
B) No, because the convictions were misdemeanors.
C) Yes, to impeach the defendant’s character for truthfulness.
D) Yes, to show that the defendant knew the prescription drugs were not an over-the-counter medication.

A

D) Yes, to show that the defendant knew the prescription drugs were not an over-the-counter medication.

Evidence that a criminal defendant previously committed a similar crime or bad act is inadmissible character evidence if it is offered to show the defendant’s propensity to commit the charged crime. But that evidence may be admissible for other relevant, noncharacter purposes—e.g., proving absence of mistake.

How well did you know this?
1
Not at all
2
3
4
5
Perfectly
80
Q

A defendant was charged with illegal possession of a firearm by a felon. At trial, a prosecution witness testified that he and the defendant were involved in a verbal altercation during which the defendant pulled a pistol. The witness further testified that the defendant shot at him with the pistol and that he returned the defendant’s fire. On cross-examination, the defendant’s attorney asked the witness if it was true that he, and not the defendant, fired first. The witness maintained that the defendant shot first. While the witness was subject to recall, the defendant called a police officer to the stand in order to impeach the witness by testifying that the witness told her that he, not the defendant, had fired first. The prosecution has objected to the officer’s testimony.

Should the court exclude the officer’s testimony as to the witness’s prior statement?

A) No, because it relates to the witness’s prior inconsistent statement.
B) No, because the witness is subject to recall.
C) Yes, because it involves a collateral matter.
D) Yes, because it is inadmissible hearsay

A

C) Yes, because it involves a collateral matter.

A party generally may not impeach the credibility of a witness by introducing extrinsic evidence of a collateral matter. Instead, the party must accept the witness’s testimony.

Here, the defendant sought to impeach the witness by introducing his prior inconsistent statement through the officer’s testimony (extrinsic evidence). The witness was subject to recall, so he could explain or deny the statement and the prosecution could question him about it. However, the issue of who fired first is collateral to the crime with which the defendant was charged—illegal possession of a firearm by a felon

How well did you know this?
1
Not at all
2
3
4
5
Perfectly
81
Q

A defendant is alleged to have been the getaway driver in a bank robbery. The prosecution has introduced evidence that a bystander made the following excited utterance as the bank robbers fled the scene: “They jumped into that black car and drove away like maniacs!” The defendant wants to call a valet from the hotel across from the bank to testify that when the bystander and the valet discussed the event the next day, the bystander told the valet that the bank robbers got into a gray car. The bystander died of natural causes shortly after speaking to the valet.

Should the bystander’s statement to the valet be admitted?

A) No, because it was not made under penalty of perjury.
B) No, because the bystander is unavailable to testify.
C) Yes, but only to impeach the bystander.
D) Yes, to impeach the bystander and as substantive evidence that the getaway car was gray.

A

C) Yes, but only to impeach the bystander.

A hearsay declarant’s credibility may be attacked using any method allowed to impeach a testifying witness, including offering evidence of the declarant’s inconsistent statement. But that statement may not be used as substantive evidence unless it is also excepted or excluded from the hearsay rule.

Note: A hearsay declarant’s inconsistent statement need not have been made under penalty of perjury to be admitted for impeachment purposes.

How well did you know this?
1
Not at all
2
3
4
5
Perfectly
82
Q

A church is being sued for negligent hiring after its bus driver got in a drunk-driving accident while driving a bus full of parishioners to a church retreat. Evidence discovered by the plaintiff suggests that the bus driver had a well-documented drinking problem. The defense called a witness from the bus driver’s last employer, who testified that no one at the driver’s last job was aware that the bus driver had a drinking problem. The plaintiff’s attorney asked on cross-examination whether the witness is a member of the defendant-church. The defense objects to the question as an improper impeachment question.

Is the plaintiff’s attorney’s question proper?

A) No, because evidence of a witness’s religious belief is not admissible to impeach the witness’s credibility.
B) No, because it would violate the witness’s First Amendment rights.
C) Yes, as proper impeachment evidence.
D) Yes, because the witness’s religious belief is relevant to her credibility under oath.

A

C) Yes, as proper impeachment evidence.

Bias: One method of establishing bias is showing that the witness is affiliated with the opposing party outside the context of the litigation. Such affiliation can be established through intrinsic or extrinsic evidence.

How well did you know this?
1
Not at all
2
3
4
5
Perfectly
83
Q

An attorney represented both a retailer and a manufacturer in a pending products liability action against them. An investigator employed by the attorney attended a meeting between the attorney, retailer, and manufacturer regarding the upcoming trial. At trial, the plaintiff called the investigator to testify regarding statements made by the manufacturer to the investigator during the meeting in which the manufacturer had admitted liability. The manufacturer objected to this testimony on the grounds that the testimony was hearsay and the statements were protected by the attorney-client privilege.

How should the court rule on this objection?

A) Overrule the objection, because the manufacturer is an opposing party.
B) Overrule the objection, because the manufacturer’s statements were made to the investigator.
C) Sustain the objection, because of the attorney-client privilege.
D) Sustain the objection, because the statements are hearsay.

A

C) Sustain the objection, because of the attorney-client privilege.

Statements made by and offered against an opposing party are nonhearsay. Additionally, the attorney-client privilege protects confidential communications made for the purpose of obtaining or providing legal assistance for the client and extends to the attorney’s agents.

Here, communications made before a coclient (the retailer) remain protected BUT the manufacturer’s statements to the investigator would not be protected by the attorney-client privilege in a subsequent action between the manufacturer and the retailer.

How well did you know this?
1
Not at all
2
3
4
5
Perfectly
84
Q

A consumer filed a products liability action in federal court on the basis of diversity jurisdiction. In complying with a discovery request, the defendant’s lawyer inadvertently, despite the lawyer’s reasonable efforts to protect privileged material, included a report prepared at the lawyer’s request by an investigator who was employed in anticipation of the litigation and was not expected to be called as a witness at trial.

Immediately upon learning of the mistake, the defendant’s lawyer sought to retrieve the report from opposing counsel, but the plaintiff’s lawyer refused, asserting that the defendant’s lawyer’s inclusion of the report in the discovery material constituted a waiver of the protection for the material. The defendant’s lawyer contended that the federal inadvertent-waiver rule applied to this disclosure.

Is the defendant’s lawyer correct?

A) No, because the federal inadvertent-waiver rule applies only to disclosures made to a federal office or agency.
B) No, because the federal inadvertent-waiver rule does not apply to a federal case based on diversity jurisdiction.
C) Yes, because the federal inadvertent-waiver rule applies to the disclosure of communications protected by the attorney-client privilege.
D) Yes, because the federal inadvertent-waiver rule applies to the disclosure of information covered by the attorney work-product doctrine.

A

D) Yes, because the federal inadvertent-waiver rule applies to the disclosure of information covered by the attorney work-product doctrine.

Federal inadvertent-waiver rule, an inadvertent disclosure of protected information does not waive the attorney-client privilege or the attorney work-product doctrine if:
(1) the disclosure was in a federal proceeding or to a federal agency and
(2) the privilege holder attempted to prevent disclosure and promptly tried to rectify the error.

Here, the report was inadvertently disclosed in a federal case, and the defendant’s lawyer had taken reasonable steps to prevent disclosure and immediately sought to retrieve it. This satisfies the four requirements of the federal inadvertent-waiver rule. Therefore, the defendant’s lawyer correctly contended that this rule applied to his disclosure of the report.

Note: The federal inadvertent-waiver rule does apply to the disclosure of communications protected by the attorney-client privilege

How well did you know this?
1
Not at all
2
3
4
5
Perfectly
85
Q

n a sexual harassment action brought by an employee against her employer, the employee alleged that her supervisor had created a hostile work environment by making repeated crude and explicit sexual comments that were unwelcome. The employer filed a motion to admit evidence that the employee had a sexual relationship with her previous supervisor to show that the employee welcomed the employer’s advances.

How should the court rule on the admissibility of this evidence?

A) The court should admit evidence of the relationship because the exclusion of evidence offered to prove a victim’s sexual conduct or predisposition applies only to a criminal case.
B) The court should admit evidence of the relationship only if its probative value substantially outweighs the danger of harm and unfair prejudice to the employee.
C) The court should refuse to admit evidence of the relationship because evidence offered to prove a victim’s sexual conduct or predisposition is inadmissible in a civil case.
D) The court should refuse to admit evidence of the relationship only if its probative value is substantially outweighed by the danger of harm and unfair prejudice to the employee.

A

B) The court should admit evidence of the relationship only if its probative value substantially outweighs the danger of harm and unfair prejudice to the employee.

Evidence of a victim’s other sexual behavior or sexual predisposition is generally inadmissible in a civil proceeding involving sexual misconduct. But such evidence may be admitted if the court determines that the probative value of the evidence substantially outweighs the danger of harm to the victim and unfair prejudice to any party.

Note: the default position is that this evidence is inadmissible, and the court need not determine that its probative value is substantially outweighed by the relevant dangers before excluding the evidence.

How well did you know this?
1
Not at all
2
3
4
5
Perfectly
86
Q

A woman sues a jewelry company for injuries stemming from an allergic reaction she had to a metal in a bracelet that she believes was manufactured by the jewelry company. The jewelry company alleges that the bracelet is a forgery made by another manufacturer and that the company is not liable. The woman seeks to introduce the following pieces of evidence: (1) trademark registrations with accompanying photographs indicating that the bracelet that allegedly caused her injuries bears the jewelry company’s trademark, (2) a bracelet the jewelry company acknowledges as its own for the jury to compare with the woman’s bracelet, (3) purchase orders for the jewelry company’s bracelets from the store from which the woman purchased her bracelet, and (4) a judgment obtained by another plaintiff against the jewelry company for a similar reaction to a bracelet made by the jewelry company.

Which of these is LEAST likely to be admissible on the issue of whether the bracelet is a forgery?

A) The evidence concerning the jewelry company’s trademark on the bracelet.
B) The evidence that the store purchased bracelets from the jewelry company.
C) The jewelry company’s bracelet, for comparison to the woman’s bracelet.
D) The judgment against the jewelry company obtained by another plaintiff

A

D) The judgment against the jewelry company obtained by another plaintiff

There is no specific hearsay exception for civil judgments.

Note: The public records exception applies to records of a public office that set out a matter observed pursuant to a legal duty to report. Since trademark registrations are required by law to be recorded in a principal registry by the U.S. Patent and Trademark Office, the registrations are admissible.

How well did you know this?
1
Not at all
2
3
4
5
Perfectly
87
Q

A plaintiff who had been injured in a car accident with a truck brought an action against the employer of the truck driver for negligent hiring. Prior to trial, the employer filed a motion for summary judgment arguing that it was entitled to judgment as a matter of law. In her response, the plaintiff submitted an affidavit by a former secretary of the employer stating that the secretary overheard the truck driver tell the employer that he had a history of accidents while interviewing for the job.

Can the court properly consider the driver’s statement in the affidavit in ruling on the employer’s summary-judgment motion?

A) No, because the affidavit constitutes double hearsay.
B) No, because the statement constitutes hearsay.
C) Yes, because it is an opposing party’s statement.
D) Yes, because it is not hearsay.

A

D) Yes, because it is not hearsay.

An out-of-court statement is not hearsay if offered for a purpose other than to prove the truth of the matter asserted.

Here, the secretary’s affidavit stated that the secretary heard the driver disclose a history of accidents to the employer. That statement is hearsay if offered for the truth of the matter asserted—because no exclusion or exception applies. But it is not hearsay if offered to show that the employer had notice of the driver’s history and may have been negligent in hiring him

How well did you know this?
1
Not at all
2
3
4
5
Perfectly
88
Q

An artist is on trial for arson of an art dealer’s gallery containing a number of pieces of artwork by the artist. The artist had a contract with the art dealer making the art dealer liable to the artist for any damage to her artwork under his care. The prosecution wants to admit the artist’s notarized contract with the art dealer. The prosecution also wants to have a bartender testify that the artist told him, “I know I’m a terrible artist. Maybe the art dealer can sell a piece or two, but I’ll make more money by burning the place to the ground.” The artist has moved to exclude the contract for lack of notice and the statement to the bartender as inadmissible hearsay.

How should the court rule on the artist’s motions?

A) Exclude both pieces of evidence.
B) Exclude only the contract.
C) Exclude only the statement to the bartender.
D) Admit both pieces of evidence.

A

D) Admit both pieces of evidence.

Notarized documents are self-authenticating and can be introduced without advance notice to an adverse party. Additionally, statements made by and offered against a party-opponent are excluded from the hearsay rule.

How well did you know this?
1
Not at all
2
3
4
5
Perfectly
89
Q

A plaintiff and a defendant were involved in a car accident in an intersection. Both parties sustained minor injuries in the accident, and the plaintiff subsequently brought an action for negligence against the defendant. Although there was evidence that both parties may have been negligent, the plaintiff sought to establish that the defendant’s vehicle did not slow down when it entered the intersection.

The plaintiff’s attorney called a passenger who was in the defendant’s vehicle at the time of the accident. The passenger testified that when she and the defendant got out of the car after the accident, the defendant told her, “It felt like the brakes failed, but that’s not possible, my mom just had the car serviced.” The defendant objected to the admission of this testimony.

Should the court sustain the defendant’s objection to the passenger’s testimony?

A) No, because the testimony can be offered by the plaintiff as a statement of a party-opponent.
B) No, because the testimony is only admissible for impeachment purposes.
C) Yes, because the defendant’s statement to the passenger was not based on personal knowledge.
D) Yes, because the defendant’s statement was not against his interest at the time it was made.

A

A) No, because the testimony can be offered by the plaintiff as a statement of a party-opponent.

Here, the statement was made by and offered against the defendant, so it constitutes a nonhearsay statement by a party-opponent. As a result, it is admissible substantively to prove that the defendant’s car did not slow down when it entered the intersection

How well did you know this?
1
Not at all
2
3
4
5
Perfectly
90
Q

In a medical malpractice case, a patient sued her surgeon for allegedly causing organ damage during a routine surgery. In a previous case, the patient sued her medical insurance company for not covering some of the medical expenses associated with the complications from the organ damage. During that trial, the patient testified that she overheard a nurse assisting the surgeon tell another assisting nurse during the surgery that it appeared the patient’s kidney had been pierced. Before the patient could testify about this statement in the medical malpractice trial, she died from complications stemming from the surgery.

Is the patient’s testimony from the previous trial admissible in the medical malpractice trial?

A) No, because the defense did not have an opportunity to question the patient in the previous trial.
B) No, because the testimony was given during a previous trial, not the current proceeding.
C) Yes, because the patient is not available and the surgeon caused the patient’s unavailability.
D) Yes, because the patient is not available and the testimony was given as a witness at trial.

A

A) No, because the defense did not have an opportunity to question the patient in the previous trial

The former testimony hearsay exception allows admission of testimony (1) given at a trial, hearing, or deposition in the same case or a different proceeding and (2) offered against a party who—or whose predecessor in interest— had an opportunity and similar motive to develop that testimony.

Here, the patient died before she could testify in the medical malpractice trial and is therefore unavailable. But the surgeon did not have the opportunity to examine the patient during the previous trial. And the insurance company—whose objective at trial was to justify its denial of coverage and not to clear the surgeon of liability—is not the surgeon’s predecessor in interest.

How well did you know this?
1
Not at all
2
3
4
5
Perfectly
91
Q

At the trial of a contract dispute, the plaintiff has offered to testify to what she heard the defendant say in a private conversation between the two of them, which the plaintiff secretly recorded on an audiotape that she did not offer in evidence.
Is the plaintiff’s testimony admissible?

(A) Yes, because the plaintiff has personal knowledge of the statement of a party-opponent.
(B) Yes, because the original document rule does not apply to audiotapes.
(C) No, because the statement must be proved by introduction of the audiotape itself.
(D) No, because of the plaintiff’s deception, even if the recording was not illegal.

A

(B) Yes, because the original document rule does not apply to audiotapes.

What the defendant said to the plaintiff, even in a private conversation, is a statement of a party-opponent and is admissible.
The plaintiff has personal knowledge of what the defendant said and can testify about it. The fact that the audiotape might be better evidence of what the defendant actually said makes no difference. The best evidence rule applies only when a witness testifies about the content of a writing or recording. Here the plaintiff would not be testifying about the content of the audiotape but rather about what she personally heard.

How well did you know this?
1
Not at all
2
3
4
5
Perfectly
92
Q

A plaintiff sued a defendant for wrongful death arising out of a traffic collision between the plaintiff’s decedent and the defendant. At trial, the investigating police officer authenticated a tape recording of her shift-end dictation of comments used in preparing the written report of her factual findings. She has testified that the tape recording was accurate when made and that she currently has no clear memory of the details of the investigation.

Is the tape recording admissible as evidence?

(A) Yes, under the recorded recollection exception to the hearsay rule.
(B) Yes, under the public records exception to the hearsay rule.
(C) No, because it is hearsay and is a police report being offered against the defendant in a wrongful death case.
(D) No, because the police report itself is the best evidence.

A

(A) Yes, under the recorded recollection exception to the hearsay rule.

The witness once had knowledge but now has insufficient recollection to testify fully and accurately about her investigation. She made the recording when the matter was fresh in her memory, and she has testified that the recording was an accurate reflection of her memory.

Note: Although the officer’s formal written report would qualify as a public record, the informal, dictated comments she made to help her prepare the report would not. The tape recording is admissible, however, under the recorded recollection exception to the hearsay rule.

How well did you know this?
1
Not at all
2
3
4
5
Perfectly
93
Q

A plaintiff sued an individual defendant for injuries suffered in a collision between the plaintiff’s car and the defendant’s truck while the defendant’s employee was driving the truck. The plaintiff sought discovery of any accident report the employee might have made to the defendant, but the defendant responded that no such report existed. Before trial, the defendant moved to preclude the plaintiff from asking the defendant in the presence of the jury whether he had destroyed such a report, because the defendant would then invoke his privilege against self-incrimination.

Should the court allow the plaintiff to ask the defendant about the destruction of the report?

(A) No, because a report that was prepared in anticipation of litigation is not subject to discovery.
(B) No, because no inference may properly be drawn from invocation of a legitimate privilege.
(C) Yes, because a party in a civil action may not invoke the privilege against self-incrimination.
(D) Yes, because the defendant’s destruction of the report would serve as the basis of an inference adverse to the defendant.

A

(D) Yes, because the defendant’s destruction of the report would serve as the basis of an inference adverse to the defendant.

If a party destroys evidence, it is proper for the jury to draw an inference that the evidence was adverse to that party’s case. It is also proper for the jury to draw an adverse inference in a civil case from a party’s assertion of the privilege against self-incrimination. Thus, the court should allow the question to be asked, because it is proper regardless of how the defendant responds.

Note: The privilege against self-incrimination may be asserted in both civil and criminal cases so long as the statement made in response to the question posed could tend to incriminate the person in a criminal prosecution.

How well did you know this?
1
Not at all
2
3
4
5
Perfectly
94
Q

A plaintiff sued a defendant for injuries allegedly suffered when he slipped and fell on the defendant’s business property. At trial, without asking that the defendant’s property manager be declared a hostile witness, the plaintiff called him solely to establish that the defendant was the owner of the property where the plaintiff fell. On cross-examination of the manager, the defendant’s attorney sought to establish that the defendant had taken reasonable precautions to make the property safe for business invitees.
Should the defendant’s cross-examination of the manager be permitted over the plaintiff’s objection?

(A) No, because cross-examination should be limited to the subject matter of the direct examination and matters affecting the credibility of the witness.
(B) No, because the court has not declared the manager hostile.
(C) Yes, because the cross-examiner is entitled to explore matters relevant to any issue in the case, including credibility.
(D) Yes, because the manager is the agent of a party, as to whom the scope of cross-examination is unlimited.

A

(A) No, because cross-examination should be limited to the subject matter of the direct examination and matters affecting the credibility of the witness.

Note: cross-examination should be limited to the subject matter of the direct examination and matters affecting credibility. Although the court has discretion under this rule to permit inquiry into additional matters, the defendant is not “entitled” to a wider scope of cross- examination.

How well did you know this?
1
Not at all
2
3
4
5
Perfectly
95
Q

A defendant was charged with assault after being involved in a barroom fight in the middle of the day. The defendant admitted to being at the bar at the time of the fight, but claimed that he was only a bystander. At the defendant’s trial, the prosecutor intended to call the defendant’s former employer. The employer was to testify that the defendant had been fired and was not working at the time of his arrest. The defendant objected to the employer’s testimony.

How should the court rule?

A) Overrule the objection, because it tends to make it more likely that the defendant was at the bar in the middle of the day and involved in the fight.
B) Overrule the objection, because the employer will be testifying based on his personal knowledge of the defendant’s employment.
C) Sustain the objection on the basis that the employer’s testimony is unfairly prejudicial.
D) Sustain the objection on the basis that the employer’s testimony is not probative of a material fact.

A

C) Sustain the objection on the basis that the employer’s testimony is unfairly prejudicial.

Relevant evidence may be excluded if its probative value is substantially outweighed by the danger of unfair prejudice. Here, the employer’s testimony includes the fact that the employee was fired. This fact is highly prejudicial, as it would tend to discredit the defendant in the minds of the jury. Moreover, the probative value of the employer’s testimony is somewhat minimal.

Note: the employer’s testimony regarding the defendant’s unemployment is relevant to whether the defendant was present at the bar (and therefore potentially involved in the fight). The mere fact that the defendant has stipulated to his presence at the bar does not automatically preclude the prosecution from presenting the employer’s testimony, which also suggests the defendant was at the bar.

How well did you know this?
1
Not at all
2
3
4
5
Perfectly
96
Q

While on her morning run, a runner was struck by a driver’s vehicle as she crossed the street, suffering a broken arm in the resulting fall to the ground. The driver was allegedly exceeding the speed limit because he was late for work. In a civil action for negligence against the driver, the runner alleges that the driver’s speeding at the time of the accident constitutes negligence per se, but the driver denies that he was speeding. The runner intends to introduce the testimony of the driver’s neighbor that the driver has a reputation for speeding regularly in their neighborhood.

Should the court allow the runner to introduce the proposed testimony of the driver’s neighbor?

A) No, because it constitutes improper character evidence.
B) No, because the driver’s tendency for speeding can only be established through specific instances of conduct.
C) Yes, because the testimony is admissible to show that the driver was likely speeding at the time of the accident.
D) Yes, because the driver’s reputation for speeding is an essential element of the runner’s claim against the driver.

A

A) No, because it constitutes improper character evidence.

In a civil case, evidence of a person’s character (or character trait) generally is inadmissible to prove that the person acted in accordance with that character (or character trait) on a particular occasion. In this case, the runner is attempting to admit testimony regarding the driver’s reputation for speeding in order to show that the driver was likely speeding at the time of the accident

How well did you know this?
1
Not at all
2
3
4
5
Perfectly
97
Q

defendant was on trial for murder. The pastor of his church was called as a character witness. The pastor testified that the defendant has a reputation in the community for peacefulness. On cross-examination, the prosecutor, seeking to impeach the pastor and having a good-faith basis for his question, asked the pastor if he had a violent temper.

Of the following, which is the defense attorney’s best basis for objecting to this impeachment question?

A) The question exceeds the scope of the direct examination of the pastor.
B) The question does not relate to the defendant’s character.
C) The question calls for impermissible character evidence.
D) The question violates the limitation imposed on evidence of a witness’s religious beliefs or opinions.

A

C) The question calls for impermissible character evidence.

The prosecutor is attempting to show that the pastor, a violent person, is a bad person and therefore should not be believed. Because this is not directly related to the pastor’s truthfulness, it is impermissible character evidence.

Note: any witness may be impeached by certain types of character evidence

How well did you know this?
1
Not at all
2
3
4
5
Perfectly
98
Q

At his trial for larceny, a defendant called his brother as a character witness. The brother testified that the defendant had a reputation in the community for being an honest man. During rebuttal, the prosecutor called the defendant’s former employer to testify that the defendant lied on his job application. The defense attorney objected, arguing that the testimony is an improper use of character evidence.

How should the judge rule on the defense attorney’s objection?

A) Sustain the objection, because the evidence has no probative value on any issue in the case.
B) Sustain the objection, because the evidence can be inquired into only during cross-examination of the brother.
C) Overrule the objection, because the defense has “opened the door” to the prosecutor’s admission of this evidence.
D) Overrule the objection, because the evidence offered involves dishonesty.

A

B) Sustain the objection, because the evidence can be inquired into only during cross-examination of the brother.

on cross-examination, the prosecution may question a defendant’s character witness about specific instances of the defendant’s conduct. Here, the prosecution could only introduce the specific instance of conduct by **asking ** the brother about the conduct on cross-examination. This evidence cannot be introduced by extrinsic evidence

Note: Although the defendant “opened the door” by having his brother testify as to his character, thus permitting the prosecution to rebut that testimony, the prosecution is limited to asking the brother about the specific instance of conduct on cross-examination, and cannot introduce extrinsic evidence of the defendant lying on a job application

How well did you know this?
1
Not at all
2
3
4
5
Perfectly
99
Q

A baker and a chef ran a catering business as a limited liability company (LLC) with the two as the only members of the LLC, and the chef as the managing member. Recently, the chef sought an equitable accounting of the assets of the LLC. The court referred the matter to a referee to hear evidence and make findings of facts with regard to the matter. One matter of contention was how much money the baker had received from the business’s proceeds in cash. The baker testified that each time he took money from the business, he logged the withdrawals in a notebook, which was offered into evidence by the baker without objection from the chef. On the witness stand, the chef admitted that she had access to the notebook, which was kept in a safe on the premises of the business, and never made nor altered any entries in it. She further testified that on several specific dates, she remembers watching the baker take $500 cash from the business. This was a larger sum of money than the amount recorded in the notebook admitted into evidence. The baker immediately objected.

Is the chef’s testimony admissible?

A) No, because the best evidence rule will not allow the baker to contradict the notebook.
B) No, because the contents of the notebook are at issue.
C) Yes, because the notebook is not a document with legal effect.
D) Yes, because the testimony is based on the chef’s firsthand knowledge.

A

D) Yes, because the testimony is based on the chef’s firsthand knowledge.

A non-expert witness must have personal knowledge of a matter in order to testify about that matter. Personal knowledge may be established by the witness’s own testimony as well as through other means.

Note: Whether the contents are at issue does not impact the chef’s ability to introduce evidence based on his personal knowledge that contradicts the document.

How well did you know this?
1
Not at all
2
3
4
5
Perfectly
100
Q

A defendant in a federal securities case introduced the testimony of a witness who had claimed on direct examination that the defendant had no prior knowledge of a change within a corporation’s executive board; the defendant’s knowledge of this fact was a central issue in the case. The prosecutor did not cross-examine the witness. On rebuttal, to impeach the defendant’s witness, the prosecutor called a witness who testified that she had heard the defendant’s witness say that the defendant knew of the change. Further, the prosecutor introduced a properly authenticated email that the defendant’s witness had sent to the witness containing the same information. The defendant’s attorney objects on the grounds that the testimony of the prosecutor’s witness and the email are inadmissible to impeach the defendant’s witness.

Should the court admit the testimony of the prosecutor’s witness and the email?

A) Yes, because the defendant’s witness may be properly impeached with them.
B) Yes as to the testimony, but no as to the email, because the prosecutor did not present the email to the defendant’s witness on her cross-examination of him.
C) No, because the defendant’s witness was not given an opportunity to explain the evidence before introduction of the prosecutor’s witness.
D) No, because the testimony and email are immaterial.

A

A) Yes, because the defendant’s witness may be properly impeached with them.

Because these statements are being used to impeach the witness and not to prove the truth of the matter asserted, they are not hearsay. Also, because the statements were not made under oath in a prior proceeding, they may not be considered as substantive evidence

How well did you know this?
1
Not at all
2
3
4
5
Perfectly
101
Q

A customer filed suit against a coffee maker’s manufacturer for injuries she sustained when the coffee maker exploded and severely injured her, under both negligence and strict liability theories. The judge granted summary judgment in favor of the manufacturer on the negligence claim and granted summary judgment in favor of the customer on the strict liability claim. The case then proceeded to jury trial on the issue of damages only. At trial, the customer qualified an expert witness who opined that there was a reasonable alternative design for the coffee maker and that the manufacturer’s failure to use that design probably rendered the product not reasonably safe. The manufacturer objects to the expert’s testimony.

What is the strongest basis for the manufacturer’s objection?

A) The testimony needlessly presents cumulative evidence.
B) The testimony concerns an ultimate issue in this case.
C) The testimony is irrelevant.
D) The expert does not possess the requisite degree of certainty in his opinion.

A

C) The testimony is irrelevant.

The judge has already ruled on the negligence and strict liability issues, and the only remaining issue is the amount of damages to which the customer is entitled. Consequently, the expert’s testimony as to whether there was design defect in the coffee maker is not relevant to this issue

Note: The issue of whether there was a reasonable alternative design was never before the jury because of the judge’s ruling at the summary judgment stage. Because the jury has not heard any other evidence on this issue, the expert’s testimony on the subject is not cumulative.

How well did you know this?
1
Not at all
2
3
4
5
Perfectly
102
Q

A plaintiff sued a defendant for fraudulent representations allegedly made by the defendant in selling an automobile to the plaintiff. At trial, the plaintiff testified that he received a letter from the defendant the day before the sale occurred in which the defendant wrote that the brakes on the automobile had just been replaced the week before. The defendant objected to the testimony.

May the court properly admit the testimony over the defendant’s objection?

A) No, because the statement constitutes hearsay.
B) No, because the letter was not properly authenticated.
C) No, because the best evidence rule applies.
D) Yes, because the statement does not constitute hearsay.

A

C) No, because the best evidence rule applies.

Here, the plaintiff is attempting to prove the contents of the letter sent by the defendant. There is no indication that the original is lost, destroyed, or in the possession of the defendant. The matter is not collateral, but central to the question of whether fraud occurred.

Note: the plaintiff was not actually trying to put the letter into evidence. Only when a document is being placed into evidence would it have to be authenticated.

How well did you know this?
1
Not at all
2
3
4
5
Perfectly
103
Q

A defendant was convicted of cocaine distribution. At the defendant’s sentencing hearing, the prosecution called the defendant’s wife to testify regarding a statement made by the defendant about the amount of cocaine the defendant had purchased from a supplier. The amount of cocaine possessed by the defendant was relevant to the length of defendant’s sentence. The defendant objected on the grounds that the statement was made during a private conversation on their honeymoon.

Should the court permit the defendant’s wife to testify as to his prior statement?

A) No, because the statement is hearsay.
B) No, because the statement is privileged.
C) Yes, because the Federal Rules of Evidence do not apply to sentencing proceedings.
D) Yes, because the testifying spouse holds the privilege not to testify regarding the communication.

A

B) No, because the statement is privileged.

Note: While the Federal Rules of Evidence generally do not apply to sentencing proceedings, the rules regarding privileges apply to all federal court cases and proceedings, including sentencing proceedings.

How well did you know this?
1
Not at all
2
3
4
5
Perfectly
104
Q

A lawyer represented both an internist and a hospital that employed him in a pending action against them based on the alleged negligence of the internist in treating a patient. A stenographer employed by the attorney attended a meeting between the lawyer, the internist, and the hospital. The stenographer recorded what was said at the meeting. Subsequently, the hospital filed a cross-claim against the internist seeking indemnity for any damages assessed against it. The lawyer was compelled to withdraw her representation of both parties before trial. At trial, the internist called the stenographer to testify regarding a statement made by the hospital’s president during the meeting that the hospital would “back the internist 100 percent.” The hospital, asserting the attorney-client privilege, objected to this testimony.

Should the court overrule this objection?

A) Yes, because the lawyer no longer represents either party.
B) Yes, because the dispute involves the hospital and the internist.
C) No, because the stenographer was an employee of the lawyer.
D) No, because the presence of the stenographer did not eliminate the confidentiality of the statement.

A

B) Yes, because the dispute involves the hospital and the internist.

Confidential communications made by a client for the purpose of obtaining or providing legal assistance for the client is generally protected by the attorney-client privilege. BUT this privilege does not apply to communications between former co-clients who are now adverse to each other.

How well did you know this?
1
Not at all
2
3
4
5
Perfectly
105
Q

During a defendant’s criminal trial for assault, a witness for the defendant testified that the defendant had told her that he had never even met the victim. The defendant was acquitted. Subsequently, the victim brought a civil assault claim against the same defendant based on the same alleged assault. The same witness was called as a witness for the victim, and she testified that, shortly after the assault took place, the defendant had admitted to her that he did assault the victim. On cross-examination, the defendant’s counsel asked the witness whether she had testified in the defendant’s prior criminal prosecution that the defendant told her that he had never even met the victim. The victim’s attorney objected to the defendant’s question.

May the court allow the question over the objection?

A) Yes, but only to prove that the defendant did not commit the assault.
B) Yes, but only to impeach the credibility of the witness.
C) Yes, both to prove that the defendant did not commit the assault and to impeach the credibility of the witness.
D) No, because the defendant did not first allow the witness the opportunity to explain or deny the statement.

A

C) Yes, both to prove that the defendant did not commit the assault and to impeach the credibility of the witness.

The witness’s prior inconsistent statement is admissible to impeach the witness’s credibility. The witness’s prior testimony is also admissible substantively as non-hearsay because her statement was made under oath at the prior criminal proceeding.

How well did you know this?
1
Not at all
2
3
4
5
Perfectly
106
Q

At a criminal trial for rape, the prosecution seeks to admit into evidence a statement made by the defendant to his friend, acknowledging that he raped the victim. The defendant made the statement shortly after the rape, but before he knew he was a suspect. The defendant testified at trial that he did not rape the victim.

Is the statement the defendant made to his friend acknowledging the rape admissible?

A) Yes, the statement is admissible as non-hearsay.
B) Yes, the statement is admissible because it falls within the statement against interest exception to the hearsay rule.
C) No, the statement is inadmissible hearsay not within any exception.
D) No, because the statement was not against the defendant’s interest at the time he made it.

A

A) Yes, the statement is admissible as non-hearsay.

Opposing Party Statement: a statement made by a party to the current litigation is not hearsay if it is offered by an opposing party.

Here, the defendant is a party to the current litigation, and the prosecution is seeking to use the defendant’s own statement against him.

Note: statement against interest exception to the hearsay rule only applies if the declarant is unavailable to testify at the present trial

How well did you know this?
1
Not at all
2
3
4
5
Perfectly
107
Q

A defendant was charged with involuntary manslaughter. The defendant testified that he did not shove the victim, but that instead she had tripped and fell down the stairs. Subsequently, the prosecution called a police officer who had talked with the defendant at the scene of the incident. The police officer testified that, on the day of the incident, the defendant told the officer that he had shoved the victim before she fell down the stairs. The prosecution had not asked the defendant about this statement when he was on the witness stand.

Is this statement admissible as substantive evidence of the defendant’s actions?

A) No, because the prosecution did not ask the defendant about the statement when the defendant was on the witness stand.
B) No, because the statement was not made under oath.
C) Yes, because it contradicts the defendant’s testimony.
D) Yes, because the defendant made the statement to the police officer.

A

D) Yes, because the defendant made the statement to the police officer.

A statement made by a party to the current litigation is not hearsay if it is offered by an opposing party, even though the statement is being offered to prove the truth of the matter asserted. Consequently, the defendant’s statement to the police officer that the defendant shoved the victim before she fell down the stairs is admissible, even though if made by someone else, it would be inadmissible hearsay

Note: Although a prior inconsistent statement made by a witness generally must have been made under oath in order to be admitted as substantive evidence, this is not the case with regard to an opposing party’s statement

How well did you know this?
1
Not at all
2
3
4
5
Perfectly
108
Q

The owner of a dog was inside while her dog was outside in a fenced-in yard. Hearing a child crying, the owner looked out a window and noticed her neighbor’s three-year-old child inside the owner’s yard. The owner went outside and discovered that the child’s face had been deeply scratched by the owner’s dog. The owner took the child to the neighbor’s home, telling the child’s mother, “My dog scratched your child’s face. I’m liable for his injuries.”

Are either of the owner’s statements admissible at trial in an action brought by the child’s parents on the child’s behalf against the owner to recover damages?

A) No, as to both statements.
B) Yes, but only the first statement because the second statement is an opinion by a lay witness about the ultimate issue in the case.
C) Yes, but only the second statement because the owner did not personally witness the event.
D) Yes, both statements are admissible because they were made by the owner.

A

D) Yes, both statements are admissible because they were made by the owner.

A statement made by a party to the current litigation is not hearsay if it is offered by an opposing party.

Note: An opposing party’s statement in the form of an opinion may be admitted even if the statement is about a matter that normally would be beyond the scope of lay witness opinion testimony, such as an opinion regarding the ultimate issue in the case.

How well did you know this?
1
Not at all
2
3
4
5
Perfectly
109
Q

A man was on trial for bank robbery and felony murder. The prosecution alleged that while the man went into a bank wearing a ski mask and wielding a gun, his getaway driver waited in a car outside. The man shot a bank patron during the robbery then got into the getaway car with the money. In the high-speed chase that followed, the getaway car crashed and the getaway driver died.

At trial, the defense called a bartender who talked to the getaway driver the night before the robbery. The bartender testified that the getaway driver had told him that because the man refused to join him in the robbery unless they used a fake gun, the getaway driver had given him a real gun to use and told him it was fake. The court ruled that the statement was admissible as a statement against the getaway driver’s interest.

The prosecution now wishes to call a woman to testify that the getaway driver had told her a week before the robbery that the man had purchased a gun and was planning on robbing a bank.

Is the woman’s testimony admissible?

A) No, because the getaway driver had no opportunity to explain or deny the statement.
B) No, because the getaway driver’s statement was not made under oath.
C) Yes, to impeach the getaway driver.
D) Yes, under a hearsay exception for an unavailable declarant.

A

C) Yes, to impeach the getaway driver.

Here, the getaway driver’s hearsay statement—that he gave the man a real gun but told him it was fake because the man had demanded that they use a fake gun—was admitted as a statement against interest. Therefore, the prosecution can offer testimony regarding the getaway driver’s inconsistent statement to the woman—that the man had purchased a real gun to use during the robbery—for the purpose of impeaching the getaway driver.

How well did you know this?
1
Not at all
2
3
4
5
Perfectly
110
Q

A defendant is on trial for felony assault. Eleven years ago, the defendant was convicted of rape. He was sentenced to three years imprisonment and served the full term. The defendant decided to testify at his current trial for felony assault, and on cross-examination, the prosecution seeks to admit evidence of the defendant’s rape conviction to impeach him.

Which of the following best states the standard that must be met to determine the admissibility of this conviction?

A) The conviction is admissible only if its probative value outweighs the prejudicial effect to the defendant.
B) The conviction is admissible only if its probative value substantially outweighs its prejudicial effect.
C) The court has discretion to exclude the conviction if the defense shows that its probative value is substantially outweighed by its prejudicial effect.
D) The court must exclude the conviction if the defense shows that its probative value is substantially outweighed by its prejudicial effect.

A

A) The conviction is admissible only if its probative value outweighs the prejudicial effect to the defendant.

A conviction for a felony not involving dishonesty that is not more than 10 years old is admissible against a criminal defendant-witness if the conviction’s probative value outweighs its prejudicial effect.

How well did you know this?
1
Not at all
2
3
4
5
Perfectly
111
Q

The user of a power tool sued the tool’s manufacturer in state court. The action was based on a strict product liability claim that the manufacturer’s failure to adequately warn the user of a defect in the power tool caused the user’s injury. The manufacturer properly removed the case to federal court. The applicable law of the state that governs the existence of the strict product liability claim also recognizes a rebuttable heeding presumption. This presumption assumes that an injured plaintiff would have heeded an adequate warning if one had been given. Under state law, this presumption does not shift the burden of persuasion on this issue to the manufacturer.

The manufacturer did not present evidence that the user would not have heeded a different warning had it been given. The court instructed the jury that it must apply the presumption that the warning, if given, would have been heeded.

Is the court’s instruction correct?

A) No, because the jury may, but is not required to, apply the presumption.
B) No, because state law presumptions are not recognized in a federal diversity action.
C) Yes, because the manufacturer failed to offer evidence to rebut the presumption.
D) Yes, because the Federal Rules of Evidence apply the bursting-bubble approach to presumptions.

A

C) Yes, because the manufacturer failed to offer evidence to rebut the presumption.

A rebuttable presumption: conclusion that can be overcome if the opposing party produces sufficient contrary evidence. However, if no contrary evidence is produced, the judge should instruct the jury that it must apply the presumption

In federal cases in which state law supplies the rule of decision for a claim or defense (i.e., in diversity cases), it also governs the effect of presumptions related to the claim or defense.

Because the manufacturer failed to offer evidence to rebut the presumption that the user would have heeded a warning on the power tool, the court correctly instructed the jury that it must apply the presumption.

How well did you know this?
1
Not at all
2
3
4
5
Perfectly
112
Q

A security guard was charged with criminal battery of a student. The prosecution alleged that the security guard used excessive force when he removed the student from a campus event after the student became disruptive. The security guard’s only argument in his defense was that he exerted lawful force to remove the disruptive student from the event pursuant to his duties as a security guard. The security guard testified that the student had a reputation on campus for disrupting campus events by starting physical fights with other students.

Is the security guard’s testimony regarding the student’s reputation admissible?

A) No, because the prosecution has not presented evidence of the security guard’s bad character.
B) No, because the security guard has not asserted a theory of self-defense.
C) Yes, because the security guard has personal knowledge of the student’s reputation for violence.
D) Yes, because the student’s reputation for violence is relevant to whether the student was the initial aggressor.

A

B) No, because the security guard has not asserted a theory of self-defense.

A criminal defendant may introduce evidence of an alleged victim’s character—but only if it is pertinent to the case.

Here, the security guard’s only defense to the criminal battery charge is that he exerted lawful force to remove the disruptive student from the campus event. In support of that defense, the guard sought to testify that the student has a reputation for disrupting campus events by starting physical fights with other students. Although that character evidence would be pertinent to a self-defense claim, it is not pertinent to whether the guard’s level of force was lawful

How well did you know this?
1
Not at all
2
3
4
5
Perfectly
113
Q

A property owner constructed a fence along what he thought was the boundary to his property. A year later, a neighbor who was selling adjoining property had a survey conducted. As a consequence of the survey, the neighbor brought an action against the property owner seeking removal of the fence.

At trial, the neighbor testified that he had orally objected to the property owner about the placement of the fence at the time it was constructed. After the neighbor left the witness stand, the property owner sought to introduce into evidence a certified copy of the official judgment and conviction of the neighbor for perjury. The neighbor was convicted 11 years ago and released from prison nine years ago. The property owner had not asked the neighbor about the conviction while the neighbor was on the stand.

Should the court permit the introduction of the judgment for the purpose of impeaching the neighbor’s testimony?

A) No, because the neighbor was convicted of perjury more than 10 years ago.
B) No, because the neighbor was not questioned about the conviction while the neighbor was on the witness stand.
C) Yes, because a conviction used to impeach a witness’s character for truthfulness may be proved by extrinsic evidence.
D) Yes, because a witness in a civil case may not be impeached with a previous conviction.

A

C) Yes, because a conviction used to impeach a witness’s character for truthfulness may be proved by extrinsic evidence.

Convictions for crimes of dishonesty are automatically admissible to impeach any witness if the conviction—or the release from incarceration—is not more than 10 years old, and use of extrinsic evidence is allowed for that purpose. Therefore, the court should permit the introduction of the judgment.

How well did you know this?
1
Not at all
2
3
4
5
Perfectly
114
Q

A plaintiff brought suit against a defendant for injuries she sustained in a car accident that she accused the defendant of negligently causing. Prior to filing suit, the plaintiff’s attorney had the plaintiff visit a physician to determine the extent of her injuries for purposes of determining the damages to be claimed in the lawsuit. After the plaintiff’s examination, while the attorney, plaintiff, and physician were discussing the extent of the plaintiff’s injuries, the plaintiff admitted that she “may have had a few beers” right before the accident. At trial, the defendant’s counsel sought to call the doctor to testify about the statement. The plaintiff properly objected to the introduction of this testimony.

How should the judge rule on the plaintiff’s objection?

A) Sustain the objection, as the attorney-client privilege is applicable.
B) Sustain the objection, as the physician-patient privilege is applicable.
C) Overrule the objection, as the statement was made by an opposing party.
D) Overrule the objection, as the physician would constitute an expert witness.

A

A) Sustain the objection, as the attorney-client privilege is applicable.

Here, the plaintiff’s attorney had the plaintiff visit a physician (an expert) to ascertain the extent of her injuries for purposes of determining the damages to be claimed in the lawsuit.

This means that the physician was hired to assist the attorney in the rendition of professional legal services by providing medical consultation. And since there is no indication that the physician was hired to testify at trial, the plaintiff’s statement in the physician’s presence remains protected by the attorney-client privilege

How well did you know this?
1
Not at all
2
3
4
5
Perfectly
115
Q

A plaintiff was shot by the defendant while the two were hunting. The plaintiff filed a negligence action against the defendant to recover damages for injuries suffered as a result of the incident. In his case-in-chief, the plaintiff seeks to introduce the testimony of a mutual friend of both the plaintiff and the defendant that, in the friend’s opinion, the defendant is habitually a careless person. Is this evidence admissible?

A) Yes, because it is has a tendency to make it more probable that the defendant was negligent in shooting the plaintiff.
B) Yes, as habit evidence.
C) No, because it is inadmissible character evidence.
D) No, because character evidence may not be presented in the form of opinion testimony.

A

C) No, because it is inadmissible character evidence.

Here, evidence that the defendant was a careless person is not admissible to prove that the defendant was careless at the time the plaintiff was shot.

Note: Although the fact that the defendant is careless may be probative in that it has a tendency to make it more probable that the defendant was negligent in the matter at hand, character evidence is inadmissible to prove that a person acted in accordance with that character on a particular occasion – improper purpose.

How well did you know this?
1
Not at all
2
3
4
5
Perfectly
116
Q

A customer at a grocery store slipped and fell on a wet spot on the floor. The customer sued the store to recover for the resulting injuries to her back. At trial, the store’s lawyer sought to prove that the damage to the customer’s back was not serious, and called the customer’s neighbor to testify regarding the severity of the customer’s injuries. The neighbor testified that on the morning after the fall, the customer walked to her car wearing a tennis dress and carrying a tennis racket. The neighbor then added, “I told my sister, ‘There goes [the customer] off for her weekly tennis match.’” The customer’s lawyer moves to the strike this statement as inadmissible hearsay.

How should the court rule on this motion?

A) Deny the motion, because, since the declarant is testifying, the statement is not hearsay.
B) Deny the motion, because the neighbor was describing an event as it was taking place.
C) Grant the motion, because the statement is being offered for its truth.
D) Grant the motion, because the sister can testify to the neighbor’s statement.

A

B) Deny the motion, because the neighbor was describing an event as it was taking place.

Present Sense Impression Exception: DESCRIBING event while perceiving it

How well did you know this?
1
Not at all
2
3
4
5
Perfectly
117
Q

The defense attorney in a criminal rape case wants to introduce testimony that the alleged victim has a reputation in the community for promiscuity. The testimony will come from one of the victim’s past sexual partners. The prosecutor objects to the introduction of the testimony, but the defense maintains that because consent is at issue in the case, the testimony is relevant.

Should the court allow the testimony?

A) Yes, because reputation testimony is an admissible form of character evidence.
B) Yes, because the victim’s past sexual behavior can be used to prove consent.
C) No, because evidence of a victim’s past sexual behavior is inadmissible.
D) No, because this evidence of the victim’s other sexual behavior is not relevant.

A

D) No, because this evidence of the victim’s other sexual behavior is not relevant.

Some instances in which a victim’s past sexual behavior may be relevant and admissible:

1) evidence of a victim’s past sexual conduct is admissible to show the victim’s past sexual behavior with the defendant in order to prove consent

2) to prove an alternate source of semen or injury

3) when the constitutional rights of the defendant require admission of the evidence

4) Civil cases –> when the probative value substantially outweighs danger of harm to the victim

5) Civil Cases –> when the victim herself has placed her reputation in controversy.

How well did you know this?
1
Not at all
2
3
4
5
Perfectly
118
Q

A few weeks before the beginning of a murder trial, a witness to the murder identified the defendant in a photo array as the person who killed the victim. Between the time of the photo array and the trial, the witness died. The witness was the only eyewitness to the crime, aside from the victim and the murderer, so the prosecution wants to admit at the trial the witness’s identification of the defendant from the photo array. The defendant’s attorney objects to the introduction of the statement, but the prosecutor claims that as a prior statement of identification, it is admissible.

Is the statement admissible as a prior statement of identification?

A) Yes, because prior statements of identification are admissible as non-hearsay.
B) Yes, because the witness is unavailable to testify.
C) No, because the witness did not testify at the present trial.
D) No, because the witness did not testify under oath at a previous trial, hearing, or deposition as to the identity of the murderer.

A

C) No, because the witness did not testify at the present trial.

A previous out-of-court identification of a person after perceiving that person is admissible as substantive evidence, but only if the witness testifies at the present trial or hearing and is subject to cross-examination concerning the identification

How well did you know this?
1
Not at all
2
3
4
5
Perfectly
119
Q

Prior to trial, a criminal defendant filed a motion to prevent the prosecution from introducing statements by an alleged co-conspirator. At the hearing on the motion, the court definitively ruled that the statements could be admitted into evidence. At the jury trial, the prosecution called the alleged co-conspirator to testify about the statements. The defendant did not object to the introduction of the statements into evidence. On appeal, the defendant claimed that the trial court erred in admitting the statements.

Should the appellate court decide the merits of this claim of error?

A) No, because the defendant failed to object at trial when the prosecution introduced the statements.
B) No, because the admissibility of evidence is properly a question for a trial court.
C) Yes, because the court made a definitive ruling on the record admitting the statements.
D) Yes, because an objection was unnecessary under the plain error rule.

A

C) Yes, because the court made a definitive ruling on the record admitting the statements.

Once the court makes a definitive ruling on the admissibility of evidence, a party need not renew an objection to the admission of the evidence, even if the ruling was made before the trial began.

Note: Plain error rule allows a party to appeal after failing to object to the introduction of evidence.

It does not apply in this case because the defendant did object to the introduction of the statements before trial. Consequently, the appellate court’s review of the admissibility of the statements is not confined to the plain error rule.

How well did you know this?
1
Not at all
2
3
4
5
Perfectly
120
Q

In a negligence action tried before a jury, a neighbor of the defendant who witnessed the plaintiff slip on the defendant’s sidewalk is called to testify as to what he saw. The neighbor, who normally wears glasses, did not have them on at the time of the incident. The neighbor is first questioned about the weather conditions at the time that the plaintiff slipped and about the plaintiff’s behavior before doing so. The neighbor is then asked whether the plaintiff’s conduct was negligent.

Of the following, which is the best ground upon which the opposing party can object to this question regarding negligence?

A) As a lay witness, the neighbor cannot testify as to an opinion.
B) The neighbor was not wearing his glasses at the time of the incident.
C) The neighbor’s opinion is not helpful to the jury’s clear understanding of his testimony.
D) The neighbor’s opinion is not rationally based on his perception.

A

C) The neighbor’s opinion is not helpful to the jury’s clear understanding of his testimony.

Here, the neighbor is being asked to give a legal conclusion without an understanding of the legal criteria for that conclusion. As such, the neighbor’s opinion is not helpful to the jury.

How well did you know this?
1
Not at all
2
3
4
5
Perfectly
121
Q

A high school teacher played on a hockey team in a local recreational league. During a league game, the teacher was involved in a fight with another hockey player. That player sued the teacher in a battery action to recover for injuries inflicted during the fight. The teacher contended that he had acted in self-defense. The teacher called his principal to testify that the teacher had a reputation within the school community for peacefulness. The plaintiff, who had not introduced evidence of the teacher’s character for violence, objected to this testimony. Should the court admit this testimony?

A)Yes, because the defendant is entitled to introduce evidence of a pertinent good character trait.
B) Yes, because character evidence may be introduced through reputation testimony.
C) No, because the plaintiff had not introduced evidence of the teacher’s character for violence.
D) No, because such evidence is not admissible in a civil action.

A

D) No, because such evidence is not admissible in a civil action.

Student SUED the teacher = civil, not criminal case

Evidence of a defendant’s character is inadmissible in a civil case to prove that the defendant acted in conformity with that character trait unless the defendant’s character is an essential element of a claim or defense. Since the defendant’s character for peacefulness is not an element of either battery or self-defense, the principal’s testimony is not admissible.

Pertinent civil traits: negligent hiring, defamation, fraud, child custody

Note: a defendant is permitted to introduce evidence of a pertinent good character trait in a criminal case, such evidence is not admissible in a civil case.

How well did you know this?
1
Not at all
2
3
4
5
Perfectly
122
Q

A plaintiff brought an action against a defendant for damages to her vehicle resulting from a car accident. To prove her damages, the plaintiff introduced a dated and authenticated invoice from the mechanic who repaired her car after the accident. After the invoice was admitted into evidence, the woman sought to enter into evidence a printout of a digital photograph of her vehicle at the scene of the accident. If admitted, she plans to testify that she called her brother immediately after the accident so he could meet her at the scene and take photographs. She will also testify that her brother took this photograph the day of the accident, and that the photograph fairly and accurately represents the condition of her vehicle and its position in the intersection after colliding with the defendant’s vehicle. Should the court rule that the photograph is admissible?

A) No, because the photograph is needlessly cumulative following the admission of the invoice.
B) No, because the plaintiff must call her brother to authenticate the photograph.
C) Yes, because a digital photograph is self-authenticating.
D) Yes, because the plaintiff has personal knowledge of the accuracy of the photograph.

A

D) Yes, because the plaintiff has personal knowledge of the accuracy of the photograph.

All tangible evidence must be authenticated. To authenticate an item, the proponent must produce sufficient evidence to support a finding that the thing is what its proponent claims it is. When reproductions (e.g., photographs, diagrams, maps, movies) are introduced into evidence, they may be authenticated by the testimony of a witness with personal knowledge that the object accurately depicts what its proponent claims it does. Here, because the plaintiff was at the scene of the accident, she has personal knowledge to support her testimony that the photograph fairly and accurately represents the condition of the vehicle and its position in the intersection after colliding with the defendant’s vehicle.

Note: Although the plaintiff has already admitted the invoice as evidence of her damages, the photograph of her vehicle at the scene of the accident could include additional evidence not available on the invoice that may be relevant to her claim for damages.

How well did you know this?
1
Not at all
2
3
4
5
Perfectly
123
Q

An attorney represents a corporation in a federal securities case. As the attorney reviewed her files before court, she discovered that—despite her diligence—a memo marked “PRIVILEGED AND CONFIDENTIAL” had inadvertently been included in a folder containing public financial documents. The attorney knew that she had copied this folder and produced it in its entirety to opposing counsel during discovery. However, the memo is detrimental to her client’s case. The attorney immediately contacted opposing counsel and requested that the memo be returned to her, that all copies be destroyed, and that the information within the memo not be used at trial; she included the judge on this correspondence. Opposing counsel refused to return the memo, and informed the attorney that they did plan to use it at trial. The memo in question was from the corporation’s chief executive officer to the attorney, and contained the chief financial officer’s thoughts and questions regarding the attorney’s trial strategy. Should the court allow opposing counsel to introduce the memo into evidence at trial?

A) Yes, because the attorney waived the privilege when she disclosed the memo to opposing counsel.
B) Yes, because the memo was not privileged to begin with.
C) No, because the attorney did not waive the privilege.
D) No, because all documents from clients to attorneys are privileged.

A

C) No, because the attorney did not waive the privilege.

FRE: disclosure of a protected communication does not operate as a waiver if
(i) the disclosure was inadvertent, (ii) the holder of the privilege took reasonable steps to prevent disclosure, and (iii) the holder promptly took reasonable steps to rectify the error, including contacting the party to whom the communication was disclosed and requesting that they return, sequester, or destroy the information.

Here, the disclosure was inadvertent and that the attorney had acted diligently during discovery. Further, the facts indicate that the attorney immediately notified opposing counsel of her error. As such, the privilege was not waived by her inadvertent disclosure, making answer

How well did you know this?
1
Not at all
2
3
4
5
Perfectly
124
Q

An expert witness was called by the defendant to testify in a murder trial. The expert was to testify that the defendant was not responsible for his actions due to a specific mental defect. On cross-examination, the prosecutor brought to the expert witness’s attention an authoritative book on psychological conditions, judicially noted to be a reliable authority in the field. The book described the symptoms of the mental defect at issue differently than the expert witness had described them, and the prosecutor read the book’s description into evidence. The prosecutor wanted the jury to be able to consider the book’s description as substantive evidence, but the defendant objected that the description could be used for impeachment purposes only, and not as substantive evidence. The prosecutor further wanted to introduce the book itself into evidence; the defendant objected to this as well. Should the court allow the jury to consider the description in the book as substantive evidence, and should the book itself be introduced as evidence?

A) The description should be considered for impeachment purposes only, and the book should not be introduced into evidence.
B) The description should be considered as substantive evidence, and the book should not be introduced into evidence.
C) The description should be considered as substantive evidence, and the book should be introduced into evidence.
D) The book should be introduced into evidence, though the description may be used only for impeachment purposes.

A

B) The description should be considered as substantive evidence, and the book should not be introduced into evidence.

Hearsay Exception: Treatises: A statement contained in published treatises or periodicals on a subject of history, medicine, or other science or art is admissible if
(i) the treatise is established as a reliable authority by the testimony of a witness, expert, or by judicial notice, and
(ii) an expert relied on it during direct examination or it was brought to the expert’s attention on cross-examination.

May be used for substance and impeachment

How well did you know this?
1
Not at all
2
3
4
5
Perfectly
125
Q

In a pre-trial hearing, a judge determined that a defendant’s confession was given voluntarily to a police detective after the detective had given Miranda warnings to the defendant. At this hearing, the defendant testified. At trial, the defense did not contest the defendant’s receipt of Miranda warnings, but sought to question the police detective about the manner in which the defendant was interrogated after receiving the warnings in order to call into question whether the confession was voluntary. The defense does not plan to call the defendant to the witness stand. Should the court permit this line of questioning?

A) No, because there had been a judicial determination that the confession was voluntary.
B) No, because the defense did not challenge the defendant’s receipt of Miranda warnings.
C) Yes, because a party may introduce evidence that is relevant to the weight and credibility of other evidence.
D) Yes, because a defendant may testify at a hearing regarding a preliminary question without being required to testify at trial.

A

C) Yes, because a party may introduce evidence that is relevant to the weight and credibility of other evidence.

Even though a judge has decided that evidence, such as a confession, is admissible, a party may nevertheless introduce other evidence that is relevant to the weight and credibility of the admitted evidence

How well did you know this?
1
Not at all
2
3
4
5
Perfectly
126
Q

A plaintiff initiated a libel action against her former boyfriend after he posted a written statement on his web site accusing the plaintiff of being able to afford a new car only because she was a con artist. At trial, the defendant called a neighbor of the plaintiff to testify that he and the plaintiff had run several cons together to earn money. The plaintiff objected to the testimony. Should the court admit this testimony for the purpose of proving that the plaintiff is a con artist?

A) No, because the introduction of specific bad acts in a civil action is prohibited.
B) No, because the testimony is inadmissible character evidence.
C) Yes, because the evidence is relevant to the defendant’s defense.
D) Yes, because character evidence must be in the form of specific acts rather than reputation or opinion testimony.

A

C) Yes, because the evidence is relevant to the defendant’s defense.

In a defamation action involving a private person and a matter that is not of public concern, as is the case here, truth is a defense. Evidence that the plaintiff has run cons would support the truthfulness of the defendant’s statement that the plaintiff is a con artist.

Note: Truth is a defense to this defamation action, and thus the plaintiff’s character is an essential element of the defense

Civil case relevant traits: negligent hiring, fraud, defamation

How well did you know this?
1
Not at all
2
3
4
5
Perfectly
127
Q

A defendant on trial for forging checks took the stand in his own defense. On direct examination, the defendant denied having forged any checks; he stated that before he graduated from college the year before, he worked in his university’s academic records office, indicating that he was “a trustworthy person.” On cross-examination, the prosecutor asked the defendant if he had falsified records while working in the academic records office. The defendant denied that he had done so. The prosecutor then wanted to call to the stand his former supervisor from the university to testify that she had to investigate the defendant after allegations of misconduct, and that when questioned, he had admitted to her that he had falsified records. The defendant was removed from his position, but no formal charges had been brought against him. Should the prosecutor be allowed to call the defendant’s former supervisor to the stand to testify as to the falsified records?

A) Yes, in order to impeach the defendant and to present propensity evidence.
B) Yes, but only to impeach the defendant.
C) No, because the testimony would contain hearsay.
D) No, because the testimony would be extrinsic.

A

D) No, because the testimony would be extrinsic.

The former supervisor may not testify about the falsified records because it would be impeachment by extrinsic evidence of a specific instance of conduct. A specific instance of conduct, if used to impeach the credibility of a testifying witness, may not be proved by the introduction of extrinsic evidence. The adverse party may cross-examine the witness about the conduct, but must take his answer as he gives it.

Note: when a person is charged with one crime, extrinsic evidence of a specific instance of conduct is inadmissible to establish that the defendant had a propensity to commit that crime. Since the facts do not indicate that the prior bad act is being used as evidence for something circumstantial and relevant, such as motive, common plan or scheme, or identity, the supervisor’s testimony is not admissible as substantive evidence.

How well did you know this?
1
Not at all
2
3
4
5
Perfectly
128
Q

A plaintiff sued a defendant for injuries she sustained when she slipped on a wet floor in the defendant’s restaurant. The plaintiff saw a physician and underwent physical therapy sessions to treat her injuries. During one session, the plaintiff said to her physical therapist, “You know, I saw the ‘Caution, Floor is Wet’ sign before I fell, but I was in such a hurry to get back to my table that I ignored it.” Another patient undergoing physical therapy with another therapist overheard the statement, and informed the defendant, who happened to be his friend. The defendant wants to introduce the testimony of his friend, as whether the plaintiff had notice of the wet floor is at issue in the case. The plaintiff objects to the testimony. Should the court allow the friend to testify as to the plaintiff’s statement?

A) No, because the statement is inadmissible hearsay.
B) No, because the statement is privileged.
C) Yes, because the statement is not hearsay.
D) Yes, because the statement was made for the purposes of medical diagnosis or treatment.

A

C) Yes, because the statement is not hearsay.

Opposing party statement: Exemption (non-hearsay)

The woman was with her physician, but did not make the statement in relation to seeking medical treatment. Thus, it is not privileged.

How well did you know this?
1
Not at all
2
3
4
5
Perfectly
129
Q

In a criminal trial for attempted murder, the prosecutor seeks to introduce a statement made by the victim immediately after he was attacked by the defendant. The victim, very seriously injured, shouted the defendant’s name and said, “I can’t believe you shot me! I’m dying!” At the time of the trial, the victim has mostly recovered from his injuries, but suffered permanent memory loss, has no recollection of the incident at all, and has no recollection of making the statement. The prosecutor seeks to introduce the statement as a dying declaration, but the defendant objects. Should this statement be admissible under the “dying declaration” exception to the hearsay rules?

A) No, the statement is not admissible as a dying declaration.
B) No, because the victim did not die.
C) Yes, because the victim is unavailable due to his inability to remember.
D) Yes, because the proceeding in which the statement will be introduced is a criminal trial.

A

A) No, the statement is not admissible as a dying declaration.

Dying declaration = only available in homicide cases (or civil cases) This was an attempted murder case.

How well did you know this?
1
Not at all
2
3
4
5
Perfectly
130
Q
  1. A customer at a fish market was leaving the store after purchasing an assortment of shrimp, oysters, and scallops. He was walking along the sidewalk in front of the store when he slipped on a piece of eel. He brought suit against the owner of the market claiming that he suffered leg and back injuries. The owner, although admitting that the customer was injured by slipping on the eel, denied negligence and claimed that the customer was contributorily negligent.
    At trial, the owner calls a witness to testify that before the fall he heard someone call out to the customer, “Watch it, buddy, you’re going to step on that piece of fish.”
    The witness’s testimony is
    (A) admissible, because it is relevant as to the customer’s contributory negligence.
    (B) admissible, as a statement of the declarant’s present state of mind.
    (C) inadmissible, as hearsay not within any exception.
    (D) inadmissible, because it was not a spontaneous utterance.
A

i. (A) Proof of utterances and writings may be made with an almost infinite variety of other purposes, not resting for their value upon the veracity of the out-of-court declarant and, hence, falling outside the hearsay classification. Here, choice (A) is correct because the witness’s testimony rests on the non-hearsay ground of evincing knowledge to show circumstantially that the customer had notice or awareness that he was about to step on the fish. Thus, the customer’s knowledge is sought to be used as the basis for a further inference that he was contributorily negligent in failing to heed the warning. Choice (B) is incorrect because the “state of mind” hearsay exception covers only statements that reveal the present mental attitude or physical condition of the declarant. Here, the statement of the declarant reveals nothing about the declarant’s own mental state or physical condition. Choice (C) is not the best answer, because choice (A) identifies a non-hearsay use for the statement. Choice (D) is a trap. The words “spontaneous utterance” tempt one to choose the excited utterance exception of FRE 803(2). The excited utterance exception requires that the declarant speak while under the stress or excitement of a startling event. There is no information in the fact pattern about the declarant’s state of mind.

How well did you know this?
1
Not at all
2
3
4
5
Perfectly
131
Q
  1. While shopping at a grocery store, a customer tripped over a broken tile, fell, and suffered a concussion. A few months after the accident, the customer’s attorney deposed a store employee. In the deposition, the employee testified, “I’d been telling the store manager for years to get that broken tile fixed, but he wouldn’t do it.” The employee died in an automobile accident after being deposed.
    At trial, the deposition should be
    (A) admitted, as a dying declaration.
    (B) admitted, as former testimony.
    (C) not admitted, because it is hearsay not within any exception.
    (D) not admitted, because the employee is not available for cross-examination.
A
  1. (B) Under FRE 804(b) (1), testimony given as a witness at another hearing of the same or different proceeding, or in a deposition taken in compliance with law in the course of the same or another proceeding is admissible as a former testimony hearsay exception. McCormick points out that the courts and textwriters generally favor the wider admission of former testimony because of its elements of special reliability, such as the oath, the solemnity of the occasion, and in the case of transcribed testimony, the accuracy of reproduction of the words spoken. Choice (A) is incorrect because the dying declaration hearsay exception of FRE 804(b)(2) requires that the hearsay statement be (1) made while the declarant’s death was imminent; and (2) concern the cause or circumstances of what the declarant believed to be impending death. Here, the statement has nothing to do with the declarant’s death. Choice (C) is incorrect. The answer meets the definition of hearsay as an out-of-court statement offered for the truth of the matter asserted, and it fits the admissibility criteria of FRE 804(b)(1). Choice (D) seems like an attractive answer, but it is deceptive:
    if testimony meets the requirements of FRE 804(b)(1), it is not necessary for the declarant to be present at triaL for cross-examination. The opportunity for cross- examination would have occurred at the deposition.
How well did you know this?
1
Not at all
2
3
4
5
Perfectly
132
Q
  1. A plaintiff was hit and injured by a falling branch on a sidewalk in front of a convenience store. The plaintiff calls a woman to testify. The woman proposes to testify that a witness who is now deceased told the woman that when he was walking past the convenience store one week before the plaintiff was hit by the branch, the witness saw an employee of the convenience store shoveling snow from the sidewalk in front of the store.
    The woman’s testimony is
    (A) admissible, because the witness is unavailable to testify.
    (B) admissible, as relevant for the limited purpose of proving that the convenience store owner retained control over the sidewalk where the accident occurred.
    (C) inadmissible, because it is hearsay not within any recognized exception.
    (D) inadmissible, because of the Dead Man’s Statute.
A
  1. (C) According to FRE 801(c) hearsay is defined as a “statement, other than one made by the declarant while testifying at the trial or hearing, offered in evidence to prove the truth of the matter asserted.” The woman is attempting to testify to what the witness alLegedly told her. Choice (C) is correct because the witness’s statement is (1) an out-of-court statement that is (2) offered for the truth of the matter asserted. The statement does not, however, fit within any recognized hearsay exceptions. This is an extremely popular Multistate Evidence example because the test maker knows many students will incorrectly choose choice (B). This choice would be correct if the woman were testifying that she saw an employee of the convenience store shoveling the snow. Rather, the woman is proposing to testify to what the witness alLegedly told her he saw. Since it is being offered to prove the truth (i.e., the convenience store owner retained control over the area in front of his store) of the matter asserted, it is inadmissible hearsay. Choice (A) is incorrect because, regardLess of the fact that the witness is unavailable to testify, his out-of-court statement fails to satisfy any recognized hearsay exceptions. Although interesting, choice (D) is wrong because Dead Man’s Statutes apply only to a limited circumstance: they prohibit claimants against estates from testifying about conversations or transactions with the decedent.
How well did you know this?
1
Not at all
2
3
4
5
Perfectly
133
Q
  1. A defendant is on trial for attempting to cash a forged check at a bank. A teller at the bank is called to testify.
    The teller testified that she was on duty when the defendant came to her station and handed her a check drawn on the account of another customer. She recognized the forgery because she knew the other customer’s signature since he was one of her regulars. The teller further testified that after becoming suspicious, she placed the check down on the counter and turned to talk to her supervisor. As she did so, the defendant picked up the check and left the bank before she could say anything. The check that the defendant presented at the bank was not recovered by law enforcement authorities and was not offered as evidence.
    The teller’s testimony regarding the forged signature on the check is
    (A) inadmissible, because the witness is at fault in allowing the loss of the original by failing to secure the check.
    (B) inadmissible, because it is not possible for the jury to compare the signature on the check since it was not presented as evidence.
    (C) admissible, because it is rationally related to the witness’s perception.
    (D) admissible, because the witness was familiar with the alleged victim’s handwriting.
A
  1. (D) Under FRE 901 (b)(2), authentication or identification as a condition precedent to admissibility is satisfied by “Non-expert opinion as to the genuineness of handwriting, based upon familiarity not acquired for purposes of the litigation.” The teller’s testimony recognizing the forgery will be admissible because she was familiar with the other customer’s signature, since he was one of her regulars. Choice (D) is correct. FRE 701 provides for lay opinion that is rationally based on the perception of the witness. Included within the proper scope of lay testimony is familiarity with handwriting. Nonetheless, choice (D) is a more specific answer than choice (C). Choice (B) is incorrect because comparison of handwriting specimens by the jury is not required as a requisite for admissibility, although it is one way to authenticate a person’s handwriting. Choice (A) alludes to FRE 1004, which allows admissibility of other evidence of contents (i.e., the teller’s testimony) where the original is lost or destroyed, unless the proponent lost the original in bad faith. Choice (A) is incorrect because the teller was not acting in bad faith when the defendant took the check from her possession and ran out of the bank.
How well did you know this?
1
Not at all
2
3
4
5
Perfectly
134
Q
  1. A plaintiff was the varsity basketball coach of the boys’ team at a high school. After the second game of the season, the plaintiff was suddenly dismissed as the coach for undisclosed reasons. After the dismissal was made public, the local newspaper ran an article discussing the plaintiff’s tenure as coach. The article, which was written by the sports editor, stated that “The plaintiff was a bad role model and morally unfit for the coaching job.”
    The plaintiff brought suit against the newspaper for defamation. At trial, the plaintiff called a player on the basketball team to testify that “the plaintiff was a good role model and respected by the players.” In presenting its evidence, the newspaper called a witness to testify. He proposes to testify that he saw the plaintiff give beer to players on three occasions after victories. Upon objection by the plaintiff’s attorney, witness’s testimony is
    (A) admissible, for impeachment purposes only.
    (B) admissible, as substantive evidence.
    (C) inadmissible, because specific instances of conduct is an improper method of proving character.
    (D) inadmissible to prove character, but admissible for the purpose of showing intent or common scheme.
A
  1. (B) Choices (C) and (D) are wrong because FRE 405(b) provides that “in cases in which character or a trait of character of a person is an essential element of a charge, cLaim or defense, proof may be made by specific instances of that person’s conduct.” The key question is whether the witness’s testimony is limited for impeachment or may be admitted substantively as proof of character. According to McCormick, a person’s character may be a material fact that, under the substantive law, determines rights and liabilities of the parties. For example, in an action of defamation for a publication to the effect that plaintiff’s character is bad, the publisher may raise the defense that the statement is true. Evidence, pg. 551. Similarly, the plaintiff is suing the newspaper for defamation based upon an article describing “The plaintiff as a bad role model.” The testimony by the witness should be admissible substantively because it relates to the truth of the assertion, which, in turn, would provide the newspaperwith a valid defense. Choice (B), therefore, trumps choice (A). Note, too, that choice (A) incorrectly states that the witness’s testimony is admissible only for purposes of impeachment.
How well did you know this?
1
Not at all
2
3
4
5
Perfectly
135
Q
  1. A plaintiff sued a defendant for injuries suffered in an automobile accident that occurred six months ago. The plaintiff claimed that the defendant drove through a red light and hit her as she was crossing the street. The posted speed limit at the scene of the accident was 25 m.p.h. In his case-in-chief, the defendant testified that the traffic light was green and he was driving carefully at 20 m.p.h. The plaintiff’s attorney did not cross-examine the defendant. Rather, she called a police officer who testified that the defendant told him immediately after the accident that he was going 30 m.p.h.
    The police officer’s testimony is
    (A) a prior inconsistent statement and, therefore, admissible to impeach.
    (B) hearsay, but admissible as a prior inconsistent statement for any purpose.
    (C) admissible, as offered against the defendant.
    (D) inadmissible, because the accident report is the best evidence of what the defendant told the police officer.
A
  1. (C) Choice (C) is the correct answer because the defendant’s statement to the police officer (that he was going 30 m.p.h.) is admissible against him as an admission. By definition, admissions are the words or acts of a party-opponent or his predecessor or representative, offered as evidence against him. This is a popular Multistate example because the test maker is distinguishing between statements (or admissions) made to a police officer and statements contained in a police report. As a general rule, statements in a police report that are attributed to witnesses or to prove conclusions of the investigating officer are inadmissibLe hearsay. Choice (A) is technically correct because FRE 613 permits the use of extrinsic evidence of prior inconsistent statements, but choice (C) is a better choice because an admission can be used substantively and is, therefore, stronger than evidence that is used only to impeach. Choice (B) is incorrect because an admission is excluded from the hearsay rule by FRE 801(d)(2). Choice (D) is incorrect because the best evidence rule, FRE 1002, applies only if the contents of a writing, recording, or photograph are in dispute.
How well did you know this?
1
Not at all
2
3
4
5
Perfectly
136
Q
  1. One year ago, while driving through an intersection, the defendant’s car collided with the plaintiff’s car. The plaintiff sued, claiming that the defendant ran a red light. In her case-in-chief, the plaintiff testified that immediately after the accident, a bystander, who was a local resident, screamed at the defendant, “You jerk.. . you drove through the red light.” In his case-in-chief, the defendant now calls a police accident investigator to testify that the plaintiff told him that she “didn’t see the color of the light when she entered the intersection because she was dialing a number on her cell phone.”
    The investigator’s testimony should be
    (A) admitted, because the plaintiff’s inconsistent statement has bearing on her credibility.
    (B) admitted, because the accident investigator gathered the information in the course of his official duties.
    (C) excluded, because the defendant failed to lay a foundation, thus not giving the plaintiff an opportunity to deny or explain her statement.
    (D) excluded, because the plaintiff’s statement to the investigator is hearsay.
A
  1. (A) In her case-in-chief, the plaintiff claimed that the defendant drove through a red light and hit her as she was crossing the street. So obviously the plaintiff must be contending that the traffic light was green (in her favor) when she proceeded to cross the intersection. As a consequence, her statement to the investigator that she “didn’t see the color of the light when she entered the intersection because she was dialing a number on her cellphone” may be receivable in two ways: (1) as an admission by a party under FRE 801(d), and (2) as an inconsistent statement to impeach the witness under FRE 613. Choice (A) is, therefore, correct. Choice (B) is a distracting answer and is incorrect. The words “official duties” might cause a test- taker to think of FRE 803(8), the public records hearsay exception. This exception, however, applies only to “matters observed” by public officials in the course of their duties and requires personal knowledge by the official of the matter observed. Here, the investigator has no personal knowledge of the light’s color at the time of the accident. Choice (C) is the wrong answer because, from the facts, it appears that the defendant has laid a sufficient foundation to introduce the plaintiff’s statement as either an admission or to impeach. The plaintiff will be afforded an opportunity to cross-examine the officer about the statement, if she so chooses. Choice (D) is incorrect because an admission is not hearsay under Rule 801(d).
How well did you know this?
1
Not at all
2
3
4
5
Perfectly
137
Q
  1. A football player sued a football team for breach of contract. He was a star player for the team before suffering a serious hip injury. Although the player attempted to continue playing for the team, he failed a physical examination and was cut from the team roster. After failing his physical, the team discontinued making contract payments to the player as per the terms of his employment contract. In his lawsuit, the player claimed that he was physically fit to continue playing but that he was cut from the team because they wanted to stop making contract payments.
    At trial, the team called its physician, an orthopedist, to testify that the player’s hip injury was, in fact, career-threatening and, in his opinion, sufficiently serious to warrant failing his physical. The team physician stated during his testimony that his diagnosis was based, in part, upon an examination of the player’s x-rays and review of an MRI test conducted by another doctor. Neither the x-rays nor the MRI test results have been admitted into evidence.
    In all likelihood, the team physician’s testimony should be
    (A) admissible, provided that the other doctor testifies as to the validity of the MRI test results.
    (B) admissible, provided the team physician based his opinion on data reasonably relied upon by other orthopedic physicians in the field.
    (C) inadmissible, because neither the x-rays nor the MRI test results had been admitted into evidence at the time he formulated his opinion.
    (D) inadmissible, because the team physician’s opinion was not based in part upon his own firsthand observation of the player’s physical condition.
A
  1. (B) FRE 703 deals with the bases of opinion testimony by experts. An expert may base her opinion on facts or data perceived by or made known to her at or before the hearing. These facts or data need not themselves be admissible in evidence if they are of a type reasonably relied upon by experts in the particular field in forming opinions or inferences upon the subject. Therefore, an expert opinion need not be based on firsthand observation—this is why choice (D) is incorrect—and instead may be based on presentation of data to the expert outside of court. The justification for this rule is based on an attempt to bring judicial practice in line with the out-of-court practice of experts themselves, who base their own life-and-death diagnoses in reliance on information from numerous sources, including statements of patients and relatives, reports and opinions of other doctors and technicians, hospital records, and x-rays. FRE 703, Advisory Committee’s notes. As Long as the team physician’s diagnosis was based on data reasonably relied upon by other orthopedic physicians in the field, it will be admissible. Choice (B) is, therefore, correct. The x-rays and the MRI test results need not be admitted in evidence at trial, so choice (C) is incorrect. Choice (A) is incorrect because the MRI test results need not be validated, provided that such data are reasonably relied upon by experts in the field.
How well did you know this?
1
Not at all
2
3
4
5
Perfectly
138
Q
  1. In an action for malpractice, a plaintiff sued a doctor, claiming that the latter failed to properly diagnose his emphysema condition. The plaintiff contends that he went to the doctor for a medical examination after experiencing chest pains and shortness of breath. However, the doctor found no evidence of the disease and advised him that he was probably suffering from indigestion. Thereafter, the plaintiff was examined by another doctor who discovered that the plaintiff in fact was suffering from a chronic case of emphysema.
    At trial, the plaintiff calls the other doctor as his first witness. The other doctor proposes to testify the x-rays of the plaintiff’s lungs showed that his respiratory organs were marked by distension, which was characteristic of emphysema. Upon objection by the defendant’s attorney, the testimony should be
    (A) admitted, because a doctor is properly qualified as an expert in medical matters.
    (B) admitted, because the other doctor followed accepted medical practice in arriving at his opinion.
    (C) excluded, because his opinion is based upon facts not in evidence.
    (D) excluded, because the x-rays are the best evidence to show the distension of the plaintiff’s lungs.
A
  1. (D) Here, it is necessary to distinguish that the other doctor is proposing to testify “that the x-rays showed a distension.” X-rays come within the best evidence rule, FRE 1002, which applies to the contents of writings, recordings, and photographs; according to FRE 1001 (2), x-rays are considered photographs. Choice (D) is, therefore, correct. Choice (A) is incorrect because there is no evidence in the fact pattern that the other doctor was tendered as an expert under FRE 702; doctors are not automatically expert witnesses at trial. There is not enough information in the fact pattern to determine whether the other doctor followed acceptable medical practices, so choice (B) is a wrong answer. Choice (C) is incorrect because, assuming that the other doctor was qualified as an expert witness, FRE 703 would permit him to base his conclusions on facts not in evidence.
How well did you know this?
1
Not at all
2
3
4
5
Perfectly
139
Q
  1. A driver was prosecuted for speeding. It is conceded that the speed limit on the stretch of the highway in question at the time of the alleged speeding was 55 m.p.h. As part of its case-in-chief, the prosecution called a police officer who testified that at about 11:30 a.m. one mile east of the town line, he set up a radar apparatus. The officer testified that, with the use of tuning forks, he found the radar apparatus was accurate. The officer also stated that he had five years of experience in operating such radar devices. Furthermore, the officer testified that the radar’s mechanism for recording the speed of vehicles by means of an ink line drawn mechanically on a roll of paper was also functioning properly. Upon further questioning, the officer stated that at about 1:30 p.m., the needle of the speed indicator dial of the radar apparatus showed the driver’s car passing his location at 65 m.p.h.
    If the driver’s attorney makes an objection to the introduction of the radar finding that the driver was traveling at a speed of 65 m.p.h., the court will most likely
    (A) sustain the objection, because the radar results are not conclusive evidence of speeding.
    (B) sustain the objection, because radar is not a technique recognized by the scientific community.
    (C) overrule the objection, because the court will take judicial notice of the accuracy of such tests.
    (D) overrule the objection, because the radar results would be construed as an admission against interest.
A
  1. (C) The majority of courts are generally in accord in holding that expert testimony is no longer needed to establish the scientific validity of radar. In People v. MacLaird, 264 Cal. App. 2d 972 (1968), the trial court required the jury to take judicial notice of the validity and accuracy of radar devices. Similar results have been reached in People v. Magri, 3 NY. 2d (1958); United States v. Preos, 156 F. Supp. 200 (D.C. Md. 1957); and State v. Dantonio, 115 A.2d 35 (1955). These are among a few of the earliest cases in which the court has taken judicial notice of the validity of radar devices. Choice (A) is incorrect, given the general acceptance of radar results by courts and the scientific community. Choice (B) is an excellent example of an answer choice that cannot be correct if any other answer choice is correct. In order for choice (B) to be correct, it would have to be true that the scientific community had rejected radar. Since case law (and, for that matter, common sense) tells us that expert testimony is no longer needed to establish radar’s validity, choice (B) cannot be correct. Finally, choice (D) is incorrect. An admission is a statement made by a party-opponent. Observations of a party’s actions made or recorded by other people or by scientific instruments are not admissions.
How well did you know this?
1
Not at all
2
3
4
5
Perfectly
140
Q
  1. In January, while he was driving on the street, a defendant lost control of his car, hitting and killing a small child. The child’s parents sued the defendant for causing the wrongful death of their child. At trial, the parents’ attorney calls a bystander to testify that the defendant’s car was traveling well over the 25 m.p.h. speed limit on that street.
    Upon objection by the defendant’s attorney, the trial judge would rule the bystander’s testimony
    (A) admissible, because the bystander’s opinion is based on his own perception.
    (B) admissible, as a declaration of existing state of mind.
    (C) inadmissible, as hearsay not within any of the exceptions.
    (D) inadmissible, because a lay witness is not qualified to testify about the speed of a vehicle.
A

ii. (A) Choice A is correct, because there are particular situations, such as estimating in miles per hour the speed of a moving vehicle, in which opinions of lay witnesses are generally admissible. Other such situations under FRE 701 in which lay opinions are admissible include (a) the general appearance or condition of a person; (b) the state of emotion of a person; (c) matters involving sense recognition; and (d) identifying the likeness or identity of handwriting. Choice (B) is inapplicable to this set of facts. An estimation of speed is not a “declaration of then-existing mental, emotional, or physical condition” within the meaning of FRE 803(2), the “state of mind” hearsay exception. Choice (C) is incorrect. The witness is testifying about his personal observation. No out-of-court statements are being introduced. Finally, choice (D) is the wrong answer because, under FRE 701, lay opinion witnesses are qualified to testify about such things as the speed of a moving vehicle.

How well did you know this?
1
Not at all
2
3
4
5
Perfectly
141
Q
  1. A city entered into a small-cost construction contract with a construction company whereby the latter obligated themselves to perform certain minor sewer repair work. Two months after completion of the work, the company sued the city for its failure to pay under the terms of the contract.
    At trial, the company’s construction supervisor who personally oversaw all the repair work was called to testify. He sought to testify about the amount of sewer pipe used, the number of hours required to complete the job, and the number of workers employed by the company during the construction repair work.
    The defendant’s attorney objected on the grounds that the company routinely recorded such factual information in their log books, which were in the company’s possession.
    As a consequence, the supervisor’s testimony should be ruled
    (A) admissible, because it is based on firsthand knowledge.
    (B) admissible, because it is a report of regularly conducted business activities.
    (C) inadmissible, because such testimony would be violative of the best evidence rule.
    (D) inadmissible, because no evidence was introduced to show that the original log records are not available for examination.
A
  1. (A) Choice (C) is incorrect because the original document rule (otherwise known as the best evidence rule) applies to those situations where in proving the terms of a writing, and the terms are material, then the original writing must be produced unLess it is shown to be unavailable for some reason other than the serious fault of the proponent. First, there are certain writings that the substantive law (e.g., the Statute of Frauds, the parol evidence rule) endow with a degree of either indispensability or primacy. Transactions to which substantive rules of this character apply tend naturally to be viewed as written transactions and writings embodying such transactions (e.g., deeds, contracts, judgments, etc.). Contrasted with the above-described types of writings are those, essentially unlimited in variety, that the substantive law does not regard as essential or primary repositories of the facts recorded. In this regard, log books would be viewed as a writing of this latter sort in that they happen to record the facts of essentially non-written transactions. Testimony descriptive of non-written transactions is not generally considered to be within the scope of the present rule and may be given without producing or explaining the absence of a writing recording the facts. [See Allen v. W. H. 0. Alfalfa Mill Co. 272 F.2d 98 (10th Cir. 1959) where costs of production were allowed without records]. As a result, choice (D) is also incorrect because the original document rule is inapplicable. Therefore, in accordance with FRE 602, “A witness may not testify to a matter unless evidence is introduced sufficient to support a finding that he has personal knowledge of the matter. .. ,“ choice (A) is the best answer. Since the construction supervisor personally oversaw the entire job, he would clearly have firsthand knowledge regarding the construction work. Choice (B) is inapplicable because the witness is testifying directly as to his firsthand knowledge and is not introducing information from the company’s records under FRE 803(6), the business records exception.
How well did you know this?
1
Not at all
2
3
4
5
Perfectly
142
Q
  1. A plaintiff sued a defendant for leg injuries stemming from a collision between their cars at an intersection. Each party was driving his own car, and the plaintiff alleged that the defendant had sped through a red light and collided with the plaintiff’s car. The defendant denied this allegation.
    During the trial, the plaintiff testified that he and the defendant had conferred after the accident and that the defendant had said, “Don’t worry about anything; I’ll see to it that your doctor’s bills are paid.” The defendant’s attorney immediately objected.
    The trial judge should rule this testimony
    (A) admissible, because the defendant admitted his negligence.
    (B) admissible, because it is a declaration against interest.
    (C) inadmissible, because the prejudice in admitting the testimony outweighs its probative value.
    (D) inadmissible, because an offer to pay medical bills is not admissible to prove negligence.
A
  1. (D) FRE 409 provides that evidence of furnishing or offering or promising to pay medical, hospital, or similar expenses occasioned by an injury is not admissible to prove liability for the injury. This rule does not extend to conduct or statements not part of the act of furnishing or offering to pay medical and similar expenses. Thus, if the defendant had said, “Since I went through the red light, I’ll pay for any injuries you may have suffered,” the defendant’s admission that he went through the red light would be admissible, although the statement regarding medical expenses would be inadmissible in accord with FRE 409. Choice (A), therefore, is an incorrect answer. Choice (B) does not apply to this fact pattern because there is no indication that the defendant is unavailable at trial, a foundational element of FRE 804(b)(3), the statement against interest exception to the hearsay rule. Choice (C) is incorrect because it misstates the balancing test of FRE 403, which states that “evidence may be excluded if its probative value is substantially outweighed by the risk of unfair prejudice…
How well did you know this?
1
Not at all
2
3
4
5
Perfectly
143
Q
  1. A pedestrian is suing a defendant for injuries she suffered to her leg when the defendant ran the pedestrian over with his motorcycle. On cross- examination of the pedestrian’s physician, the defendant’s attorney asked him whether or not the pedestrian had told him that the leg that she claimed was injured had been permanently stiff as a result of a former field hockey injury the pedestrian suffered in high school.
    Upon objection by the pedestrian’s attorney, the trial judge should
    (A) sustain the objection, as hearsay not within any recognized exception.
    (B) sustain the objection, as violative of the physician-patient privilege.
    (C) overrule the objection, as a statement made for the purpose of diagnosis or treatment.
    (D) overrule the objection, as a statement against interest.
A
  1. (C) Statements made for purposes of medicaL diagnosis or treatment and describing medical history, or past or present symptoms, pain, or sensations, or the inception or general character of the cause or external source thereof insofar as reasonably pertinent to diagnosis or treatment, are not excluded by the hearsay rule under FRE 803(4). Choice (A) is the wrong answer. The statement is hearsay, but it falls under a recognized hearsay exception. Choice (B) is incorrect. There is no standard physician-patient privilege under the FRE. Instead, under FRE 501, state law provides the basis for the privilege in federaL courts. In this fact pattern, there is no information gWer about the parameters of or exceptions to the state privilege. Although the pedestrian’s statement to her doctor might well be against her interest, the fact pattern gives no information about the physician’s unavailability, which would be required in order to trigger FRE 804(b) (3), the statement against interest exception. Thus, choice (D) is incorrect.
How well did you know this?
1
Not at all
2
3
4
5
Perfectly
144
Q
  1. A newly elected mayor was charged with tax fraud. The prosecutor served a trial subpoena directing the mayor’s attorney to produce private bank records that the mayor delivered to the attorney. The attorney refused the request. The prosecutor in a hearing then sought to compel compliance with the subpoena.
    The court should
    (A) compel production, because it would not violate the attorney-client privilege.
    (B) compel production, because there is no element of confidentiality.
    (C) not compel production, because it would violate the mayor’s privilege against self- incrimination.
    (D) not compel production, because it would violate the attorney-client privilege.
A
  1. (A) This question deals with the issue of what type of communications are within either the attorney-client priviLege or the Fifth Amendment privilege against selfincrimination. Pre-existing written documents (contracts, leases, memos) do not become privileged by simply being handed over to the attorney. Written documents are comm unications protected by the attorney-client privilege if they were prepared specifically from the client to the attorney or vice versa. In light of this discussion, choice (A) is correct, and the court should compel production of the mayor’s private bank records. Choice (B) confuses the issue and is incorrect. Certainly, the mayor intended the bank records to be confidential; that’s why he gave them to his attorney. However, no privilege is available to him under these circumstances. Choice (C) is incorrect. Similarly, pre-existing records, especially those of a business nature, which were voluntarily prepared by or for the owner at an earlier time, may generally be obtained by the government without violating the privilege against self-incrimination. Lilly, Law of Evidence, pp. 339—340. Whereas ordinary business records may be seized pursuant to a valid search warrant because such a seizure does not involve any incriminating assertion by the owner of the records [Andresen v. Maryland, 427 U.S. 463 (1976)], even production of specified business records is generally allowed on the theory that the gathering and submitting procedure is not, standing alone, an incriminating assertion [Fisher v. United States, 425 U.S. 391 (1976)]. Private items, such as a diary, present a more difficult question. However, “the fact that a pre-existing, non-coerced communication is private in nature is probably not dispositive so long as it can be obtained and presented at trial without compelling the individual resisting disclosure to make incriminating statements or verify the truth of the contents.” Lilly, Law of Evidence, pp. 340—341. Choice (D) is the wrong answer, for the same reasons that choice (A) is the correct answer.
How well did you know this?
1
Not at all
2
3
4
5
Perfectly
145
Q
  1. A guitarist and lead singer for a rock and roll band was performing a concert when an overhead strobe light fell on stage and struck him. The singer suffered a fractured skull and was hospitalized for an extended period of time. A lighting company was hired by the venue to perform the strobe lighting show at the concert.
    During his hospital stay, the singer sent a letter to the lighting company’s president threatening to sue and holding the lighting company responsible for the accident. After receiving the singer’s letter, the company’s attorney visited the singer at the hospital where he was being treated. The attorney entered the singer’s hospital room and told him, “The company will pay your medical expenses if you will give a release.” The singer remained silent, and the attorney then left the room.
    Thereafter, the singer filed a lawsuit against the lighting company to recover damages for his injury. At trial, the singer seeks to introduce into evidence the attorney’s statement at the hospital.
    Upon objection, the attorney’s statement should be
    a. admitted, as a vicarious admission.
    b. admitted, as a declaration against interest.
    c. excluded, as an offer to compromise.
    d. excluded, as a privileged attorney-client communication.
A

i6. (C) One of the most important Evidence distinctions tested on the bar exam is statements made in compromise or settlement discussions (FRE 408) and admissions made in connection with an offer to pay medical or hospital expenses (FRE 409). Statements made in compromise negotiations are not admissible. On the other hand, under FRE 409, we sever and admit the admission while the offer to pay medical expenses is not admissible. Based on the given answer choices, it is necessary to assume that the attorney’s statement was made in connection with settlement negotiations. Since the facts state that the singer has threatened to sue the lighting company, there is an actual dispute between the parties. Under the circumstances, the lighting company’s lawyer is attempting to engage in compromise negotiations. Thus, choice (C) is correct, and choice (A) is incorrect. Choice (B) is incorrect, fortwo reasons: (1) an offer to compromise is not necessarily a statement against interest (here, there is no admission of liability); and (2) the facts contain no evidence about the declarant’s unavailability, a critical foundational element for FRE 804(b) (3). Choice (D) does not apply here because the attorney was authorized by his client to make this communication to a third party.

How well did you know this?
1
Not at all
2
3
4
5
Perfectly
146
Q
  1. An experienced equestrian rider sued a woman, claiming that her horse kicked and injured the rider. At trial, the rider testified that he was attending a horse show when, without provocation, he was kicked by a gray stallion with a black tail. The rider then called the woman as an adverse witness and asked her the following question: “Isn’t it true that you are the owner and trainer of a gray stallion with a black tail?” After answering affirmatively, the woman was not asked any further questions on direct examination. Whereupon in cross- examination, the woman was then asked, “Isn’t it true that your horse is very gentle and would not kick anyone unless provoked?”
    Upon objection by the rider’s attorney, the rider’s strongest argument would be that the woman’s proffered testimony is
    (A) inadmissible as an improper form of impeachment.
    (B) inadmissible as substantive evidence to show that the horse must have been provoked.
    (C) inadmissible, because the horse’s general propensity cannot be used to prove what occurred on a particular occasion.
    (D) inadmissible as beyond the scope of the woman’s direct testimony.
A
  1. (D) FRE 611(b) provides that “cross-examination should be limited to the subject matter of the direct examination and matters affecting the credibility of the witnesses. The court may, in the exercise of discretion, permit inquiry into additional matters as if on direct examination.” In the rider’s suit against the woman, by calling the defendant as an adverse witness, the plaintiff tactically limited the scope of direct examination only to the issue of ownership of the gray stallion. As a result, the ensuing cross-examination as to the temperament of the horse goes beyond the scope of direct examination. Subject to discretion of the court, such questioning will be inadmissible. Choice (D) is therefore correct. Note: FRE 611(b) will govern procedurally, despite the fact that the defense may use other proper means (e.g., calling other witnesses or calling the defendant at a later time) to impeach the rider’s testimony. Choice (A) is inapplicable here because the question focuses more on scope of direct than impeachment. Had the rider’s attorney not limited the scope of direct, the question on cross might very well have been permissible impeachment. Choice (B) is also inapplicable. The evidence of the horse’s temperament would be admissible to show it must have been provoked, but as pointed out earlier in this paragraph, the defense would have to introduce that evidence in some other way than cross-examination. Choice (C) is wrong, because the character rules for propensity (contained in FRE 404 and 405), apply to human beings, not animaLs.
How well did you know this?
1
Not at all
2
3
4
5
Perfectly
147
Q
  1. A truck driver was assigned to drive a huge tractor- trailer loaded with logs. The logs were being sold by a lumber company, which had loaded the tractor- trailer. After four hours of steady driving, the truck driver stopped at a lounge and consumed five bottles of beer. He left the tavern in an intoxicated condition and then drove off to make his final delivery.
    As he came to the top of a steep incline in the highway and started his descent, an 11-year-old girl suddenly darted out into the road directly in front of him. The truck driver slammed on his powerful air brakes and swerved sharply to the left to avoid hitting her, but the girl was struck by the tractor- trailer and was seriously injured.
    The girl’s parents, on behalf of their daughter, assert a claim against the lumber company to recover damages for the girl’s injuries.
    At trial, which of the following is LEAST likely to be admissible?
    (A) Evidence that the truck driver’s reputation for driving is poor, if the lumber company and truck driver argue that the latter was not negligent.
    (B) Evidence that the truck driver pleaded guilty to a criminal charge of DUI arising from this incident.
    (C) Evidence that the lumber company carried liability insurance on the tractor-trailer, if an investigator for the lumber company’s insurance company testified for the lumber company that they deny ownership of the truck.
    (D) Evidence that the lumber company carried liability insurance on the tractor-trailer, if the lumber company argues that the truck driver was an independent contractor.
A
  1. (A) Where the owner of a motor vehicle is charged with liability for the acts of a person using it on the grounds of negligent entrustment, then the character of the custodian is “in issue” (to show negligence in entrusting the vehicle to an incompetent driver). This is a difficult question because the facts do not indicate that this is a negligent entrustment case. The facts simply state that the girl’s parents assert a claim against the lumber company. Consequently, choice (A) is correct because, if thisisanegligenceaction (which itwould be if the lumbercompanywere introducing evidence to show that the truck driver was not negligent on this occasion), then the defendant’s character would not be “in issue,” and character evidence would be inadmissible. Choice (B) is incorrect, because the truck driver’s DUI guilty plea would be admissible to help prove negligence. Choice (C) is incorrect. Although evidence of insurance coverage is generally inadmissible under FRE 411 to prove negligence, it is admissible to prove other things, such as proof of ownership. For similar reasons, choice (D) is incorrect. Proof of insurance coverage can be introduced to show “agency, ownership, or control.” Under these facts, the evidence could be admissible to prove agency or control.
How well did you know this?
1
Not at all
2
3
4
5
Perfectly
148
Q
  1. Ajudge attended a local men’s club meeting. In introducing the judge to the club members, the club president said: “Gentlemen, I take great pleasure in presenting the judge, who we all know is the best judge money can buy in the city.”
    The judge sued the club president for slander. If the club president pleads truth as a defense, he may not properly introduce evidence to show
    (A) the judge has a reputation for dishonesty in the community.
    (B) the judge was convicted of bribery three years earlier.
    (C) the judge, though not convicted, embezzled money from a former employer before being elected to the bench.
    (D) the judge was convicted for a felony assault 12 years earlier.
A
  1. (0) Character evidence is generally not admissible in civil actions unless character is “in issue” (i.e., where it is a material element of a charge, claim, or defense). In the following tort actions, character is said to be “in issue” and, thus, character evidence is admissible: (1) defamation, (2) deceit, (3) negligent entrustment, and (4) assault/battery where the defendant claims he acted in self-defense, in which case the plaintiff’s reputation for violence or turbulence is in issue. Since this is a defamation action, the judge’s character for honesty is “in issue” especially since the club president is pleading truth as a defense. In this regard, FRE 405 provides “In all cases in which evidence of character or a trait of a person is admissible, proof may be made by testimony as to reputation or by testimony in the form of an opinion.” Thus, evidence that supports the statement in choice (A) may properly be offered, since the club president can prove that the judge has a reputation for dishonesty. FRE 405(2) states, “In cases in which character or a trait of character of a person is an essential element of a charge, claim, or defense, proof may also be made of specific instances of his conduct.” Therefore, evidence that supports the statement in choices (B) and (C) may properly be admitted because the club president can show specific instances of (mis)conduct on the judge’s part (namely that (a) he was convicted of bribery, and (b) he embezzled money) to prove his defamatory statements were truthful. The assertion in choice (D) is the only one that the club president may not introduce. The judge’s character for honesty is in issue, but the felony conviction for assault does not speak to his character for honesty. Witness impeachment by prior conviction is generally acceptable for certain crimes, but such evidence is generally inadmissible if 10 years or more have elapsed.
How well did you know this?
1
Not at all
2
3
4
5
Perfectly
149
Q
  1. Two weeks before an election, a local newspaper published an article implying that one of the mayoral candidates was a thief. The candidate subsequently lost the election and sued the newspaper for defamation. The newspaper defended on the grounds of truth. At trial, the candidate took the stand and testified that he was not a thief. Thereafter, the newspaper called a witness to testify that three years ago the candidate committed a larceny while employed at his former job.
    Upon objection by the candidate’s attorney, the witness’s testimony should be
    (A) excluded, because bad acts may not be proved by extrinsic evidence.
    (B) excluded, because it is improper character evidence.
    (C) admitted as probative evidence of the candidate’s character for veracity.
    (D) admitted as relevant evidence of the candidate’s character as a thief.
A
  1. (D) FRE 405(b) provides, “In cases in which character or a trait of character of a person is an essential element of a charge, claim, or defense, proof may also be made of specific instances of his conduct.” Many students will incorrectly choose choice (B); however, choice (B) is wrong because although character evidence is generally not admissible in a civil action, there are certain exceptions. For example, character evidence is admissible in those cases in which a person’s character is “in issue” (e.g., defamation, negligent entrustment, and hiring). Since this is a defamation action, the candidate’s character is in issue and, therefore, all three forms of character evidence (opinion, reputation and specific acts) are admissible. On this very point, Lilly provides the following example: Assume that the defendant states that the plaintiff is “corrupt and dishonest”; in the resulting suit for defamation, the defendant bases his defense upon the truth of his statement. Plaintiff’s character for dishonesty and corruption is directly in issue, and the inferential chain stops with the establishment of these traits. Here, the distinguishing characteristic is that the candidate’s character trait for stealing, not veracity, is in issue. In other words, the newspaper wants to produce evidence to show that the candidate is a thief in order to sustain its defense of truth in this defamation action. That’s why choice (D) is a better answerthan choice (C). Choice (A) is incorrect, because FRE 405 permits proof of specific instances of a person’s conduct in cases in which character or a trait of character of a person is an essential element of a charge, claim, or defense, as it is in this case.
How well did you know this?
1
Not at all
2
3
4
5
Perfectly
150
Q
  1. A man and a school entered into a written contract providing that the man would be employed for one year in various capacities (athletic director, teacher, football coach) in return for an annual salary of $37,750. Shortly thereafter and prior to the July commencement date of employment, the school elected to terminate its varsity football program. The school publicly attributed the decision to economic factors. Upon hearing this news, the man sought out the headmaster of the school to inquire as to the effect, if any, of this decision on his salary and responsibilities.
    In order to resolve the matter, the headmaster arranged a meeting among various school officials to discuss the man’s situation. Afterward, the school sent written notice to the man of its intent to terminate his employment contract. The man then commenced suit to recover the full balance of his salary calculated on the basis of the contract. At trial, the headmaster was called to testify. On direct examination, he testified that six people (including himself) attended the meeting where it was decided to terminate the man’s employment contract. When asked to identify the attendees, the headmaster gave the names of four but, despite trying, was unable to remember the name of the fifth person.
    The attorney who called the headmaster to testify now seeks to show him her handwritten notes that the attorney prepared during her pretrial interview with the headmaster, in which he provided all five names.
    The trial court is likely to consider the showing of the notes taken as
    (A) a proper attempt to refresh the headmaster’s recollection.
    (B) a proper attempt to introduce recorded recollection.
    (C) an improper attempt to support the headmaster’s credibility.
    (D) an improper attempt to lead the witness.
A
  1. (A) FRE 612 clearly states that “Except as otherwise provided . . . if a witness uses a writing to refresh his memory for the purpose of testifying, either (1) while testifying, or (2) before testifying, if the court in its discretion determines it is necessary in the interest of justice, an adverse party is entitled to have the writing produced at the hearing, to inspect it, to cross-examine the witness thereon, and to introduce in evidence those portions which relate to the testimony of the witness.” Refreshing may be accomplished by handing the writing to the witness, accompanied by a request to the witness to read the writing to himself. Thereupon counsel retrieves the writing and repeats the question. At this point, the witness testifies from his own jogged present recollection. In this question, the headmaster’s present recollection has been exhausted, since he is unable to remember the name of the fifth person who attended the meeting to terminate the man’s employment contract. Use of the handwritten notes prepared by examining counsel in her pretrial interview is a proper attempt to refresh the headmaster’s recollection. Choice (A) is correct. Choice (B) is incorrect in this case. As the Multistate Nuance Chart below demonstrates, a key foundational element for using someone’s recorded recollection under FRE 803(5) is a failed attempt to refresh the individual’s memory. There is nothing in the fact pattern to suggest that the attorney’s effort to refresh the headmaster’s recollection has failed. Choice (C) is wrong underthese facts, because there is no indication in the fact pattern of anyone attempting either to attack or buttress the headmaster’s credibility. Choice (D) is also wrong. Showing a witness a document to refresh his or her recoLlection is not “leading” within the meaning of FRE 611.
How well did you know this?
1
Not at all
2
3
4
5
Perfectly
151
Q
  1. On Thursday, March 7, an owner’s home was destroyed by fire. The owner’s house was insured against such fire loss by an insurance company. According to the policy contract, the insured was required to notify the insurance company within 10 days following any loss due to fire or other catastrophe.
    The owner sued the insurance company to recover $750,000, which was the face amount that his home was insured for. The insurance company admitted that the owner’s house was insured for that amount, though the company denied liability because it claimed that it had not received notice of the fire within the 10-day period as required by the policy.
    At trial, the owner calls his bookkeeper to testify. The bookkeeper proposes to testify that on March 11, the owner handed her an envelope and told her it contained a notice form for the insurance company, and that she mailed the envelope on his behalf.
    If the bookkeeper intends to establish that the envelope that the owner handed her contained the notice of fire loss, her proposed testimony is
    (A) admissible, because the owner’s statement to her is non-hearsay.
    (B) admissible, because a letter properly mailed is presumed received in due course.
    (C) inadmissible, because the owner’s statement to the bookkeeper is not the best evidence.
    (D) inadmissible, because the owner’s statement to the bookkeeper is hearsay not within any exception.
A
  1. (D) Hearsay evidence is testimony in court, or written evidence, of a statement made out of court, the statement being offered as an assertion to show the truth of matters asserted therein, and thus resting for its value upon the credibility of the out-of-court asserter. Here, the owner’s out-of-court statement to the bookkeeper (that the envelope contained an insurance notice form) is being offered to prove the truth of the matter stated. As a consequence, it is inadmissible hearsay. Choice (A) is, accordingly, wrong, because it states that the evidence is not hearsay. Choice (B) is interesting, but wrong. Choice (B) implicates the “mailed letter presumption,” which is that one can presume a letter reached its destination if it was properly posted. The point here is subtle, but important:
    the bookkeeper is testifying about what the owner told her concerning the purpose of the letter, not the fact that she posted it on a particular date. Thus, choice (B) is incorrect. Choice (C) is also incorrect. The contents of the letter are not at issue, which is a basic requirement of FRE 1002, the best evidence rule.
How well did you know this?
1
Not at all
2
3
4
5
Perfectly
152
Q
  1. A husband is charged with the murder of his wife. The prosecution alleges that on December 17, the husband murdered his wife by giving her a massive injection of succinyicholine while she was asleep. Succinylcholine is a drug used in small quantities by anesthesiologists as a muscle relaxant. The prosecution claims that the fatal dose given to the wife so totally relaxed her lung muscles that she suffocated. Further allegations claim that the husband’s principal motive was to collect his wife’s $500,000 life insurance policy under which he was the named beneficiary.
    The defense called the physician who signed the wife’s death certificate as its first witness. The defendant’s attorney asked her, “Is it not true that the cause of death was a heart attack?” The physician answered in the negative and stated that the cause of death was unknown. The husband’s attorney then assailed her testimony as a recent fabrication and asked her if she had not stated at the coroner’s inquest that the cause of death was a heart attack. The prosecution immediately objected to this question.
    The trial judge should rule that this question is
    (A) objectionable, because a party may not impeach its own witness.
    (B) objectionable, because the husband’s attorney did not lay a proper foundation.
    (C) unobjectionable, because a party may impeach its own witness by a prior inconsistent statement.
    (D) unobjectionable, because the physician’s testimony at a proceeding that occurred shortly after the death would be more reliable.
A
  1. (C) In accordance with FRE 607, the credibility of a witness may be attacked by any party calling him (to testify). The common law rule against impeaching one’s own witness has been abandoned. Where the impeachment is by a prior statement, it is free from hearsay dangers. For this reason, choice (C) is correct, and choice (A) is incorrect. Choice (B) is wrong under these facts, because there is no indication that the defense failed to lay a proper foundation for the physician’s testimony. The physician’s expert testimony would be admissible under FRE 702 whereby “a witness qualified as an expert by knowledge, skill, expertise, training or education may testify thereto in the form of an opinion or otherwise.” Although choice (D) is closely related to choice (C), after all, a benefit of impeaching the physician with her prior statement is the inference that a statement made closer in time to an event is more accurate than a statement made later. Choice (D) is not the best choice because it fails to specifically address the doctrine that any party may impeach a witness.
How well did you know this?
1
Not at all
2
3
4
5
Perfectly
153
Q
  1. A toxicologist employed at the state crime laboratory testifies at a murder trial about the autopsy he performed on the victim. His testimony is that he detected traces of succinic acid and choline, the components of succinylcholine, in the victim’s brain tissues. Over defense objections, he testified further that, in his opinion, the victim’s cause of death was succinylcholine poisoning. On cross-examination, the toxicologist admitted that up until his findings, it had been universally accepted in the medical community that succinylcholine would be undetectable in the brain within a few minutes of its injection. He also admitted that his methods of detecting succinylcholine were experimental and that his results were not presently publishable because they were not complete enough. The defense attorney then makes a motion to strike the toxicologist’s testimony regarding the cause of death.
    The court will most likely
    (A) grant the motion, because the toxicologist’s opinion could not be substantiated.
    (B) grant the motion, because the toxicologist was not properly qualified as an expert.
    (C) deny the motion, because the toxicologist was qualified as an expert in medical matters.
    (D) deny the motion, because he was qualified to give an opinion on the ultimate issue of the case.
A
  1. (A) With regard to facts ordata upon which expert opinions are based, FRE 703 requires that the facts or data “be of a type reasonably relied upon by experts in the particular field.” In the present case, the toxicologist admitted that his methods of detecting succinylcholine were “experimental” and that his results were incomplete and not publishable. Therefore, since the toxicologist could not substantiate his theory of death by succinylcholine poisoning, it would be struck as falling outside FRE 703. Choice (A) is also correct for another reason. FRE 702 states that an expert may give an opinion in a case if”(1) the testimony is based on sufficient facts or data, (2) the testimony is the product of reliable principles and methods, and (3) the witness has applied the principles and methods reliably to the facts of the case.” Under these facts, the toxicologist’s testimony likely fails prongs (1) and (2) of the FRE 702 test. Experimental methods that have not been published and subjected to peer review are suspect under FRE 702, as well as the seminal Supreme Court case on expert testimony, Daubert v. Merrell Dow Pharmaceuticals, 509 U.S. 579 (1993). Choice (B) is incorrect, because the toxicologist meets the requirements to be qualified as an expert under FRE 702 by virtue of his knowledge, training, and experience. Choice (C) is misleading: even if an individual is qualified as an expert under FRE 702, his testimony must still satisfy the FRE 702 three-prong reliability test and FRE
    703’s requirement that the underlying facts or data be of a type reasonably relied on by experts in the field. Choice (D) is aLso a misleading answer. Even though FRE 704 permits experts to testify on the ultimate issue in a case, their testimony must still satisfy FRE 702’s reliability test.
How well did you know this?
1
Not at all
2
3
4
5
Perfectly
154
Q
  1. A collector told a local coin dealer that he wished to purchase an 1804 silver dollar. The coin dealer told the collector that he would locate one of these rare coins and the cost would be approximately $40,000 plus a commission. The coin dealer successfully located and arranged for the purchase of the 1804 silver dollar. After the collector purchased the coin, a disagreement arose as to the amount of commission. The coin dealer brought suit, claiming that the agreed-upon amount was 15%. The collector contends that they agreed upon 12%.
    At trial, the coin dealer introduces photocopies of company records showing that in all his previous dealings with the collector, and all other customers as well, he charged a 15% commission when asked to locate rare coins.
    The court should find that the offer of this evidence is
    (A) admissible, under the business records exception.
    (B) admissible as past recollection recorded.
    (C) inadmissible as a violation of the best evidence rule.
    (D) inadmissible as hearsay not within any recognized exception.
A
  1. (A) Since the coin dealer’s records are made in the course of regularly conducted business activities, they can be admitted under the business records exception. Choice (B) is incorrect because this is not an example of past recollection recorded, under which an authenticated memorandum, made by a witness, may be read into evidence where the witness is unable to recall the facts recorded in the document. Choice (C) is wrong because under the FRE, photocopies are as admissible as original documents and, therefore, do not violate the best evidence rule. Choice (D) is incorrect, since the company records are within a recognized exception.
How well did you know this?
1
Not at all
2
3
4
5
Perfectly
155
Q
  1. A grain broker specializes in exotic grains for the growing organic-food market. Using his network of contacts across the globe, he is able to find sources for almost any kind of grain, grown anywhere in the world, to be shipped anywhere in the world. Last year, an organic food co-op asked the grain broker to find quinoa for them. The grain broker said, “I’ll do it. It’ll cost you market rate plus the commission.” The grain broker found the quinoa and had it shipped to the co-op at a cost of $15 per bushel plus a 20% commission. The co-op paid $15 per bushel plus an 8% commission. The grain broker brought suit, claiming that the appropriate commission was 20%. The grain broker introduced business records and other documentary evidence establishing that he normally charges a commission of between 10% and 20% for grains, with a higher commission for more exotic grains. The owner of the co-op called another grain broker, a competitor of the plaintiff, to testify. The witness testified that he never charges more than an 8% commission when asked to locate exotic grains for his customers.
    The trial judge should rule this testimony
    (A) admissible as evidence of customary business practices in the community.
    (B) admissible as a proper lay opinion.
    (C) inadmissible as hearsay.
    (D) inadmissible as irrelevant.
A
  1. (A) This is a classic example of a Multistate cross-over testing pattern involving an interrelationship of both Evidence and Contracts principles. When answering a Contracts question that involves buyers and sellers, apply UCC rules when presented with a hypothetical involving a merchant. In this regard, course of dealing, usage of trade, or course of performance evidence may explain or supplement language in a writing, for it shows how the parties themselves interpreted their own deal. Thus, since the grain broker was a merchant, UCC Section 1-205(4) would permit evidence of customary business practices to aid in interpreting the agreement between the parties. Choice (B) is incorrect because the witness is giving fact testimony about business practices in the area, not rendering an opinion. Choice (C) is wrong because there are no out-of-court statements being offered for their truth. Choice (D) is incorrect. FRE 401 defines relevant evidence as having “any tendency to make the existence of any fact that is of consequence to the action more probable or less probable than it would be without the evidence.” The witness’s testimony about prevailing trade practices is highly relevant on the issue of the agreed-upon commission.
How well did you know this?
1
Not at all
2
3
4
5
Perfectly
156
Q
  1. A buyer filed a lawsuit against a seller based on a written contract allegedly executed at the time of the sale of the seller’s hot dog stand to the buyer. Under the terms of the contract, the seller agreed not to open a hot dog stand for a period of two years within a distance of two miles of the old hot dog stand, now owned by the buyer. The buyer alleged in his complaint that the seller violated the contract by opening a hot dog stand one-half mile from the old location. The original contract was lost prior to trial. At trial, the buyer wants to introduce a duplicate carbon copy of the written contract. The seller objects as to the authenticity of the original written contract.
    The copy is
    (A) inadmissible, because the seller contests the authenticity of the written contract.
    (B) inadmissible, because the best evidence rule requires that the original contract always be used as evidence.
    (C) admissible, because the duplicate is a carbon copy.
    (D) admissible, because the original contract was lost.
A
  1. (A) Under FRE 1003, a duplicate is admissible to the same extent as an original unless (1) a genuine question is raised as to the authenticity of the original, or (2) in the circumstances it would be unfair to admit the duplicate in lieu of the original. Therefore, if the seller contests the authenticity of the original written contract, the duplicate introduced by the buyer would not be admitted into evidence in accordance with subsection (1) of FRE 1003. Choice (B) is an incorrect rule of law. The best evidence rule does notalways require that the original document be used as evidence. If the original document is (1) lost or destroyed, (2) not obtainable by judicial process, or (3) in the possession of an opponent who refuses to produce the original at trial, other “secondary” evidence of the writing, recording, or photograph is admissible. See FRE 1004. Choice (C) is not the best answer. Although duplicate copies are generally admissible to the same extent as originals under FRE 1003, there are exceptions to this rule. Choice (A) correctly states an exception that applies to this fact pattern, whereas choice (C) merely states the general rule. Choice (D) is misleading. The key issue is that the seller contests the authenticity of the original, an issue that brings this question squarely within FRE 1003’s rule on the use of duplicates.
How well did you know this?
1
Not at all
2
3
4
5
Perfectly
157
Q
  1. One afternoon a motorist was driving her car when she struck a pedestrian who entered the intersection. The pedestrian was seriously injured in the accident and sued the motorist to recover damages for personal injuries. There were no witnesses to the accident. The motorist claims that the traffic light was green at the time she entered the intersection. The pedestrian now has amnesia and cannot remember how the accident occurred.
    At trial, the pedestrian’s attorney calls a witness to testify. The witness proposes to testify that she was in the emergency room at the hospital when the pedestrian was brought in following the accident. The witness further testifies that while the pedestrian was being treated for his injuries, the nurse asked him, “How did you get hurt?” The pedestrian replied, “The car ran the red light and struck me as I was walking through the intersection.”
    Upon objection, this evidence is
    (A) admissible as a statement made for purposes of medical treatment and diagnosis.
    (B) admissible as a present sense impression.
    (C) inadmissible, because the witness was eavesdropping on a confidential communication between the pedestrian and a hospital representative.
    (D) inadmissible, because it is hearsay not within any recognized exception.
A
  1. (D) Hearsay is defined under FRE 801(c) as a “statement, other than one made by the declarant while testifying at the trial or hearing, offered in evidence to prove the truth of the matter asserted.” The witness’s testimony as to the pedestrian’s out-ofcourt reply to the nurse’s question is hearsay, since it is being offered for its truth; namely, to prove that the motorist’s car ran a red light. By process of elimination, choice (D) is correct. The pedestrian’s reply is inadmissible as hearsay, since no exceptions apply. Choice (A) is incorrect because, under FRE 803(4), a statement made for purposes of medical diagnosis or treatment must describe “medical history, or past or present symptoms, pain, or sensations, or the inception or general character of the cause or external source thereof insofar as reasonably pertinent to diagnosis or treatment.” However, statements pointing to persons responsible for the condition are considered irrelevant to medical diagnosis or treatment and do not fall within the exception. Federal Rules of Evidence Manual p.830. The pedestrian’s statement implicated the motorist and will, therefore, be inadmissible as a statement relating to the cause of the accident. Choice (B) is incorrect because, under FRE 803(1), a present sense impression must describe or explain an event or condition white the declarant was perceiving the event or condition or immediately thereafter. Here, the pedestrian’s statement was made in the emergency room fottowingthe accident. Choice (C) is incorrect because the presence of a non-essential third person, such as the witness, would destroy whatever confidentiality existed between the pedestrian and the nurse.
How well did you know this?
1
Not at all
2
3
4
5
Perfectly
158
Q
  1. A driver has been charged with leaving the scene of an accident. The driver allegedly hit another car in a parking lot and drove away without leaving proper insurance and contact information. A bystander wrote down her license plate number and called the police. At trial, the driver proposes to testify that the day after the accident, she met with the police officer investigating the accident, and told him that she did not leave until after she had inspected both cars and determined there was no damage. The bystander is not available to testify at trial.
    The driver’s testimony is
    (A) admissible, because it is the statement of the witness herself, who is subject to cross-examination.
    (B) admissible, because it is a statement based on firsthand knowledge.
    (C) inadmissible, because the bystander is unavailable and, therefore, cannot contradict the driver.
    (D) inadmissible, because it is hearsay not within any recognized exception.
A
  1. (D) In this case, the driver is the out-of-court declarant proposing to testify as to the truth of the matter asserted that she only left the scene after determining there was no damage to either vehicle. As in the previous example, the statement itself is hearsay and will be inadmissible, since no exceptions apply. Choice (D) is, therefore, correct. Choice (A) is incorrect because the driver is proposing to testify in court as to what she said at another prior time out of court. The driver’s prior statement is, therefore, hearsay, despite the fact that she is both the witness, as well as the out-of-court declarant. By the same reasoning, choice (B) is incorrect because the driver is not actually testifying as to firsthand knowledge; rather, she is quoting herself. Choice (C) is incorrect because the rules of evidence do not predicate the admissibility of evidence on the availability of witnesses to contradict it.
How well did you know this?
1
Not at all
2
3
4
5
Perfectly
159
Q
  1. While walking across the street one afternoon, a pedestrian was hit by a car and thrown down in the middle of the intersection. The driver of the vehicle did not stop. Although he never lost consciousness, the pedestrian was in excruciating pain and has been taking strong painkillers since the accident that have affected his memory of the accident. At trial, the pedestrian calls a renowned hypnotist to testify. The hypnotist proposes to testify that after he placed the pedestrian under a hypnotic spell, the pedestrian gave a detailed account of the accident and told the hypnotist that the driver ran a red light and that the driver’s license plate number was GIJO-7 14. The driver concedes that his license plate number is
    GIJO-7 14.
    The hypnotist’s proposed testimony will most likely be held to be
    (A) admissible, because the license plate identification substantiates that the other information is accurate and trustworthy.
    (B) admissible, because it is a statement of past recollection refreshed.
    (C) inadmissible, because it is hearsay not within any recognized exception.
    (D) inadmissible, because statements procured through hypnosis cannot be authenticated as truthful.
A
  1. (C) The hypnotist’s testimony as to what the pedestrian told him while under the hypnotic spell, is an out-of-court assertion being offered for its truth. The statement will be inadmissible as hearsay, since no exceptions apply. Because the hypnotist’s testimony is inadmissible hearsay, the corroborating evidence of the driver’s license plate number does not matter. Thus, choice (A) is incorrect. Choice (B) is wrong because the foundational elements of past recollection refreshed have not been met. Past recollection refreshed under FRE 612 applies only when an attempt is made to refresh the recollection of a testifying witness. Here, the pedestrian is not testifying. Choice (D) asserts that hypnotically refreshed testimony that cannot be authenticated may be true, but authentication (orthe lackthereoO does not matter if a statement is inadmissible hearsay under the rules. Choice (0) is incorrect.
How well did you know this?
1
Not at all
2
3
4
5
Perfectly
160
Q
  1. Under the Federal Rules of Evidence, which of the following items of evidence is NOT self- authenticating?
    (A) A pamphlet on air safety regulations that a plaintiff claims is an official publication of the Federal Aviation Administration.
    (B) A copy of a magazine that a plaintiff claims contains a libelous picture.
    (C) A holographic will without attesting signatures that a plaintiff claims was written by her mother in her own handwriting.
    (D) A certified copy of his car registration form that, according to a plaintiff, verifies his ownership of the vehicle.
A
  1. (C) Choice (C) is correct, since an unattested holographic will of the plaintiffs mother will not be self-authenticating. Under FRE 902, extrinsic evidence of authenticity as a condition precedent to admissibility is not required with respect to the following: (1) domestic public documents under seal; (2) certified genuine foreign public documents; (3) certified copies of public records—such as the certified copy of the car registration form in choice CD); (4) official publications—including books and pamphlets issued by public authority—such as the Federal Aviation Administration pamphlet on air safety regulations in choice (A); and (5) newspapers and periodicals—such as the copy of a magazine in choice (B).
How well did you know this?
1
Not at all
2
3
4
5
Perfectly
161
Q
  1. An owner of a storage facility stores flammable gases under high pressure in large spherical tanks. The facility was constructed for the owner by a company that specializes in the construction of such facilities. After the facility had been in use for three months, one of the tanks ruptured, causing a spillage and loss of the chemicals. The owner sued the company for strict liability, claiming that the storage tank had been defectively manufactured.
    At trial, the owner called a photographer who testified she was hired by the owner to photograph the ruptured tank after the accident. She further testified that the roll of film was sent to a commercial film lab where the photographs were developed. Thereupon, the owner’s attorney showed the photographer the photographs, which she identified as the ones she had taken and given to the Owner.
    The owner’s attorney offers 10 photographs into evidence. Upon objection by the company’s counsel, which of the following, if true, is the best reason to exclude the photographs from evidence?
    (A) The photographer was not shown to be an expert or professional photographer.
    (B) There was an unaccountable break in the chain of custody from the time when the photographs were taken to the period when the negatives were processed.
    (C) There was no testimony that the photographs accurately portrayed the ruptured tank.
    (D) The photographs are not the best evidence to prove how the tank ruptured.
A
  1. (C) To authenticate a photograph, it is always required that the proponent establish that the pertinent parts of the picture are a reasonable and accurate representation of the subject pictured. This requirement may be satisfied if a witness (not necessarily the photographer) has observed the scene and testifies that the picture is an accurate representation. Lilly, Law of Evidence, p.423. Choice (C) is the best answer to exclude the 10 photographs. Lack of proper authentication would preclude admissibility if no testimony were offered that the photographs accurately portrayed the ruptured tank. Choice (A) is incorrect because there is no requirement under the rules that a photographer be qualified as an expert or professional photographer. Choice (B) is wrong for two reasons: (1) it is predicated on facts that are not contained in the fact pattern; and (2) even if there were a break in the chain of custody, it would still be possible to authenticate the photographs as reasonably and accurately representing the ruptured tank. Choice (D) is wrong. The best evidence rule only applies when the contents of a writing, recording, or photograph are at issue. See FRE 1002.
How well did you know this?
1
Not at all
2
3
4
5
Perfectly
162
Q
  1. A woman was sailing in a bay when she was struck by a motorboat. The woman, who was seriously injured in the accident, sued the motorboat operator in a personal injury action. During the course of the lawsuit, an important question of consequence was the wind speed on the afternoon the woman was sailing.
    At trial, the woman calls an orthopedic surgeon to testi1’. The surgeon, who is an amateur boating enthusiast, testified that he was navigating his sailboat into the bay when the motorboat hit the woman. Furthermore, the surgeon testified that in the cockpit of his boat he maintains a sophisticated electronic weathering device that measures wind speed, temperature, and barometric pressure at periodic intervals. The woman then offers into evidence the computer printout from the surgeon’s weathering device measuring the wind speed at the time of the accident.
    Upon objection by the motorboat operator, the printout is
    (A) admissible as past recollection recorded.
    (B) admissible as the record of regularly conducted activity.
    (C) inadmissible as hearsay not within any recognized exception.
    (D) inadmissible, unless there is foundation testimony as to the accuracy and good working condition of the surgeon’s electronic weathering device on the afternoon in question.
A
  1. (D) In the woman’s personal injury action against the motorboat operator, her witness, the surgeon, is offering into evidence a computer printout from his sophisticated electronic weathering device measuring wind speed. Regarding this computer printout, students should begin by understanding that under FRE 1001(3), “If data are stored in a computer or similar device, any printout or other output readable by sight, is an ‘original’.” There is no best evidence problem as to the admissibiLity of the computer printout. Next, a foundation for the authenticity of the scientific wind device must be laid. Under the original Frye v. United States standard, scientific evidence is admissible where the device or principle from which the deduction is made is “sufficiently established to have gained general acceptance in the particular fieLd.” McCormick, pp. 610—614. Choice (D) is correct. Choice (A) is incorrect because the “past recollection recorded” hearsay exception requires a human witness. Choice (B) is wrong; it describes the business records exception of FRE 803(6), which is not at issue here. Choice (C) is wrong because hearsay under the rules requires either a human declarant or, in the case of computer records, input generated by a human being. According to Mueller & Kirkpatrick’s treatise on evidence, “when information provided by machines is mostly a product of mechanical measurement or manipulation of data by common scientific or mathematic techniques, hearsay concerns are usually addressed by requiring the proponent to show the machine and its functions are reliable, that it was correctly adjusted or calibrated, and that basic data put into the machine are accurate.” Christopher Mueller& Laird Kirkpatrick, Evidence § 8.11 (3d ed. 2003). Mueller& Kirkpatrick go on to explain that when the output of a machine depends on human input, regular hearsay concerns apply.
How well did you know this?
1
Not at all
2
3
4
5
Perfectly
163
Q
  1. A shopper suffered injuries to her back and legs when she slipped on a loose piece of carpeting in a hallway connecting a department store and a cafeteria.
    The shopper brought suit against the owner of the entire building. The owner claimed that since he leased the cafeteria, he no longer retained any control over the connecting hallway.
    If the shopper offers into evidence the fact that two days after the accident, the owner had the hallway carpet re-laid, the court should rule that the evidence is
    (A) admissible, because it is relevant to the issue of the owner’s knowledge of the unsafe condition.
    (B) admissible, because it shows that the owner retained ownership and control.
    (C) inadmissible, because it shows subsequent repair, which is against public policy.
    (D) inadmissible, because the evidence is irrelevant to the accident.
A
  1. (B) ALthough evidence of subsequent repair is usually excluded because its basic relevancy is outweighed by public policy considerations, evidence of subsequent remedial measures is admissible for the Limited purpose of showing ownership or control. Choice (A) is incorrect. One of the policies underlying Rule 407 is to encourage owners to repair unsafe conditions. Permitting a jury to infer that repair of an unsafe condition equates to knowledge it was unsafe at the time of the accident would discourage owners from making repairs. Although choice (C) is a true statement, it is not the best answer in this case because the exception to show ownership or control applies more specifically to the fact pattern. Choice (D) is incorrect. The evidence is relevant under FRE 401.
How well did you know this?
1
Not at all
2
3
4
5
Perfectly
164
Q
  1. A man and a woman are tennis partners who play regularly. Last January, the man slipped and fell while restocking grocery shelves at work. The man is suing the grocery store for damages, claiming that he suffers from pain in his back and legs as a result of the accident. At trial, the woman testified that five days prior to the accident, the man told her, “I can’t complete the tennis match because of pains in my back and legs.”
    The trial judge should rule this testimony
    (A) inadmissible as hearsay.
    (B) inadmissible as opinion testimony.
    (C) admissible as a proper lay opinion.
    (D) admissible as a statement of bodily condition.
A
  1. (D) As a general rule, a declaration of a physical condition (e.g., “I have pains in my back and legs”) is admissible as an exception to the hearsay rule under FRE 803(3) (then-existing mental, emotional, or physical condition). Choice (D) is, therefore, correct. Choice (A) is wrong. The statement is hearsay, but it is admissible under a recognized hearsay exception. Choices (B) and (C) are incorrect for similar reasons. The man’s statement is a description of his physical condition, not a statement of opinion.
How well did you know this?
1
Not at all
2
3
4
5
Perfectly
165
Q
  1. A man works at a tire warehouse, where his job duties consist of loading tires onto trucks. The job is exhausting, and the warehouse is always short of personnel. One day, while loading a heavy tractor tire onto a truck by himself, he experienced a sharp and excruciating pain in his lower back. He did not report the incident to his employer, but continued to work. A week later, after work, he went to the doctor for treatment. The man then sues his employer, alleging that the employer failed to provide him with adequate assistance, safety equipment, and training to load heavy tractor tires, thereby contributing to his injury. The employer alleges that the man is merely trying to obtain compensation for an old back injury, unrelated to his employment.
    At trial, the man’s doctor testified that the man told her that his back began hurting while he was loading the tire on the truck. The doctor’s testimony is
    (A) admissible as expert testimony.
    (B) admissible as a statement for the purposes of medical diagnosis.
    (C) inadmissible as a violation of the physicianpatient privilege.
    (D) inadmissible as hearsay not within any recognized exception.
A
  1. (B) Statements made to assist a physician in diagnosing or treating a physical condition are hearsay and require the exception listed in choice (B) in order for them to be admissible. Choice (A) is wrong because the doctor is repeating the man’s statement and not giving his expert opinion. Choice (C) is incorrect because the doctor’s testimony, in such a situation, does not violate the doctor-patient privilege. Choice CD) is incorrect; although the statement is hearsay, it does meet an exception and is admissible.
How well did you know this?
1
Not at all
2
3
4
5
Perfectly
166
Q
  1. A defendant was being prosecuted on a charge of manslaughter in the first degree for causing the death of a 15-year-old girl on whom he had allegedly performed an abortion in his private clinic.
    A witness for the defendant testified to the defendant’s non-culpability at the grand jury proceeding. The witness was later called as a defense witness at the trial. During his direct examination, defendant’s attorney asked him, “Is it not true that the defendant could not have performed the abortion on the decedent since he was assisting you in another abortion at the time?” The witness answers in the negative and states that the defendant did, in fact, perform the abortion on the girl.
    The defendant’s attorney then assails the witness’s testimony as a recent fabrication, and asks the witness if he had not stated to a grand jury that the defendant was not guilty of performing the abortion, thus causing the death of the girl. The prosecutor objects to this question.
    The trial judge should rule that this question is
    (A) objectionable, because the defendant’s attorney did not lay a proper foundation for impeaching his own witness.
    (B) objectionable, because a party may not impeach his own witness.
    (C) unobjectionable, because a witness’s testimony before a grand jury is more reliable because it occurred shortly after the abortion was allegedly performed.
    (D) unobjectionable, because a party can impeach his own witness by a prior inconsistent statement.
A
  1. (D) In accordance with FRE 607, the credibility of a witness may be attacked by any party, including the party calling him. The common law rule against impeaching one’s own witness has been abandoned. Where the impeaching is by a prior statement in a grand jury proceeding, it is free from hearsay dangers and is excluded from the category of hearsay under FRE 801 (d) (1)(A). For similar reasons, choice (B), which states that a party may not impeach its own witness, and choice (A), which claims that the attorney did not lay a proper foundation for impeachment, are the wrong choices. Choice (C) is incorrect. The issue in impeachment by prior inconsistent statements is not the reliability of the prior statement, but the fact that the witness has made inconsistent statements on the same matter.
How well did you know this?
1
Not at all
2
3
4
5
Perfectly
167
Q
  1. While undergoing a routine plastic surgery procedure, a patient died on the operating table. The patient’s husband is suing the plastic surgeon alleging malpractice. Two years later, at trial, the defense called another doctor as an expert witness. The expert witness testified that the patient’s death was a freak accident and that the surgeon’s performance met the highest standards for medical care. On cross-examination, the plaintiff’s attorney brings out the fact that, one week before trial, the expert witness and the surgeon signed an agreement to become partners in an outpatient plastic surgery clinic. The plaintiff’s attorney suggests that the expert’s testimony was awfully convenient and likely motivated by his desire to maintain a good relationship with the defendant. The defendant then calls another surgeon at the hospital as a witness. He is asked to tell the jury about a conversation he had with the expert inunediately following the patient’s death. The witness stated that “the expert told me the patient’s death was a textbook example of a freak accident and there was nothing anyone could have done to prevent it.”
    On objection to this testimony, defendant’s attorney seeks a ruling on the admissibility of this question and answer. The trial judge should
    (A) sustain the objection, because the testimony is hearsay.
    (B) overrule the objection, because a witness’s veracity may be rehabilitated by a prior consistent statement.
    (C) sustain the objection, because a prior consistent statement cannot be used to rehabilitate a witness’s testimony.
    (D) overrule the objection, because the expert’s statement is a statement against interest.
A
  1. (B) Generally, the prior consistent statements of a witness are not admissible to support the witness’s testimony at trial. However, where the testimony of a witness is assailed as a “recent fabrication,” it may be confirmed by proof of declarations of the same tenor before the motive to falsify existed. Prior consistent statements used for this purpose are not hearsay under FRE 801(d) (1). Because prior consistent statements are not hearsay under FRE 801(d)(1), choice (A) is incorrect. Choice (C) is wrong: as discussed above, prior consistent statements can be used to rehabilitate a witness’s testimony. Choice (D) is incorrect, for two reasons: (1) the statement was not against the expert’s interest; and (2) in orderto use the statements-againstinterest hearsay exception of FRE 804(b)(3), the decLarant must be unavailable. The expert, having just testified in the trial, is available.
How well did you know this?
1
Not at all
2
3
4
5
Perfectly
168
Q
  1. A defendant was charged with vehicular battery after driving through a red light and crashing into a woman’s small convertible. The woman suffered massive internal injuries and lapsed into a coma for several hours after she reached the hospital. The woman’s best friend, who was a passenger in the car, miraculously received only a few minor bruises. After the collision, the passenger stayed with the woman trying to comfort her until the ambulance arrived.
    At trial, the passenger is called to testify. ‘While on the witness stand, she gazes with sorrow at the woman, who is still wearing a cast on her leg. The passenger testifies that the first thing she remembered seeing after the accident was the woman’s bloodied body trapped in the mangled wreckage. The state’s attorney then asked the passenger if the woman said anything to her before being taken to the hospital. The passenger answered, “Yes, she told me: ‘I know I’m dying. Why did he have to run that red light?”
    Upon objection by the defendant’s attorney, the court should rule the passenger’s testimony
    (A) admissible, because it is irrelevant that this is not a homicide case.
    (B) admissible, because the woman believed her death was imminent when she made the declaration.
    (C) inadmissible, because of the close relationship between the woman and the passenger.
    (D) inadmissible, because it is hearsay not within any recognized exception.
A
  1. (A) In order for a dying declaration to be admitted, the dec(arant must be unavailabLe to testify. In this example, the dying declaration exception is inapplicable because the woman is not unavailable. In accordance with FRE 804, the dying declaration exception is not excluded by the hearsay rule if the declarant is unavailable as a witness. Since the woman is in court and available as a witness, choice (B) is wrong. Consequently, choice (A) is correct because her statement, “I know I’m dying,” is admissible under FRE 803 as a statement of then-existing physical condition. In addition, her statement, “Why did he have to run the red light?” is not excluded by the hearsay rule because, under FRE 803, it qualifies as a present sense impres. sion. Note that in regard to the hearsay exceptions classified under FRE 803, the availability of the declarant is immaterial. Choice (C) is incorrect. The relationship between a hearsay declarant and a witness is immaterial in determining the admissibility of the hearsay statement. Choice (D) is also incorrect because, although the statement is hearsay, it fits within recognized hearsay exceptions.
How well did you know this?
1
Not at all
2
3
4
5
Perfectly
169
Q
  1. A victim was standing on a street corner waiting to make a drug buy. The drug dealer was always late, so the victim continued to wait outside in the freezing cold weather because he needed a quick fix. Suddenly a car rounded the street corner and a shot rang out. The victim slumped over and fell to the sidewalk. Immediately after being shot and before he died, the victim exclaimed, “The defendant did it!” The defendant was indicted and is now on trial for the victim’s murder.
    Based on these facts alone, the admissibility of the victim’s statement should be determined by
    (A) the jury, after a preliminary determination by the judge.
    (B) the jury, without a preliminary determination by the judge.
    (C) the judge, with instruction to the jury that it may consider the dying declaration if it determines that the victim knew he was dying.
    (D) the judge, without assistance from the jury.
A
  1. (D) FRE 804 (B) (2), also referred to as the “dying decLaration exception” to the hearsay rule, provides that “in a prosecution for homicide or in a civil action or proceeding, a statement made by a declarant while believing that his death was imminent, concerning the cause or circumstances of what he believed to be his impending death is not excluded by the hearsay rule if the declarant is unavailable as a witness.” Note two important points in regard to the dying declaration exception: (1) the FRE has broadened the traditional use of dying declarations in prosecutions for homicide by now allowing such statements to be utilized in civil cases; and (2) in accordance with the FRE, the declarant does not need to die in order for his statement “made under belief of impending death” to be deemed admissible. Rather, the requirement that must be met is that the declarant be unavailable to testify. Choices (A), (B), and (C) all misstate the proper roles of the judge and the jury pertaining to the admissibility of evidence. According to FRE 104(a), questions concerning the admissibility of evidence “shall be determined by the court [judge].” Choice (D) is the only answer that correctly states the role of a judge in determining the admissibility of evidence.
How well did you know this?
1
Not at all
2
3
4
5
Perfectly
170
Q
  1. A defendant, on trial for robbery, took the stand and testified in his own behalf. On cross-
    examination, the prosecuting attorney asked the defendant whether he had committed false pretenses by making long-distance telephone calls by the unauthorized use of a credit card number. The defendant replied, “That’s a lie, I never did.” In rebuttal, the prosecuting attorney called the court officer to testif’ that the defendant admitted committing false pretenses. The court officer further testified, however, that the defendant was granted immunity from prosecution in exchange for his testimony against others.
    The court officer’s testimony is
    (A) admissible, as bearing on the defendant’s credibility.
    (B) admissible, as bearing on the defendant’s guilt.
    (C) inadmissible, because the testimony is extrinsic evidence.
    (D) inadmissible, because the court order is the
    best evidence.
A
  1. (C) In accordance with FRE 608 (b), specific instances of the conduct of a witness, for the purpose of attacking or supporting his credibility, otherthan conviction of crime as provided in FRE 609, may not be proved by extrinsic evidence. They may, however, in the discretion of the court, if probative of truthfulness or untruthfulness, be inquired into on cross-examination of the witness (1) concerning his character for truthfulness or untruthfulness, or (2) concerning the character for truthfulness or untruthfulness of another witness as to which character the witness being cross- examined has testified. Choice (A) is incorrect, because of the prohibition against using extrinsic evidence. Choice (B) is also incorrect. Although the evidence does have a bearing on the defendant’s guilt, the rules prohibit using it. Finally, choice (D) is wrong because the contents of the court order are not in dispute; therefore, the best evidence rule is inapplicable.
How well did you know this?
1
Not at all
2
3
4
5
Perfectly
171
Q
  1. In a suit by an investor against a speculator, a relevant fact is the price of gold as quoted on a specific date 20 years ago. The investor calls a librarian to authenticate the microfilm copy of a newspaper from that date. This copy, kept in the archives of the public library, is the only record of that particular issue of the newspaper.
    The microfilm is
    (A) admissible, to prove the price of gold on that day.
    (B) admissible as an ancient document.
    (C) admissible as past recollection recorded.
    (D) inadmissible as not the best evidence.
A
  1. (A) Under FRE 1003, duplicates of a document are generally admissible to the same extent as the original. According to FRE 1001(4), a duplicate includes a photographic miniature. Thus, a microfilm of the newspaper would be considered a duplicate and would be admissible as an original under FRE 1003. Choice (B) is not a bad answer—the ancient documents exception of FRE 803(16) permits the admissibility of statements in a document more than 20 years old, the authenticity of which is established, but choice (A) is a better answer because the use of a microfilm copy of the newspaper raises issues regarding the admissibility of duplicates. Choice (C) is incorrect. The past recollection recorded exception applies when a witness on the stand is unable to remember an event but made a record of it close to the time of its occurrence. Choice (D) is wrong because it misstates the best evidence rule.
How well did you know this?
1
Not at all
2
3
4
5
Perfectly
172
Q
  1. For which of the following preliminary questions of fact will the trial court judge, not the jury, determine admissibility?
    (A) Whether the voice on a sound recording is that of the defendant.
    (B) Whether defendant’s copy of a document accurately reflects the contents of the original writing.
    (C) Whether a conspiracy existed and defendant and declarant were members of the conspiracy before admitting an alleged declaration by a co-conspirator.
    (D) A plaintiff sues on a lost writing, and the defendant contends that it was not lost because it never existed. The question to be decided is whether the original writing ever existed.
A
  1. (C) Lawyers sometimes use the expression that the jury decides questions of fact, and the judge resolves questions of law. But there are numerous occasions when the judge makes factual determinations. In the pre-trial process, for example, he resolves factual disputes that may arise in connection with discovery proceedings. The judge also decides preliminary factual questions that accompany the application of the exclusionary rules of evidence. Here, choice (D) is wrong because FRE 1008 states that “when an issue is raised (a) whether the asserted writing ever existed, or (b) whether another writing, recording, or photograph produced at trial is the original, the issue is for the trier of fact to determine as in the case of other issues of fact.” Choices (A) and (B) are wrong because they also involve factual issues to be decided by the jury. Choice (C) is correct because, according to McCormick, “the judge should make the preliminary determination whether a conspiracy existed and defendant and declarant were members of it, before admitting an alleged declaration by a co-conspirator.” Evidence, pg. 139.
173
Q
  1. A defendant is on trial for robbery. The defendant’s attorney plans to call a witness. Nine years earlier, the witness had been convicted of misdemeanor battery. The defendant’s attorney has filed a motion disallowing the prosecution from questioning the witness regarding this prior conviction.
    The motion will likely be
    (A) granted, as a matter of right.
    (B) granted, because the prejudice to the defendant substantially outweighs the probative value of admitting the witness’s conviction.
    (C) denied, because the credibility of a witness may be attacked by any party.
    (D) denied, because a period of less than 10 years has elapsed since the date of conviction.
A
  1. (A) Under FRE 609 (a)(2), “For purposes of attacking the credibility of a witness, evidence that he has been convicted of a crime shall be admitted if elicited from him or established by public record during cross-examination but only if the crime (2) involved dishonesty or false statement, regardless of the punishment.” The only way a conviction of a misdemeanor can be used to impeach a witness under this rule is if the misdemeanor involves dishonesty or false statement. Since battery does not, the prosecution may not question the witness regarding this prior conviction. Choice (A) is thus correct. Choice (B) is wrong because the “balancing test” under FRE 609 applies only to convictions more than 10 years old. Choice (C) is wrong. Although any party may attack the credibility of a witness, FRE 609 specifically controls the use of convictions evidence. Choice (D) is misleading. The question is designed to attract students because everyone remembers the 10-year time limit of FRE 609. It is important to remember that misdemeanor offenses not involving dishonesty or false statements aren’t admissible under FRE 609 to impeach a witness.
174
Q
  1. A plaintiff is bringing an action against a defendant to quiet title to determine ownership of lakefront riparian property. At issue is the mean water level of the lake. For the past several years, a commercial fisherman has kept daily logs of the lake’s water level. In preparation for trial, the plaintiff hired a graphic artist to prepare charts from the fisherman’s logs to establish the property’s boundary line. Based upon the information contained in the fisherman’s logs, the graphic artist compiled a number of charts, which the plaintiff now seeks to introduce into evidence. At trial, it was determined that the fisherman is an expert on water level computations.
    The defendant objects to the charts being offered into evidence. The court will most likely rule the charts
    (A) admissible, because the fisherman is an expert on water level computations.
    (B) admissible, because they are summaries of voluminous records.
    (C) inadmissible, because the charts are hearsay not within any recognized exception.
    (D) inadmissible, because the fisherman’s daily logs are the best evidence of the lake’s water level.
A
  1. (B) According to FRE 1006, the contents of voluminous writings, recordings, or photographs that cannot conveniently by examined in court may be presented in the form of a chart, summary, or calculation.The originals, or duplicates, shall be made available for examination or copying, or both, by other parties at a reasonable time and place. In this question, the fisherman has kept daiLy Logs for severaL years on the water level of the lake. Clearly, these logs are voluminous records that may not be conveniently reviewed in court by the jury. Consequently, the charts drafted by the graphic artist may be admitted as summaries of voluminous records. Choice (A) is incorrect. There is no requirement that the compiler of records be qualified as an expert. Choice (C) is incorrect. The fisherman’s logs themselves are probably hearsay, but would be admissible under the business records exception of FRE 803(6). The reason for using a summary under FRE 1006 is that the fisherman’s records themselves could not be conveniently reviewed in court; a summary would be much better. Choice (D) misstates the best evidence rule and is wrong.
175
Q
  1. A man is on trial for securities fraud. He takes the stand to testify in his own behalf. On direct examination, the man’s attorney proffers evidence of his good character.
    Upon objection by the prosecution, which of the following statements is correct?
    (A) The defendant may introduce evidence to show truthfulness.
    (B) The defendant may introduce evidence to show honesty.
    (C) The defendant may not introduce evidence to show honesty because character evidence is inadmissible.
    (D) The defendant may not introduce evidence of truthfulness unless it is inquired into on cross- examination.
A
  1. (B) A trial for fraud is a criminally related civil proceeding. As a general rule, character evidence is not admissible in a civil case unless character is “in issue.” According to FRE 405, character is in issue if it is an essential element of a charge, claim, or defense. For the exam purposes, remember that character is generally in issue and, thus, admissible in the following civil actions: (1) defamation, (2) child custody, (3) negligent entrustment, and (4) negligent hiring. Another civil action where character is in issue is an action for fraud. In regard to a fraud or deceit action, the defendant’s character for honesty is in issue. Therefore, the defendant may introduce evidence to show his character trait for honesty. Choice (B) is, therefore, correct. Choice (A) is wrong because, in a fraud action, it is the defendant’s character trait for honesty, not truthfulness, that is in issue. Choice (C) is wrong for the same reasons that choice (B) is correct. Choice (0) is wrong because it suggests that the defendant cannot respond unless his character has been attacked on cross-examination. If the defendant testifies, he becomes a witness. His character for truthfulness may be attacked “by opinion or reputation evidence or otherwise.” FRE 608(a). Once his character for truthfulness is attacked, he can respond with reputation or opinion evidence of his positive character for truthfulness.
176
Q
  1. A gardener sued a homeowner for breach of contract. The gardener claims that the homeowner hired him to perform landscape work at his home. After performing the gardening services, the homeowner paid him $100. The gardener, who speaks only English, contends that the homeowner, who speaks only Japanese, originally agreed to pay him $200 for the work. When the parties entered into their agreement, the gardener was accompanied by his friend, who is now deceased, who spoke both English and Japanese. This jurisdiction has a typical Dead Man’s Statute in effect.
    At trial, the gardener now seeks to testify to the terms of his agreement with the homeowner as communicated to him by his friend. Upon objection by the homeowner’s attorney, the gardener’s proposed testimony is
    (A) admissible as an admission by a party-opponent.
    (B) admissible as non-hearsay to corroborate evidence of a verbal act.
    (C) inadmissible, because of the Dead Man’s Statute.
    (D) inadmissible, unless it is established that the gardener accepted the friend as the homeowner’s agent in communicating the terms of the contract.
A
  1. (D) Under FRE 801(d)(2)(D), a statement offered against a party by his agent or servant concerning a matter within the scope of his agency or employment, made during the existence of the relationship is admissible as a vicarious admission. If it is established that the friend was the homeowner’s agent in communicating the terms of the contract, then the friend’s statement may be offered as a vicarious admission against the party, the homeowner, by his opponent, the gardener. Choice (D) is correct. Choice (C) is incorrect. A Dead Man’s Statute typically provides that in any action commenced or defended on behalf of a decedent, any other party surviving shall be disabled from testifying as to any transaction involving the decedent. The practical consequence of this type of statute is that if a survivor has rendered services or furnished goods to a person he trusted, without an outside witness or admissible written evidence, he is helpless if the other dies and the representative of his estate declines to pay. McCormick, Evidence, p. 159. In these facts, the contract action was not commenced or defended on behalf of the friend, the deceased, so the Dead Man’s Statute will not apply. Choice (A) is incorrect because the gardener is seeking to testify as to the friend’s statements. The fact that the friend is not a party to the lawsuit precludes the use of his statements as an admission. Choice (B) is incorrect because the friend’s statements concerning the terms of the homeowner’s agreement would be inadmissible as hearsay, unless a basis for a vicarious admission is established.
177
Q
  1. A victim and a defendant both worked as longshoremen at a shipyard. After the victim was shot to death, the defendant was tried for murder and acquitted. Following the acquittal, the victim’s estate sued the defendant in a wrongful death action. During the civil trial, the victim’s estate called a witness to testify. The witness, who worked with both men at the shipyard, testified that two weeks before the shooting, the victim came to work with a broken nose and said that the defendant had caused it. The attorney for the victim’s estate then asked the witness the following question, “Was the defendant present during your conversation with the victim, and if so, did he say anything about the victim’s broken nose?” The witness replied, “Yes, the defendant was present, and after the victim told me that the defendant broke his nose, the defendant said, ‘And that’s only the beginning.”
    Upon objection by the defendant’s attorney, the witness’s testimony is
    (A) admissible, because it reports a declaration against interest.
    (B) admissible, because it reports the defendant’s adoptive admission of the victim’s assertion.
    (C) inadmissible, because of the principle of collateral estoppel.
    (D) inadmissible, because it is hearsay not within any recognized exception.
A
  1. (B) FRE 801(d) (2) (B), which defines an adoptive admission, provides that a statement is an admission if it is offered against a party and is “a statement of which (the party) has manifested his adoption or belief in its truth,” under circumstances where a reasonable person would deny the statement. The defendant’s silence when the victim told the witness that the defendant broke his nose, coupled by his assertion, “And that’s only the beginning,” clearly evidence the defendant’s admission that he did, in fact, break the victim’s nose. Such a “tacit admission” of a party, namely, the defendant, will be admissibLe against him. Choice (B) is correct. Choice (A) is incorrect, since a declaration against interest is a hearsay exception. But the statement being offered is not hearsay, since it is an admission. Choice (C) is incorrect, since the question of whether or not the defendant broke the victim’s nose would not have to be a necessarily determined (or even actuaLly litigated) issue concerning the murder of the victim. Choice (D) is incorrect, since admissions are defined as non-hearsay.
178
Q
  1. A man was prosecuted for assault and battery after he admitted striking a victim with a pool cue during a barroom argument. The man claimed that he acted in self-defense after he was attacked by the victim, who was drunk and belligerent.
    As his first defense witness, the man calls his neighbor to testify that the man is a good neighbor. The neighbor’s testimony is
    (A) admissible, because it is relevant to show the improbability of the man’s having committed an unprovoked attack.
    (B) admissible, because it is relevant to support the man’s credibility.
    (C) inadmissible, because it is merely the opinion of a character witness.
    (D) inadmissible, because it is not directed toward a pertinent trait of the man’s character.
A
  1. (D) FRE 404 provides that evidence of a person’s character or a trait of his character is not admissibLe for the purpose of proving that he acted in conformity therewith on a particular occasion, except: (1) when evidence of a pertinent trait of his character is offered by the accused, or (2) by the prosecution to rebut the same. The neighbor’s testimony that the man is a “good neighbor,” however, is not admissible because it is not directed to a pertinent trait of the man’s character (e.g., peacefulness, honesty, and the like). On the contrary, the neighbor can testify that the man is “non-violent” or that he has a reputation for “peacefulness” because such traits are pertinent to show circumstantially the improbability that the man was the aggressor in support of his claim of self-defense. Choice (A) is incorrect because evidence of being a good neighbor is not relevant to whether a defendant would commit an unprovoked attack. Choice (B) is wrong, because being a good neighbor is not relevant to the issue of credibility. Finally, choice (C) is inapplicable to this fact pattern. In addition, under FRE 405, character witnesses are required to testify in the form of reputation or opinion testimony. If otherwise proper under the ruLes, there is no problem with a character witness testifying as to his or her opinion.
179
Q
  1. A defendant was prosecuted for aggravated assault after he stabbed a man in the chest during a dispute over a gambling debt. The defendant claimed he acted in self-defense after the man drew a knife on him and made menacing gestures with it. The defendant called a witness to the stand to testify that the defendant had a reputation in the community for being a peaceful person.
    On cross-examination of the witness, the prosecuting attorney asked the witness if he had heard that the defendant had often engaged in fights and brawls.
    Upon objection by the defendant’s attorney, the trial judge should rule the question
    (A) proper, because it tends to show the witness’s knowledge of the defendant’s reputation.
    (B) proper, because it is relevant to prove the defendant’s violent propensities.
    (C) not proper, because it puts into evidence separate and unrelated offenses.
    (D) not proper, because character cannot be proved by generalities.
A
  1. (A) FRE 405 provides that “In all cases in which evidence of character or a trait of character of a person is admissible, proof may be made by testimony as to reputation or by testimony in the form of an opinion. On cross-examination, inquiry is allowable into relevant specific instances of conduct.” Accordingly, the witness may be asked whether he had heard or whether he knew “that the defendant had often engaged in fights and brawls.” This is allowable as to whether the witness (who is a character witness) had knowledge of the defendant’s reputation for fighting. Choice (B) is incorrect because the purpose of the cross-examination is not to contest the defendant’s character, but rather to test the witness’s knowledge. Choice (C) is wrong because the defendant’s prior fights and brawls are related to the character trait of peacefulness upon which the witness testified. Choice (D) is also wrong. Character can be provided through opinion and reputation testimony on pertinent character traits. To an extent, every character trait is a generality, a shorthand expression of a series of actions and interactions overtime.
180
Q
  1. A defendant was charged with illegally selling arms to a foreign country without a government license. During the trial, the defendant admitted selling the weapons, which included missiles, rocket launchers, bazookas, and ammunition, to agents of the foreign country. In his defense, however, the defendant testified that he did not sell the arms as a private citizen, but rather for his employer, a federal agency. In rebutting the defendant’s testimony, the government called a high-ranking official from the federal agency to the stand. He testified that if the defendant were, in fact, a member of the agency, that information would ordinarily be contained in the agency’s employment records. Asked whether the defendant’s employment records were on file with the agency, the official then testified, “I have searched the agency employment records diligently, but I haven’t found any documentation that the defendant was employed by the agency.”
    Upon objection by the defendant’s attorney, the official’s testimony is
    (A) admissible, because it shows the absence of an entry in the record of a public agency.
    (B) admissible, because the official had firsthand knowledge of the information contained in the agency’s employment records.
    (C) inadmissible, because it is hearsay not within any recognized exception.
    (D) inadmissible, because the agency employment records are the best evidence to substantiate whether the defendant was actually in the employ of the governmental intelligence agency.
A
  1. (A) In United States v. De Georgia, 420 F.2d 889 (1969), proof that a car rental agency’s records showed no lease or rental activity regarding a certain vehicle was admissible as tending to show that the defendant in possession of the vehicle had stolen it. FRE 803(10) provides that “to prove the absence of a record, report, statement, or data compilation, in any form, or the nonoccurrence or nonexistence of a matter of which a record, report, statement, or data compilation, in any form, was regularly made and preserved by a public office or agency, evidence in the form of a certification in accordance with FRE 902, or testimony, that diligent search failed to disclose the record, report, statement, or data compilation, or entry” is admissible as a hearsay exception. As a result, the official’s testimony is admissible under this hearsay exception, so choice (C) is incorrect. Choice (D) is likewise incorrect because the mere non-existence of the employment records, not their contents, is at issue; therefore, the best evidence rule does not apply. Choice (B) is wrong because a witness has firsthand knowledge when he actually observes an event (e.g., a car accident). If a witness testifies to someone else’s statement or to a writing prepared by someone else, then a hearsay issue arises. Since the official is testifying to the contents of the employment records (which obviously were compiled by someone else), this presents a hearsay problem.
181
Q
  1. An incumbent was running for re-election as the mayor of a city. During one of his opponent’s campaign speeches, he stated that the mayor had once propositioned a prostitute. A television correspondent who was covering the election was present when the opponent made the statement. She recorded the opponent’s remark about the mayor in her notebook. In a stunning upset, the opponent defeated the mayor in the election. Many of the pollsters attributed the mayor’s loss to the question of his moral character following the opponent’s smear campaign depicting the mayor as a client of prostitutes.
    Following the election, the mayor sued the opponent for defamation, basing his suit on the opponent’s statement that the mayor had “once propositioned a prostitute.” At trial, the mayor calls the television correspondent to testify concerning her recollection of what the opponent said during the campaign speech attended by the television correspondent.
    The television correspondent’s testimony is
    (A) admissible, because the television correspondent recorded the opponent’s statement in her business capacity.
    (B) admissible, because the television correspondent had firsthand knowledge of the opponent’s statement.
    (C) inadmissible, because the notebook entry is the best evidence.
    (D) inadmissible, because it is hearsay not within any recognized exception.
A
  1. (B) Where personal knowledge is offered as testimony to prove an event or statement, the best evidence rule does not apply. In other words, the television correspondent’s testimony is not “reliant” on the notes she recorded in her notebook; she’s testifying as to her own recollection. Therefore, the notebook entry is not the best evidence of the opponent’s remark. Choice (C) is incorrect. Generally, testimony descriptive of non-written transactions is not considered to be within the best evidence ruLe and may be given without producing or explaining the absence of a writing recording the fact. McCormick, p. 707. Likewise, in a defamation case, the actual “tortious words” of slander are oral acts which, by definition, are non-hearsay. Therefore, choice (D) is incorrect. Choice (A) is incorrect because, inasmuch as the opponent’s defamatory statement is non-hearsay, it cannot be admissible under the business record exception. Choice (B) is correct.
182
Q
  1. A witness lived next door to a victim. Late one night, the witness overheard the victim scream, “The defendant, please don’t do it!” The next day the victim was found dead in her bedroom. The defendant was arrested and charged with murdering the victim.
    At trial, the witness proposes to testify to the victim’s statement. Upon objection by defendant’s counsel, the court should rule the witness’s testimony regarding the victim’s statement
    (A) admissible as a dying declaration if the jury determines that the victim believed that her death was imminent.
    (B) admissible as a dying declaration if the judge, by preponderance of the evidence, determines that the victim believed that her death was imminent.
    (C) inadmissible, because the probative value is substantially outweighed by the danger of unfair prejudice.
    (D) inadmissible as hearsay not within any recognized exception.
A
  1. (B) Under FRE 104 (a), “Preliminary questions concerning the qualification of a person to be a witness, the existence of a privilege, or the admissibility of evidence shall be determined by the court.” The judge is not bound by the rules of evidence at this point, except those with respect to privileges. To the extent that these inquiries are factual, the judge acts as a trier of fact. The judge’s decision is final and is not subject to a contrary determination by the jury. On the other hand, the jury is to determine how much, if any, probative value or “weight” to accord to the admitted evidence, as well as to decide issues of credibility (i.e., whether to believe, wholly or in part, the witness’s testimony). The judge not only decides factual issues, he also determines the appLicability of any technical evidentiary rules. He decides, for example, if a dying declarant had a sense of impending death; if an entry was made promptly in the regular course of business; if there was the necessary state of excitement to qualify a declaration as an excited utterance; if a witness is unavailable; or if an original document is unavailable so as to justify the admission of a copy under the best evidence rule. Lilly, Law of Evidence, 2d Ed., p. 459. Finally, note that preliminary questions of fact are ordinarily resolved in both criminal and civil trials by using a preponderance of the evidence standard. The witness’s testimony as to the victim’s statement will be admissible as a dying declaration if the judge, by a preponderance of the evidence, determines the victim believed her death was imminent. Choice (B) is correct. Choice (A) is incorrect because it misstates the role of the judge and jury in determining preliminary questions of admissibility under FRE 104. Choice (C) is also incorrect. There is not enough information in the fact pattern to balance the probative value and prejudicial impact of this evidence. Finally, choice (D) is wrong because the dying declaration is a recognized hearsay exception.
183
Q
  1. A defendant is on trial for attempted murder. The alleged victim is called by the prosecution to testify. During her testimony, the victim recounted the incident and described how the defendant savagely beat her with a baseball bat. She was not asked by the prosecution whether she made any statements during the attack.
    After the victim was excused and left the witness stand, the prosecution called another witness to testify. The witness proposes to testify that when the beating stopped, the victim screamed, “I’m dying. Don’t let the defendant get away with this.”
    Upon objection by the defendant’s attorney, the witness’s proffered testimony is
    a. admissible as an excited utterance.
    b. admissible as a dying declaration.
    c. inadmissible as hearsay not within any recognized exception.
    d. inadmissible, because the victim was not questioned about the statement before being excused.
A
  1. (A) Under FRE 803 (2), an excited utterance is defined as a statement relating to a startling event or condition made while the declarant was under the stress of excitement caused by the event or condition. The witness’s testimony of the victim’s statement, “I’m dying. Don’t let the defendant get away with this,” related to the savage beating (startling event) and was made when the beating stopped (while the victim was still under the stress of excitement). The testimony will be admissible substantively as an excited utterance. Choice (A) is correct. Clearly, the distractor answer was Choice (B). Under FRE 804 (b) (2), a dying declaration is a statement in a criminal homicide case or any civil proceeding made by the declarant while believing his death was imminent and concerning the cause or circumstances of what the declarant believed to be his impending death, provided the declarant is unavailable. First, the victim, the declarant, was not unavailable, since she testified at the trial. Second, the defendant is being charged with attempted murder, which is a non-homicide criminal case, so the federal rules do not allow for a dying declaration to be admissible in this situation. Choice (C) is wrong because the excited utterance is a recognized hearsay exception. Choice (D) is incorrect. There is no requirement that a hearsay declarant, even one who has already testified or will testify at trial, be questioned about their out-of-court statements.
184
Q
  1. A man was on trial for murder. Following a recess, the man was in the hallway outside the courtroom speaking with his attorney. During their conversation, the man said, “So what if I killed him, big deal.” The man’s statement was overheard by the presiding judge as he was walking down the corridor to the rest room. The judge then informed the prosecuting attorney of the man’s comment. After trial is reconvened, the prosecutor calls the judge as a witness to testify to what he heard the defendant tell his attorney.
    Upon objection by defendant’s attorney, may the judge be called as a witness?
    (A) No, because the man’s statement was a confidential communication and protected under the attorney-client privilege.
    (B) No, because a judge may not testify in a matter over which he or she is presiding.
    (C) Yes, because a judge has a duty to disclose incriminating evidence to the prosecution.
    (D) Yes, because the man’s statement was not a confidential communication.
A
  1. (B) According to FRE 605, a judge presiding at the trial may not testify in that trial as a witness. No objection need be made in order to preserve the point. Choices (A) and (D) are incorrect because the man made the statement in a public hallway area. If another individual (other than the judge) had overheard the statement, she would be permitted to testify. The statement would no longer be considered confidential. Choice (C) is wrong in this context because it has nothing to do with the evidentiary issue of whether a judge presiding over a trial can testify at that trial.
185
Q
  1. A shopper purchased a can of sardines, which had been produced by a fish company. A short while later at home, the shopper was preparing a sardine sandwich when she opened the can and, to her astonishment, she saw a decomposed mouse. The shopper became nauseated and vomited upon seeing the mouse.
    After consulting her attorney, the shopper initiated a strict liability action against the company. At trial, the shopper presented into evidence the sardine can with the company’s label, along with other corroborating evidence. The attorney representing the company objected to the introduction of the label on the can.
    The trial court should
    (A) overrule the objection, because the label is self-authenticating.
    (B) overrule the objection, because the label is the best evidence showing that the can was produced by the company.
    (C) sustain the objection, because there is no corroborating evidence that the company placed the label on the can.
    (D) sustain the objection, because the shopper’s proper cause of action is for infliction of emotional distress, not strict liability.
A
  1. (A) According to FRE 902(7), “inscriptions, signs, tags, or labels purporting to have been affixed in the course of business and indicating ownership, control or design” are self-authenticating. In this regard, McCormick states that a manufacturer’s certificate affords prima fade authenticity to the contents of the product. Evidence, pg. 700. Choice (B) is wrong because the best evidence rule applies only when the proponent is attempting to prove the contents or terms of a writing. Here, the plaintiff is not trying to prove the contents of a writing (such as the terms of a contract) but rather is introducing the label as prima fade evidence that the company produced the can of sardines. Choice (C) is incorrect; there is no requirement to corroborate self-authenticating evidence. Choice CD) is wrong because the fact pattern and question pertain to an evidentiary issue and not the appropriate cause of action for the case.
186
Q
  1. A bank was robbed by a person wearing a yellow ski mask, goggles, and a fluorescent yellow jumpsuit. Three weeks after the robbery, a defendant was arrested and charged with the crime. At trial, the prosecuting attorney seeks to introduce evidence that the defendant had previously committed a robbery wearing a yellow ski mask, goggles, and a fluorescent yellow jumpsuit.
    Upon objection by the defendant’s attorney, the proffered evidence should be
    (A) admissible as circumstantial evidence that the defendant committed the crime as charged.
    (B) admissible, to show the identity of the defendant.
    (C) inadmissible, because the probative value of the evidence is outweighed by the danger of unfair prejudice.
    (D) inadmissible, because it constitutes improper character evidence.
A
  1. (B) Under highly tested FRE 404(b), evidence of other crimes, wrongs, or acts is not admissible to prove the character of a person in order to show that he acted in conformity therewith. It may, however, be admissible for other purposes, such as proof of motive, opportunity, intent, preparation, plan, knowledge, identity, or absence of mistake or accident. This so-called MIMIC rule allows admission of other criminal acts as circumstantial character evidence to show conduct in conformity therewith, provided that the purpose for admission of such evidence is not solely to show criminal disposition or propensity to commit the crime being tried. Choice (A) is a good answer, but it is not as complete as choice (B). Without a direct identification of the defendant, the prosecution will necessarily have to present a circumstantial case. The indirect evidence of identity is circumstantial evidence. Choice (C) is incorrect. The fact pattern does not provide sufficient information to weigh the probative value and prejudicial effect of the evidence. Finally, choice (0) is wrong because the evidence is being offered for a non-character purpose.
187
Q
  1. After being passed over for a promotion, an aeronautic engineer became a whistleblower. He contacted a government agency to state that the company for which he worked was submitting false safety reports on newly built aircraft. When the company learned that the engineer was leaking this information to the government agency, he was fired from his job.
    Afterward, the engineer sued the company for wrongful termination of employment. During the discovery stage of litigation, the engineer was deposed by the company’s attorney. In his deposition, the engineer stated that the company submitted false safety reports to the government agency to cover up structural defects in its aircraft.
    A pilot was injured when one of the company’s
    airplanes he was piloting crashed. The pilot asserted
    a strict products liability tort against the company.
    At thal, the pilot seeks to introduce into evidence portions of the engineer’s deposition from his wrongful termination lawsuit against the company. Assume that the engineer is unavailable to testify at trial.
    Upon objection by the company, the trial court judge should rule the engineer’s deposition testimony
    (A) admissible as former testimony.
    (B) admissible as a vicarious admission.
    (C) inadmissible as hearsay not within any recognized exception.
    (D) inadmissible, because the company did not have the opportunity to cross-examine the engineer on the liability issue for which the statement is now being offered.
A
  1. (A) Usually, formertestimony questions will involve the same parties where each had the opportunity to interrogate the witness at the earlier trial. In this question, we have a plaintiff (who was not party to the first suit) seeking to introduce former testimony against the same defendant. According to Lilly, “using the testimony against the same defendant, which was a party to both suits and had an opportunity at the first trial to interrogate the witness, appears to fit comfortably within the exception.” Evidence, p. 286. On the other hand, Lilly points out that if a plaintiff seeks to use all or part of the former testimony against a new defendant (who did not have an opportunity to interrogate the witness), then the testimony should be excluded. For these reasons, choice (A) is the correct answer, and choice (D) is incorrect. Choice (B) is wrong. Under FRE 801(d)(2)(D), a vicarious admission by an employee can occur only if made duringthe employment relationship. At the time of this deposition, the engineer no longer worked for the company. Choice (C) is incorrect. While the evidence is hearsay, it is admissible under a recognized hearsay exception.
188
Q
  1. A plaintiff was driving her car when she stopped at a red light at an intersection. A defendant, who was behind her, did not see the red light and failed to stop. He crashed into the rear of the woman’s car and pushed her onto the curb and into a tree.
    When the plaintiff got out of her car, she felt fine physically, but was emotionally upset when she saw that the hood and trunk of her car were severely damaged. The next day she received an estimate from the mechanic for $1,950 to repair her car. She consulted an attorney the following afternoon and, that evening, suddenly developed shooting back pains.
    The plaintiff filed a lawsuit against the defendant based on a theory of negligence. One day prior to trial, two years after the accident, the plaintiff went to take a photograph of the accident scene.
    At trial, the photograph is
    (A) inadmissible, if the defendant objects, because the photograph is non-verbal hearsay.
    (B) inadmissible, if the defendant objects, because the photograph was taken two years after the accident.
    (C) admissible, if the plaintiff identifies the photograph at trial.
    (D) admissible, if the photograph correctly and accurately portrayed the accident scene.
A
  1. (D) As a general rule, a photograph is viewed merely as a graphic portrayal of oral testimony and becomes admissible only when a witness has testified that it is a correct and accurate representation of relevant facts personally observed by the witness. Moreover, the witness who lays the foundation need not be the photographer, but he needs to know about the facts represented or the scene photographed. Once this knowledge is revealed, the witness can testify as to whether the photograph correctly and accurately portrays these facts. Choice (A) is wrong because a photograph is not an “out-of-court statement.” Choice (B) is incorrect. The photograph would be admissible if it met the foundational elements, even over the defendant’s objection. Choice (C) is wrong because it suggests that admissibility depends only on the plaintiff’s identification of the photograph. In fact, she (or, for that matter, any other witness familiar with the location), must establish the foundational elements of “correct and accurate representation” of the location.
189
Q
  1. A man and a woman were involved in a car accident that occurred when the man abruptly switched lanes in front of the woman without signaling. Unable to stop, the woman hit the man’s car into a median. Immediately after the accident, as the two drivers emerged from their vehicles, a pedestrian, obviously emotional and upset from having just observed an accident, ran up to the man and shouted, “You careless driver, you. Didn’t you ever learn that you’re supposed to use a turn signal when you switch lanes?” The pedestrian is never identified and is not present at trial.
    The pedestrian’s statement is
    (A) admissible as non-hearsay.
    (B) admissible, even though it is hearsay.
    (C) admissible under the excited utterance exception.
    (D) inadmissible, because the bystander cannot be identified and is not present at trial.
A

6o. (C) Under FRE 803(2), a statement relating to a startling event or condition made while the declarant was under the stress of excitement caused by the event or condition is not excluded by the hearsay rule. The theory behind the “excited utterance” exception is simply that circumstances may produce a condition of excitement that temporarily stills the capacity of reflection and, thus, produces utterances free of conscious fabrication. Spontaneity is the key factor in determining whether the statement was, indeed, made without any conscious reflection. It is important to note here that choice (B) is not incorrect. Choice (A) is wrong. The statement would not qualify as an admission under FRE 801. Choice (D) is incorrect. If the proper foundational elements are met (as they are here), there is no requirement either to identify the declarant or produce him at trial.

190
Q
  1. An investor sued a corporation for stock fraud. In presenting his case-in-chief, the investor sought to introduce an issue of a newspaper to show the corporation’s stock price on that given day.
    Upon objection by the corporation’s attorney, this evidence should be
    (A) admitted, under the business records exception.
    (B) admitted, under the market reports exception.
    (C) excluded, because the newspaper copy does not fit within any established exception to the hearsay rule.
    (D) excluded, because it violates the best evidence rule.
A

6i. (B) This exception encompasses “newspaper market reports, telephone directories, city directories, and mortality and annuity tables used by life insurance companies.”
Federal Rules of Evidence 803 (17). The basis of trustworthiness is general reliance by the public, or by a particular segment of it, and the motivation of the compiler to foster reliance by being accurate. Choice (A) is not the best answer because business records contemplate the existence of “both an informer who supplies the information from personal knowledge and an entrant-recorder who makes the written entry in the regular course of the business activity.” This, obviously, is not the case with the publication of newspaper market prices. Choice (C) is wrong for the same reasons that choice (B) is the correct answer. Choice CD) is incorrect. The best evidence rule applies when the contents of a document (in other words, what the document actually said) are at issue, which does not appear to be the case here.

191
Q
  1. A teenager shot and killed a man she said tried to attack her as she was walking home from an all night market. The teenager, 18 years old and 6 feet 3 inches tall, said a very short man placed a knife against her throat and unzipped her dress as she was walking down the street. She pulled a gun from her purse and shot him. The man died of a chest wound. Police said the man, who was 5 feet 5 inches tall and weighed 130 pounds, was an ex-felon, twice convicted of attempted rape. The teenager is charged with the murder of the man, as a result of the above described incident.
    At trial, the teenager wishes to offer evidence of the fact that the man was previously convicted of attempted rape. The evidence would be
    (A) admissible, because it proves the man’s disposition to commit rape.
    (B) admissible, because the victim’s prior convictions would be relevant on the issue of the defendant’s self-defense.
    (C) inadmissible, because the evidence is being used to prove propensity.
    (D) inadmissible, because it violates the Dead Man’s Statute.
A
  1. (B) Past crimes may be introduced to show anything but disposition to commit a present crime or wrong; because no other legitimate reason, beyond disposition, appears to be raised by these facts, choice (C) is the correct answer. While there are special rules concerning sexual assault cases, this question does not involve a defendant being charged with a sexual assault crime (the defendant here is the teenager, not the man). Moreover, in a criminal case, where self-defense is at issue, a criminal defendant may offer evidence of the victim’s violent character to prove that the victim was the first aggressor; however, this may only be offered in the form of reputation or opinion evidence, not specific act evidence. Finally, the fact that the prior acts in question are convictions does not appear to impact this question, since while convictions may be specialized in terms of how they may be used to attack the credibility of a witness, this issue is not raised by the facts (the man is not a witness). For these reasons, choices (A) and (B) are incorrect. Choice (D) is incorrect, since the Dead Man’s Statute is not a rule of evidence and is inapplicable here.
192
Q
  1. A man was arrested and charged with sexually assaulting a child. At trial, in its case-in-chief, the prosecution seeks to introduce evidence that the man had twice been previously involved in similar cases with other children. The prosecution argues that these past crimes demonstrate the likelihood that the man molested this child.
    Defense counsel objects to the evidence regarding other children. The judge should find the evidence
    (A) admissible, because the man’s past crimes may be introduced to show that he is more likely to have committed the crime with which he is charged.
    (B) admissible, because a common plan or scheme may be proven by other similar criminal acts.
    (C) inadmissible, because past crimes may not be introduced to prove disposition to commit certain acts.
    (D) inadmissible, because the prosecution cannot initiate evidence of bad character.
A
  1. (A) According to FRE 413, in a criminal case in which the defendant is accused of an offense of sexual assault, evidence of the defendant’s commission of another offense or offenses of sexual assault is admissible and may be considered for its bearing on any matter to which it is relevant. For this same reason, choice (C) is the wrong answer. Choice (B) is a good answer, but not as strong as choice (A) in this case because FRE 413 permits the introduction of this evidence for any purpose. Choice (D) is incorrect. Under FRE 413, the prosecution can introduce this evidence on its case-in-chief.
193
Q
  1. A defendant has been charged with committing the armed robbery of a convenience store. During the defendant’s case-in-chief, the defendant called several character witnesses to testify as to his good character. As part of the prosecution’s rebuttal, the defendant’s ex-wife was called to testify that during their marriage the defendant had a violent temper, beat her frequently, and once threatened her with a knife.
    The ex-wife’s testimony should be ruled
    (A) admissible, because it is relevant to the crime charged.
    (B) admissible, because the defendant opened the door by introducing evidence of his good character.
    (C) inadmissible, because the defendant will be able to prevent the testimony using the spousal testimony privilege.
    (D) inadmissible, because character evidence may not be introduced in this manner.
A
  1. (D) Under the FRE, once the defendant in a criminal trial opens the door by introducing evidence as to their good character, the prosecution may so rebut. However, the prosecution is constrained in the same manner as the defendant in that the evidence of character must be introduced through reputation or opinion evidence only, no specific acts. Therefore, choice (A) is incorrect. While the ex-wife’s testimony may be relevant to the crime charged, it is in the form of specific acts and, therefore, inadmissible underthe FRE. Choice (B) is incorrect for the same reasons stated above. While it is true that the defendant properly opened the door, the prosecution is attempting to rebut with specific acts, which are inadmissible. Choice (C) is incorrect because (1) the spousal testimony privilege ends at divorce, and (2) where applicable, the spousal testimony privilege allows the witness-spouse to refuse to testify, but may not be used by the defendant-spouse to prevent the testimony. Choice CD) is, therefore, the correct answer and the evidence is inadmissible.
194
Q
  1. A famous comedian was acting as guest host of a popular late-night television talk show. Also appearing on the show was a professional football player. During the course of the television show, the comedian continually referred to the player as a “bench warmer.” Angered by the comedian’s insults, the player told him on the telecast, “Listen, I am a starter, not a bench warmer.”
    The football player sued the comedian for slander. At trial, the football player calls a witness who saw the show in question to describe what he had heard on the broadcast. The comedian’s attorney objects, claiming that the witness’s testimony would be hearsay.
    The trial judge should
    (A) sustain the objection, because the witness’s testimony is not the best evidence.
    (B) sustain the objection, because the witness does not have firsthand knowledge whether the comedian was reading from a script.
    (C) overrule the objection, because defamatory statements aren’t hearsay.
    (D) overrule the objection, because the witness is qualified to render a lay opinion as to what he heard.
A
  1. (C) The hearsay rule forbids evidence of out-of-court assertions to prove the facts asserted in them. Clearly, proof of utterances and writings may be made with an almost infinite variety of other purposes, not resting for their value upon the veracity of the out-of-court declarant and, hence, falling outside the hearsay classification. A few of the more common types of non-hearsay utterances include (1) verbal acts, (2) utterances and writings offered to show effect on hearer or reader, (3) declarations offered to show circumstantially the feelings or state of mind of the decLarant, and (4) knowLedge. Choice (C) is correct because (a) defamation, (b) slander, and (c) deceit are admissible as non-hearsay verbal acts. Choice (A) is incorrect because it misstates the best evidence rule, which applies only when the contents of documents, photographs, or recordings are in dispute. Choice (B) is irrelevant to the issue and, therefore, incorrect. Choice (D) is also wrong. The witness is not rendering an opinion, but rather testifying about something he personally heard and observed.
195
Q
  1. A college student and a man got into a fight at a bar while playing pooi. The college student allegedly started the fight when he struck the man in the head with a pool cue after the man sank a game-winning shot. At the college student’s trial for assault, the prosecution called one of the college student’s classmates to testify. The classmate was not at the bar during the fight. The classmate offered to testify that the next day at class, he approached the college student and said, “It was wrong of you to start a fight with the man over a game of pool,” at which point, the college student bowed his head and walked away.
    The classmate’s testimony is
    (A) admissible as part of the res gestae.
    (B) admissible as an admission.
    (C) inadmissible as hearsay not within any recognized exception.
    (D) inadmissible as self-serving.
A
  1. (B) If a statement is made by a person in the presence of a party to the action, containing assertions of facts which, if untrue, the party would, under all circumstances, naturally be expected to deny, his failure to speak has traditionally been receivable against him as an admission. See Wigmore, Evidence, Section 1071. Choice (A) is wrong because a conversation the day after an incident is not part of the incident’s res gestae. Choice (C) is also incorrect. By definition, an admission is not hearsay under FRE 801(d)(2). Finally, choice (D) is incorrect. The critical aspect of this fact pattern is the college student’s admission by failure to respond to the classmate, not the classmate’s motives for asking the question or testifying at trial.
196
Q
  1. While a ski lift was ascending the mountain, the overhead cable broke, dropping a skier 15 feet to the ground. The skier suffered a broken pelvis on account of the fall. Thereafter, the skier sued the ski resort alleging negligent maintenance of the ski lift machinery.
    The ski resort’s skier’s attorney took a discovery deposition of a witness who was seated behind the skier at the time of the accident. The witness died shortly after the deposition. At trial, the skier offers the witness’s deposition testimony into evidence.
    Upon objection by the ski resort’s attorney, the deposition is
    (A) admissible as former testimony.
    (B) admissible, provided that the witness’s statements are more probative on the point than any other evidence the skier can procure through reasonable efforts and that the skier has given the ski resort reasonable notice that she would offer the deposition.
    (C) inadmissible, because it violates the ski resort’s right to confront the witnesses against itself.
    (D) inadmissible, because it is hearsay not within any recognized exception.
A
  1. (A) Former testimony is defined under FRE 804(b) as “Testimony given as a witness at another hearing of the same or a different proceeding, or in a deposition taken in compliance with law. In the course of the same or another proceeding, if the party against whom the testimony is now offered, or in a civil action or proceeding, a predecessor in interest, had an opportunity and similar motive to develop the testimony by direct, cross, or redirect examination” . . . if the declarant is unavailable as a witness. Choice (A) is correct because the ski resort, the party against whom the deposition is being offered, deposed the deceased (i.e., unavailable) declarant, the witness. At that time, the ski resort had the opportunity to examine the witness, thereby satisfying the requirements for admissibility of former testimony. Choice (B) is incorrect because the former testimony exception does apply. Choice (C) is incorrect because a defendant’s right to confrontation allows an accused in a criminal case the right to effectively confront and cross-examine those who testify against him at trial. Choice (D) is incorrect because the statement is hearsay within a recognized exception.
197
Q
  1. A plaintiff files suit in federal district court against a lawnmower manufacturer, alleging strict liability, negligent manufacture, and breach of warranty. The suit is to recover damages for injuries. suffered by the plaintiff when his gasoline-powered lawnmower exploded, while the plaintiff was mowing his front lawn. At trial, the plaintiff calls a man who witnessed the explosion to testify as an eyewitness. The witness suffers from a mental illness that affects his perception of reality.
    Which of the following is true regarding the witness’s mental condition?
    (A) It disqualifies him from testifying
    (B) It maybe proved to impeach him as a witness
    (C) It has no bearing on either his competence or his credibility.
    (D) It must not be referred to because it is unfairly prejudicial.
A
  1. (B) The FRE specify no qualifications for mental competency. FRE 601 effectively eliminates as grounds for competency the following: age, religious belief, mental incapacity, conviction of a crime, marital relationship, and interest in the litigation as a party or attorney. Such matters, generally regarded at common law as grounds for disqualification, survive under the FRE as avenues for impeachment. In this question, the witness’s mental state does not disqualify him. Choice (A) is incorrect, but it does bear upon his competence and credibility. Choice (C) is incorrect, since the defendant can use the witness’s mental illness for purposes of impeachment. Choice (B) is correct. There are two basic competency requirements under the FRE:
    under FRE 602, a witness must possess personal knowledge; under FRE 603, every witness must declare to testify truthfully (i.e., be capable of understanding the obligation to tell the truth). Such a two-fold approach to competency is often referred to as a test of minimum credibility.
198
Q
  1. A testator died and through his will gave his entire estate to charity, but the gift failed because the will was executed 10 days prior to the testator’s death. A man then filed a claim against the estate, alleging that he was the son and heir of the testator. At the trial on the man’s claim, the man was unable to produce any proof of a ceremonial marriage, but established the following by undisputed evidence: (a) that his mother and the testator had cohabited for a number of years; and (b) that his mother had predeceased the testator by one day. Assume that this jurisdiction has a relevant Dead Man’s Statute in effect.
    At trial, the man’s attorney called a clerk at a winter resort hotel. The clerk testified that the testator and the man’s mother had come to the hotel for many years and he often saw the testator sign the hotel register as “Mr. and Mrs. Testator.”
    The trial court should rule the clerk’s testimony
    (A) admissible, because the clerk had personal knowledge of the matter.
    (B) admissible as a pedigree exception to the hearsay rule.
    (C) inadmissible, because the clerk’s testimony is not the best evidence.
    (D) inadmissible, under the Dead Man’s Statute.
A
  1. (A) The clerk’s testimony would be deemed admissible, since he had personal knowledge of the fact that the testator signed the hotel register as “Mr. and Mrs. Testator.” The clerk actually observed the testator signing the hotel register. Choice (B) is wrong because the pedigree exception covers community reputation of a person’s pedigree. In this case, the clerk is offering direct evidence, not reputation evidence, that the testator signed the hotel register in a particular way. Choice (C) is also wrong. The best evidence rule is applicable only where the terms or contents of a writing/document are in issue. Here, the terms of the hoteL registry are not in issue; the man is trying to prove that he is the testator’s son and heir to his estate. Moreover, in accordance with McCormick, Law of Evidence, Second Edition, Section 233, testimony descriptive of non-written transactions is not generaLly considered to be within the scope of the present rule and may be given without producing or explaining the absence of a writing recording the facts, such as evidence of a marriage, without production of the marriage certificate. Choice (D) is incorrect and somewhat misleading. Dead Man’s Statutes prohibit claimants to an estate from testifying about conversations they had with the decedent. In this case, the clerk is not a claimant to the estate.
199
Q
  1. A year ago, a very wealthy man died in a car accident. A woman, whose mother worked as a secretary to the wealthy man for years, filed a claim against the estate, alleging that she was the wealthy man’s daughter. At trial, the woman’s attorney called the wealthy man’s butler to testify. The butler testified that it was Common knowledge in the household that the wealthy man and the woman’s mother were having an affair. The butler also testified that the wealthy man had often confided in him and told him shortly after the woman’s birth that the woman was the wealthy man’s daughter.
    Upon objection by the attorney for the executor of the wealthy man’s estate, the court will most likely
    a. sustain the objection bc butler’s testimony would be violative of hearsay rule.
    b. sustain, bc butler’s testimony would be violative of Dead man’s statute.
    (C) overrule the objection because the butler’s testimony would be relevant to the ultimate issue in the case.
    (D) overrule objection because the butler’s testimony would qualify under the pedigree exception to the hearsay rule.
A
  1. (D) Under FRE 803 (19), the butler’s testimony would be admitted under the pedigree exception to the hearsay rule as “reputation among members of his family by blood, adoption, or marriage, or among his associates, or in the community, concerning a person’s birth, adoption, marriage, divorce, death, legitimacy, relationship by blood, adoption, or marriage, ancestry, or other similar fact of his personal or family history.” Choice (A) is wrong. Even though the evidence is hearsay, it satisfies a recognized hearsay exception. Choice (B) is incorrect because the butler is not a claimant to the estate and, therefore, is not prohibited by a Dead Man’s Statute from testifying about conversations with the decedent. Choice (C) is a good answer, but the wrong choice. The butler’s testimony is, indeed, relevant to the ultimate issue in the case. Choice (D) is a better choice than choice (C) because it specifically addresses the hearsay issues raised by the use of this evidence.
200
Q
  1. A nephew brings an action against the administrator of his uncle’s estate based upon a contract for services rendered in the management of his uncle’s property. In order to rebut the presumption that the services rendered were gratuitous, since the nephew was a relative of his uncle, the nephew called his sister-in-law as a witness. Assume that this jurisdiction has a relevant Dead Man’s Statute in effect.
    The sister-in-law testified that she had lived in the nephew’s house, knew his uncle, and that she was familiar with the uncle’s handwriting. The nephew’s attorney asked her to look at a letter and to tell the court whether it was written by the uncle.
    Upon objection by the attorney for the administrator of the estate, the trial judge would most likely
    (A) sustain the objection, because the sister-in-law is not a handwriting expert.
    (B) sustain the objection, because of the Dead Man’s Statute.
    (C) overrule the objection, because the letter qualifies as a past recollection recorded, an exception to the hearsay rule.
    (D) overrule the objection, because an authenticating witness need not be an expert if familiar with the handwriting of the person in question.
A
  1. (D) It is generally heLd that anyone familiar with the handwriting (or signature) of a given person may supply authenticating testimony in the form of his opinion that a writing or signature is in the handwriting of that person. In this regard, adequate familiarity may be present if the witness has seen the person write, or if he has seen writings purporting to be those of the person in question. FRE 901 (b) (2). Choice (A) is incorrect because a Lay witness can authenticate handwriting. Choice (B) is wrong because the sister-in-law is not a claimant to the estate. Choice (C) is incorrect. The past recollection recorded hearsay exception applies when (1) an in-court witness has forgotten something; (2) the attorney has unsuccessfully attempted to refresh the witness’s recollection; and (3) a writing is available containing matters “shown to have been made or adopted by the witness when the matter was fresh in the witness’s memory and to reflect that knowledge correctly.”
201
Q
  1. A tenant lived in a small apartment building that was owned by a landlord. In July, the landlord was killed in an accident. In August, the tenant brought an action against the administrator of the landlord’s state. The tenant alleged that he and the landlord had a contract under the terms of which the landlord was supposed to pay him $50 a week to mow the lawn and perform small maintenance repairs for the other tenants. He claimed that the landlord had not paid him for the past six months. He testified that there was a written contract and that the landlord kept the only copy of it in his home. At trial, he called the landlord’s accountant to testify about the contract. The accountant testified that she had seen the contract. She further testified about the terms of the contract, consistent with the tenant’s testimony. She testified that the contract, along with many other documents pertaining to the landlord’s business, had most likely been destroyed when the landlord’s business office was robbed a few months earlier.
    The trial judge should rule that the accountant’s. testimony is
    (A) admissible as secondary evidence under the circumstances.
    (B) admissible as a written admission by the party opponent.
    (C) inadmissible under the Dead Man’s Statute.
    (D) inadmissible, because the accountant’s testimony is not the best evidence.
A
  1. (A) Under FRE 1004, secondary evidence of the contents of a writing, recording, or photograph is admissible if the originals are lost or destroyed. Most courts hold that the exclusion of secondary evidence should be confined to cases where the writings are purposely destroyed to avoid producing them. Thus, choice (A) is correct and choice (D) is incorrect. Choice (B) is wrong because there is no written admission to enter into evidence. Choice (C) is incorrect because the accountant is not a claimant to the estate and, in any event, is not testifying about any conversations with the decedent.
202
Q
  1. A minor-plaintiff suffered permanent head injuries as a result of being hit by a car driven by a defendant. The plaintiff’s father commences this action as a parent in his own right and on behalf of his son.
    At trial, the plaintiff’s attorney called the only witness to the accident, the plaintiff’s friend, aged seven. The friend was four years of age at the time of the accident. It was determined that the friend lacked the capacity to perceive and relate the accident at the time it actually occurred.
    The court should rule that the friend would be
    (A) competent to testify, because he had personal knowledge of the accident.
    (B) competent to testify, because a seven-year-old is presumed to be chronologically mature.
    (C) incompetent, because he lacked the capacity to perceive and relate the accident at the time of its occurrence.
    (D) incompetent, because all children under the age of 10 are deemed incompetent to testify.
A
  1. (C) It is necessary to measure the competency of an infant witness at the time of the occurrence or event (about which he is called to testify), and not at the time of trial. There is no rule that excludes an infant of any specified age from testifying, but in each case, the traditional test is whether the witness has intelligence enough to make it worthwhile to hear him at all and whether he feels a duty to tell the truth. Choices (B) and (D) are wrong because they both incorrectly state presumptions that do not exist under the law. Choice (A) is incorrect. Although witnesses are presumed competent under FRE 601, personal knowledge alone does not make a witness competent to testify. Competency factors under the law include whether the witness can make an adequate oath or affirmation and communicate with a jury.
203
Q
  1. One day, while riding his motorcycle through a residential area, a man hit and injured a child who darted into his path. The child’s mother files suit against the man. At trial, the mother is then called to testify as to her conversation with the man at the hospital where her son was rushed immediately following the accident. According to the mother, the man told her, “Don’t worry, my insurance company will pay for all of your son’s medical expenses.”
    If offered into evidence, the court will most likely rule that the mother’s statement is
    (A) admissible, on the issue of the defendant’s negligence.
    (B) admissible, to show that the defendant is able to pay for the medical expenses.
    (C) inadmissible, because the testimony was highly prejudicial.
    (D) admissible, only to prove ownership or control of the motorcycle.
A
  1. (D) Under FRE 411, evidence that a person was or was not insured against liability is not admissible upon the issue of whether he acted negligently or otherwise wrongfully. The rule does not require the exclusion of evidence of insurance when offered for another purpose, such as proof of agency, ownership, or control of the vehicle. Therefore, choice (D) is correct, and choices (A) and (B) are incorrect. Choice (C) is also incorrect because it incorrectly states the standard for excluding evidence under FRE 403. Under Rule 403, evidence may be excluded if its probative value is substantially outweighed by the danger of unfair prejudice. Evidence is not excluded merely because it is “highly prejudicial.”
204
Q

skip

A

skip

  1. (D) Generally, a spouse (whether or not a party) has a privilege during the marital relationship and afterward to refuse to disclose and to prevent another from disclosing a communication if he claims the privilege, and that the communication was made in confidence between him and his spouse while they were husband and wife. However, the privilege is lost if the communication was made, in whole or in part, to enable or aid anyone to commit, or plan to commit, a crime or fraud. Thus, choice (D) is correct, and choice (A) is incorrect. Choice (B) is also wrong, since the marital privilege does not apply to fraud actions. Finally, choice (C) is incorrect because either spouse can claim the privilege.
205
Q
  1. A plantiff sued a defendant for injuries suffered m a fall on the sidewalk of the defendant’s home. The plaintiff’s complaint alleged that the walk was covered by a thick sheet of ice, which had been negligently left there for several days by the defendant. In his answer, the defendant set forth that the ice formed overnight and that the plaintiff fell before the defendant had a chance to remove it. During the trial, a physician, whose office was next door to the defendant’s home, testified that he saw the plaintiff fall and that the ice, which had been there for several days was at least two inches thick. On cross-examination, counsel for the defendant asked the physician the following question: “During your treatment of the plaintiff on the day in question, is it not true that he told you his fall resulted from a loss of equilibrium after he suffered from dizziness?”
    Upon objection by the plaintiff’s attorney, the physician’s testimony will be ruled
    (A) admissible, because it is reasonably pertinent to diagnosis or treatment.
    (B) admissible, because the physician had personal knowledge of the plaintiff’s injuries.
    (C) inadmissible, because the defendant’s counsel failed to lay a proper foundation.
    (D) inadmissible, because the physician’s testimony is not relevant to prove that the plaintiff’s alleged injuries are false or exaggerated.
A
  1. (A) The FRE allow not only statements of past symptoms and medical history as exceptions under the hearsay rule, but also the cause or source of a patient’s past physical condition, insofar as reasonably pertinent to diagnosis or treatment. In this regard, the plaintiff’s statements to the physician would be admissible as reasonably pertinent to diagnosis and/or treatment regarding the cause or source of his injuries. Choice (B) is not the best answer here. While the physician might have personal knowledge of the injuries the plaintiff suffered from the fall, he is also privy to knowledge about past symptoms and medical history. Choice (C) is incorrect. The question occurred on cross-examination and required no foundation. Choice (D) is wrong because the physician’s evidence is highly relevant in this case.
206
Q

skip

A

skip

  1. (C) In accordance with FRE 503 (b), the attorney-client privilege provides that a client has a privilege to refuse to disclose and to prevent any other person from disclosing confidential communications made for the purpose of facilitating the rendition of professional legal services to the client, between himself or his representative and his attorney or his attorney’s representative. Choice (A) is incorrect. Whatever the man said to his attorney might have been an admission, but it was covered by the privilege. Choice (B) is also wrong. The attorney-client privilege survives the attorney-client relationship. Finally, choice (D) is an incorrect statement. Like most evidence precluded by privilege, this statement is highly relevant.
207
Q
  1. A shopper slipped and fell on a wet spot at the local mall, suffering injuries. The shopper has sued the mall owner for negligence. At trial, the shopper’s housekeeper testified that the shopper was bedridden and unable to work for a month because of the sprained back she suffered when she fell at the mall. The housekeeper also testified that about one week after the shopper returned home from the hospital, the shopper told her, “My back is really killing me. The pain is excruciating. It’ll be a miracle if I can ever stand up again.” The mall’s counsel then moved to strike the housekeeper’s testimony.
    The motion should be
    (A) granted, because the housekeeper’s testimony is hearsay not within any recognized exception.
    (B) granted, because the housekeeper’s testimony would be self-serving.
    (C) denied, because the housekeeper’s testimony would be admissible as an adoptive admission.
    (D) denied, because the shopper’s statement to her housekeeper would be admissible as a declaration of present bodily condition.
A
  1. (D) Choice (D) is correct, since the shopper’s statement, “My back is really killing me,” is admissible as a declaration of present bodily condition. Under FRE 803(3), such statements need not have been made to a physician. Thus statements to hospital attendants, ambulance drivers, or even members of the family might be included. Choice (A) is wrong. Although the statement is hearsay, it falls within a recognized exception. Choice (B) is incorrect. The self-serving nature of hearsay testimony does not affect its admissibility. Choice (C) is also incorrect. A statement of present bodily condition is not an admission.
208
Q
  1. A defendant was arrested and charged with forgery for unlawfully signing her ex-husband’s signature on his pay check. Before trial, the defendant entered into plea bargain negotiations with the prosecution. During the course of these negotiations, the defendant sent a letter to the prosecuting attorney in which she stated: “I don’t want to go to jail, so if you recommend a suspended sentence, I’ll plead guilty and admit that I forged my ex-husband’s signature on the check.” Subsequently, the plea bargain negotiations fell through.
    Shortly thereafter, the defendant was arrested and charged with forging her ex-husband’s signature on his next pay check. She has denied culpability regarding the second forgery charge. At trial for the alleged second forgery, the prosecution seeks to introduce the letter the defendant sent to the prosecution during her earlier plea bargain negotiations.
    The defendant’s attorney objects to the admissibility of the letter. The trial court judge should rule the letter is
    (A) admissible as evidence of the defendant’s propensity for committing the crime.
    (B) admissible as an admission.
    (C) inadmissible, because it was written during the course of plea bargain negotiations.
    (D) inadmissible, because specific instances of misconduct are not admissible in a criminal case.
A
  1. (C) FRE 410 provides that evidence of the following is not, in any civil or criminal proceeding, admissible against the defendant who made the plea: “…(4) any statement made in the course of plea discussions with an attorney for the prosecuting authority which does not result in a plea of guilty or which results in a plea of guilty later withdrawn.” The thrust of the rule is to promote the plea bargaining process by freeing the accused from the apprehension that his declarations may be used to his detriment in the pending criminal trial or in subsequent litigation. In this question, the defendant’s letter to the prosecution admitting to the forgery of the first check was written during the course of plea bargaining negotiations and, therefore, may not be used by the prosecution as an admission in the subsequent trial for alleged forgery of the next pay check. Choice (C) is the correct answer, and choice (B) is incorrect for the same reasons; plea agreements and negotiations often contain admissions. Choice (D) is incorrect because character evidence in the form of specific acts in a criminal case may be offered by the prosecution under the so-called MIMIC rule, but two similar instances of conduct do not establish a common plan or scheme. Choice (A) is incorrect because FRE 404 prohibits the use of propensity evidence in criminal cases.
209
Q
  1. A plaintiff sued an insurance company to recover life insurance proceeds. At trial, the issue before the court is the insured’s correct date of birth.
    Which of the following is inadmissible if offered to prove the insured’s correct date of birth?
    (A) A family portrait engraved with the ages of the family members.
    (B) A family bible with the insured’s birth date inscribed on the cover.
    (C) A photocopy of the insured’s birth certificate.
    (D) A sworn affidavit from the insured’s brother verifying the insured’s birth date.
A
  1. (D) One of the oldest exceptions to the hearsay rule encompasses statements concerning family history, such as the date and place of births and deaths of members of the family and facts about marriage, descent, and relationship. Under FRE 803 (13), “statements of fact concerning personal or family history contained in family Bibles, genealogies, charts, engravings on rings, inscriptions on family portraits, engravings on urns, crypts, or tombstones” are admitted. Therefore, choices (A) and (B) are wrong. Choice (C) is incorrect because the photocopy of the insured’s birth certificate would be admissible under FRE 1003, since a duplicate is admissible to the same extent as the original. Choice (D) is the best choice because the affidavit is inadmissible hearsay, since it is an out-of-court statement that is being offered for its truth.
210
Q
  1. After two men robbed a local bank, they left the bank and went in different directions with a plan to meet three days later at the train station. Two days after the robbeiy, one of the men was involved in an auto accident. The man was rushed to the hospital and taken to the x-ray room. A doctor positioned the man so that the x-ray technician would be able to take x-rays of his lower back area. Based on the man’s complaints about the area of pain, the doctor stated, “It seems as if you might have a fractured pelvis. If you do, you’ll have to stay in bed for several weeks.” The man responded that he couldn’t do that because he had to meet the other man tomorrow. A technician who was in the room at the time overheard the man’s response to the doctor. Subsequently, both men are charged with robbery and conspiracy.
    At the man’s trial, as evidence of a conspiracy, the prosecution attempts to introduce the doctor’s testimony that the man said, “I have to meet the other man tomorrow.” The defense objects.
    This evidence is
    (A) inadmissible, because it violates the physician- patient privilege.
    (B) admissible, because the technician overheard the conversation.
    (C) admissible, because of the nature of the man’s statements.
    (D) admissible, because the statement was not confidential, under the circumstances.
A
  1. (C) The physician-patient privilege is not available to the man under the facts presented. In order fora statement to be deemed admissible under the physician-patient privilege, the information obtained from or the communication from the patient must be necessary to enable the physician to prescribe or act for the patient. Since the man’s statement to the doctor does not relate to treatment, it is not protected by the privilege. Thus, choice (A) is incorrect, and choice (C) provides the best answer. Choice (B) is an incorrect rule of law, since the privilege is not destroyed if it is overheard by an x-ray technician. Choice (D) is also wrong because the communication here remains confidential, even though it is not privileged.
211
Q
  1. A man and a woman worked for an armored car company, picking up and delivering cash from banks and businesses. They figured out a plan to fake a holdup and robbery of the man’s armored car. After faking the holdup, they planned to purchase a boat with the proceeds and travel to a remote island to deposit the money in a bank account there. The police got wind of the plan before it could be carried out, and the man and the woman were charged with conspiracy. At trial, the prosecution wants to introduce a letter from the man to the woman before the robbery in which the man wrote:
    “Dear woman,
    I hope our plan for the heist works. If it does, we’ll have a giant pile of bills to divide between us. (signed) the man”
    Before the contents of the letter can be admitted into evidence, the prosecution must
    (A) have an expert witness identify the man’s signature as genuine.
    (B) bring the original letter into court.
    (C) demonstrate the relevance of the letter.
    (D) present evidence of the authenticity of the letter.
A
  1. (D) In accordance with FRE 901, choice (D) is the best answer, since the letter must be properly authenticated by the proponent before it will be admitted into evidence. FRE 901 (a) states that “the requirement of authentication or identification as a condition precedent to admissibility is satisfied by evidence sufficient to support a finding that the matter in question is what its proponent claims.” A letter must be authenticated, but not necessarily by an expert under the FRE. Thus, choice (A) is incorrect. Choice (B) is incorrect because the contents of the letter are not in issue. Choice (C) is a good answer; all evidence must satisfy the relevance requirements of FRE 401 to be admitted at trial. Choice (D) is a better answer, however, because even if the letter is relevant, it won’t come into evidence unless it is authenticated.
212
Q
  1. A witness testified under a state grant of immunity about statewide gambling activities being investigated by a state grand jury. Five months later, the witness was subpoenaed by a federal grand jury investigating gambling related activities.
    Which of the following statements is correct with respect to the witness’s rights before the federal grand jury?
    (A) The witness’s grant of immunity by the state would extend to all subsequent investigations.
    (B) Federal authorities have the burden of showing that they have an independent source for their evidence against the witness.
    (C) The witness’s Fifth Amendment protection against double jeopardy would extend to federal prosecution.
    (D) In order for the witness’s grant of immunity to apply in the federal investigation, it must be broader than the protection afforded by his Fifth Amendment privilege against selfincrimination.
A
  1. (B) In Murphy v. Waterfront Commission, 378 U.S. 52 (1964), once a defendant demonstrates that he has testified, under a state grant of immunity, to matters related to the federal prosecution, the federal authorities have the burden of showing that their evidence is not “tainted,” by establishing that they had an independent, legitimate source for the disputed evidence. Choice (A) is incorrect because the state does not have the authority to bind the federal government with a grant of use immunity, and choice (0) is incorrect for similar reasons. Choice (C) is incorrect because jeopardy is not triggered until a jury is impaneled and sworn or, in the case of a bench trial, the first witness is sworn.
213
Q
  1. During a federal grand jury investigation into money laundering, a man’s business and tax records were subpoenaed by the government. The man invokes his Fifth Amendment privilege against self- incrimination to prevent production of the records in his possession.
    The court would most likely
    (A) compel production of the records, because the Fifth Amendment is inapplicable.
    (B) order a preliminary hearing to determine if the man has grounds to invoke the Fifth Amendment.
    (C) permit the man to exercise his Fifth Amendment privilege, since such records may have the same incriminating effect as testimony.
    (D) admit the records under the business records exception to the hearsay rule.
A
  1. (A) In Couch v. United States, 409 U.S. 322 (1973), the U.S. Supreme Court ruled that a taxpayer may not invoke his Fifth Amendment privilege against compulsory self- incrimination to prevent the production of his business and tax records. The Court noted that “it is important to reiterate that the Fifth Amendment privilege is a personal privilege: it adheres basically to the person, not to information which may incriminate him.” Choice (B) is incorrect because the rule stated in Couch is clearly applicable to the man’s case. Choice (C) is incorrect. Business and tax records are not considered testimonial for the purposes of Fifth Amendment protection. Choice (0) is also incorrect. The FRE do not apply to grand jury proceedings.
214
Q
  1. A driver had a new car and, as he was carefully driving within the posted speed limit, a girl, aged 11, suddenly darted into the street in front of his car. Although\he driver immediately applied his brakes and swerved to avoid the girl, the cat hit the girl, fracturing her legs. As the girl was lying in the street awaiting an ambulance, the driver rushed over to her and said: “Im terribly sorry. This is the first time I ever drove this car. I don’t know what happened, but it must have been my fault. Send me all your hospital bills. I’fl pay for everything.”
    When the girl was later released from the hospital, her parents contacted the driver who refused to pay anything. The driver told the girl’s mother, “Since your daughter ran into the street, it was her fault. I have witnesses who saw what she did. If I weren’t such a nice guy, I’d sue you for the damage to my new car.”
    If the girl’s parents, on her behalf, sue the driver in tort, which of the following is the most accurate statement regarding the driver’s post-accident statements?
    (A) The driver’s statement regarding his operation of the car, as well as his statement concerning payment of the hospital bills, are both admissible on the issue of negligence.
    (B) The driver’s statement regarding payment of the hospital bills is admissible, but his statement concerning his operation of the car is not admissible.
    (C) The driver’s statement regarding his operation of the car is admissible on the issue of negligence, but his statement concerning payment of the hospital bills is not admissible in accordance with public policy considerations.
    (D) Neither the driver’s statement regarding his operation of the car nor his statement concerning payment of the hospital bills is admissible on the issue of negligence.
A
  1. (C) The driver’s statement regarding his operation of the car is admissible, but his statement concerning payment of the hospital bills is not admissible. FRE 409 excludes evidence of offers or promises to pay medical, hospital, or similar expenses occasioned by injury to prove liability for the injury. However, if the reference to the medical expenses is severable from the defendant’s admission of liability, the courts will permit such severance. Thus, choice (C) is correct, and choices (A) and (D) are incorrect. Choice (B) is wrong because it states the rule on severance exactly wrong.
215
Q
  1. Late one evening a man was driving home from his job as a security guard at a jewefry store. As the man approached a railroad crossing he noticed that the barriers to the crossing were open. The man cautiously looked both ways for any train. Assuming that no train was coming, he proceeded across the railroad tracks. Suddenly, an oncoming train struck the man’s car, completely demolishing it. The man was dead on arrival at the local hospital. Unknown to the man, the railroad crossing guard, in a drunken stupor, had fallen asleep at the barrier controls. He failed to see or hear the signals from the oncoming train that struck and killed the man. The railroad company was well aware of the crossing guard’s alcoholism, but continued to employ him. However, two weeks after the accident occurred, the railroad company fired the crossing guard.
    The executor of the man’s estate institutes a wrongful death action against the railroad company. During the trial, the attorney for the man’s estate offered into proof the fact that the defendant railroad company dismissed the crossing guard shortly after the accident.
    Upon objection by the railroad company’s attorney, the court should
    (A) overrule the objection, as the employee’s dismissal was an admission that the railroad company was aware of the crossing guard’s chronic alcoholism.
    (B) overrule the objection, as an adoptive admission by conduct.
    (C) sustain the objection, because the probative value of the evidence would be outweighed by its prejudicial effect.
    (D) sustain the objection, as evidence of subsequent remedial repairs is inadmissible.
A
  1. (D) FRE 407 states that “When, after an event, measures are taken which, if taken previously, would have made the event less likely to occur, evidence of the subsequent measures is not admissible to prove negligence or culpable conduct in connection with the event.” The evidence that the defendant railroad company dismissed the crossing guard would thus be inadmissible to prove that they were admitting negligence in connection with the accident that resulted in the man’s death. The exclusion of evidence of subsequent remedial repairs under the FRE rests specifically today on a social policy of encouraging people to take, or at least not discouraging them from taking, steps in furtherance of added safety and repairs. For these reasons, choice (D) is correct, and choices (A) and (B) are incorrect. Choice (C) is wrong for two reasons: (1) it misstates the FRE 403 standard for excluding evidence; and (2) there is no information in the fact pattern that would permit weighing the probative value and prejudicial effect of the evidence.
216
Q
  1. A plaintiff was seriously injured in a collision involving three trucks. A sideswipe occurred between the plaintiff’s pickup truck and a trailer truck as they passed in opposite directions. The sideswipe caused the plaintiff’s truck to careen down the road and into the path of a second trailer truck. The driver of the second trailer truck was immediately killed. Both of the trailer trucks were owned by the same transport company.
    At trial, the plaintiff called a witness to the collision to testify that he heard an unidentified female witness scream, “Oh my God, that trailer truck sideswiped that little pickup truck.”
    The trial judge should rule the witness’s testimony
    (A) admissible as a declaration of existing state of mind.
    (B) admissible as an excited utterance.
    (C) inadmissible, because it contains inadmissible opinion evidence.
    (D) inadmissible as hearsay not within any recognized exception.
A
  1. (B) Under FRE 803 (2), the statement of the unidentified bystander, made in the presence of the witness, should qualify as an “excited utterance” or “spontaneous exclamation” within the meaning of that exception to the hearsay rule. The theory of this exception is simply that there has been some occurrence startling enough that temporarily stills the capacity of reflection and produces utterances free of conscious fabrication. Choice (A) is wrong because the statement describes an observation made by the declarant and not the declarant’s then-existing state of mind. Choice (D) is incorrect. The statement is hearsay but, as discussed above, meets a recognized exception. Choice (C) is incorrect. The statement expresses a factual observation and not opinion testimony.
217
Q
  1. While driving her company vehicle near a pedestrian mall, a woman came upon the scene of a three-car accident. She was so busy gawking at the damaged vehicles that she failed to see one of the victims lying on the road in front of her car. She hit and ran over the victim, who survived and sued the woman’s company. The victim offers the testimony of a witness to the incident. Referring to the woman, the witness stated, “The driver of that car ran over the victim as he was lying on the ground awaiting an ambulance, and said ‘It is all my fault, I should have been paying more attention to my driving.” Assume for this question that the woman is available to testify.
    The trial judge should rule that the testimony is
    (A) admissible as a declaration against interest.
    (B) admissible as a present sense impression.
    (C) admissible as an admission.
    (D) inadmissible as hearsay not within any recognized exception.
A
  1. (C) A statement will not be barred from admission by the general rule against hearsay where the statement is offered against an opposing party and was made by the party’s agent or employee on a matter within the scope of that relationship and while it existed. Indeed, there is a substantiaL trend favoring the admission of statements relating to matters within the scope of the agency or employment. Therefore, choice (C) is correct, because the statement will be admitted as a vicarious admission. Choice (A) is wrong because a declaration against interest can only be admitted if the declarant is unavailable at trial. Choice (B) is incorrect. The woman is admitting liability and negligence, not “describing an event or explaining an event or condition” within the meaning of FRE 803(1). Finally, choice (D) is wrong because the statement is hearsay that falls within a recognized exception.
218
Q
  1. A husband and wife had been married for 10 years. The wife obtained a divorce from the husband after learnitg that he was engaging in an extramarital love affair with one of his co-workers. Following the divorce, the husband was arrested and charged with embezzling funds from his employer during a two- year period. In preparing for trial, the prosecuting attorney interviewed the wife, who stated that during their marriage, the husband admitted committing the embezzlement. This jurisdiction has adopted both the common-law marital and spousal privileges.
    At the husband’s trial, the wife is called to testify regarding statements the husband made to her during the marriage regarding commission of the crime.
    Upon objection by the husband’s attorney, the wife’s testimony will be
    (A) admissible, under both the marital and spousal privileges.
    (B) admissible, under the spousal privilege but not admitted under the marital privilege.
    (C) admissible, under the marital privilege but not admitted under the spousal privilege.
    (D) inadmissible, under either the marital or spousal privileges.
A
  1. (B) On the MBE, students are frequently tested on the distinction between the spousal and marital privileges. At common law, the spousal privilege (or incapacity) prevented one spouse from testifying against the other in a criminaL case. According to Lilly, “the trend has been to abolish the privilege entirely, or to make the witness the holder, leaving the accused without the power to object to his spouse’s voluntary adverse testimony.” Lilly further notes that “a majority of states now follow the ruLing in Trammel.” Evidence, p. 391. Therefore, underthe marital privilege, the wife may be prevented from testifying but not under the spousal privilege. ft is important to note that the marital privilege survives the termination of marriage either by death or divorce. For these reasons, choices (A) and (C) are incorrect. Choice (D) is incorrect because the evidence would be admissible under the spousal privilege.
219
Q
  1. A defendant was arrested and charged with arson, which in this jurisdiction is defined as the “malicious or intentional burning of any structure or dwelling.” When the defendant was arrested, he was inside a warehouse that was engulfed in flames. At the time he was apprehended, the police found in his possession a book of matches with four matches missing.
    At trial, the prosecution introduced the book of matches into evidence. Following closing arguments, and before the jury retired for deliberations, the judge gave instructions concerning the law of the case. The judge instructed the jury that it could infer the defendant’s intent to set the fire based on the fact that four matches were missing from the matchbook. This instruction was specifically objected to by the defense counsel.
    The judge’s jury instruction concerning the defendant’s intent was
    (A) proper, because it constituted a rebuttable presumption of fact.
    (B) proper, because it constituted a permissible inference.
    (C) improper, because the prosecution must prove each element of the crime beyond a reasonable doubt.
    (D) improper, because the instruction shifts the burden of persuasion from the prosecution to the defendant defense.
A
  1. (B) Here is another example of why the Evidence questions are among the trickiest on the exam. By now, you can probably see that the Multistate exam is not testing rote outline memorization but, rather, it homes in on subtle nuances and fine-line distinctions. In this regard, you need to differentiate between inferences and presumptions. According to LiLly, “when a genuine presumption is accepted as true by the trier of fact, it gives rise to a mandatory inference, which is properly called a presumed fact.” Evidence, pg. 56. Once the basic facts are believed, the resuLting presumed fact must be accepted by the trier of fact unless it is rebutted by contravening evidence. Consequently, in criminal cases, presumptions are disfavored and subject to close constitutional scrutiny because they tend to shift the burden of producing evidence to the accused. On the other hand, an inference never has such a compulsory effect. The trier of fact always is at liberty either to accept or reject an inferred fact. In this regard, Lilly further states that “a presumption creates a compulsory finding that remains obligatory untiL the presumed fact is rebutted.” However, this is not true of an inference, which results only in creating a jury question as to whether the inferred fact exists. As a result, choice (B) trumps choice (A) because an inference is favored over a presumption in criminal cases. Choices (C) and (D) are wrong because an inference does not shift the burden of proof away from the prosecution.
220
Q
  1. A plaintiff was suffering from a respiratory infection and his doctor prescribed for him a drug that was manufactured by a pharmaceutical company. With every prescription of this drug distributed, the company inserted a written warning, which stated:
    “Use of this drug may cause dizziness.”
    In accordance with the prescription, the plaintiff received a four-day supply of the drug from his neighborhood pharmacy, which passed along the warning to the plaintiff. After two days, the plaintiff felt much better and went back to work at a home improvement store, where he was the manager. The plaintiff continued to take the drug until his supply was exhausted. On the fourth day, the plaintiff took his final capsule at 2:00 p.m. While he was driving his car home at 5:45 that afternoon, he became dizzy and crashed into a tree. As a result, he received multiple injuries.
    The plaintiff decided to consult an attorney, who filed a lawsuit on the plaintiff’s behalf against the pharmaceutical company, alleging that the company was responsible for the accident and resulting injuries on the theories of negligence and strict liability.
    At trial, the plaintiff’s attorney attempts to introduce evidence that subsequent to this accident, the company expanded its warning to read: “Use of this drug may cause dizziness. Do not drive for five hours after use.”
    Such evidence is
    (A) admissible as an indication that the company did not use due care in wording its original warning.
    (B) admissible as a declaration against interest, to prove the possibility of alternate warnings.
    (C) inadmissible to show that the company was negligent in wording its original warning.
    (D) inadmissible, because the subsequent warning is not relevant to the feasibility of a modified warning.
A
  1. (C) The evidence of the company’s expanded warning for use of the drug is inadmissible to show that the company was negligent in wording the original warning. Evidence of the company’s revised warning creates the obvious inference that the manufacturer was negligent in failing to use the subsequent warning earlier. This type of subsequent measures (revising original warning), which is being introduced to prove negligence, is inadmissible under the FRE. FRE 407 states, “when, after an event, measures are taken which, if taken previously, wouLd have made the event less likely to occur, evidence of the subsequent measures is notadmissible to prove negligence or culpable conduct in connection with the event.” This rule does not require the exclusion of evidence of subsequent measures when offered for another purpose, such as (1) proving ownership; (2) control; or (3) feasibiLity of precautionary measures, if controverted by the defendant. Choice (A) is the wrong answer because of the policy concerns implicated by FRE 407 to encourage remedia’ measures. For similar reasons, choice (B) is also incorrect. Choice (D) is a correct outcome, but for the wrong reason. A subsequent warning would be relevant to show the feasibility of modifying the original warning. However, the policy concerns of FRE 407 trump the relevance of this evidence.
221
Q
  1. In an automobile accident case, the defendant claimed that the plaintiff was contributorily negligent by driving at an excessive speed. The defendant’s attorney calls its first witness, who was standing along the side of the road when the plaintiff’s and defendant’s cars collided. The defendant’s attorney asks the witness: “Is it a fact that in your estimation, plaintiff was traveling at a speed of 45 m.p.h. in a 35 m.p.h. speed zone at the time of the accident?” The plaintiff objects.
    The trial judge should
    (A) overrule the objection, because the judge may take judicial notice of the speed limit within the jurisdiction.
    (B) overrule the objection if the witness has already been shown to be an experienced driver who was able to estimate the approximate speed of moving vehicles.
    (C) sustain the objection if the witness has not been qualified as an expert witness.
    (D) sustain the objection, because the witness is not an adverse witness.
A
  1. (D) Choice (D) is the best answer, since leading questions in accordance with FRE 611 are generally inadmissible on the direct examination of a witness. This question requires a two-step analysis. First, you must determine if the objection should be sustained or overruled. Since the question posed by defendant’s attorney on the direct examination of the witness is leading, the trial judge should, thus, sustain plaintiff’s objection. Second, you must determine the correct reasoning for the judge’s sustaining of the objection. Choice (D) is correct, since one of the exceptions in which leading questions may be asked on direct examination is where the witness is adverse or hostile. Choice (C) is incorrect because one need not be an expert witness to testify to speed and, furthermore, the question is still objectionable as leading. Therefore, choice (D) states the correct ruling by the trial judge, as well as the best legal reasoning of the two “sustain-the-objection” alternatives. There are several exceptions to the general rule that leading questions are not permissible on the direct examination of a witness. The exceptions include the use of leading questions (a) to bring out preliminary matters, such as the name and occupation of a witness; (b) to elicit matters not substantially in dispute; (c) to question a hostile witness or an adverse party; and (d) to aid a witness in testifying because of the witness’s loss of memory, immaturity, or mental weakness. Choice (B) is incorrect; lay witnesses can testify about such things as speed estimates without having to be qualified as experienced drivers. Finally, choice (A) is wrong because the issue isn’t the speed limit itself, but how fast the plaintiff was driving.
222
Q
  1. George was charged with the murder of a well- known criminal. The victim was found shot to death in his car. The only witness to the shooting was the victim’s pet parrot, which was found hovering over the victim’s body repeating over and over, “George don’t. . . George don’t.”
    To prove the identity of the victim’s killer, the prosecution has a witness testify that about the time of the murder, he was in the area and heard the victim’s parrot calling George’s name.
    The best objection to the testimony is that it is
    (A) hearsay.
    (B) irrelevant.
    (C) opinion testimony.
    (D) self-serving.
A
  1. (B) Choice (A) is incorrect because the FRE require that the hearsay rule apply to the out-of-court statements or declarations of individuals. Since the statement in the present question was made by a parrot, the best objection would be that the testimony is irrelevant, not hearsay. Choice (C) is incorrect because the rules on opinion testimony refer to human witnesses, not animals. Choice (D) is also wrong; it is difficult to conceive of an animal testifying at trial, let alone giving self-serving
    testimony.
223
Q
  1. As part of his defense to a murder charge, a defendant offered testimony that he was committing a bank robbery in another state on the day that the victim was killed.
    The testimony is
    (A) admissible as not hearsay.
    (B) admissible as an admission.
    (C) admissible as a declaration against interest.
    (D) admissible as then-existing state of mind.
A
  1. (A) Under FRE 801(d)(2)(A), an admission is a statement offered against a party by a party opponent. Here, the statement by the defendant is not an admission because the defendant was simply testifying to his actions on the day the victim was killed. The question is tricky because the defendant is, of course, testifying that he was committing another offense. This fact could cause a test-taker to forget that under the rules, admissions must be offered by a party opponent. Because the defendant is testifying to his location and what he was doing, the statement is not hearsay. Since the statement is not hearsay, choices (B), (C), and (D) are not correct.
224
Q
  1. A man is charged with murder. During the trial, defense counsel offers the testimony of a man on death row admitting he hired another man to shoot the victim.
    The testimony is
    (A) not hearsay.
    (B) hearsay, but admissible as an admission.
    (C) hearsay, but admissible as a dying declaration.
    (D) hearsay not within any recognized exception.
A
  1. (A) Since the man is testifying, the confession would not be hearsay in that hearsay is an out-of-court statement. As a consequence, choices (B), (C), and (D) are incorrect. Furthermore, this is not an admission of the defendant or a dying declaration, in that the statement does not deal with the cause or circumstances of the declarant’s death (and there is no fear of death being impending).
225
Q
  1. A defendant was charged with murder. The defendant was a butcher by trade, and it was crucial to the prosecution’s case against the defendant to prove that the victim was stabbed to death with a butcher’s knife. The prosecution called a police officer to the stand. The assistant district attomey marked a colored photograph of the dead murder victim for identification and was ready to question the police officer. Before the prosecutor proceeded further, the defense counsel objected to any use of the victim’s photo, which pictured 14 stab wounds to his body.
    The photo is
    (A) admissible, for the limited purpose of showing that the stab wounds resulted from a butcher’s knife.
    (B) admissible, because it is relevant to the prosecution’s case, and all relevant evidence is, by its nature, admissible.
    (C) admissible, because the photo would neither mislead the jury nor waste time.
    (D) inadmissible.
A
  1. (D) Choice (D) is the best answer when compared to the alternatives. In accordance with FRE 403, the danger of unfair prejudice to the defendant would outweigh the relevance of the photo depicting the victim’s blood-spattered body. FRE 403 provides that “although relevant, evidence may be excluded if its probative value is substantially outweighed by the danger of unfair prejudice, confusion of the issues, or misleading the jury, or by considerations of undue delay, waste of time or needless presentation of cumulative evidence.” The prejudice to the defendant results from the facts that the photo was in color and the body was blood-spattered from the butcher’s knife wounds. Moreover, the prosecution can use other methods to attempt to prove that the wounds are those from a butcher’s knife (medical testimony from an autopsy report regarding the victim’s wounds). Choice (A) is wrong since, if the photo is admissible, there is no reason to believe it must be limited to that single purpose. It could also be relevant to indicate the angle of wounds, a possible struggle, etc. Choice (B) is incorrect, since all relevant evidence is not necessarily admissible. Choice (C) is incorrect because it reaches a conclusion about the effect of the evidence on the jury that is not necessarily correct under this fact pattern.
226
Q
  1. A woman has been charged with felony murder for holding up a convenience store and shooting the clerk during the course of the robbery. The prosecutor now wishes to introduce the testimony of a bystander who was in the parking lot and saw the shooting through a nearby window. The bystander proposes to testify that he heard a customer in the store scream out, “Knock that gun away! Don’t let her do it again!”
    This evidence is admissible
    (A) as non-hearsay.
    (B) if the bystander can be shown to have been a co-conspirator.
    (C) as an excited utterance.
    (D) as a statement made under belief of impending death.
A
  1. (C) Under the excited utterance exception to the hearsay rule, the bystander’s testimony about the customer’s statements would be admissible. FRE 803 (2) provides that an “excited utterance” is not excluded by the hearsay rule, even though the declarant is available as a witness. An excited utterance is a statement relating to a startling event or condition made while the declarant was under the stress of excitement caused by the event or condition. The customer’s statements clearly relate to the startling event of the shooting that he was observing. Choice (A) is wrong, since the testimony is hearsay. Choice (B) is wrong, since the co-conspirator’s admission rule, FRE 801 (d)(2)(E), is incorrectly interpreted here. Choice (D) is not correct, since the unavailability requirement has not been met.
227
Q
  1. A recent law school graduate applied for a legal position with a law firm. A senior partner in the law firm contacted a professor at the law school seeking information about the graduate’s qualifications. The professor replied, “Don’t be misled by that ‘A’ I gave the graduate in entertainment law. I later found out that he had obtained the grade by cheating.”
    The graduate soon received a letter from the law firm notifying him that he was not suitable to become a member in the prestigious tradition of the firm. After the graduate received the rejection letter, he phoned the senior partner to inquire why he was not suitable for employment. The senior partner told him what the professor had said and then stated, “We do not hire cheaters who are dumb enough to get caught.”
    The graduate brings suit against the professor for defamation for falsely charging the graduate with being a cheater.
    If the professor pleads truth as a defense, he should not be permitted to introduce evidence that shows that
    (A) the graduate did, in fact, cheat on an examination in the entertainment law class.
    (B) another professor told him that the graduate had cheated on an admiralty law final examination.
    (C) the findings of the law school indicated that the graduate had submitted a plagiarized paper in a moot court brief.
    (D) the reputation among the graduate’s fellow classmates was that he frequently cheated on law school examinations.
A
  1. (B) Character evidence is generally not admissible in civil actions unless character is “in issue” (i.e., where it is a material element of a charge, claim, or defense). In the following tort actions, character is said to be “in issue” and, thus, character evidence is admissible: (1) defamation, (2) deceit, (3) negligent entrustment, and (4) assault/battery where the defendant claims that he acted in self-defense, in which case the plaintiff’s reputation for violence or turbulence is in issue. Since this is a defamation action, answer choices (A), (C), and (D) are admissible character evidence. Choice (B) is inadmissible as hearsay because it is an extra-judicial (or out- of-court) statement, which is being offered to prove the truth of the matter asserted.
228
Q
  1. A defendant was arrested and charged with burglarizing a home. At trial, the main issue of contention centered on the identity of the burglar. The state called the homeowner to testify, but his identification testimony was inconclusive. The homeowner testified that he was asleep during the burglary and awoke as the burglar was fleeing the premises. The homeowner admitted that he didn’t see the face of the burglar and couldn’t even tell whether the person was male or female. Thereafter, the prosecution offers evidence that at the time of the burglary, the defendant needed money to defend himself against other burglary charges.
    This proffered evidence should be
    (A) admitted without restriction as probative evidence of motive.
    (B) admitted, provided the court instructs the jury to limit use of the evidence to the defendant’s motive to burglarize.
    (C) excluded as lacking any probative value. (D) excluded as substantially more unfairly prejudicial than probative.
A
  1. (D) FRE 403 states that “Although relevant, evidence may be excluded if its probative value is substantially outweighed by the danger of unfair prejudice, confusion of the issues, or misleading the jury, or by consideration of undue delay, waste of time, or needless presentation of cumulative evidence.” The policy behind the often utilized balancing standard is best viewed by pitting the maximum probative value of evidence against the likely prejudicial effect (i.e., in close cases, the decision should be to let in the evidence). This question is clearly not a close case. The prosecutor’s proffered evidence as to the defendant’s motive for the burgLary (that he needed the money to defend against other burglary charges) is more prejudicial than probative because the defendant’s identity is in issue, not his motive. Therefore, choice (D) is correct, and choice (A) is incorrect. FRE 404(b) was also being tested, but it did not apply. Specifically, Rule 404(b) excludes admissibility of other crimes, wrongs, or acts as to character evidence to prove conduct in conformity therewith; however, such crimes may “be admissible for other purposes, such as proof of motive, opportunity, intent, preparation, plan, knowledge, identity, or absence of mistake or accident.” Again, evidence of the defendant’s motive to commit burglary should be excluded, since its prejudicial effect outweighs its probative value. Thus, choice (B) is incorrect. Finally, choice (C) is incorrect because the evidence does have probative value; it’s just that the danger of unfair prejudice substantially outweighs its probative value.
229
Q
  1. A man was charged with tax fraud. He wished to offer evidence, on his behalf, of his honesty while running for mayor. During his political campaign, he refused to lie about a psychiatric problem in his past, and his honesty cost him the election. The man was quoted as saying, “If I lied, I’d be mayor today, but that’s no way to win an election.”
    The BEST argument of the objector to such evidence in the tax fraud case would be which of the following?
    (A) Character evidence is not admissible in such cases.
    (B) One incident doesn’t prove character.
    (C) This is an impermissible way to prove character.
    (D) The evidence is irrelevant.
A
  1. (C) Since character evidence is admissible in a criminal case (when the defendant “opens the door” and places her character in issue), choice (A) is incorrect. Choice (B) is not the best argument, since it applies to evidence of habit rather than character. Choice (C) is the best choice because specific acts are not admissible in a criminal trial for the purpose of proving character. Choice (D) is incorrect because the standard for relevance under FRE 401 is relativeLy low: evidence having any tendency to make the existence of a determinative fact more or less probable than it would be without the evidence.
230
Q
  1. A salesman, who had worked 20 years for the same company, was suddenly terminated for no apparent reason. Thereafter, the salesman sued the company, alleging age discrimination. At trial, he wants to call an employee of the company as an adverse witness. The salesman seeks to have the employee testify that she was present at a company board meeting when the company’s president allegedly said, “Now, I’m sure that everyone agrees that the salesman is too old, and he really doesn’t typify the image we want our employees to project.” It is the common practice of the company to tape record all such board meetings. Moreover, it is customary for the company’s secretary to transcribe the tapes following the board meetings.
    Upon objection by the company’s attomey, the employee’s proposed testimony will be held
    (A) admissible, because the employee was present during the board meeting.
    (B) admissible, because the president’s statement was an admission by a company representative.
    (C) inadmissible, because the tape of the meeting is the best evidence.
    (D) inadmissible, because the secretary’s transcribed notes are the best evidence.
A
  1. (B) FRE 801 (d)(2)(D) allows a statement by a party’s agent or servant concerning a matter within the scope of the agency or employment, made during the existence of the relationship, to be admissible against the party as a vicarious admission. In O’Donnell v. Georgia Osteopathic Hospital, 748 F.2d 1543 (1984), the plaintiffs claim based upon age discrimination was upheld where the Court admitted (as a vicarious admission) the testimony of the person who repLaced the plaintiff describing discussions he had had with the hospital’s executive director concerning the plaintiffs treatment. The statements made by the company’s president concerning the salesman’s age not typifying the company’s image were made within the scope of his empLoyment and during the existence of the reLationship. As such, his testimony will be admissible as a vicarious admission. Choice (B) is correct. Choice (A) is incorrect because the mere fact that the employee was present at the board meeting and had personal knowledge is insufficient in itself to admit an out- of-court statement which that otherwise be hearsay. Allowing the statement as an admission however, under choice (B), addresses this issue, since admissions, by definition, are non-hearsay. Choices (C) and (D) are incorrect. The best evidence rule is not relevant under these facts. The employee’s testimony of what the president said is based on personal knowledge and is not reliant on any tape recording or transcribed notes.
231
Q
  1. A customer sued a printing company for negligence, claiming that the holiday cards he specially ordered were of poor quality and had his name misspelled. At trial, the customer shows the court a printed statement from a printer’s association defining the minimum standard of skill for the printing industry in the community. The customer asks the court to take judicial notice that the printed statement constitutes the applicable standard of care for printing companies in the area.
    The court should
    (A) grant the request, because it promotes expeditious and economical disposition of the proceedings.
    (B) grant the request, because the standard of care is amply supported by information supplied to the court.
    (C) deny the request, because judicial notice cannot be taken of matters that are ultimate issues in the case.
    (D) deny the request if the judge determines that the statement is subject to reasonable dispute.
A
  1. (D) A judicially noticed fact must be one not subject to reasonable dispute in that it is either (1) generally known within the territorial jurisdiction of the trial court or (2) capable of accurate and ready determination by resort to sources whose accuracy cannot reasonably be questioned. FRE 201. Choice (D) is the best choice because, according to the rule, a judge will not take judicial notice of a fact over which there is a reasonable dispute. Choice (C) is incorrect because it is too conclusory. While courts are hesitant to take judicial notice over matters that are the ultimate issues in a case, that is not to say that courts will always refrain from doing so. Choice (A) is wrong. If the statement in (A) were correct, a judge could take judicial notice of nearly every disputed issue at trial in order to save time and promote efficiency. Choice (B) is also incorrect. There simply isn’t enough information about this document to determine whether it amply supports or defines the standard of care for printers.
232
Q
  1. A grocery store is sued by a customer for damages for injuries allegedly suffered to the face and forearms, which resulted from his collision with the store’s automatic doors. The customer contended that at the time of the mishap, the doors were programmed to swing outward, taking would-be customers entering the store, by surprise. At trial, the customer testified to both the nature of his injuries and the events leading up to their occurrence. The grocery store denied that the doors to its store were in any way unsafe or that the customer’s injuries were caused by his encounter with the doors. The customer’s sister testified at trial that she accompanied her brother to the store the day of the accident and witnessed the outward swinging doors, which struck him. She further alleged that when she returned to the store the following week, the doors had been re-programmed to swing inward.
    The trial judge should rule the sister’s testimony
    (A) admissible as a commonsense impression for which a lay opinion is entirely proper.
    (B) admissible as a tacit admission of a party opponent.
    (C) inadmissible, on the grounds of irrelevancy.
    (D) inadmissible, on the grounds of public policy considerations.
A
  1. (D) Evidence of subsequent remedial measures is generally excluded on grounds that the possible relevancy of the evidence is outweighed by public policy considerations (e.g., encouraging repairs after accidents). Therefore, choices (A), (B), and (C) are incorrect.
233
Q
  1. A man is suing a store for damages for injuries allegedly suffered when a can of soup fell on his toe while he was shopping. At trial, the store calls a member of the local community to testif’ that the man’s reputation for honesty is poor, and that he is known far and wide as the biggest storyteller in
    town.
    This testimony is
    (A) admissible, but only to show the likely exaggeration of the man’s alleged injuries.
    (B) admissible, but only to discredit the man’s testimony that the accident happened in the manner in which he claims it did.
    (C) inadmissible character evidence.
    (D) inadmissible, because the man’s testimony has not yet been rebutted by the store, and his credibility is, therefore, not yet susceptible to attack.
A
  1. (B) Note that the witness’s testimony (regarding the man’s reputation) is inadmissible character evidence. As a general rule, character evidence is not admissible in a civiL case unless defendant’s character is “in issue” (e.g., in defamation or negligent entrustment cases). Thus, choice (C) does state a correct rule of law. However, choice (B) is a better answer because even though the witness’s testimony is not admissible as character evidence, it is admissible for purposes of impeachment. According to FRE 608, for impeachment purposes opinion and reputation evidence of character (subject to the limitation that such evidence may refer only to character for truthfulness or untruthfulness) are admissible. In this case, testimony about the man’s character for truthfulness is properly admissible to impeach him. Choice (A) is wrong because it lists an incorrect purpose for the admission of the evidence. Choice (D) is also wrong. Once a witness in a case testifies, his or her credibility is at issue. There is no requirement to first rebut the witness’s testimony in order to attack his credibility.
234
Q
  1. A plaintiff alleges in a lawsuit against a store that he purchased a hot dog with chili that gave him food poisoning and caused him excruciating pain. The store offers into evidence certified copies of court proceedings revealing that the plaintiff has filed three similar lawsuits in the last 18 months against other convenience stores, claiming food poisoning resulting from the purchase of food from those stores.
    These copies should be ruled
    (A) inadmissible, because of the best evidence rule.
    (B) inadmissible, because they are irrelevant to the present claim.
    (C) inadmissible, because there is the danger of undue prejudice to the plaintiff, which outweighs the probative value of the evidence.
    (D) admissible, because they establish a pattern of similar actions.
A
  1. (C) Since evidence of prior tort claims is generally inadmissible because of the chance for prejudice and conclusion, choice (C) is correct. Choice (A) is incorrect because certified copies may be admitted in Lieu of original documents. Choice (B) is wrong because the evidence may have some probative value. Choice (0) is incorrect because the probative value is outweighed by the prejudicial effect of the evidence.
235
Q
  1. While shopping at a mall, a woman suffered injuries when an escalator malfunctioned, crushing her foot. The woman brings suit against the mall. At trial, she calls another shopper to testif’ that he rode the same escalator just moments before the woman’s accident and had to jump in the air at the last minute to avoid getting his foot crushed in the escalator.
    This testimony is
    (A) inadmissible, because it is irrelevant.
    (B) inadmissible, because it is not known whether the other shopper the woman was exercising reasonable care in riding the escalator.
    (C) admissible, because it tends to prove that a dangerous condition was present at the time the woman’s foot was crushed in the escalator.
    (D) admissible, because of its probative value in establishing that the woman’s injury was caused by the malfunctioning escalator.
A

io6. (C) The testimony is admissible to prove the existence of the condition that the woman claimed caused her injuries. When there appears to be sufficient proof of similarity in conditions and proximity in time to the accident, such evidence is admissible. Choice (A) is wrong, since the information is, indeed, relevant. Choice (D) is an incorrect statement, since the evidence is only admissible to prove the existence of the condition, but not that the accident happened. Choice (B) is incorrect because it establishes an unnecessary condition precedent for the use of this evidence.

236
Q
  1. A witness is scheduled to testify in a criminal case. To prove that he was a juvenile on a given date, evidence is offered that on that date he was confined in a juvenile detention facility.
    If a party wished to argue that this evidence is hearsay, whom would he point to as the hearsay declarant?
    (A) The witness on the stand.
    (B) The party offering the evidence.
    (C) The juvenile authorities.
    (D) The witness.
A
  1. (C) In this question, the facts tell you that someone will argue that this evidence is hearsay. In every hearsay statement, the out-of-court statement is made by the declarant. Here, the statement wouLd be that the witness was confined in a juvenile detention facility. That statement would have been made originaLLy by the juvenile authorities. So choice (C) is the best choice. Choice (A) is not as good because it is not necessarily true. The witness on the stand might have had nothing to do with the witness’s juvenile records. Choice (B) is wrong. The party offering hearsay evidence is rarely the declarant. Choice (D) is incorrect because the witness would not have been responsible for the creation and maintenance of his juvenile records.
237
Q
  1. During a custody dispute, a court granted the request of the defendant and appointed a clinical pathologist to conduct the blood grouping tests of the child, the complainant, and the defendant. After first stating all of his qualifications at the non-jury trial, the pathologist testified that he and his associates made five separate blood grouping tests and that all proper safeguards were taken to protect the integrity and accuracy of the tests. The pathologist also offered more detailed testimony about the testing procedures and stated that no discrepancies were found in the testing methods. Finally, the pathologist made the statement that the blood grouping tests were conclusive in proving that the defendant could not be the father of the child.
    If the complainant’s attorney makes an objection to the introduction of the blood tests into evidence, the court will most likely
    (A) sustain the objection, because the blood tests are not conclusive evidence of paternity.
    (B) sustain the objection, because blood tests have not been generally recognized by the scientific community.
    (C) overrule the objection, because the court will take judicial notice of the accuracy of such tests.
    (D) overrule the objection, because the blood tests are an admission.
A
  1. (C) The court is warranted in taking judicial notice of the correctness of such blood tests excluding paternity where it is shown that the person making the tests is qualified and that no discrepancies were found in the testing method. See FRE 201. Choice (A) states an incorrect standard. Evidence does not have to provide conclusive proof of the matter at issue to be admissible. Choice (B) is wrong because blood tests are, in fact, generally recognized as accurate by the scientific community. Choice (D) is incorrect because a blood test is not an admission within the meaning of FRE 801(d).
238
Q
  1. An accounting professor has been called as a defense expert witness in a white-collar criminal case. He testified that accounting records conclusively demonstrated that the defendant had not committed any type of fraud. On cross-examination of the professor, the prosecuting attorney asked him if he was familiar with a particular treatise. The professor responded affirmatively. The attorney then asked him if the treatise was accepted as authoritative and if he was aware of the conclusions regarding the ability of accountants to detect fraud. The professor responded in the affirmative. The prosecutor now attempts to read the following section of the treatise into evidence: “It is virtually impossible to state with conclusive certainty the existence, or lack thereof, of fraud in the majority of white-collar crime cases, given the sophistication of defendants and the variety of accounting methods available.”
    On objection by the defendant’s attorney, the trial judge will
    (A) overrule the objection and permit the entire treatise to be introduced into evidence.
    (B) overrule the objection and permit only the contradictory statements to be read into evidence.
    (C) sustain the objection, because the statements in the treatise are hearsay not within any recognized exception.
    (D) sustain the objection, because the professor did not rely on the treatise in making his conclusions.
A
  1. (B) Under FRE 803 (18), the following are not excluded by the hearsay rule “… to the extent called to the attention of an expert witness upon cross-examination or relied upon by him in direct examination statements contained in published treatises, periodicals, or pamphlets on a subject of history, medicine, or other science or art, established as a reliable authority by the testimony or admission of the witness or by other expert testimony or by judicial notice. If admitted, the statements may be read into evidence but may not be received as exhibits.” Choice (A) is wrong because only the entire treatise itself cannot be placed into evidence. Choice (C) is incorrect. The statements in the treatise are hearsay, but fall within a recognized hearsay exception. Choice (D) is also wrong. One purpose of this hearsay exception is to test the knowledge of expert witnesses on cross-examination and provide contradictory testimony to the fact-finder.
239
Q
  1. During a paternity trial, the mother’s attorney asks to have the baby girl, now 36 months old, exhibited for the purpose of calling attention to certain facial resemblances between the child and the reputed father. The defendant’s attorney objects to the offering of the baby into evidence for comparison.
    The trial judge should
    (A) permit the child to be exhibited as a non-objectionable form of demonstrative evidence.
    (B) not permit the child to be exhibited, because such an exhibition would be highly prejudicial to the defendant.
    (C) permit the child to be exhibited as within the hearsay exception of pedigree.
    (D) not permit the child to be exhibited, because such an exhibition would be self-serving to the complainant.
A
  1. (A) The majority of states hold that a trial court may, in its discretion, permit a child to be exhibited (e.g. demonstrative evidence) in a proceeding for the purpose of showing a resemblance to the putative father. Judway v. Kovacs, 239 A.2d 556 (1967). Accordingly, choice (A) is correct, and choice (B) is incorrect. Choice (C) is incorrect. The hearsay exception for pedigree requires an out-of-court statement. The live exhibition of a child at trial cannot constitute an out-of-court statement by any stretch of the imagination. Choice (D) is wrong. Much of the evidence admitted in an adversarial trial is self-serving to the offering party.
240
Q
  1. A man was driving his car when he suddenly hit a pedestrian as she was crossing the street. Following the accident, an ambulance was summoned and the pedestrian was taken to a nearby hospital. The man, who also drove to the hospital, approached the pedestrian in the emergency room while she was being treated for her injuries. When the pedestrian saw the man, she told him, “You went through that red light. You could have killed me.” She then asked the man for $5,000 to cover her anticipated medical expenses. The man replied that he would have to think about it. Shortly thereafter, the man sent a letter to the pedestrian in which he wrote, “I realize I drove through the red light. Naturally, I would like to settle this without any litigation. I’m willing to pay all your medical expenses and give you $2,000 if you don’t sue me.” The pedestrian did not accept the man’s offer and brought suit against him.
    At trial, the pedestrian’s attorney offers the first sentence of the letter into evidence, and the man objects. The trial judge should rule the evidence
    (A) admissible as an admission.
    (B) inadmissible as an admission made in connection with an offer to pay medical expenses.
    (C) inadmissible as an admission made in connection with settlement negotiations.
    (D) inadmissible, because documents must be offered in their entirety.
A

iii. (C) In accordance with FRE 408, evidence of (1) furnishing or offering or promising to
furnish, or (2) accepting or offering or promising to accept, a valuable consider
ation in compromising or attempting to compromise a claim that was disputed as
to either validity or amount, it is not admissible to prove liability for the claim or
the amount. Two grounds for the rule of inadmissibility may be advanced: (1) lack
of relevancy, and (2) the public policy aspect is to promote the settling of disputes,
which would be discouraged if offers of compromise were admitted in evidence.
Choices (A) and (B) are incorrect because of the public policy considerations of FRE 408. Choice (D) is incorrect. There is no requirement for a proponent to offer documents in their entirety. The Rule of Completeness in FRE 106 does, however, permit an adverse party to demand the contemporaneous admission of any part of a recording or writing that ought to, in fairness, be considered, along with the portion offered by the proponent of the evidence.

241
Q
  1. In a suit between a plaintiff and a defendant arising out of an automobile accident, a bystander was
    called to the stand to testify that the defendant had driven through a red traffic light immediately before the collision. On cross-examination by the defense, the bystander admitted to having made a prior statement to a friend that he was not sure if the traffic signal was yellow or red when the defendant drove through the intersection.
    The trial judge should
    (A) permit the jury to consider the prior statement as substantive evidence.
    (B) permit the defendant to call other witnesses to confirm the fact that the prior statement had been made.
    (C) on request by the defendant, instruct the jury that the bystander’s testimony that the defendant had driven through the red light be
    disregarded.
    (D) on request by the plaintiff, instruct the jury that the prior statement may be used only to
    impeach the bystander’s credibility, and not as substantive evidence.
A
  1. (D) This is an example of another key Multistate testing area, namely, whether a prior
    inconsistent statement should be admissible as substantive evidence or for the
    limited purpose of impeaching the witness. Under the Multistate viewpoint, these
    prior statements will often be inadmissible as evidence of what they state because
    they constitute hearsay and are not within any exceptions to the hearsay rule. Even
    though inadmissible hearsay as evidence of the facts stated, they are nevertheless
    admissible for the limited purpose of impeaching the witness. See McCormick on
    Evidence, pp. 66—67, 601—602. Consequently, choice (D) is correct. Choice (A) is
    incorrect. The evidence can be considered for its impeaching value, but not sub
    stantively. Choice (B) is incorrect. If a witness denies making an inconsistent state
    ment, the opposing party can offer extrinsic evidence of it under FRE 613. Since
    this witness admitted the inconsistency, there is no need for extrinsic evidence of
    it. Choice (C) is wrong because it is inconsistent with the purposes of impeachment
    by prior inconsistent statements. Impeaching statements are not admitted to prove
    their truth, but rather to demonstrate that the witness is inconsistent.
242
Q
  1. A defendant is charged with murder and relies upon a claim of self-defense. At trial, the defendant attempts to show that prior to the killing, he was told by a drinking buddy that “The victim has killed five men and would as soon kill a man as look at him.”
    If the statement by the drinking buddy is offered into evidence to prove that the victim, in fact, killed the five men, the statement should be ruled
    (A) admissible, because the statement is offered to demonstrate its effect on the defendant.
    (B) admissible, because it qualifies under the state of mind exception to the hearsay rule.
    (C) inadmissible, because it is hearsay not within any recognized exception.
    (D) inadmissible, because it is self-serving.
A
  1. (C) Choice (C) is correct since, under FRE 801, hearsay is any extra-judicial statement
    offered in evidence to prove the truth of the matter stated. Choice (A) is incorrect
    because the fact pattern tells us that the statement is being offered to prove that
    the victim killed five men. It is not being offered to prove its effect on the defendant.
    Choice (B) is wrong because the statement does not satisfy the foundational elements of FRE 803(3): it is not a statement of the declarant’s then-existing physical or mental condition. Finally, choice (D) is incorrect. “Self-serving” is not a valid evidentiary objection.
243
Q
  1. A fan attended a minor league hockey game in his hometown. Unfortunately, he was only able to obtain tickets in the visitor’s section. While at the game, he became involved in an altercation with a fan of the visiting team. When the fan cheered for a home team goal, the visiting fan turned around and threatened to kill the home fan if he didn’t shut up. The home fan pulled a knife out of his pocket and stabbed the visiting fan in the arm. At his trial for aggravated assault, the home fan wants to introduce a statement from a witness who was standing next to the visiting fan at the game. The statement, which the witness had made earlier in the game when the home fan cheered for the home team, was, “You’d better watch out. At a hockey game last week, the visiting fan put two guys in the hospital when they wouldn’t shut up. One of them had 33 stitches after the visiting fan bashed his head against the steps.”
    Assume that the witness’s statement is offered as proof of the effect it produced in the home fan’s mind. In this regard, the statement would most likely be
    (A) admissible as non-hearsay.
    (B) admissible as a present sense impression.
    (C) inadmissible as hearsay not within any recognized exception.
    (D) inadmissible, because the statement is selfserving.
A
  1. (A) Choice (A) is correct, since the witness’s statement that the visiting fan had put
    two men in the hospital at a hockey game the week before is being offered to show
    its effect on the defendant’s state of mind and not offered to prove the truth of
    the assertion. Thus, the witness’s statement bears upon the reasonableness of the
    home fan’s fear to justify his self-defense plea. Choice (B) is wrong. The statement
    does not satisfy the foundational elements of FRE 803(3): it is not a statement of
    the declarant’s then-existing physical or mental condition. Choice (C) is incorrect
    because the statement is not being offered as hearsay. FinaLly, choice (D) is incor
    rect. “Self-serving” is not a valid evidentiary objection.
244
Q
  1. A customer purchased a mattress from a furniture store. The mattress had a defective spring, which one of the manufacturer’s workers had not properly tied down. When the customer slept on the mattress, she was wounded in the back by the sharp point of the spring. The customer sued the furniture store for negligence and breach of warranty.
    At trial, the customer testified that the salesman assured her that the furniture store inspected each mattress before delivery. Furthermore, the customer testified that she remembered the salesman telling her that “The furniture store stands behind all of its mattresses with a lifetime guarantee.” She is then asked if the salesman made any other representations about the mattress. The customer stated, “Yes, in fact the day before the mattress was delivered, I received a letter from him thanking me for my patronage at the furniture store. As I recall, he also made some assurances about the manufacturer’s mattresses.” The customer’s attorney then asked, “What assurances were made in the letter?”
    The customer’s answer is
    (A) admissible as an admission.
    (B) admissible as a declaration against interest.
    (C) inadmissible, under the best evidence rule.
    (D) inadmissible as hearsay not within any recognized exception.
A
  1. (C) This is a classic Multistate example where the “best evidence” ruLe is the correct
    answer. The best evidence (or, commonly, the original document) rule provides that
    in proving the terms of a writing, where the terms are material, the original writing
    must be produced unless it is shown to be unavailable for some reason other than
    the serious fault of the proponent. Here, since the customer is attempting to testify
    as to the assurances (or representations) made in the salesman’s letter to support
    her breach of warranty action, the writing itself would be the “best evidence” for
    proving the terms contained therein. Choice (A) is incorrect because there is no admission involved in the contents of the letter. Choice (B) is aLso wrong. Even if the salesman is unavailable to testify, this is not a statement against interest, but rather a thank-you letter and, possibly, a warranty declaration. Choice (D) is also incorrect. The answer to the question would not be offered for the truth of the matter asserted, but rather to establish what was said.
245
Q
  1. A man suffered frostbite when he spilled a soft drink on his lap that he had purchased from a restaurant. The man sued the restaurant for negligence and damages. At trial, the man’s attorney calls a witness to testify that the witness also suffered frostbite when she spilled a soft drink in her lap that she had purchased at the restaurant two weeks earlier. The witness also states that she told the restaurant about her injury immediately after suffering it.
    Upon objection by the restaurant’s attorney, the trial judge should rule the witness’s testimony
    (A) admissible to prove the restaurant’s negligence, provided that the court gives a cautionary instruction that the testimony should not be considered as bearing on the issue of damages.
    (B) admissible to prove that the restaurant should have been aware of the danger of frostbite posed by its soft drinks.
    (C) inadmissible, because it is not probative of a fact in issue.
    (D) inadmissible, because it seeks to put into evidence separate, unrelated transactions with a third party.
A

ii6. (B) One of the most commonly tested areas in Evidence comes under relevancy, where
evidence is offered to prove the existence of a particular physical condition, situa
tion, or defect. In a suit alleging fraud or breach of warranty, for example, the pLain
tiff must prove that the defendant knew of the defective product, or ought, in the
exercise of reasonable care, to have learned of it. Here, the plaintiff will want to
prove directly that the defendant had knowledge of other accidents, injuries, or
complaints as circumstantial evidence that the defendant was aware of the defect.
Choice (A) is incorrect because the evidence cannot prove negligence by itself. Choice (C) is incorrect; as explained above, the evidence is probative. Choice CD) is an incorrect statement of the law of evidence.

246
Q
  1. In a breach of contract action brought by a supplier against a grocery store for refusing to buy his artisanal bread and goat cheese, the supplier calls his ex-wife to testify about the business the supplier operated and the financial effect of the breach on his business and personal finances. On cross-examination she admits that she is the supplier’s former spouse. Thereafter, on re-direct, the supplier’s attorney seeks to have the ex-wife testify that she and the supplier have not spoken to each other since their bitter divorce proceeding three
    years ago.
    The ex-wife’s testimony is
    (A) admissible under the family history exception to the hearsay rule.
    (B) admissible, because the ex-wife’s answer might rebut the inference of bias.
    (C) inadmissible, because it relates to a collateral matter.
    (D) inadmissible, because it is irrelevant to any substantive issue in the case.
A
  1. (B) Certainly, the Law recognizes the slanting effect upon human testimony of the emo
    tions or feelings of the witness toward the parties or the self-interest of the witness
    in the outcome of the case. Partiality, or any acts, relationships, or motives reason
    ably likely to produce it, may be proved to impeach credibility. In most states the
    impeacher may inquire as to the facts of bias on cross-examination as the first step
    in impeachment. By the same token, the reply to bias (or any new matter) drawn
    out on cross-examination is the normal function of the re-direct, and examination
    for this purpose is a matter of right, though its extent is subject to the control of the
    judge’s discretion. Choice (A) is incorrect. The purpose of the re-direct is to attack
    the charge that the ex-wife’s testimony is biased, not to establish the fact of her
    former marriage. Choice (C) is wrong because witness motive and bias are never a
    collateral matter at trial. For similar reasons, choice (D) is also incorrect; bias and
    motive are always relevant.
247
Q
  1. A plaintiff sued a defendant for injuries that the plaintiff suffered when he was struck by the defendant’s car. At trial, the plaintiff testified that he was walking across the street inside a crosswalk at the time the defendant’s car hit him. This action takes place in a jurisdiction that has a statute in effect requiring all motorists to stop and permit pedestrians to cross streets at designated crosswalks.
    The defendant contended that the plaintiff ran into the street outside the crosswalk. Furthermore, the defendant testified that he immediately applied his brakes when he saw the plaintiff dart into the street, but couldn’t stop his car in time to avoid hitting him. In support of the defendant’s testimony, the defendant calls a neighbor to testify that she has observed the plaintiff frequently during the years that she has lived on that street, and that the plaintiff “always crosses the street outside the crosswalk.”
    The neighbor’s testimony is likely to be
    (A) admitted, because it tends to show that the plaintiff was careless.
    (B) admitted, because it is probative of the plaintiff’s habit of crossing the street outside the crosswalk.
    (C) excluded, because the neighbor was not an eyewitness to the accident.
    (D) excluded, because her testimony is impermissible evidence of character to show defendant’s plaintiff’s conduct on a particular occasion.
A
  1. (B) According to FRE 406, “Evidence of the habit of a person or of the routine practice
    of an organization, whether corroborated or not and regardless of the presence of
    eyewitnesses, is relevant to prove that the conduct of the person or organization on
    a particular occasion was in conformity with the habit or routine practice.” Words
    such as “aLways” or “invariably” are necessary to prove habit. Choice (A) is true,
    but it is a weaker answer than choice (B) because habit under FRE 406 is more spe
    cific to the fact pattern. Choice (C) is wrong because the neighbor is not testifying
    about the accident itself, but rather to the pLaintiff’s habits. Choice (D) is incorrect.
    Although one can argue that habit evidence is a form of character evidence, its use
    is permissible under FRE 406 if the foundational elements are met.
248
Q
  1. A husband and wife had a stormy relationship. The husband, who was an alcoholic, frequently beat his wife when he became drunk. One day, the husband had been drinking beer while watching football on television. After his favorite team lost a close game, the husband went into a violent rage. As he was storming around the apartment, he suddenly grabbed his wife and led her outside to their car. He then ordered her to get inside the vehicle.
    The husband was driving around aimlessly when he negligently collided with another car. The other driver, who was injured in the accident, brought suit against the husband to recover damages. Thereafter, the husband retained an attorney to represent him. At his first consultation with the attorney, the husband was accompanied by his wife. During the conference, the attorney’s secretary took notes of the meeting. Two weeks later, the wife separated from the husband. She then hired a lawyer to represent her in a suit against the husband for battery and false imprisonment. At the trial between the husband and the other driver, the other driver’s attorney calls the wife to testify. She proposes to testify that her husband was intoxicated at the time of the accident.
    Upon objection by the husband’s attorney, the wife’s proposed testimony will most likely be
    (A) admissible as a proper lay opinion. (B) inadmissible, under the attorney-client
    privilege.
    (C) inadmissible, under the marital privilege. (D) inadmissible, because the wife is biased.
A
  1. (C) At common law, under the marital privilege, a husband or wife shall not be required
    or, without consent of the other, if living, allowed to disclose a confidential communication made by one to the other during marriage. A majority of courts have construed “communications” to extend to facts, conditions, and transactions, as well. McCormick even points out that the marital privilege has been said to apply to “any information secured by the wife (or husband) as a result of the marital relation and which would not have been known in the absence of such relation.” Accordingly, “information secured by one spouse through observation during the marriage as to the health, or intoxication, habitual or at a particular time, or the mental condition of the other spouse would be protected by the privilege.” Evidence, pp. 163—164. Note: at the time of trial, even though they are separated, the husband and wife are still married, so the privilege still applies. Choice (A) is wrong. Although the wife’s observation about the husband’s intoxication is likely lay opinion testimony, the marital privilege will prohibit her from sharing her observation and opinion at trial. Choice (B) is incorrect. There is no indication that the information the wife wants to share occurred during an attorney-client consultation. Finally, choice (D) is incorrect. Biased witnesses frequently testify at trial, and it is the job of the advocate in the adversarial system to bring the bias to the attention of the fact-finder.
249
Q
  1. A husband visits an attorney seeking a divorce because of his wife’s infidelity. At the client consultation, the attorney’s secretary took notes. With the permission bf the husband, the attorney’s law clerk also sat in on the consultation to see how the attorney conducted these meetings. Shortly after this initial consultation with the attorney, the secretary quit. During the divorce trial, the wife’s attorney seeks to call the secretary to testify about what the husband told the attorney during the initial client consultation.
    Upon objection, the secretary’s proposed testimony will most likely be
    (A) admissible, because her presence during the meeting destroyed the attorney-client privilege.
    (B) admissible, because the law clerk’s presence during the meeting destroyed the attorney- client privilege.
    (C) inadmissible, because the attorney-client privilege disqualifies the secretary from testifying to such confidential communications.
    (D) inadmissible, because it is hearsay not within any recognized exception.
A
  1. (C) It is the essence of the attorney-client privilege that it is limited to those communications that the client either expressly made confidential or which she could reasonably assume, under the circumstances, would be understood by the attorney as so intended. In cases where the client has one of his agents attend the conference, or the lawyer calls in his clerk or confidential secretary, the presence of these intermediaries will be assumed not to militate against the confidential nature of the consultation. McCormick, pg. 189. Thus, choices (A) and (B) are incorrect answers. Choice (D) is a wrong answer because the fact pattern tells us nothing about the nature of the statements made during the consultation.
250
Q
  1. A husband and his wife are involved in a contested divorce and child custody battle. Ignorant of the adversarial system, they both visited a family law attorney together for an initial consultation. The attorney advised them that he could not represent them both. The wife found another attorney. During the trial, the wife’s attorney calls the wife to testify. She states that during the initial consultation she and her husband had with his attorney, she privately told the attorney, when her husband was taking a bathroom break, that her husband had a bad drinking problem, which was one of her major reasons for seeking a divorce and custody of the children.
    Upon objection by the husband’s attorney, the wife’s testimony should be
    (A) admitted, because the spousal privilege is inapplicable, since the parties are living in separation.
    (B) admitted, because the marital communication privilege is inapplicable in a lawsuit involving the parties.
    (C) excluded, because the wife is incompetent to testify to marital communications because she is not the holder of the privilege.
    (D) excluded, because confidential statements made during a joint consultation are privileged communications between clients and their attorneys.
A
  1. (B) When two or more persons, each having an interest in some problem or situation, jointly consult an attorney, their confidential communications with the attorney will be privileged in a controversy of either or both of the clients with the outside world. But McCormick notes that it will often happen that the two original clients will fall out between themselves and become engaged in a controversy in which the communications at their joint consultation with the attorney may be vitally material. In such a situation, it is clear “that the privilege is inapplicable.” Evidence, pg. 189. Furthermore, the marital privilege does not apply to either, (1) prosecutions for crimes committed by one spouse against the other, or (2) actions by one spouse against the other. Consequently, choice (B) is correct, and choice (D) is incorrect. Choice (A) is wrong because the spousal privilege applies, regardless of the current marital status of the parties (and separation is not divorce). Choice (C) is incorrect. Either party can claim the marital privilege.
251
Q
  1. A plaintiff brought an action against a defendant for personal injuries resulting from a car accident in which the defendant’s car, negligently driven by his cousin, struck the telephone booth in which the plaintiff was resting. Liability is based on a statute making owners of automobiles liable for the negligent actions of those driving with the consent of the owner of the vehicle. The plaintiff offered into evidence the testimony of his doctor that the plaintiff had said to the doctor, when consulting the doctor for treatment, that he felt pain in his back immediately after the accident, and that pain persisted.
    The trial court should rule this testimony
    (A) admissible as a spontaneous declaration.
    (B) admissible under the hearsay exception of declaration of present bodily condition.
    (C) inadmissible as hearsay not within any recognized exception.
    (D) inadmissible as conclusions.
A
  1. (B) Due to the fact that the statement was not made immediately following the occurrence, choice (A) is incorrect. Choice (C) is incorrect because, although hearsay, it falls within a recognized exception. Choice (D) is not accurate, since the plaintiff did not state a legal conclusion (e.g., the ailment was caused by the accident). But rather, he stated that “he felt” pain in his back following the accident. Choice (B) is correct, and the evidence would be admissible as a declaration of present bodily condition, a recognized exception to the hearsay rule.
252
Q
  1. During his lunch break one day, an employee took a company vehicle to a fast-food restaurant and got into an accident on the way back. The driver of the other car sued the company. The company, in its defense, claimed that the employee’s use of the vehicle was unauthorized. At trial, the company calls the employee’s boss. The boss offers to testify that the employee called him five minutes after the accident and stated, “I hope you won’t be sore, because I took the company car without permission.”
    The court should rule that this evidence is
    a. admissible as a declaration against interest.
    b. admissible as an admission.
    c. inadmissible, because the employee was not unavailable.
    d. inadmissible, because it violates the employee’s Fifth Amendment rights.
A
  1. (C) Choice (A) is incorrect because there is no showing that the dectarant is unavailable. Choice (B) is incorrect because the employee is not a party opponent and there is no showing of agency. Choice (D) is an incorrect statement of law. Choice (C) is the correct answer, since unavailability would be a requirement to bring the statement in as a declaration against interest.
253
Q
  1. Following their law school graduation party, a graduate and his roommate were driving home in an automobile that was owned and operated by the graduate. As they were approaching an intersection, their vehicle collided with a motorcycle whose rider was seriously injured in the accident. The rider sued the graduate for operating his motor vehicle in excess of the speed limit. In addition, the roommate brought suit against the rider for operating his motorcycle negligently and driving through a red light. The roommate subsequently reached an out- of-court settlement with the graduate and dismissed the lawsuit against him.
    At trial against the rider, the roommate testified that he observed the rider drive his motorcycle through a red light and then swerve into the path of the graduate’s automobile. In rebuttal, the rider offered into evidence the roommate’s complaint against the graduate alleging that the latter was driving “at an excessively high rate of speed at the time the accident occurred.”
    The complaint is
    (A) admissible as an admission.
    (B) admissible as a declaration against interest.
    (C) admissible under the former testimony exception to the hearsay rule.
    (D) inadmissible, because the suit was dismissed as part of a compromise.
A
  1. (A) A party’s pleadingin onecase, whether a final one orone laterwithdrawn, amended, or superseded, is freely usabLe against him as an evidentiary admission in any other litigation. Choice (B) is incorrect because there is no showing of unavailability. Choice (C) is incorrect because a pleading is not “former testimony.” Choice (D) is an incorrect statement of the law.
254
Q
  1. A driver was severely injured when his car burst into flames after he was rear-ended in a minor auto accident. The driver filed suit against the manufacturer. To prove negligent design by the manufacturer, the driver offers the portion of the transcript from a prior trial in which an expert witness testified that in his opinion the car was unsafe as designed.
    The testimony could qualify under the hearsay exception for former testimony
    (A) only if the manufacturer had been a party to the former proceeding.
    (B) whether or not the manufacturer had been a party to the former proceeding.
    (C) if this exception has been held incompatible with the confrontation clause of the Sixth Amendment.
    (D) if the manufacturer had an opportunity to cross-examine the expert at the former preceding.
A
  1. (B) Although the FRE make former (reported) testimony admissible when proffered against a party, such testimony is also admissible against a predecessor in interest to the former proceeding. Choice (A) is too limited an answer, while choices (C) and (D) are simply incorrect. Thus, choice (B) is the best answer.
255
Q
  1. In order to establish the unavailability of a witness at trial
    (A) it is sufficient to show that the witness is not presently in court.
    (B) it is necessary to show that he has died.
    (C) it is necessary to show that every reasonable effort has been made to procure his attendance.
    (D) if living, it is necessary to show that he is insane.
A
  1. (C) Absence from the courtroom is not enough; thus, choice (A) is not the best answer. Choice (B) is incorrect, since death is not the only circumstance for unavailability. Choice (D) is incorrect, as insanity is only one of the conditions that make a person unavailable. Therefore, the best alternative is choice (C).
256
Q
  1. While riding her bicycle along the street, a woman was struck by a vehicle that she didn’t see. Subsequently, the woman sued the driver of the vehicle to recover damages for her injuries.
    At trial, the woman calls a police officer to testify that a few minutes after the accident, a driver stopped him and said, “Hey, officer, I just saw an accident involving a red truck that hit this girl who was riding a bicycle. The truck left the scene of the accident and I followed it to a warehouse.” The police officer then testified that he immediately drove to the warehouse and saw the defendant sitting in a red truck that was parked in the lot. The driver is available to testify at trial.
    Upon objection by the defendant’s attorney, the police officer’s testimony regarding the driver’s statement should be
    (A) admissible as a statement of recent perception.
    (B) admissible as a present sense impression.
    (C) inadmissible, because the driver was available to testify at trial.
    (D) inadmissible as hearsay not within any recognized exception.
A
  1. (D) A favorite Multistate testing area deaLs with distinguishing between the hearsay rule and the rule requiring firsthand knowledge. The distinction is whether the witness purports to give the facts directly upon his own credit or whether he purports to give an account of what another has told him and this is offered to evidence the truth of the other’s report. Here, you must read the question carefully. You’re being asked to determine the admissibility of the police officer’s testimony regarding the driver’s statement. Although the police officer can properly testify to his own observations, he cannot testify to what the driver told him because it is being offered to prove the truth of the driver’s statement and, thus, inadmissible hearsay. Choices (A) and (B) are, therefore, incorrect because both of them discuss hearsay exceptions. Choice (C) is also incorrect. It suggests that the hearsay would be admissible if only one could establish the unavailability of the driver. The driver’s statement, however, doesn’t fit into any hearsay exceptions under FRE 803, 804, or 807.
257
Q
  1. In a prosecution of a defendant for receiving stolen property, an informer testified that the defendant sold him a stolen stereo, which the defendant knew had been stolen. During direct examination, the informer testified that he was unemployed but sometimes worked part-time as a substitute teacher.
    On cross-examination, the defendant’s attorney asked the informer if he had recently filed an application to become a police officer. The informer responded affirmatively. The defendant’s attorney then asked the informer the following question:
    “Isn’t it true that you hope that by acting as an undercover agent, the police department will overlook the fact that you had two misdemeanor convictions for possession of marijuana?” The prosecuting attorney immediately objected.
    The trial judge should rule that the defendant’s attorney’s inquiry concerning the informer’s hopes and misdemeanor convictions is
    (A) improper, as evidence of conduct not related to truthfulness.
    (B) improper, as relating to convictions of crimes not punishable by imprisonment in excess of one year.
    (C) proper, as tending to show the informer’s bad character for truthfulness.
    (D) proper, as relevant to the informer’s possible bias.
A
  1. (D) Partiality, or any acts, relationships, or motives reasonably likely to produce it, may be proved to impeach credibility. McCormick notes that self-interest may be shown in a criminal case when the witness testifies for the state and it is shown that an indictment is pending against him. By analogy, in this case, the defense attorney may show the informer’s seLf-interest, or bias, because he filed an application to become a police officer and had two misdemeanor convictions on his record. Choice (A) is incorrect because the issue here is bias and not necessarily truthfuLness. Choice (B) is a wrong answer, and somewhat deceptive. The rule it states is true if one is trying to accomplish an impeachment by proof of prior conviction under FRE 609. That is not the purpose of the impeachment by evidence of bias in this case. Choice (C) is incorrect. Evidence of motive or bias does not necessarily prove character for untruthfulness; it merely provides a vehicle for suggesting that the witness’s testimony on this occasion is less worthy of belief.
258
Q
  1. A wife is the beneficiary of a policy issued by an insurance company, insuring the life of her husband, now deceased. The policy contained a clause providing that double indemnity is payable in the event that death of the insured “results directly, and independently of all other causes, from bodily injury effected solely through external violent and unexpected means.”
    The husband was found dead in the chicken shed of his farm. His death resulted from wounds caused by a shotgun blast. The wife filed the necessary papers with the insurance company concerning proof of her husband’s death. The insurance company admitted liability for the face amount of the policy but rejected the wife’s claim for double indemnity.
    The wife then instituted suit against the insurance company demanding judgment according to the double indemnity provisions of the husband’s insurance policy.
    At trial, the wife was called to testify about the events on the day of her husband’s death. The wife said that she was in the kitchen when she heard a gunshot in the shed. As she rushed out of the house, she saw their neighbor running from the shed. The neighbor is present in court.
    As a witness, the wife was
    (A) competent, because she had personal knowledge of the matter.
    (B) competent, because the neighbor is available to testify.
    (C) incompetent, because she had a personal interest in the outcome of the lawsuit.
    (D) incompetent, because she was testifying to facts occurring after her husband’s death.
A
  1. (A) Under FRE 602, a witness must have personal knowledge of the matter he is to testify about. The requirement of “personal knowledge” means that the witness must have observed the matter and must have a present recollection of his observation. Choice (B) is incorrect, since the neighbor’s availability is irrelevant with respect to the wife’s competency to testify. Choice (C) is incorrect, since one having a personal interest is competent to testify. Choice (D) is wrong because, under the circumstances, the Dead Man’s Statute is inapplicable.
259
Q
  1. At a defendant’s trial for the murder of his former employer, the prosecutor offers into evidence a note that was found during a lawful search of the defendant’s apartment. The note states: “The demon compels me to kill my former employer. I thirst for his bloody spirit.” At trial, the defendant’s former girlfriend is called by the prosecution to testify that the handwriting was, in fact, the defendant’s. The defendant’s attorney objects.
    The trial judge should
    (A) sustain the objection on the grounds that the former girlfriend’s testimony would be inadmissible opinion evidence.
    (B) sustain the objection on the grounds that identification of handwriting requires expert testimony.
    (C) overrule the objection on the grounds that the former girlfriend qualifies as an authenticating witness.
    (D) overrule the objection on the grounds that the letter qualifies as a past recollection recorded exception to the hearsay rule.
A
  1. (C) In accordance with FRE 901 (a)(2), a lay person with sufficient familiarity with the handwriting of another person may be called upon to prove or disprove the genuineness of such handwriting. Here, one may properly infer that a former girlfriend would have sufficient familiarity with the handwriting of her boyfriend. Thus, the former girlfriend would be afforded a basis for identifying the defendant’s handwriting specimen. Given the authentication rules of FRE 901, choices (A) and (B) are both incorrect. Choice (D) is incorrect because none of the foundational elements for past recollection recorded apply to this fact pattern.
260
Q
  1. A customer is suing a car dealer for selling him a salvaged vehicle that the car dealer had represented as being brand new. A few weeks before trial, the car dealer approached his sister and said, “Sis, I need some sympathy. I sold a salvaged vehicle to a customer, and now he’s suing me. I didn’t mean any harm by it. I inspected the vehicle and everything.” Unknown to either the car dealer or his sister, the sister’s boyfriend was at the front door and overheard this conversation. When the time for trial came around, the car dealer left the country and refused to attend the trial, telling his attorney to handle it. The customer’s attorney attempted several times to secure the car dealer’s attendance at trial, but was unsuccessful. At trial, the sister’s boyfriend is called to testify about the conversation he overheard.
    On objection by the car dealer’s attorney, the court will most likely rule the testimony
    (A) admissible as a statement of then-existing mental or emotional condition.
    (B) admissible as a declaration against interest.
    (C) admissible as a statement of present sense impression.
    (D) inadmissible as hearsay not within any recognized exception.
A
  1. (B) In accordance with FRE 804(b)(3), the car dealer’s statement, “…I sold a salvaged vehicle to the customer,” may properly be admissible as a declaration against interest. Whenever you are dealing with a declaration against interest, there are two things to remember. First, it must be a declaration against one’s (i.e., the declarant’s) (1) penal, (2) pecuniary, or (3) proprietary interest(s). Second, the declarant must be unavailable to testify at trial. In the present case, the car dealer’s out-of- court statement subjected him to pecuniary liability. As a result, it would qualify as a declaration against interest. According to FRE 804, a person can be considered unavailable if he “is absent from the hearing and the proponent of a statement has been unable to procure [his] [attendance or testimony].” Choice (A) is wrong because the car dealer’s statement does not relate to his mental or emotional condition. Choice (C) is incorrect because the statement describes a past act and not a current impression. And finally, choice (D) is wrong because, even though the statement is hearsay, it fits within a recognized exception.
261
Q
  1. A defendant is on trial for false pretenses. He is charged with selling worthless stock in a dummy corporation to unwitting investors. The defendant is alleged to have masterminded a scheme wherein he set up a nonexistent corporation that never conducted business. The victims were sent prospectuses containing false financial data, which induced them to purchase stock in the phony corporation.
    At trial, the prosecution seeks to introduce into evidence proof that the defendant had set up 10 other so-called dummy corporations that never existed.
    This evidence is
    (A) admissible, to show defendant’s character trait for dishonesty.
    (B) admissible, to show his intent to defraud.
    (C) inadmissible, because character cannot be proved by specific instances of misconduct.
    (D) inadmissible, because the evidence is not relevant.
A
  1. (B) In accordance with FRE 404(b), evidence of other crimes, wrongs, or acts is not admissible to prove the character of a person in order to show that he acted in conformity therewith. It may, however, be admissible for other purposes, such as proof of motive, opportunity, intent, preparation, plan, knowledge, identity, or absence of mistake or accident. It may be helpful to remember this exception by the acronym MIMIC: M-motive, I -intent, M-mistake (absence of which), I-identity, and C-common scheme or plan. Choice (A) is wrong because the evidence is being offered for intent and not to show character. Choice (C) is a true statement, but the wrong answer here because the evidence is being offered for a non-character purpose. Choice (D) is incorrect. The evidence is relevant to show intent.
262
Q
  1. A husband and his passenger were killed in an auto collision involving a defendant. The husband’s wife now brings an action to recover damages for loss of society and companionship. The wife testified that she and her husband had a close relationship and that they were happily married for over 15 years. Then on cross-examination, defendant’s counsel asked her if she “was aware of the affair that her husband was having with his passenger?”
    The wife’s attorney immediately objected to this question. The judge should rule that the question is
    (A) objectionable, because the question is beyond the scope of direct examination.
    (B) objectionable, because it was highly prejudicial.
    (C) unobjectionable, because plaintiff’s knowledge of such an affair was probative of the value to be placed on her loss.
    (D) unobjectionable, because it was relevant on the issue of negligence.
A
  1. (C) Choice (C) is correct since, in plaintiff’s action for loss of society and companionship, the existence of her husband’s infidelity and plaintiff’s knowledge of it would be probative of the value to be placed upon husband’s loss. Moreover, the question would be directly related to, and in impeachment of, hertestimony on direct examination. Choice (A) is wrong; the statement is squarely within the scope of direct. Choice (B) is incorrect. Some of the most effective evidence at trial is highly prejudicial. FRE 403 only excludes prejudicial evidence if its prejudicial impact substantially outweighs its probative value. Choice (D) is wrong. The evidence is relevant for loss, but has nothing to do with negligence.
263
Q
  1. A defendant in an automobile accident case is being sued by the estate of the driver of the other car. At trial, the defendant calls an eyewitness to the collision. The eyewitness testifies that after the crash, he immediately ran to the other driver’s car to try to render assistance. The eyewitness observed the other driver covered with blood from the top of his head down to his toes. He was moaning, gasping, and crying out, “I did not see the other car coming!” The other driver died 10 minutes later. The estate’s attorney objects to the eyewitness’s testimony.
    The trial judge should rule that his testimony is
    (A) admissible as a declaration against interest.
    (B) admissible as a dying declaration.
    (C) inadmissible as hearsay not within any recognized exception.
    (D) inadmissible, because this testimony cannot be admitted in civil cases.
A
  1. (B) In accordance with FRE 804 (b), in a prosecution for homicide or in a civiL action or proceeding, a statement made by a declarant, while believing that his death was imminent, concerning the cause or circumstances of what he believed to be his impending death is not excluded by the hearsay rule. While the common law required that the “dying declaration” be that of the homicide victim, the Federal Rules of Evidence have expanded the theory of admissibility to apply equally in civil cases. Thus, choice (B) is correct, and choice (D) is incorrect. Choice (A) is incorrect because there is nothing to indicate that the statement was against a penal, pecuniary, or proprietary interest. Choice (C) is wrong because the statement does satisfy a recognized hearsay exception.
264
Q
  1. A defendant was arrested and charged with involuntary manslaughter for causing a vehicular homicide. A grand jury was convened, and various witnesses were subpoenaed. At the grand jury, a witness was asked the following question by the prosecuting attorney: “Is it not true that 20 minutes after the accident you told the police that the defendant’s car went through the red light?” He answered, “Yes.”
    The defendant was also subpoenaed to appear before the grand jury. The defendant was represented by counsel at the grand jury hearing. During questioning he was shown the witness’s statement from his testimony. No objection was made to the introduction of the witness’s statement.
    At the defendant’s trial, the prosecuting attorney seeks to introduce into evidence the witness’s statement from the deposition taken at the grand jury hearing. The defendant’s attorney objects to this proffer of evidence.
    The objection at trial will be
    (A) granted, because the statement is hearsay not within any recognized exception.
    (B) granted, because the questioning of the witness was leading.
    (C) denied, because the defendant’s attorney did not object at the grand jury hearing that the witness’s testimony was hearsay.
    (D) denied, because the defendant’s attorney did not object at the grand jury hearing that the questioning of the witness was leading.
A
  1. (A) Grand jury proceedings have traditionally been shrouded in secrecy. Substantial controversy has surrounded the issue of disclosure of testimony given by grand jury witnesses. Clearly, the defense has a right of access to such testimony under Fed. Rule Crim. Pro. Section 26.2(a) once a government witness has testified on direct. Here, the question presented is whether a defendant is required to make objections at the grand jury proceeding in order to preserve the issue at trial. As a general rule, objections that are “substantive” in nature (e.g., relevancy and hearsay) may be raised for the first time when the deposition is offered in evidence at trial. McCormick, pg. 127. Therefore, choice (A) is correct and, for the same reasons, choice (C) is incorrect. However, objections that go to the “manner and form” of questions! answers (e.g., nonresponsive or leading) must be made at the time the deposition is taken. Choice (B) is, therefore, incorrect because no objection was lodged at the proceeding to the form of the question. Although it is a true statement, choice (D) is not the best answer here. Choice (A) is a stronger answer because it provides a contingency (if the defendant’s attorney objects on hearsay grounds) that, if true, would ensure the objection was sustained.
265
Q
  1. A man was arrested and charged with robbery. Upon being taken into custody, he was given his Miranda rights and then taken to the police station for booking. At the stationhouse, the man told a police officer that he was prepared to make a confession. The police officer then turned on a video recorder and videotaped the man’s confession.
    At trial, the prosecution called the police officer to testify to the incriminating statements that the man made in his confession.
    Upon objection by the man’s attorney, the police officer’s proposed testimony is
    (A) inadmissible, because the videotape is the best evidence of the man’s confession.
    (B) inadmissible, because it is hearsay not within any recognized exception.
    (C) admissible, because the police officer had firsthand knowledge of the confession.
    (D) admissible, because the man was given his Miranda rights before the confession was elicited.
A
  1. (C) The best evidence rule applies only when the proponent is attempting to prove the contents or terms of a writing. Sometimes, a writing recites or records a perceivable event or condition, such as a marriage (marriage certificate), payment of money (receipt), or the utterance of certain words (transcript). Here, the proponent wishing to prove the underlying event may offer testimony as an observer. This does not involve the best evidence rule because the proponent is not attempting to prove the terms of a writing, but merely is presenting evidence of an event perceived by a witness with firsthand knowledge. Therefore, choice (A) is a wrong answer. In this question, the police officer’s testimony is not reliant on the writing orthe videotape. He had firsthand knowledge of the man’s confession because he was present and overheard it. Choice (B) is wrong because the police officer is testifying based on personal knowledge and not introducing a hearsay statement. Choice (D) is incorrect because the issue of proper Miranda warnings is not relevant to admissibility under these facts.
    Exam Tip: This distinction between the best evidence rule and firsthand knowLedge is frequently tested on the MBE. Remember, the best evidence rule applies only when the proponent is attempting to prove the contents or terms of a writing.
266
Q
  1. A heavyset man with long red hair robbed a liquor store. Thereafter, a man was arrested and charged with the armed robbery. At the man’s trial, the owner of the liquor store was called to testify. He admitted that he was unable to identify the man, who now had a shaven head, as the robber. The prosecuting attorney then handed the owner six photographs. He proposed to testify, over defense objections, that he had previously told the prosecuting attorney that picture #4, admittedly a picture of the man before he shaved his head, was a picture of the person who robbed his store.
    The owner’s proffered testimony should be adjudged
    (A) admissible as a prior identification by the witness.
    (B) admissible as past recollection recorded.
    (C) inadmissible, because it is hearsay not within any recognized exception.
    (D) inadmissible, because it is a violation of the man’s right of confrontation.
A
  1. (A) FRE 801(d)(1)(C) defines as non-hearsay a prior statement by a witness of identification of a person made after perceiving him, if the person making the identification is available to testify at the trial or hearing and subject to cross-examination concerning the statement. There is no requirement that the witness first be impeached. Nor does a prior identification require that the identifying witness make a positive in- court identification. Choice (A) is correct, since the owner had previously identified the man in picture #4. Choice (C) is incorrect, since prior identifications are defined as non-hearsay. Choice (D) is incorrect because the man’s right to confrontation is not violated, since the owner is available in court and subject to cross-examination. Choice (B) is incorrect because the past recoLlection recorded hearsay exception involves first, an attempted refreshing of the witness’s recollection that fails, followed by the witness reading the words of a memorandum or record into evidence. In this case, the only writing is a photograph.
267
Q
  1. A supermarket had just reopened after a two- week closing for renovations. On the morning of the reopening, a woman was shopping when she suddenly fell and suffered a broken leg. Thereafter, the woman sued the supermarket for personal injuries. In her suit, the woman claimed that she tripped over a paint bucket that had been lying in one of the aisles.
    At trial, the woman called another customer to testify that while he was shopping he heard a thud, turned around and saw the woman fall on the floor. He also testified that he saw an empty paint bucket lying a few feet from the woman’s leg. On cross- examination, the customer got into an argumentative exchange with the supermarket’s attorney. When asked his opinion of what caused the woman’s fall, the customer testified, “She fell because she tripped on the bucket.” He admitted on cross-examination, however, that he didn’t actually see the woman trip on the bucket. The supermarket’s attorney then asked the customer the following question:
    “If you didn’t see the woman trip on the bucket, how do you know that she fell because of it?” The customer answered, “Because my girlfriend who was shopping with me told me later that she saw the woman trip over the bucket.”
    The supermarket’s attorney moves to strike the customer’s last answer. If the trial judge overrules the defendant’s motion and permits the customer’s answer to stand, it will most likely be because
    (A) the customer’s answer was invited by the supermarket’s attorney’s question.
    (B) the customer’s answer was based on firsthand knowledge.
    (C) the customer’s answer was a statement of recent perception.
    (D) the customer’s answer was a present sense impression.
A
  1. (A) Choice (B) is wrong because the customer’s answer was not based upon firsthand knowledge. In order to have firsthand knowledge, a witness must have actuaLly observed the fact (to which he is testifying). The facts clearly indicate that the customer did not see the woman fall. Choice (C) is incorrect because the customer did not perceive the woman falling. Choice (D) is wrong because declarations concerning a present sense impression require that the declarant observe the event at the time the statement is made. In other words, the declaration must be made contemporaneously with the observation. By process of elimination, choice (A) is the best answer.
268
Q
  1. Four men entered into a lease as tenants of a five- bedroom house. They each had a separate bedroom and used the fifth bedroom as a laboratory to manufacture the illegal drug methamphetamine.
    One of the men was the so-called money man; he provided the funds to make the rental payments and to purchase the laboratory equipment. One of the other men was the chemist; he had both an undergraduate and graduate degree in chemistry and knew the formula and procedure for producing methamphetamine. Another of the men had sold drugs in the past and knew potential buyers of the methamphetamine. Finally, the fourth man was the enforcer; his role in the scheme was to make sure that no unauthorized persons entered onto the premises of the house.
    The four men had been involved in this drug venture for seven months when they were finally arrested and charged with conspiracy to manufacture and distribute methamphetamine, and distribution and manufacture of methamphetamine. During the trial, the prosecution wishes to introduce as evidence against the money man, a statement made by the enforcer to the landlord at the time of the signing of the lease, in which the enforcer said, “No matter what you charge us for rent, you better O.K. it with the money man, because he’s the one who will really be paying it.” The money man’s attorney objects.
    The judge’s ruling on admissibility will depend on
    (A) whether the enforcer actually paid all of the rent.
    (B) whether the enforcer had a legal obligation to pay the rent under the terms of the lease.
    (C) whether the landlord may be considered a party opponent.
    (D) whether the statement was made during the course of and in furtherance of the conspiracy.
A
  1. (D) If the enforcer’s statement was made during the course of and in furtherance of the conspiracy, the statement would be admissible as an admission. Under FRE 801(d) (2), “A statement is not hearsay if the statement is offered against a party and is (subsection E) a statement by a co-conspirator of a party during the course and in furtherance of the conspiracy.” The enforcer’s statement concerning the fact that the money man would be paying the rent for the premises where the methamphetamine was being manufactured would certainly fall within the “course of and furtherance of” the conspiracy to manufacture and distribute methamphetamine. All other statements would be incorrect as to prove the admissibility of the enforcer’s statement against co-conspirator, the money man.
269
Q
  1. A defendant has been charged with manufacturing illegal firearms in violation of federal firearms laws. In its case-in-chief, the government wants to introduce a copy of the defendant’s transcript from a technical college to show his certificate as gunsmith. The college is no longer in existence, though, and the whereabouts of the previous officials and employees are unknown. However, the transcript bears a seal above the registrar’s signature, and the court has taken judicial notice of the existence of the college and has found that it is normal for a college to compile a transcript. Finally, the information about the defendant on the transcript has been corroborated by a government witness. Assume the prosecution provided the defendants with a copy of the transcript before trial and informed them of its intent to introduce it as evidence.
    The transcript will be admitted
    (A) as a business record, because the custodian can testifS’ as to how the records of the college were kept.
    (B) under the general hearsay exception, as the transcript in these circumstances has sufficient circumstantial guarantees of trustworthiness.
    (C) only if the registrar’s signature can be authenticated by an expert witness.
    (D) only if the defendant waives his right to object to its admission.
A
  1. (B) The transcript would be admitted under FRE 807, which is the residual “catch-all” exception since, under the circumstances, the circumstantial guarantees of trustworthiness would be met. The court here took (1) judicial notice of the existence of the college and of the fact that colleges normally compile such records, (2) found that the copy of the transcript was an authentic copy because of the registrar’s seal, and (3) received corroborating testimony concerning the contents of the transcript. Therefore, the transcript was properly authenticated and reliable under the circumstances. Choice (A) is incorrect, since the custodian of the record was not available to testify as required by FRE 806 (6). Choice (C) is wrong, since an expert witness is not required under the FRE for authenticating a document. Choice (D) is incorrect, since it implies that the transcript is not admissible. Refer to United States v. Hitsman eta!., 604 F.2d 443 (5th Cir. 1979), concerning one of the first instances wherein a federal court has invoked the residual “catch-all” hearsay exception.
270
Q
  1. A victim was killed by a gunshot in the presence of three men. The fatal wound was inflicted by a single shot, and there was no suggestion of a conspiracy among those present. One of the three men was charged with the killing.
    The prosecution called one of the other men as its first witness. His testimony, if true, established that the defendant fired the shot that killed the victim. The defendant’s attorney then called the man’s longtime neighbor to impeach him.
    Which of the following offers of evidence is most likely to be admitted for the impeachment of the man?
    (A) Testimony that the witness is a cocaine addict.
    (B) Testimony that the witness embezzled money from his employer last year.
    (C) Testimony that the witness is not, in the neighbor’s opinion, worthy of belief.
    (D) Testimony that the witness is an atheist.
A
  1. (C) Without question, Evidence is one of the toughest areas on the Multistate exam. Although the Evidence fact patterns are rather short and simple, the questions themselves are very tricky. Here, for example, is an extremely difficult question dealing with impeachment. Most students will be tempted to choose choice (B). With regard to impeachment, FRE 609 provides, “For the purpose of attacking the credibility of a witness, evidence that he has been convicted of a crime shall be admitted but only if the crime (1) was punishable by death or imprisonment in excess of one year, or (2) involved dishonesty or false statement” Choice (B) is wrong, then, because the neighbor testified that the witness “embezzled money,” not that he was convicted of embezzlement; FRE 609 does not apply. Choice (C) is the best answer because FRE 608 states, “The credibility of a witness may be attacked or supported by evidence in the form of opinion or reputation, but subject to these limitations: (1) the evidence may refer only to character for truthfulness or untruthfulness.” You are being tested here on the distinction between impeachment by opinion and by conviction of a crime.
271
Q
  1. A man and a woman were passengers on a plane. They were seated next to one another in the first- class section of the plane. Midway through the flight, the woman excused herself and went to the restroom. While she was gone, the man rummaged through the woman’s purse and stole $100 in cash. Unknown to the man, a flight attendant saw him steal the money. As is customary in such situations, the flight attendant radioed ahead, and police officers arrested the man when the plane landed.
    The man was subsequently charged with violating an applicable federal larceny statute. During the trial, the prosecuting attorney called the man’s wife as a willing witness against her husband. She proposed to testify that the man confided to her that he did steal the woman’s money during the flight. The man’s attorney objected on the grounds of hearsay and privilege.
    The wife’s proposed testimony is
    (A) admissible, because it is neither hearsay nor privileged.
    (B) inadmissible, because it is hearsay not within any recognized exception, though it is not privileged.
    (C) inadmissible, because it discloses a privileged communication, though it is not hearsay.
    (D) inadmissible, both because it discloses a privileged communication and because it is hearsay not within any recognized exception.
A
  1. (C) This is an extremely tricky question because, in light of the holding in Trammel v. California, 445 U.s. 40 (1980), you might be tempted into choosing choice (A). This is incorrect, however, because although in federal courts one spouse may testify against the other in criminal cases, with or without the consent of the party spouse, there is one very important limitation. According to footnote 42 in Trammel, confidential communications between the spouses are nevertheless privileged. Since the facts indicated that “the man confided” to his wife, his communication is deemed to be confidential and, therefore, remains privileged. The statement is not hearsay because it is an admission under FRE 801(d). Choice (A) is wrong because the statement is privileged. Choice (B) is wrong because the statement is not hearsay within the meaning of FRE 801(d), but it is privileged. Choice (D) is incorrect because the statement is privileged, but is not hearsay.
272
Q
  1. A defendant was prosecuted for assault and battery after he admitted striking the victim with a pool cue during a barroom argument. The defendant claimed that he acted in self-defense after he was attacked by the victim, who was drunk and belligerent. At trial, the defendant called a witness who testified that the victim was the aggressor in the altercation. On cross-examination of the witness, the prosecuting attorney asked the witness the following question:
    “Isn’t it true that when you filed your federal income tax return last year, you failed to report the interest income from your savings accounts?” The prosecuting attorney was informed of this fact by the witness’s accountant.
    Upon objection by the defendant’s attorney, the prosecutor’s question is
    (A) improper, because it is not relevant to the issues in the case.
    (B) improper, because the defendant has not been convicted of any crime in connection with the tax return.
    (C) within the court’s discretion to allow, because filing a false income tax return is an act that bears on the witness’s truthfulness.
    (D) within the court’s discretion to allow, because federal income tax evasion is a crime punishable by imprisonment in excess of one year.
A
  1. (C) This question deals with the highly tested area of “bad act” impeachment. FRE
    608(b) states that “Specific instances of the conduct of a witness, for the purpose ofattackingorsupporting his credibility, otherthan conviction of crime as provided in Rule 609, may not be proved by extrinsic evidence. They may, however, in the discretion of the court, if probative of truthfulness or untruthfulness, be inquired into on cross-examination of the witness (1) concerning his character for truthfulness or untruthfulness, or (2) concerning the character for truthfulness or untruthfulness of another witness as to which character the witness being cross-examined has testified.” The prosecuting attorney’s impeachment of the witness, the defense witness, is proper in this case, since the questioning inquires into unconvicted acts bearing on untruthfulness — i.e., failure to report income interest on the witness’s tax return. Since admissions of such specific instances of conduct are discretionary, choice (C) is the correct answer. Choice (A) is incorrect because, as discussed above, the evidence could be admissible on the issue of credibility. Choice (B) is incorrect. Forthe proposed use of this evidence, a prior conviction is not necessary. Choice (D) is wrong. Allegations of serious offenses that have not yet resulted in a conviction cannot be used for impeachment unless they can be tied into an issue, such as truthfulness, as in this example.
273
Q
  1. A plaintiff sued a trucking company for injuries allegedly suffered when a speeding truck jackknifed and struck her car.
    Which of the following pieces of evidence, if offered by the plaintiff, is most likely to be admitted by the court?
    (A) Testimony concerning subsequent repairs to the truck paid for by the trucking company where they have stipulated to ownership.
    (B) Color pictures of a plastic surgeon operating on the plaintiff showing the incision and bloody surgical tools.
    (C) Testimony from the plaintiff’s boss concerning the amount of time she has missed work, when payroll records had already been admitted as evidence of the plaintiff’s lost wages.
    (D) Testimony of a surprise witness to the accident when the court is willing to grant the trucking company a continuance to prepare for crossexamination.
A

izt. (D) Choice (D) is the best choice. This has high probative value, and the prejudice that could result from a surprise witness can be overcome by the continuance. The easiest way to answer this question is by process of elimination. Choice (A) is wrong. Even though evidence of subsequent remedial measures is sometimes admissible to show ownership and control (though not for an inference of negligence), it is unnecessary here to prove that the defendant owned the truck, since that issue was stipulated. Choice (B) is incorrect. A court would most probably sustain an objection under FRE 403 on the grounds that the evidence is too inflammatory and would encourage the jury to decide the issue on an emotionaL basis. In other words, the pictures are unduly prejudicial, even though they may have some limited probative value as to the extent of injury. Note: The examiners often use gross, disgusting evidence to test you on FRE 403. Choice (C) is wrong. This evidence is cumulative, and a judge could exclude it under Rule 403 on the grounds that it is time-consuming and adds little to the case.

274
Q
  1. One night, a defendant went to play bingo. After losing at bingo, the defendant went on a violent rampage. He stole a car and then picked up a woman at a local bar. After she rejected his advances, the defendant stabbed her to death. The defendant was subsequently arrested and charged with felony- murder under an appropriate federal criminal statute.
    The defendant admitted committing the crime but pleaded not guilty by reason of insanity. At trial in federal court, the prosecuting attorney calls the defendant’s psychiatrist to testify as to the defendant’s mental state at the time of the killing. The defendant’s attomey objects, claiming that the testimony would violate the psychiatrist-patient privilege.
    The objection should be
    (A) sustained, provided the state law recognizes the psychiatrist-patient privilege.
    (B) sustained, provided the court concludes that the privilege should be recognized as part of modern common law.
    (C) overruled, because no such privilege is specifically provided in the Federal Rules of Evidence.
    (D) overruled, because the right to a fair trial overrides the use of a privilege to prevent full exploration of the facts in federal court.
A
  1. (B) The FRE do not contain a specific physician-patient (or any other) privilege. However, FRE 501, the onLy federal rule on privilege, is often tested on the MBE. It provides as follows: “Except as otherwise required by the Constitution of the United States or provided by Act of Congress or in rules prescribed by the Supreme Court pursuant to statutory authority, the privilege of a witness, person, government, State, or political subdivision thereof shall be governed by the principles of the common law as they may be interpreted by the courts of the United States in the light of reason and experience. However, in civil actions and proceedings with respect to an element of a claim or defense as to which State law supplies the rule of decision, the privilege of a witness, person, government, State, or political subdivision thereof shaLL be determined in accordance with State law.” The defense attorney’s objection to the testimony of the defendant’s psychiatrist may not be sustained under choice (A) because the defendant’s trial is a criminal case, and in accordance with FRE 501, the principles of common law (not state Law) wiLl govern. Choice (B) is correct because, under FRE 501, common law “as interpreted . . . in the Light of reason and experience,” wilL determine the privileges applicable in federal question and criminal cases. See McCormick, Evidence, pg. 182. Choice (C) is wrong for the reasons discussed above. Choice (D) is also incorrect. Frequently, privilege law operates to exclude relevant and valuable evidence, thereby interfering with the full exploration of the facts in court. For sound policy reasons, however, we have decided, as a society, that this is an acceptable price to pay.
275
Q
  1. A man and a defendant were college roommates. With the defendant’s permission, his roommate borrowed the defendant’s baseball bat to use in an intramural baseball game. During the course of the game, the roommate struck out with the bases loaded. Angry at himself, the roommate took his bat and flung it into the stands. The bat struck a fan in the face and fractured his nose.
    The fan sued the defendant for his injury, alleging that the defendant was negligent in lending his baseball bat to his roommate when he knew that his roommate was irresponsible with bats. At trial, the fan offers evidence that on four separate occasions during the past year the roommate had negligently thrown bats during other baseball games.
    The fan’s proffered evidence is
    (A) admissible to show that the roommate was negligent on the occasion when the fan was injured.
    (B) admissible to show that the roommate was irresponsible in the use of bats.
    (C) inadmissible, because it is evidence of character.
    (D) inadmissible, because character must be proved by evidence in the form of reputation or opinion.
A
  1. (B) FRE 404(a) states the general rule for character evidence in a civil case: “Evidence of a person’s character or a trait of his character is not admissible for the purpose of proving that he acted in conformity therewith on a particular occasion.” However, under FRE 405(b), ‘in cases in which the character or a trait of character of a person is an essential element of a charge, claim, or defense, proof may also be made of specific instances of his conduct.” In the fan’s suit against the defendant, he is alleging that the defendant was negligent in (ending his bat to his roommate, knowing that he was irresponsible with bats. Since the cause of action here is negligent entrustment, character is in issue. Therefore, FRE 405(b) applies, allowing evidence of the roommate’s specific acts to be admissible to show the character trait that is in issue, specifically, the roommate’s irresponsible use of bats. Evidence of this trait will then be relevant to the defendant’s negligent entrustment. Choice (B) is, thus, correct. Choice (A) is incorrect; the fan’s testimony is not being used to show the roommate’s negligence on this occasion, but rather to show the defendant’s negligence in lending his roommate his bat. Choice (D) is incorrect, since character evidence need not be limited to reputation or opinion evidence when character is in issue.
276
Q
  1. A husband and a wife are filing for divorce. At issue is the market value of the house they own. During the divorce proceedings, the husband’s lawyer calls a neighbor to testify that the market value of the home exceeds $100,000. She states that her testimony is based on an estimate contained in an appraisal report that she saw. The appraisal had been prepared at the husband’s request by a well-known real estate company in the area.
    Upon objection by the wife’s attorney, the neighbor’s testimony will be held
    (A) admissible as proper lay opinion. (B) admissible, because she had firsthand knowledge of the estimate contained in the appraisal report.
    (C) inadmissible, because it involves a privileged communication between husband and wife.
    (D) inadmissible, because the neighbor has not been shown to be an expert on real estate market values.
A
  1. (D) FRE 701 states that “if the witness is not testifying as an expert, his testimony in the form of opinion or inference is limited to those opinions or inferences which are (a) rationally based on the perception of the witness and (b) helpful to a clear understanding of his testimony or the determination of a fact in issue.” Lay opinion under FRE 701 must be based on personal knowledge. Generally, a property owner is qualified to give an opinion as to the value of his property. However, the facts here state that the witness is a neighbor, not the property owner. Moreover, she has no personal knowledge of the fact on which the appraisal report based the market value. Therefore, the neighbor’s testimony will be inadmissible. Choices (A) and (B) are incorrect, since FRE 602 requires evidence “sufficient to support a finding that the witness has personal knowledge of the matter.” The neighbor’s testimony as to real estate market value would require an expert opinion (since experts need not have personal knowledge of the subject matter of their testimony). Choice (D) thus provides the correct rationale.
277
Q
  1. A woman was driving to work when her car was struck by another car. At the time of the accident, the other driver had momentarily taken his eyes off the road while he placed a CD in his dashboard player. Following the collision, which resulted in the woman’s car being extensively damaged, the woman demanded that the other driver pay for the repair work. Whereupon, the other driver said to the woman, “If you will take $2,000, I’m certain my insurance company will pay for it.” The woman refused the other driver’s offer and sued him for damages.
    If, at trial, the woman seeks to testify to the other driver’s statement, this proffered evidence should be ruled
    (A) admissible as an admission by a party-opponent.
    (B) admissible as a statement against interest. (C) inadmissible as a statement made in conjunction with settlement negotiations.
    (D) inadmissible as a self-serving declaration.
A
  1. (C) Under FRE 408, “Evidence of (1) furnishing, offering, or promising to furnish, or (2) accepting, offering, or promising to accept, a valuable consideration in compromising or attempting to compromise a claim which is disputed as to either validity or amount, is not admissible to prove liability. . .“ Since the other driver’s statement to the woman offering $2,000 to pay for the damage to her car was an attempt to settle, it would be inadmissible for public policy reasons. Therefore, choice (C) is correct. Note: Be aware that any statement made in connection with a settlement offer will be inadmissible to prove fault or value of the claim. For example, where the defendant says, “I know I drove through the red light. Anyway, I’m willing to settle and pay for the damage to your car,” the entire statement is inadmissible to prove fault. Choice (A) is incorrect because the other driver’s offer to settle is not an admission. Choice (B) is incorrect. First, FRE 408 would preclude the statement and, second, declarations against interest require a showing of unavailability of trial, which is not present in this fact pattern. Choice (D) contains a rule that is nowhere to be found in evidence law and is, therefore, incorrect as a distracter answer choice.
278
Q
  1. Two cars were driving within the legal speed limit as they approached an intersection. There are no traffic lights, stop signs, or warnings posted at this intersection. Both vehicles entered the intersection simultaneously and collided. In the accident, one of the drivers suffered a broken pelvis and internal injuries. Although the other driver’s truck was slightly damaged, he did not suffer any physical injuries.
    Thereafter, the injured driver sued the city for negligence, claiming that the city failed to provide adequate warning at the intersection, which she contended was particularly dangerous and heavily traveled. At trial, the city attorney offers evidence that the intersection has been the same for 14 years, and no other accidents have occurred during that time.
    Upon objection by the injured driver’s attorney, this evidence should be
    (A) admitted as relevant evidence of a fact of consequence to the action.
    (B) admitted, provided that the jury is instructed about the danger of inferential negative evidence.
    (C) excluded, because it is improper negative evidence.
    (D) excluded, because the dangers of unfair prejudice and confusion of the issues may tend to mislead the jury.
A
  1. (A) A commonly tested area on both the MBE and state essays deals with absence of similar accidents. FRE 403 states that “although relevant, evidence may be excluded if its probative value is substantially outweighed by the danger of unfair prejudice, confusion of the issues, or misleading the jury, or by considerations of undue delay, waste of time, or needless presentation of cumulative evidence.” Application ofthis rule poses considerable difficulty for a defendant wishing to establish the nonexistence of an unduly dangerous condition through the lack of previous accidents. The key determination required to establish relevancy in this case is a showing of substantial identity of material circumstances. By showing fixed or stable conditions, safe use, and an extensive number of experiences, the similarity requirement may be satisfied and, on balance with the probative value, will justify admission. Lilly, Introduction to the Law of Evidence, p. 149. In the classic case of Erickson v. Wa!green Drug Co., 120 Utah 31 (1951), evidence that over 4,000 people a day, for 15 years, in all weather conditions, had used a terrazzo entranceway without anyone slipping, was held admissible negative evidence. Similarly, where a heavily traveled intersection has remained the same for 14 years without any accidents during that time, such evidence should be found relevant and admissible to show nonexistence of a dangerous condition. Choices (B) and (C) are, therefore, incorrect. Choice (A) is correct and a better answer than choice (B), since a jury instruction would not necessarily be required under these facts, but would be subject to the discretion of the court.
279
Q
  1. A relevant fact in a plaintiff’s suit is the magnitude of an explosion at an oil refinery one evening. At trial, the plaintiff was asked on direct examination if he remembered the explosion. He replied, “I recall my son running into the home and screaming that the car windows just shattered.” The son was available to testify at trial.
    The defendant’s attorney makes a motion to strike the plaintiff’s answer. The trial judge should rule the plaintiff’s testimony
    (A) admissible, because the son is available as a witness.
    (B) admissible, whether or not the son is available as a witness.
    (C) inadmissible as hearsay not within any recognized exception.
    (D) inadmissible under the excited utterance exception, because it can be a product of reflection and deliberation.
A
  1. (B) FRE 803(1) provides that a present sense impression is admissible as a recognized hearsay exception. Thus, a statement describing or explaining an event or condition made while the declarant was perceiving the event or condition or immediately thereafter is not excluded by the hearsay rule. Hearsay exceptions under FRE 803 apply regardless of the declarant’s availability at trial. Thus, choice (B) is correct, and choice (A) is incorrect. Choice (C) is incorrect because the statement of the son is, in fact, admissible as a present sense impression. Choice (D) misstates the excited utterance exception, which does not apply if the statement is the result of reflection and deliberation. Choice (D) is, therefore, incorrect.
280
Q
  1. A commissioner of a sports league was charged with embezzling funds by increasing the amount of his allocated travel expenses without the approval of a majority of the team owners. In accordance with the bylaws of the league’s charter, the commissioner was required to obtain the approval of at least half the owners before raising his expense allocation.
    At trial, the commissioner seeks to testify that his predecessor told him that he had authority under the league’s charter to raise his expense allocation.
    The commissioner’s testimony is
    (A) admissible, to show that the commissioner lacked criminal intent.
    (B) admissible as evidence of the routine practice of an organization.
    (C) inadmissible as hearsay not within any recognized exception.
    (D) inadmissible, because the league’s charter is the best evidence.
A
  1. (A) This is a Multistate example of non-hearsay because the predecessor’s statement or utterance is being offered to show its effect on the commissioner, the listener. That is to say, the commissioner is attempting to prove that he acted honestly (without criminal intent) in increasing his expense allocation based upon the information provided him by the predecessor. Consequently, the commissioner’s testimony is not hearsay because its value does not rest upon the truth of the predecessor’s statement. Therefore, choice (C) is incorrect. On the contrary, the predecessor’s utterance is being offered only to show the effect it had on the commissioner’s subsequent conduct (i.e., he reasonably believed that he had the power as commissioner to raise his expense allocation). Choice (B) is incorrect because the foundational elements of FRE 406 on habit and routine practice are not met here: there is no evidence of what the organization “always” or “invariably” did. Choice (D) is wrong because the contents of a writing are not at issue here.
281
Q
  1. A defendant was arrested at an airport when the small suitcase he was carrying was found to contain heroin. The defendant, who did not challenge the legality of the airport search, was subsequently prosecuted for possession of heroin.
    At trial, the defendant testified on his own behalf and said that the suitcase belonged to his girlfriend who was accompanying the defendant when he was arrested. The girlfriend died in a skydiving accident two weeks before the defendant’s trial. Moreover, the defendant testified that although he was a former heroin addict, he had not used any heroin in the past three years. On cross-examination, the prosecuting attorney asked the defendant to roll up the sleeves of his shirt and exhibit his arms to see if there were any needle marks.
    This request is
    (A) objectionable, because the defendant has a privilege against self-incrimination.
    (B) objectionable, because the probative value is substantially outweighed by the danger of unfair prejudice.
    (C) permissible, because such evidence is relevant to the defendant’s credibility.
    (D) permissible, because the defendant waived his privilege against self-incrimination by taking the stand.
A
  1. (C) Choices (A) and (D) are wrong because the privilege against self-incrimination applies only to evidence that is testimonial in nature. Choice (C) is correct because the prosecuting attorney is attempting to attack the defendant’s credibility. According to FRE 806, “When a hearsay statement, or a statement defined in Rule 801(d) (2) has been admitted in evidence, the credibility of the declarant may be attacked, and if attacked may be supported, by any evidence which would be admissible for those purposes if declarant had testified as a witness.” In our case, the defendant testified that he “had not used any heroin for the past three years.” Consequently, it is proper for the prosecution to attack his credibility by seeing if there are any needle marks on his arms. Choice (B) is wrong. For most criminal defendants, the unfortunate reality of trial includes the introduction in evidence against them of highly prejudicial evidence. This fact pattern contains insufficient information to balance probative value and prejudicial impact.
282
Q
  1. A professional football player was seriously injured when the football helmet he was wearing shattered as he was being tackled during a game. The player subsequently asserted a claim against the manufacturer of the helmet to recover damages for his injury.
    At trial, the player’s attorney calls a former employee of the manufacturer to testify that three weeks after the player suffered his injury, the manufacturer devised a new design for its football helmets.
    The former employee’s testimony is
    (A) admissible as an admission.
    (B) admissible as circumstantial evidence that the player’s injury was traceable to the defective design of the football helmet.
    (C) inadmissible, because of the public policy behind taking safety precautions.
    (D) inadmissible, because the former employee is no longer authorized to speak on behalf of the manufacturer.
A
  1. (C) In accordance with FRE 407, “When, after an event, measures are taken which, if taken previously, would have made the event less likely to occur, evidence of the subsequent measures is not admissible to prove negligence or culpable conduct in connection with the event.” Choice (B) is wrong because, although some courts permit evidence of subsequent repairs in strict liability cases, this is not the majority rule. Furthermore, you should not assume that this is a strict liability action, since the facts simply state that the player is asserting a claim against the manufacturer. Quite conceivably, the player may be bringing suit for negligent manufacture or breach of warranty, in which case evidence of subsequent repairs is clearly inadmissible. Choice (A) is wrong because the former employee is no longer an employee of the company and cannot make vicarious admissions on its behalf under FRE 801(d) (2)(D). Choice (D) is incorrect because the former employee is testifying as a fact witness and is not speaking on behalf of the manufacturer.
283
Q
  1. A large man with red hair robbed a liquor store. Thereafter, a defendant was arrested and charged with the armed robbery. At the defendant’s trial, several eyewitnesses testified that they had seen a large redheaded man pull out a gun and rob the owner of the liquor store. The defendant appeared at trial with a shaven head. The prosecution calls a corrections officer to testify that the defendant had red hair when he was first brought to jail. The defendant’s counsel objects.
    The trial judge should rule the correction officer’s testimony
    (A) admissible as a prior identification. (B) admissible, for the limited purpose of clarifying the discrepancy in the witnesses’ testimony.
    (C) inadmissible as hearsay not within any recognized exception.
    (D) inadmissible, because it is opinion testimony.
A
  1. (B) Choice (B) is a better answer than choice (A) because the corrections officer’s testimony does not qualify as a prior identification. A prior identification requires that on a prior occasion, a statement of identification be made. For example, if A testifies that on a prior occasion B pointed to the accused and said, “That’s the man who robbed me,” the testimony, even though it is hearsay, would be admissible as a prior identification if B is present in court and available for cross- examination. Similarly, if B has himself testified to the prior identification. In either case, a prior statement of identification was made before trial. Here, on the other hand, no prior statement was made. Rather, the corrections officer is being called to testify at trial (for the first time) regarding the defendant’s appearance when he was arrested. Choice (C) is incorrect because no out-of-court statements are involved in the corrections officer’s testimony. Choice (D) is wrong. The corrections officer is testifying about a factual observation and not an opinion.
284
Q
  1. In a false imprisonment action by a plaintiff against a defendant, the plaintiff calls a witness to testify that the incident occurred on particular date. The witness was not questioned about an affidavit he made for trial in which he stated that the incident occurred on a different date. After the witness left the stand, but before he was excused, the defendant’s attorney offers into evidence the affidavit.
    Assuming that the affidavit is properly authenticated, the trial judge should rule it
    (A) admissible as substantive evidence.
    (B) admissible for impeachment purposes only.
    (C) inadmissible, because the defendant’s attorney failed to question the witness about the affidavit while on the stand.
    (D) inadmissible as hearsay not within any recognized exception.
A
  1. (B) In accordance with FRE 607, the credibility of a witness may be attacked by any party calling him (to testify). According to McCormick, there are five main lines of attacking the credibility of a witness. The first, and probably the most effective and most frequently employed, is an attack by proof that the witness, on a previous occasion, has made statements inconsistent with his present testimony. Therefore, choice (B) is the best answer. The second is an attack by a showing that the witness is biased. The third is an attack upon the character of the witness. The fourth is an attack by showing a defect of capacity in the witness to observe, remember, or recount the matters testified about. The fifth is proof by other witnesses that material facts are otherwise than as testified to by the witness. Choices (A) and (0) are wrong because, under the traditional view of hearsay, previous statements are inadmissible as substantive evidence (i.e., as evidence of what they state) because they constitute hearsay and are not within any exceptions to the hearsay rule. Choice (C) is incorrect because, under these facts, the witness has not been excused. Therefore, the witness can be afforded an opportunity to explain or deny the inconsistence under FRE 613.
285
Q
  1. In which of these situations are the communications set forth privileged?
    (A) An attorney was a member of a seven-attorney law firm. On a busy day, three of his partners had clients waiting to see them, and all of the chairs in the office waiting room were filled. A woman had an appointment with the attorney, and the attorney came out to greet her in the crowded waiting room. As the woman saw the attorney she said, “Am I glad to see you! I need a will made up right away, because I don’t want my rotten son to get any of my money. He will just throw it away at the race track.” The attorney replied, “Let’s go back to my office to discuss this matter.” The attorney prepared a will for the woman.
    (B) A woman arrived at a law office for an appointment and was escorted to her attorney’s office. She sat down and said to the attorney, “You know, I pay you a large retainer every year for your valued counsel. I need some advice now. There was a fire in my house yesterday. It was minor, but I’m covered for $15,000 under my fire insurance policy. I didn’t have any furniture in the room where the fire took place, but I want to put in a claim for a sofa and love seat. How should I go about it?” The attorney refused to give her any advice.
    (C) A woman engaged an attorney to represent her in adoption proceedings. During the course of his representation, she told the attorney some very personal facts about her lifestyle that the attorney needed in order to prepare a petition for adoption. The adoption took place, but the woman never paid her agreed fee to the attorney. The attorney sued the woman to collect his fee.
    (D) A woman consulted an attorney regarding a sale of 70% of the stock of a closely held corporation in which the woman was the majority shareholder. She explained the reason for the sale and divulged the financial condition of the corporation and its prospects for the future. The attorney determined that the transaction would be complicated. Since he felt he was inexperienced in handling such a transaction, the attorney decided to decline to represent the woman. The woman nevertheless sold her stock with the assistance of another attorney.
A
  1. (D) Under the FRE, there are no specific privilege provisions. FRE 501 provides that the privilege of a witness shall be governed by the principles of common law as they may be interpreted by the courts of the United States in light of reason and experience. However, in civil actions and proceedings, with respect to an element of a claim or defense as to which state law applies the rule of decision, the privilege of a witness shall be determined in accordance with state law. Thus, the common Law attorney- client privilege is applicable in our factual situations. The client has a privilege to refuse to disclose and to prevent other persons from disclosing any confidential communications between himself and his attorney. Choice (D) is the best answer, since the woman intended that her disclosures to the attorney about the corporation were confidential and in furtherance of the rendition of legal services that she sought from the attorney. McCormick, in The Handbook on Evidence, p. 179, states that “Communications in the course of preliminary discussion with a view to employing the lawyer are privileged though the employment is in the ‘upshot’ not accepted by the attorney. . . . Payment, or agreement to pay a fee, however, is not essential.” In choices (A), (B), and (C), the communications between the attorney and his client are not confidential and, therefore, not protected by the attorney- client privilege. In choice (A), the communication is made publicly and, therefore, there is no expectation of confidentiality. In choice (B), the communication was made in connection with the woman’s attempt to get the attorney to assist her in a fraud. Such statements are not confidential. In choice (C), the statements are not confidential because they were integrated into the pleadings of the case.
286
Q
  1. One afternoon, a pilot was flying a small airplane when it suddenly ran out of gas. As he was coming in for an emergency landing, the plane crossed into a neighboring state at a very low altitude. At this time, a 9-year-old boy was walking to school when he was struck and injured by an object, which may have fallen from the plane.
    In federal court, a negligence suit was brought against the pilot by the father of the boy for his son. Accompanied by his father, the boy had visited an attorney for preliminary discussions regarding the case. However, the father did not retain the attorney to represent his son in the lawsuit. Instead, the father hired another lawyer to handle the case.
    At trial, the pilot’s attorney calls the consulting attorney to testify what the boy had said to him regarding his physical condition during the consultation that the attorney had had with the boy and his father.
    The attorney’s testimony is
    (A) admissible, because the attorney-client privilege was waived by the filing of the lawsuit.
    (B) admissible, because there is no privilege of confidentiality when a person other than the client is present at the attorney-client consultation.
    (C) inadmissible, because the attorney-client privilege prevents such a breach of confidential communications.
    (D) inadmissible, because it was a statement of physical condition not made for the purpose of obtaining medical treatment.
A
  1. (C) The privilege for communications of a client with his attorney hinges upon the client’s belief that he is consulting a lawyer in that capacity and has manifested intention to seek professional legal advice. According to the hoLding in the case of In re Dupont’s Estate, 140 R2d 866 (1943), communications in the course of preliminary discussion with a view of employing the attorney are privileged even though the attorney is never hired. Choice (A) is wrong; the privilege cannot be defeated by the filing ofa lawsuit. Choice (B) is incorrect. The father’s position as father and guardian ad 1/tern of his minor child puts him on the same footing as the child for purposes of determining the existence of the privilege. Choice CD) is incorrect. Choice (0) misstates the medical hearsay exception, which permits the admissibility of information provided for the purpose of obtaining medical treatment.
287
Q
  1. A taxpayer was notified by the government that her individual income tax was underpaid by $1,012.69. The taxpayer retained an attorney to represent her in contesting the assessment. During the preparation of his client’s case, the attorney suggested that it might be a good idea if the taxpayer hired an accountant to organize her records and prepare a financial statement for the year in question. Following the attorney’s advice, the taxpayer consulted an accountant, an old family friend, who prepared a financial statement, which the attorney referred to at trial.
    During the trial, the attorney representing the government called the accountant to testif’ about statements that the taxpayer made to him.
    The accountant’s proposed testimony is
    (A) inadmissible, because it would violate the attorney-client privilege.
    (B) inadmissible, because it would violate the taxpayer’s privilege against self-incrimination.
    (C) inadmissible as violative of the work-product rule. (D) admissible as an admission.
A
  1. (0) In Fishery. United States, 425 U.S. 391 (1976), the court held that compliance with a subpoena for an accountant’s records (that defendant had given his attorney in preparation for trial) did not violate the defendant’s 5th Amendment privilege against self-incrimination. For this same reason, choice (B) is incorrect. Choices (A) and (C) are incorrect because the accountant was not a representative of the attorney. The facts do not state that the attorney hired the accountant. Rather, the accountant was employed by the taxpayer. A “representative of the lawyer” is one employed by the attorney to assist the attorney in the rendition of professional Legal services.
288
Q
  1. Late one afternoon, a woman was hitchhiking when she was picked up by a man. Shortly thereafter, the man stopped and parked his car in a roadside rest area. They were smoking marijuana when another car skidded and crashed into the man’s car. The collision damaged the man’s car and inflicted personal injuries upon him. The woman was likewise injured. In fact, the impact produced a state of unconsciousness in her that lasted several minutes.
    The woman sued the other driver seeking to recover for his alleged negligence. At trial, the woman testified to her injuries and to the other driver’s negligence. In defense, the other driver called an attorney who lived next door to the woman. The attorney proposed to testify that after the accident, the woman consulted him about her claim and asked the attorney in confidence how she could falsely testify that she wasn’t smoking marijuana at the time of the accident.
    Upon objection by the woman’s attorney, the attorney’s testimony is
    (A) admissible as an admission.
    (B) admissible as a statement against interest.
    (C) inadmissible, because it violates the woman’s privilege against self-incrimination.
    (D) inadmissible, because it violates the attorney-client privilege for confidential
    communications.
A
  1. (A) Under the attorney-client privilege, a client has a priviLege to refuse to disclose and to prevent any other person from disclosing confidential communications made for the purpose of facilitating the rendition of professional legal services to the client between himself (or his representative) and his attorney (or his attorney’s representative). However, there are exceptions when there is not privilege. For example, the privilege is lost if the services of the attorney were sought or obtained to enable or aid anyone to commit or plan to commit what the client knew or reasonably should have known to be a crime or fraud. Thus, choice (A) is correct, and choice (D) is incorrect because the woman sought the attorney’s advice about committing perjury or falsely testifying about the incident. Choice (B) is incorrect because there has been no showing of unavailability in this case. Choice (C) is incorrect. Voluntary statements made to others outside the courtroom are not covered by the privilege against self-incrimination.
289
Q
  1. A shopper sued a supermarket for injuries allegedly suffered from a collision with the store’s automatic doors. The shopper contended that the doors, which were programmed to swing inward, swung outward and injured him as he attempted to enter the store one afternoon. The shopper’s brother-in- law, who was an eyewitness to the accident, was not called to testify at trial. Moreover, the shopper’s attorney failed to depose the brother-in-law, who redomiciled out of state shortly after the accident.
    With respect to the shopper’s failure to offer the brother-in-law’s testimony at trial, on request by the supermarket’s attorney, the court should
    (A) instruct the jury that it raises the presumption that the brother-in-law’s testimony would have been unfavorable to the shopper.
    (B) instruct the jury that it constitutes an adoptive admission that the brother-in-law’s testimony would have been unfavorable to the shopper.
    (C) permit the supermarket’s attorney to argue that it raises a presumption that the brother-in-law’s testimony would have been unfavorable to the shopper.
    (D) neither instruct the jury on the matter nor permit the supermarket’s attorney to argue the matter.
A

i6o. (0) A popular Multistate testing area deals with whether the failure of a party to call a particular witness (or the failure to take the stand himself as a witness) allows his adversary to use this failure as a basis for invoking an adverse inference. Although there are a large number of cases supporting the inference, under the prevailing view, if the witness is “equally available” to both parties, no inference springs from the failure of either to call him. McCormick, Law of Evidence, pg. 657. McCormick points out that “the possibility that the inference may be drawn invites waste of time in calling unnecessary witnesses or in presenting evidence to explain whythey were not called.” Most important, McCormick notes that “the availability of modern discovery procedures serves to diminish both the justification and the need for the inference.” Law of Evidence, pg. 657. For this reason, answers (A), (B), and (C) are all incorrect.

290
Q
  1. During the murder trial of a defendant, the prosecution presented four witnesses to the brutal slaying of the victim. The evidence pointed to the fact that the defendant beat her about the head and neck with a baseball bat, causing severe injuries to her brain and her ultimate death.
    The prosecution rested, and the defendant presented two witnesses, his brother and his girlfriend, who testified that the defendant was dining at an elegant restaurant on the other side of town at the time of the alleged murder. The defendant presented no other witnesses.
    During his closing argument to the jury, the assistant district attorney called attention to the fact that the prosecution witnesses had no apparent reason to have any bias toward the prosecution or against the defendant. He then noted that the defendant’s witnesses had clear motives to falsify their testimony and favor the defendant. The assistant district attorney added, “If the defendant was on the other side of town, why didn’t he tell us himself? Why didn’t he get on the stand? What was he hiding? Those are questions for you, the jury, to answer.”
    The defendant was convicted of first-degree murder and sentenced to life imprisonment.
    On appeal, his conviction should be
    (A) reversed, because the prosecutor improperly referred to the possible motives or interests of the defense witness.
    (B) reversed, because the defendant’s constitutional rights were violated during the closing argument.
    (C) reversed, because the assistant district attorney referred to the defendant’s failure to testify.
    (D) reversed, because the assistant district attorney’s argument violated the defendant’s rights under the Fifth and Fourteenth Amendments.
A

i6i. (D) In Griffin v. California, 380 U.S. 609 (1965), the U.S. Supreme Court held that the self-incrimination guarantee of the 5th Amendment, as applicable to the states under the 14th Amendment, forbids either comment by the prosecution of an accused’s silence or instructions by the court that such silence is evidence of guilt. The comments by the prosecutor during his closing argument that the defendant failed to take the stand would be violative of the defendant’s right against self- incrimination. Choices (B) and (C) are also correct, but choice (D) is a better answer because it is more specific and on point to the facts of the question. Choice (A) is incorrect. The motives and bias of a party’s witnesses are fair game in closing argument.

291
Q
  1. A woman sued a man for personal injuries that she suffered when she was struck by the man’s car. The man’s car hit the woman as she was walking across the street. Immediately after the accident, the man ran over to the woman and said, “I know I was driving fast, but you weren’t paying attention where you were walking. Anyhow, I’m willing to pay all your medical expenses.”
    At trial, the woman calls an eyewitness to the accident. The eyewitness proposes to testify that he heard the man tell the woman, “I know I was driving fast.” The man’s attorney objects.
    If the eyewitness’s testimony is admitted, it will most likely be because the proffered evidence is
    (A) admissible as an opinion.
    (B) admissible as an admission.
    (C) admissible as a present sense impression.
    (D) admissible as a declaration against interest.
A
  1. (B) If the eyewitness’s testimony is admitted, it will most likely qualify as an admission. According to FRE 409, “Evidence of furnishing or offering or promising to pay medicaL, hospital, or similar expenses occasioned by an injury is not admissible to prove liability for the injury.” As is so often the case on the MBE, an admission may be made in connection with an offer or promise to pay medical or hospital expenses. In this instance, the admission of liability is admissible, but evidence of offering or promising to pay medical (or similar) expenses is inadmissible, in accordance with public policy considerations. Choice (A) is incorrect. The statement, even if it is in the form of an opinion, is an admission by a party opponent. Choice (C) is wrong because the statement does not “describe or explain an event or condition made while the decLarant was perceiving the event or condition, or immediately thereafter.” Choice (D) is wrong because there is no indication of the man’s unavailability at trial.
292
Q
  1. One day, while visiting a plastic surgery clinic for a routine Botox treatment, a patient received an excessive dose of Botox from a new medical assistant on the staff. Her face frozen in a grotesque grimace, she demanded to speak to the doctor. The doctor said, “Listen, why don’t you go over to the emergency room and send the bills to me? I’ll take care of your medical expenses.” The patient subsequently sued the doctor for negligent hiring of the medical assistant and for damages. At trial, she proposes to call the doctor’s administrative assistant, who was present during her conversation with the doctor. The patient wants the administrative assistant to testify to the doctor’s offer to pay her medical bills.
    The doctor’s attorney objects. The trial judge should
    (A) sustain the objection, because the patient’s medical records are the best evidence of the doctor’s negligence.
    (B) sustain the objection as an offer to pay the medical bills.
    (C) overrule the objection, because the evidence is relevant as to the question of the doctor’s negligent hiring.
    (D) overrule the objection, because an offer to pay medical bills is an inferential admission.
A
  1. (B) As noted previously, evidence of payment (or offering or promising to pay) of medical, hospital, or similar expenses of an injured party by the opposing party, is not admissible. The reason often given is that such payment or offer is usually made from human impulses and not from an admission of Liability, and that to hold otherwise would tend to discourage assistance to the injured person. Choice (B) is correct because the aforesaid rule is often phrased in terms of “humanitarian motives.” See FRE 409 Advisory Committee’s Note. For the same reasons, choice (D) is incorrect. Choice (A) is wrong because it incorrectly states the best evidence rule. Choice (C) is incorrect. The doctor’s offerto pay medical bills might be relevant to the issue of negligent hiring, but this tenuous connection will be trumped by the policy considerations of FRE 409.
293
Q
  1. A surgeon performed an appendectomy on a patient in an operating room at the local hospital. The surgeon was assisted by a nurse who was assigned to the operation by the hospital. During the patient’s early convalescence, he complained of pain not explicable as an ordinary post-operative symptom. On investigation, it turned out that the surgeon, who had bandaged him following the operation, had done so in such a manner as to constrict certain blood vessels. The faulty bandaging had caused acute pain and retarded the patient’s recovery.
    After the patient’s eventual recovery, he sued the surgeon for malpractice, claiming $25,000 in damages. In his case-in-chief, the patient called the nurse to testifS’ that shortly after the surgery, she saw the surgeon destroy the postoperative x-rays of the patient’s abdomen.
    Upon objection by the surgeon’s attorney, the trial judge should rule the nurse’s testimony
    (A) admissible, provided that the judge determines that the surgeon destroyed the x-rays as a cover-up.
    (B) admissible, but leave the weight of the nurse’s testimony to be determined by the jury.
    (C) inadmissible, because the probative value is substantially outweighed by the danger of unfair prejudice.
    (D) inadmissible, because it is extrinsic evidence of a collateral matter.
A
  1. (B) The nurse’s testimony as to seeing the doctor destroy the postoperative x-rays of the patient’s abdomen should be admissible. Since the patient’s suit for malpractice is based on the faulty bandaging following the operation, the postoperative x-rays would be relevant, not collateral, to show the effects of the constricted blood vessels on the patient’s retarded recovery, under FRE 401, relevant evidence is that which tends to make the existence of a fact of consequence more (or less) probable than it would otherwise be. ReLevant evidence is generally admissible under FRE 402. Once admitted, the jury then determines the weight of the evidence. Choice (B) is the correct answer, since the relevance of the postoperative x-rays goes to the malpractice issue being litigated, namely the patient’s postoperative pain and slow recovery. Proof of a cover-up is not a foundational prerequisite to admissibility. Choice (C) is incorrect, since the probative value of the nurse’s testimony far outweighs the danger of unfair prejudice; testimony regarding destruction of the x-rays will not confuse the issues, mislead the jury, or cause undue delay. FRE 403. Choice CD) is incorrect because the destruction of the x-rays is not a collateral matter.
294
Q
  1. A plaintiff is the beneficiary of a policy issued by an insurance company insuring the life of his wife, now deceased. The policy contains a clause providing that double indemnity is payable in the event that death of the insured “results directly, and independently of all other causes, from bodily injury effected solely through external violent and accidental means.”
    The plaintiff’s wife met her death in the silage shed of her farm. The death resulted from wounds caused by the discharge of a double-barreled shotgun. The plaintiff was arrested and prosecuted for the murder of his wife. After a lengthy trial, the plaintiff was acquitted of the charge.
    After the insurance company refused to pay the plaintiff’s insurance claim, the plaintiff instituted this civil action. The complaint sets forth the policy, alleges the facts surrounding the death of the insured, avers that the death was within the policy provisions for double indemnity payment, and demands judgment accordingly.
    At trial, the insurance company’s attorney proposes to introduce excerpts from the plaintiff’s murder trial. The plaintiff’s attorney objects to the introduction of such evidence.
    The trial judge should rule the proffered evidence
    (A) admissible as a declaration against interest.
    (B) admissible as former testimony.
    (C) inadmissible, because of collateral estoppel.
    (D) inadmissible, because of double jeopardy.
A
  1. (B) This is an extremely tricky Evidence example. Many students will recognize that unavailability (as a witness) is a requirement for the former testimony exception to the hearsay rule. Although this is true, former testimony may often be given in evidence without meeting the requirements of unavailability and confrontation. McCormick states that these requirements are applicable only when admission of the evidence is sought under the hearsay exception. However, when the former testimony is offered for some non-hearsay purpose, as to show the commission of the act of perjury, to refresh recollection, or impeach a witness at the present trial by proving that he testified differently on a former occasion, the restrictions of the hearsay exception do not apply. See McCormick, pg. 615. Since all other choices are clearly inapplicable, choice (B) is the only conceivably correct answer.
295
Q
  1. A rescuer was driving on an isolated portion of a country road. His headlights caught a figure lying at the side of the road. The rescuer stopped to investigate and found a victim, who was bleeding from head wounds and appeared to have been severely beaten.
    The rescuer then lifted the victim into his car and drove her to the hospital, a half-hour trip. When they arrived at the hospital, the rescuer carried the victim into the emergency room. He left her with a nurse and then returned home. Although the victim recovered from her injuries, she sued the hospital for malpractice, claiming that she was not promptly given medical attention.
    At trial, the nurse proposes to testify that when the victim was first brought to the hospital, she was unconscious. The victim’s attorney objects and moves to strike the nurse’s testimony.
    The trial judge should
    (A) sustain the objection, because it goes to an ultimate issue in the case.
    (B) sustain the objection, because the nurse is not qualified to render an expert opinion.
    (C) overrule the objection, because it is a shorthand rendition of what she observed.
    (D) overrule the objection, because there are independent grounds to show a present sense impression.
A

i66. (C) According to FRE 701, “If the witness is not testifying as an expert, his testimony in the form of opinions or inferences is limited to those opinions or inferences which are (a) rationally based on the perception of the witness and (b) helpful to a clear understanding of his testimony or the determination of a fact in issue.” Choice (C) is correct because the federal rules have adopted the so-called shorthand rendition rule, which incorporates the more liberal notion of sanctioning the admission of opinions on grounds of “expediency” and “convenience,” rather than “necessity.” Choice (A) is incorrect because an opinion witness can testify to the ultimate issue in a case under FRE 704. Choice (B) is incorrect. The nurse is not offering an expert opinion but, rather, an opinion within the province of lay witnesses. Choice (D) is incorrect. The nurse is not testifying to an out-of-court statement by a hearsay declarant, but rather to her own observations and conclusions.

296
Q
  1. A doctor is charged with the murder of his wife. The prosecution alleges that he murdered his wife by giving her a massive injection of succinylcholine while she was asleep. Succinylcholine is a drug used in small quantities by anesthesiologists as a muscle relaxant. The prosecution claims that the fatal dose given to the wife so totally relaxed her lung muscles that she suffocated.
    During the trial, a toxicologist, who had no previous knowledge of the case, sat in court and heard all of the evidence about the wife’s death and autopsy.
    As part of the doctor’s defense, his attorney calls the toxicologist to give his opinion on the cause of the wife’s death.
    May the toxicologist so testify?
    (A) Yes, because he can identify the data upon which his opinion is based.
    (B) Yes, because an expert may base his opinion on facts made known to him at the trial.
    (C) No, because he has no personal knowledge of the wife’s death.
    (D) No, because the cause of death is an issue to be decided by the jury.
A
  1. (B) FRE 703 provides that the “facts or data in the particular case upon which an expert bases an opinion or inference may be those perceived by or made known to him at or before the hearing.” Since an expert may base his opinion on facts made known to him at the trial, choice (B) is correct. Choice (A) is incorrect because FRE 705 states that an expert can testify to her opinion or inferences without having to first testify to the underlying facts or data. Choice (C) is incorrect. Under FRE 703, an expert is not required to have firsthand knowledge of the case. Finally, FRE 704 permits expert opinion on the ultimate issue in a case. Therefore, choice (D) is an incorrect answer.
297
Q
  1. For nearly three months, a supermarket underwent extensive renovations. The store was temporarily closed during the renovation period. The day the supermarket reopened, the store manager noticed that small fragments of plaster had fallen from a section of the ceiling. He promptly posted signs warning shoppers of the hazardous condition. The signs, which were printed in bold letters, read: “ATTENTION SHOPPERS - BE ON THE LOOKOUT FOR FALLiNG PLASTER.”
    That same afternoon, a shopper was shopping in the supermarket and noticed the signs. She looked at the ceiling but didn’t see any plaster falling. Moments later, she was placing some squash in a bag when a section of the ceiling suddenly fell on her head. She suffered a concussion and head lacerations.
    Thereafter, the shopper brought a tort action against the supermarket to recover for the injuries she suffered. Her attorney hired a physician to examine the shopper in order to assist the attorney in preparing the case.
    At trial, the supermarket’s attorney calls the physician that the shopper’s attorney hired as a witness and seeks to ask the physician about statements concerning the injuries that the shopper had made to the physician in confidence and that the physician had in turn communicated to her attorney.
    The physician’s testimony should be
    (A) admitted, because the shopper waived the physician-patient privilege by placing her physical condition in issue.
    (B) admitted, because the shopper’s statements are deemed admissions of a party-opponent.
    (C) excluded, because the shopper’s statements are protected by the physician-patient privilege.
    (D) excluded, because the shopper’s statements are protected by the attorney-client privilege.
A

i68. (D) Choices (A) and (C) are both incorrect, as the physician is an attorney representative and not the shopper’s physician. Choice (D) is the best answer because the physician is a “representative” of the shopper’s attorney and, therefore, the communications between the shopper and the physician will be privileged under the attorney-client privilege. Choice (B) is wrong because the shopper’s statements describe her injuries and are not admissions.

298
Q
  1. A defendant was charged with killing a victim during a barroom fight. The defendant claimed that he acted in self-defense when he was attacked by the victim. At trial, the defendant called a witness who testified that he witnessed the altercation and that the victim was the aggressor. The witness further testified that he has known the defendant for 10 years and that the defendant is a peaceable man who was acting reasonably in self-defense. The prosecution then offered the testimony of the victim’s brother-in-law, who proposed to testify that the victim was a peaceable and law-abiding person.
    Upon objection by the defendant’s attorney, the brother-in-law’s proposed testimony is
    (A) admissible, for the purpose of rebutting the contention that the defendant was a peaceable person.
    (B) admissible, for the purpose of repudiating the defendant’s claim of self-defense.
    (C) inadmissible, because the victim’s character cannot be proved by the brother-in-law’s testimony.
    (D) inadmissible, because the defendant did not directly attack the victim’s character.
A
  1. (B) When the accused has produced evidence that the deceased attacked him, thus grounding a claim of self-defense, this, when met by counter-evidence, raises an issue of conduct: Was the deceased or the accused the first aggressor? It is universally held that when such evidence has been produced, the accused may introduce testimony of the reputation of the deceased for turbulence and violence. Moreover, McCormick points out that “it is equally well settled of course that the prosecution may meet this by rebutting testimony of his (the victim’s) good reputation for peacefulness.” Evidence, pp. 460—461. Thus, choice (B) is correct. Choice (A) is incorrect because evidence of the victim’s peaceful character does not rebut the testimony that the defendant is a peaceful person. Choice (C) is incorrect. According to FRE 405, reLevant character evidence can be established by reputation or opinion testimony, as was done in this case. Choice (D) is incorrect. In order for the prosecution to introduce evidence of the victim’s peaceful character in a homicide case, there is no requirement that the defense directly attack the victim’s character. Instead, it is enough for the defense to introduce evidence that the victim was the first aggressor.
299
Q
  1. A man claims to have inherited property from a wealthy businessman. The man’s birth certificate shows that he was born out of wedlock. The record of a proceeding for a support order shows that the businessman was adjudged to be father of the man. However, the man can establish his status as heir of the businessman only if he was legitimated by the marriage of his mother and the businessman subsequent to his birth. Thus, the man must prove that this marriage occurred.
    The man’s attorney has not been able to discover any marriage license or certificate. However, the attorney does have a photograph that shows a couple dressed in wedding clothes. The scene is the front of a church. Bystanders are seen to be throwing rice at the couple and they seem to be responding by smiling and bowing down. The attorney was given the photograph by the man. He found it in his mother’s effects following her death. The man believes that the bride in the picture is his mother. He cannot identify the groom. The attorney was informed by a former acquaintance of the businessman who has seen the snapshot that he thinks the groom is the businessman.
    If the attorney seeks to introduce the photograph as proof that the mother and the businessman were married, the trial judge should rule the photograph
    (A) admissible, only if the photographer is available to testify concerning the circumstances under which the photograph was taken.
    (B) admissible, only if a witness verifies that it is a correct and accurate representation of the relevant facts.
    (C) inadmissible as non-verbal hearsay not within any recognized exception.
    (D) inadmissible as not the best evidence.
A
  1. (B) McCormick instructs thatthe principle upon which photographs are most commonly admitted into evidence is the same as that underLying the admission of illustrative drawings, maps, and diagrams. Under this theory, a photograph is viewed merely as a graphic portrayal of oraL testimony, and becomes admissible only when a witness has testified that it is a correct and accurate representation of relevant facts personally observed by the witness. Under this theory, the witness need not be the photographer; therefore, choice (A) is incorrect. Choice (C) is an incorrect statement of law and is, therefore, a wrong answer. Choice (D) is the wrong answer because the photograph itself is being introduced into evidence. Therefore, the best evidence rule is not implicated.
300
Q
  1. A defendant was charged with attempting to possess and distribute narcotics. The defendant was arrested after allegedly trying to purchase several kilos of heroin from an undercover government agent. At trial, the undercover agent testified about the defendant’s efforts to arrange the drug transaction. The defendant’s defense, on the other hand, consisted entirely in attacking the undercover agent’s credibility.
    During his summation, the prosecuting attorney, over defendant’s objections, stated that he had produced the only witness who was willing to take the stand and testify to what occurred at the meeting when the undercover agent arrested the defendant. Based on the undercover agent’s uncontradicted testimony, the prosecuting attorney urged the jury to convict the defendant. The defendant was convicted and, on appeal, argued that the judge erred in permitting the prosecutor’s statement.
    The defendant’s conviction should be
    (A) reversed, because the prosecutor’s remarks constituted improper comment about the defendant’s right to testify.
    (B) reversed, because the prosecutor’s remarks were not proper rebuttal to defense attacks on the undercover agent’s credibility.
    (C) affirmed, because it is immaterial; the probative value of the defendant’s failure to testify is outweighed by the danger of unfair prejudice.
    (D) affirmed, because the prosecutor had the right to express the strength of the evidence he had pursued.
A
  1. (A) Under the prevailing view, the failure of a defendant to take the stand himself as
    a witness does not allow his adversary to use this failure as a basis for invoking
    an adverse inference. See McCormick, Law of Evidence, pg. 656. Choice (B) is the
    wrong answer because the issue here concerns improper comment on the defendant’s failure to testify and not the proper scope of rebuttal evidence. Choice (C)
    is incorrect as a matter of Law. Choice (D) is incorrect. Prosecutors may strike hard
    blows, but they must be fair ones.
301
Q
  1. A national association brought suit in federal court against a city. The suit charged the city with violating several federal statutes. During the trial, the association’s attorney called the city attorney as an adverse witness. The city attorney asserted the attorney-client privilege and objected that the matters communicated to him were intended to be confidential.
    The issues raised by the objections of the city attorney should be resolved under
    (A) federal privilege law.
    (B) the privilege law of the forum state.
    (C) either federal privilege law or the privilege law of the forum state as determined by the court.
    (D) either federal privilege law or the privilege law of the forum state, whichever the court determines is more likely to admit the evidence.
A
  1. (A) FRE 501 states that “except as otherwise required by the Constitution of the United
    States or provided by Act of Congress or in ruLes prescribed by the Supreme Court
    pursuant to statutory authority, the privilege of a witness, person, government, State
    or political subdivision thereof shall be governed by the principles of the common
    law as interpreted by the courts of the United States in the light of reason and experience.” In other words, on matters governed by federal substantive law—and this
    will generally be true in criminal cases, civiL actions brought by the United States, and private federal question cases—federal courts are to apply and develop a federal common law of privilege. Since this question deals with a federal question, the issues should be resolved under federal privilege law. Choice (A) is, therefore, correct. Choices (B) and (C) are wrong because they state the wrong rule about the choice of privilege law. Choice (D) is incorrect in its suggestion that a court should select evidentiary law based on its greater likelihood to ensure the admission of evidence at trial.
302
Q

x173. Two defendants were prosecuted for conspiracy to rob a bank. An undercover officer of the local police department was the most important government witness. The defendants’ principal defense was entrapment.
The undercover officer testified for the government that he was present at a meeting with the defendants during which they plotted to rob the bank. He further testified that the idea of robbing the bank had first been suggested by one of the defendants, and that afterward, the undercover officer stated that he thought it was a good idea.
Thereafter, the defendants’ counsel called a witness who testified that the undercover officer had a bad reputation for truthfulness. The defense then called a second witness who proposed to testify that the undercover officer once perpetrated a hoax on the police department and was reprimanded.
The second witness’s proposed testimony is
(A) admissible, because the hoax resulted in a reprimand of the undercover officer.
(B) admissible, because a hoax is probative of the undercover officer’s untruthfulness.
(C) inadmissible, because it is essentially cumulative impeachment.
(D) inadmissible, because it is extrinsic evidence of a specific instance of misconduct.

A
  1. (D) FRE 608(b) provides that “specific instances of the conduct of a witness, forthe purpose of attacking or supporting his credibility, otherthan conviction of crime as provided in FRE 609, may not be proved by extrinsic evidence.” Choice (A) is incorrect. The fact pattern tells us nothing about the maximum penalty for a hoax. Furthermore, according to FRE 609(a)(1), evidence of an accused’s convictions for serious offenses must be more probative than prejudicial to be admitted. The exception for crimes involving deception and false statement is narrowly interpreted, and it is not at all clear whether perpetrating a hoax would qualify. Choice (B) is also incorrect because it is not necessarily true that a hoax is probative of untruthfulness. Finally, choice (C) is the wrong answer because it is not likely that two witnesses would be considered cumulative. Under FRE 403, a judge can exclude the needless presentation of cumulative evidence.
303
Q
  1. One morning, a woman telephoned her next-door neighbor and asked if she could borrow her car. The woman explained that her car was being serviced and would not be ready for a couple of days. The woman told her neighbor that she had a doctor’s appointment that afternoon and would return the car immediately afterwards. The neighbor agreed and gave the woman permission to use her car. As the woman was driving to her doctor’s office, she collided with another car.
    As a result of the accident, the other driver brought suit against the woman and the neighbor to recover for her personal injuries. The other driver asserted a claim against the neighbor for negligent entrustment of an automobile and charged the woman with negligent operation of a motor vehicle.
    In her case-in-chief, the other driver called a witness to testify to three incidents of careless driving on the woman’s part during the past six months.
    The trial judge should rule the testimony
    (A) admissible as circumstantial evidence that the woman was negligent on this occasion.
    (B) admissible, because the witness had personal knowledge of the woman’s poor driving record.
    (C) admissible against the neighbor as evidence of the woman’s lack of fitness.
    (D) inadmissible, because specific acts are not admissible except to rebut evidence of good character.
A
  1. (C) According to FRE 405(b), “In cases in which character or a trait of character of a person is an essential element of a charge, claim, or defense, proof may also be made of specific instances of his conduct.” Therefore, choice (D) is incorrect. In connection with FRE 405 are situations where the owner of a motor vehicle or any dangerous object is charged with liability for the acts of a person using it on grounds of negligent entrustment of the vehicle or object to an incompetent or unfit person. In such cases, the character of the custodian is in issue and his acts come in to show
    it. Thus, the witness’s testimony is admissible to show the neighbor’s negligence (namely, that she was aware or should have been aware that the woman was a careless driver) in entrusting her vehicle to the woman. Therefore, choice (C) is the correct answer. Choice (A) is incorrect. Character evidence is propensity evidence, not circumstantial evidence. Choice (B) is incorrect. Personal knowledge alone is insufficient to admit character evidence at trial.
304
Q
  1. A college student owns an expensive sports car. His friend called him up one afternoon and asked to borrow his car for a date he had that night. The college student generously lent the car to his friend. On his way home from the date, basking in the memories of a beautiful evening, the friend lost control of the car and hit a pedestrian.
    As a result of the accident, and after learning that the friend had no money and no insurance, the pedestrian brought suit against the college student to recover for his injuries. He asserted a claim against the college student for negligent entrustment of a vehicle. During his case, the pedestrian introduced evidence that the friend had been in several previous accidents. In his defense, the college student offered the testimony of his girlfriend that the college student frequently lent his car to his friend. The girlfriend further testified that the college student gave his friend permission to use his car only after determining that the friend was a careful and attentive driver.
    The girlfriend’s testimony is
    (A) admissible as evidence of habit.
    (B) admissible as a present sense impression.
    (C) inadmissible, because it goes to the college student’s character.
    (D) inadmissible, because she is biased.
A
  1. (C) This is a highly tested Multistate area dealing with negligent entrustment. Where the owner of a motor vehicle or any dangerous object is charged with Liability for the acts of a person using it on grounds of negligent entrustment, the character of the custodian is in issue, and his acts come in to show it. Such character evidence is probative on the issue of whether the entrustor was negligent in lending the instrumentaLity to the custodian or entrustee. Note, however, that the entrustor’s character is not in issue. Therefore, the college student’s character is not in issue, since he is the person making the entrustment. That’s why character evidence is not admissible in this question. Choice (A) is wrong because habit evidence applies to a person’s regular response to a repeated specific situation. The words “always,”“invariably,”“automatically,” or “without fail” will tip off a habit issue. Conversely, “frequently” or “generaLly” will not. Choice (B) is wrong because the testimony describes past acts directly observed by the witness and is not a hearsay statement. Finally, choice (D) is incorrect. Biased evidence is not per se inadmissible. In an adversarial system, it is up to the advocate to convince the fact- finder that a biased witness is less worthy of belief.
305
Q

In a civil action for breach of an oral contract, the defendant denied having entered into a contract with the plaintiff, although he admitted that he had discussed doing so. Which of the following standards of admissibility should the court apply to evidence proffered as relevant to prove whether a contract was formed?

(A) Whether a reasonable juror would find the evidence determinative of whether the contract was or was not formed.
(B) Whether the evidence has any tendency to make the fact of contract formation more or less probable than without the evidence.
(C) Whether the evidence is sufficient to prove, absent contrary evidence, that the contract was or was not formed.
(D) Whether the evidence makes it more likely than not that a contract was or was not formed.

A

(B) Whether the evidence has any tendency to make the fact of contract formation more or less probable than without the evidence.

306
Q

In a prosecution for aggravated battery, a police officer testified that when he arrested the defendant, he took a knife from the defendant and delivered it to the medical examiner. The medical examiner testified that the knife blade was consistent with the victim’s wound but admitted on cross-examination that any number of other knives could also have caused the wound. Should the judge grant a motion to strike the medical examiner’s testimony?

(A) No, because the probative worth of this evidence is for the jury to assess.
(B) Yes, because in light of the medical examiner’s admission, his testimony has insufficient probative value.
(C) Yes, because the medical examiner could not state the probability that the wound was caused by the defendant’s knife.
(D) Yes, because the probative value is substantially outweighed by the danger of unfair prejudice.

A

(A) No, because the probative worth of this evidence is for the jury to assess.

307
Q

A defendant’s house was destroyed by fire, and she was charged with arson. To prove that the defendant had a motive to burn down her house, the government offered evidence that the defendant had fully insured the house and its contents. Should the court admit this evidence?

(A) No, because the probative value of the evidence of insurance on the issue of whether the defendant intentionally burned her house down is substantially outweighed by the dangers of unfair prejudice and confusion of the jury.
(B) No, because evidence of insurance is not admissible on the issue of whether the insured acted wrongfully.
(C) Yes, because evidence of insurance on the house has a tendency to show that the defendant had a motive to burn down the house.
(D) Yes, because any conduct of a party to the case is admissible when offered against the party.

A

(C) Yes, because evidence of insurance on the house has a tendency to show that the defendant had a motive to burn down the house.

308
Q

A defendant is on trial for attempted fraud. The state charges that the defendant switched a price tag from a cloth coat to a more expensive fur-trimmed coat and then presented the latter for purchase at the cash register. The defendant has testified in her own behalf that someone else must have switched the tag. On cross-examination, the prosecutor asks whether the defendant was convicted on two prior occasions of misdemeanor fraud of a retailer by the same means of switching the price tag on a fur-trimmed coat. Is the question about the prior convictions proper either to impeach the defendant or to prove that the defendant committed the crime?

(A) It is not proper for either purpose.
(B) It is proper for both purposes.
(C) It is proper to impeach the defendant, but not to prove that the defendant committed the crime.
(D) It is proper to prove that the defendant committed the crime, but not to impeach the defendant

A

(B) It is proper for both purposes.

309
Q

At a defendant’s trial for a gang-related murder, the prosecution introduced, as former testimony, a statement by a gang member who testified against the defendant at a preliminary hearing and has now invoked his privilege against self-incrimination. If the defendant now seeks to impeach the credibility of the gang member, which of the following is the court most likely to admit?

(A) Evidence that the gang member had three misdemeanor convictions for assault.
(B) Testimony by a psychologist that persons with the gang member’s background have a tendency to fabricate.
(C) Testimony by a witness that, at the time the gang member testified, the gang member was challenging the defendant’s leadership role in the gang.
(D) Testimony by a witness that the gang member is a cocaine dealer.

A

C) Testimony by a witness that, at the time the gang member testified, the gang member was challenging the defendant’s leadership role in the gang.

310
Q

In a civil trial for fraud arising from a real estate transaction, the defendant claimed not to have been involved in the transaction. The plaintiff called a witness to testify concerning the defendant’s involvement in the fraudulent scheme. To the plaintiff’s surprise, however, the witness testified that the defendant was not involved and denied having made any statement to the contrary. The plaintiff has now called a second witness to testify that the first witness had stated, while the two were having a dinner conversation, that the defendant was involved in the fraudulent transaction. Is the testimony of the second witness admissible?

(A) No, because a party cannot impeach the party’s own witness.
(B) No, because it is hearsay not within any exception.
(C) Yes, but only to impeach the first witness.
(D) Yes, to impeach the first witness and to prove the defendant’s involvement.

A

(C) Yes, but only to impeach the first witness.

311
Q

A defendant was charged with aggravated assault. At trial, the victim testified that the defendant beat her savagely, but she was not asked about anything said during the incident. The prosecutor then called a witness to testify that when the beating stopped, the victim screamed: “I’m dying—don’t let [the defendant] get away with it!” Is the testimony of the witness concerning the victim’s statement admissible?

(A) No, because it is hearsay not within any exception.
(B) No, because the victim was not asked about the statement.
(C) Yes, as a statement under belief of imminent death, even though the victim did not die.
(D) Yes, as an excited utterance.

A

(D) Yes, as an excited utterance.

312
Q

At the trial of a contract dispute, the plaintiff has offered to testify to what she heard the defendant say in a private conversation between the two of them, which the plaintiff secretly recorded on an audiotape that she did not offer in evidence. Is the plaintiff’s testimony admissible?

(A) Yes, because the plaintiff has personal knowledge of the statement of a party-opponent.
(B) Yes, because the original document rule does not apply to audiotapes.
(C) No, because the statement must be proved by introduction of the audiotape itself.
(D) No, because of the plaintiff’s deception, even if the recording was not illegal.

A

(A) Yes, because the plaintiff has personal knowledge of the statement of a party-opponent.

313
Q

A plaintiff sued her employer, alleging that poor working conditions had caused her to develop a stomach ulcer. At trial, the plaintiff’s medical expert testified to the cause of the plaintiff’s ulcer and stated that his opinion was based in part on information in a letter the plaintiff’s personal doctor had written to the plaintiff’s employer, explaining why the plaintiff had missed work.
When offered to prove the cause of the plaintiff’s condition, is the letter from the plaintiff’s doctor admissible?

(A) No, because it is hearsay not within any exception.
B) No, because the plaintiff’s doctor is not shown to be unavailable.
(C) Yes, because it was relied upon by the plaintiff’s medical expert.
(D) Yes, under the business records exception to the hearsay rule.

A

(A) No, because it is hearsay not within any exception.

314
Q

A plaintiff sued a defendant for wrongful death arising out of a traffic collision between the plaintiff’s decedent and the defendant. At trial, the investigating police officer authenticated a tape recording of her shift-end dictation of comments used in preparing the written report of her factual findings. She has testified that the tape recording was accurate when made and that she currently has no clear memory of the details of the investigation.
Is the tape recording admissible as evidence?

(A) Yes, under the recorded recollection exception to the hearsay rule.
(B) Yes, under the public records exception to the hearsay rule.
(C) No, because it is hearsay and is a police report being offered against the defendant in a wrongful death case.
(D) No, because the police report itself is the best evidence.

A

(A) Yes, under the recorded recollection exception to the hearsay rule.

315
Q

A defendant was charged with conspiracy to possess cocaine with
intent to distribute. While on bail with travel restricted to his home
state, he purchased an airplane ticket to another country by using an
alias. At trial, the prosecution seeks to introduce evidence of the
defendant’s ticket purchase. Should the court admit this evidence?

A) No, because the evidence does not make any fact of consequence
to the trial more or less probable than it would have been without
the evidence.
(B) Yes, because the evidence is relevant both to show the
defendant’s consciousness of guilt and to show his motive to commit
the crime.
(C) Yes, because the evidence is relevant to show the defendant’s
consciousness of guilt.
(D) Yes, because the evidence is relevant to show the defendant’s
motive to commit the crime.

A

(C) Yes, because the evidence is relevant to show the defendant’s
consciousness of guilt.

316
Q

A woman has sued a man for defamation, alleging that he sent a letter to her employer making false and injurious statements about her. At trial, she proposes to testify about the statements in the letter but does not produce the letter itself or show that it is unavailable. Should the court allow the woman’s testimony about the statements in the letter?

(A) No, because the statements were made out of court and are
hearsay not within any exception.
(B) No, because the best evidence rule requires an original or
duplicate of the letter to prove the letter’s contents.
(C) Yes, because the statements are not being offered for their truth
and thus are not hearsay.
(D) Yes, because the statements are those of a party-opponent and
therefore are admissible under a hearsay exception.

A

(B) No, because the best evidence rule requires an original or duplicate of the letter to prove the letter’s contents.

317
Q

A defendant was charged with murder. At trial, the prosecutor called a witness who testified that a friend had told him that there was “bad blood” between the defendant and the victim. After the witness was excused, the defense
attorney moved to strike the portion of the testimony that included the friend’s statement, arguing that the statement was inadmissible. Has the issue of admissibility been preserved for appeal?

(A) No, because an objection must be made before the evidence is
presented to the jury.(B) No, because the defense failed to make a timely objection stating
the specific ground for objection.
(C) Yes, because admission of the statement affected the defendant’s
right to confront witnesses.(D) Yes, because the defense objected to the statement during the
trial.

A

(B) No, because the defense failed to make a timely objection stating the specific ground for objection.

318
Q

D is charged with possession of heroin. Prosecution witness W, an
experienced dog trainer, testified that he was in the airport with a
dog trained to detect heroin. As D approached, the dog
immediately became alert and pawed and barked frantically at
D’s briefcase. D managed to run outside and throw his briefcase
into the river, from which it could not be recovered. After W’s
experience is established, he is asked to testify as an expert that
the dog’s reaction told him that D’s briefcase contained heroin.
W’s testimony is;
(A) admissible, as evidence of Dahle’s guilt.
(B) admissible, because an expert may rely on hearsay.
(C) inadmissible, because it is based on hearsay not within any
exception.
(D) inadmissible, because of the unreliability of the reactions of an
animal.

A

(A) admissible, as evidence of D’s guilt.

319
Q

P sued D for copyright infringement for using in D’s book some
slightly disguised house plans on which P held the copyright. P
is prepared to testify that he heard D’s executive assistant for
copyright matters say that D had obtained an advance copy of the
plans from P’s office manager.
P’s testimony is;
(A) admissible as reporting a statement of an employee of a party
opponent.
(B) admissible as a statement of a co-conspirator.
(C) inadmissible, because it is hearsay not within any exception.
(D) inadmissible, because there is no showing that the assistant
was authorized to speak for D.

A

(A) admissible as reporting a statement of an employee of a party
opponent.

320
Q

D is on trial for theft. At trial, the prosecutor called J and M Wong.
They testified that, as they looked out their apartment window,
they saw thieves across the street break the window of a jewelry
store, take jewelry, and leave in a car. M Wong telephoned the
police and relayed to them the license number of the thieves’ car
as J Wong looked out the window with binoculars and read it to
her. Neither of them has any present memory of the number. The
prosecutor offers as evidence a properly authenticated police tape
recording of M Wong’s telephone call with her voice giving the
license number, which is independently shown to belong to D’s
car.
The tape recording of M Wong’s stating the license number is;(A)
admissible, under the hearsay exception for present sense impressions.
(B) admissible, as nonhearsay circumstantial evidence.
(C) inadmissible, because it is hearsay not within any exception.
(D) inadmissible,

A

(A) admissible, under the hearsay exception for present sense
impressions.

321
Q

P sued D for an assault that occurred March 5 in Ca. To support his
defense that he was in Utah on that date, D identifies and seeks to
introduce a letter he wrote to his sister a week before the assault
in which he stated that he would see her in Utah on March 5.
The letter is:
(A) admissible, within the state of mind exception to the hearsay
rule.
(B) admissible, as a prior consistent statement to support Dover’s
credibility as a witness.
(C) inadmissible, because it lacks sufficient probative value.
(D) inadmissible, because it is a statement of belief to prove the
fact believed.

A

(A) admissible, within the state of mind exception to the hearsay
rule.

322
Q

D was prosecuted for murder of V, whose body was found one
morning in the street near D’s house. The state calls W, a neighbor,
to testify that during the night before the body was found he heard
D’s wife scream, “You killed him! You killed him!”
W’s testimony is
(A) admissible as a report of a statement of belief.
(B) admissible as a report of an excited utterance.
(C) inadmissible, because it reports a privileged spousal communication.
(D) inadmissible on spousal immunity grounds, but only if the wife
objects.

A

(B) admissible as a report of an excited utterance.

323
Q

In a prosecution of D for murdering V, D testified that the killing had
occurred in self defense when V tried to shoot him. In rebuttal, the
prosecution seeks to call Walter, V’s father, to testify that the day
before the killing, V told W that she loved D so much she could
never hurt him.
W’s testimony is
(A) admissible within the hearsay exception for statements of the
declarant’s then existing state of mind.
(B) admissible, because V is unavailable as a witness.
(C) inadmissible as hearsay not within any exception.
(D) inadmissible, because V’s character is not an issue.

A

(A) admissible within the hearsay exception for statements of the
declarant’s then existing state of mind.

324
Q

D is on trial for robbing a bank in State A. She testified that she was
in State B at the time of the robbery. Defendant calls her friend,
W, to testify that two days before the robbery D told him that she
was going to spend the next three days in State B.
W’s testimony is;
(A) admissible, because the statement falls within the present
sense impression exception to the hearsay rule.
(B) admissible, because a statement of plans falls within the
hearsay exception for then-existing state of mind.
(C) inadmissible, because it is offered to establish an alibi by
Defendant’s own statement.
(D) inadmissible, because it is hearsay not within any exception.

A

(B) admissible, because a statement of plans falls within the
hearsay exception for then-existing state of mind.

325
Q

P sued Officer D for false arrest. D’s defense was that, based on a
description he heard over the police radio, he reasonably believed
P was an armed robber. Police radio dispatcher B, reading from a
note, had broadcast the description of an armed robber on which
D claims to have relied.
D offers the following items of evidence:
I. D’s testimony relating the description he heard.
II. B’s testimony relating the description he read over the radio.
III. The note containing the description B testifies he read over the
radio.
Which of the following are admissible on the issue of what description
D heard?
(A) I and II only.
(B) I and III only.
(C) II and III only.
(D) I, II, and III.

A

(D) I, II, and III.

326
Q

P Corp sued D for ten fuel oil deliveries not paid for. D denied that
the deliveries were made. At trial, P calls its office manager, W, to
testify that P employees always record each delivery in duplicate,
give one copy to the customer, and place the other copy in P’s files;
that he (W) is the custodian of those files; and that his examination
of the files before coming to court revealed that the ten deliveries
were made.
W’s testimony that the invoices show ten deliveries is;
(A) admissible, because it is based on regularly kept business
records.
(B) admissible, because W has first-hand knowledge of the contents
of the records.
(C) inadmissible, because the records must be produced in order
to prove their contents.
(D) inadmissible, because the records are self-serving.

A

(C) inadmissible, because the records must be produced in order
to prove their contents.

327
Q

The chart contained a notation made by a medical resident that
an aortic clamp had broken during P’s surgery. The resident made
the notation in the regular course of practice, but had no personal
knowledge of the operation, and cannot remember which of the
operating physicians gave him the information.
The document is;
(A) admissible as a record of regularly conducted activity.
(B) admissible as recorded recollection.
(C) inadmissible as a violation of the best evidence rule.
(D) inadmissible, because it is hearsay within hearsay.

A

(A) admissible as a record of regularly conducted activity.

328
Q

P sued D for injuries sustained in an automobile collision. During
P’s hospital stay, D, a staff physician, examined P’s X rays and
said to P, “You have a fracture of two vertebrae, C4 and C5.”
Intern, who was accompanying D on her rounds, immediately
wrote the diagnosis on P’s hospital record. At trial, the hospital
records custodian testifies that P’s hospital record was made and
kept in the ordinary course of the hospital’s business.
The entry reporting D’s diagnosis is;
(A) inadmissible, because no foundation has been laid for Doctor’s
competence as an expert.
(B) inadmissible, because D’s opinion is based upon data that are
not in evidence.
(C) admissible as a statement of then existing physical condition.
(D) admissible as a record of regularly conducted business activity.

A

(D) admissible as a record of regularly conducted business activity.

329
Q

In an arson prosecution the government seeks to rebut Defendant’s
alibi that he was in a jail in another state at the time of
the fire. The government calls Witness to testify that he diligently
searched through all the records of the jail and found no record
of Defendant’s having been incarcerated there during the time
Defendant specified.
The testimony of Witness is;
(A) admissible as evidence of absence of an entry from a public
record.
(B) admissible as a summary of voluminous documents.
(C) inadmissible, because it is hearsay not within any exception.
(D) inadmissible, because the records themselves must be produced.

A

(A) admissible as evidence of absence of an entry from a public
record.

330
Q

At D’s trial for theft, W, called by the prosecutor, testified to the
following:
A.)that from his apartment window, he saw thieves across the
street break the window of a jewelry
store, take jewelry, and leave in a car;
B.)That his wife telephoned the police and relayed to them the
license number of the thieves’ car as W looked out the window
with binoculars and read it to her; andC.)That he has no present memory of the number, but that immediately
afterward he listened to a playback of the police tape
recording giving the license number (which belongs to D’s car)
and verified that she had relayed the number accurately.
Playing the tape recording for the jury would be:
(A) proper, because it is recorded recollection.
(B) proper, because it is a public record or report.
(C) improper, because it is hearsay not within any exception.
(D) improper, because his wife lacked firsthand knowledge of the
license number.

A

(A) proper, because it is recorded recollection.

331
Q

D was prosecuted for bankruptcy fraud. D’s wife, now deceased,
had testified adversely to D during earlier bankruptcy proceedings
that involved similar issues. Although the wife had been cross-examined,
no serious effort was made to challenge her credibility
despite the availability of significant impeachment information. At
the fraud trial, the prosecutor offers into evidence the testimony
given by D’s wife at the bankruptcy proceeding.
This evidence should be
(A) admitted, under the hearsay exception for former testimony.
(B) admitted, because it is a statement by a person identified with
a party.
(C) excluded, because it is hearsay not within any exception.
(D) excluded, because D has the right to prevent use of his
spouse’s testimony against him in a criminal case.

A

(A) admitted, under the hearsay exception for former testimony.

332
Q

At D’s trial for sale of drugs, the government called W to testify,
but W refused to answer any questions about D and was held in
contempt of court. The gov’t then calls Officer to testify that, when
W was arrested for possession of drugs and offered leniency if he
would identify his source, W had named D as his source.
The testimony offered concerning W’s id of D is;
(A) admissible as a prior inconsistent statement by W.
(B) admissible as an identification of D by W after having perceived
him.
(C) inadmissible, because it is hearsay not within any exception.
(D) inadmissible, because W was not confronted with the statement
while on the stand.

A

(C) inadmissible, because it is hearsay not within any exception.

333
Q

D and M were charged with conspiracy to dispose of a stolen diamond
necklace. M jumped bail and cannot be found. Proceeding
to trial against D alone, the prosecutor calls W, M’s girlfriend, to
testify that M confided to her that “D said I still owe him some of
the money from selling that necklace.”
W’s testimony is;
(A) admissible as evidence of a statement by party-opponent D.
(B) admissible as evidence of a statement against interest by M.
(C) inadmissible, because M’s statement was not in furtherance
of the conspiracy.
(D) inadmissible, because M is not shown to have firsthand knowledge
that the necklace was stolen.

A

(B) admissible as evidence of a statement against interest by
Melville.

334
Q

Mrs. P sued D for shooting her husband from ambush. Mrs. P
offers to testify that, the day before her husband was killed, he
described to her a chance meeting with D on the street in which
D said, “I’m going to blow your head off one of these days.”
The witness’s testimony concerning her husband’s statement is;
(A) admissible, to show D’s state of mind.
(B) admissible, because D’s statement is that of a party-opponent.
(C) inadmissible, because it is improper evidence of a prior bad
act.
(D) inadmissible, because it is hearsay not within any exception.

A

(D) inadmissible, because it is hearsay not within any exception.

335
Q

In a jurisdiction without a Dead Man’s Statute, P’s estate sued D
claiming that D had borrowed from P $10,000, which had not been
repaid as of P’s death. P was run over by a truck. At the accident
scene, while dying from massive injuries, P told Officer S to “make
sure my estate collects the $10,000 I loaned to D.”
S’s testimony about P’s statement is;
(A) inadmissible, because it is more unfairly prejudicial than probative.
(B) inadmissible, because it is hearsay not within any exception.
(C) admissible as an excited utterance.
(D) admissible as a statement under belief of impending death.

A

(B) inadmissible, because it is hearsay not within any exception.

336
Q

A woman is on trial for a burglary that took place at about six in the evening on November 1. A surveillance video from a local gas station shows that the woman visited the gas station at seven in the evening on November 1. The woman alleges as an alibi that she was actually at the gas station at six in the evening. She argues that the gas station failed to change the time display on its camera to reflect the end of Daylight Savings Time on the morning of November 1. The judge, on his own initiative and after first giving the prosecutor the opportunity to object, took judicial notice of the fact that Daylight Savings Time did end in the year in question on November 1. The judge instructed the jury that it may or may not accept any judicially noticed fact as conclusive. Were the judge’s actions with regard to judicial notice of this fact proper?

A. Yes, because the court may take judicial notice on its own initiative.

B. Yes, because the court gave the prosecution an opportunity to be heard on the propriety of taking judicial notice before doing so.

C. No, because the fact is not one that is generally known within the territorial jurisdiction of the trial court.

D. No, because the court should have instructed the jury that it is required to accept the noticed fact as conclusive.

A

Answer choice A is correct. A court may take judicial notice at any time during a proceeding, including on appeal, whether upon request of a party or by the court’s own initiative.

Answer choice B is incorrect. When a party makes a timely request, the judge must give the party an opportunity to be heard on the propriety of taking judicial notice and the nature of the fact to be noticed. However, a judge is not required to provide this opportunity before taking judicial notice of an adjudicative fact.

Answer choice C is incorrect. Although judicial notice may be taken of an adjudicative fact that is not subject to reasonable dispute because it is generally known within the territorial jurisdiction of the trial court, judicial notice may also be taken of an adjudicative fact that can be accurately and readily determined from sources whose accuracy cannot reasonably be questioned. Here, the date on which Daylight Savings Time ends is such a fact.

Answer choice D is incorrect because this is a criminal case, and in a criminal case, the jury must be instructed that it may or may not accept any judicially noticed fact as conclusive, as the judge properly did here.

337
Q

A plaintiff sued a defendant for negligence arising out of an automobile accident that occurred just prior to noon. At trial, the defendant took the stand. On direct examination, the defendant’s attorney asked the defendant whether he stopped at a stop sign prior to entering the intersection where the accident with the plaintiff occurred. After the defendant responded that he had, the defendant’s attorney indicated that he had no further questions for the defendant. On cross-examination, the first question posed by the plaintiff’s attorney to the defendant was, “Even though it was raining heavily, you didn’t have your headlights on at the time of the accident, correct?” The defendant’s attorney objected to the question.

Of the following, which is the best basis for this objection?

A. The question is irrelevant.

B. The question does not impeach the defendant’s testimony.

C. The question is a leading question.

D. The question is a compound question.

A

Answer choice D is correct. The question contains independent issues—whether it was raining at the time of the accident, and whether the defendant had his headlights on at the time of the accident. Consequently, the question is a compound question. By objecting, the defense attorney can compel the plaintiff’s attorney to separately inquire about each issue.

Answer choice A is incorrect because, while the issue of the defendant’s use of his headlights would generally be irrelevant to an accident that occurred during the day as this one had, the question suggests that it was raining at the time of the accident. Consequently, the issue of the defendant’s use of his headlights would be relevant if it is established that it was raining at the time of the accident.

Answer choice B is incorrect because cross-examination may address substantive issues as well as the credibility of witness. The scope of cross-examination generally is limited to the subject matter of the direct examination and the credibility of the witness. It is at least arguable that, based on a narrow definition of the subject matter of the direct examination, this question deals with a matter (i.e., the failure to use headlights during rain) that is beyond the matter addressed in direct examination (i.e., the failure to stop at the stop sign). However, even under this narrow definition, FRE 611(b) permits the court to allow inquiry into additional matters. Consequently, the best basis for objecting to this particular question is the compound question basis.

Answer choice C is incorrect because there is generally no restriction on the use of leading questions during cross-examination.

338
Q

A son and a daughter are opposing parties in federal court. At trial, the daughter presented evidence that her father has been missing for ten years, and that no one has heard from him in that time. The son testified that he received a phone call three years ago from a person that he believes was his father. In the jurisdiction, a rebuttable presumption arises that a person is dead when a party establishes that the person has been missing and not heard from for more than seven years. Which of the following is correct?

A. The jury must find that the father is dead.

B. The jury may find that the father is dead.

C. The burden has shifted to the son to persuade the jury that the father is alive.

D. The judge must instruct the jury to conclude that the father is dead.

A

Answer choice B is correct. A presumption is a conclusion that the trier of fact is required to draw upon a party’s proof of an underlying fact or set of facts (i.e., basic facts). A rebuttable presumption shifts the burden of production, but not the burden of persuasion, to the opposing party. However, a rebuttable presumption may be overcome by evidence to the contrary. If no contrary evidence is introduced, the judge must instruct the jury to accept the presumption. If contrary evidence is introduced, as is the case here, then the presumption no longer has a preclusive effect. At this point, the jury may, but is not required to, draw the conclusion from the basic facts. Thus, the jury may determine the weight and credibility of all of the evidence. For this reason, answer choice A is incorrect.

Answer choice C is incorrect because a rebuttable presumption shifts the burden of production, but not the burden of persuasion, to the opposing party.

Answer choice D is incorrect because it is an incorrect statement of law. After a rebuttable presumption no longer has a preclusive effect due to the introduction of contrary evidence, a judge may instruct the jury that it may, but is not required to, draw the conclusion from the basic facts.

339
Q

A man was prosecuted for the false imprisonment of a woman after he allegedly pretended to have a broken arm, asked the woman to help him carry a box into the back of his van, and then pushed her into the van and locked it. At trial, the prosecution attempted to introduce evidence during a female witness’s direct examination that five years ago, the man had impersonated a policeman, entered the witness’s home, and made serious threats of harm if the witness did not stay in the home and answer his questions.

Is this evidence admissible?

A. Yes, because it shows that the man has the propensity to falsely imprison women.

B. Yes, because it is relevant evidence that shows the man’s preparation and planning.

C. No, because it is improper character evidence.

D. No, because it can only be introduced during cross-examination.

A

Answer choice C is correct. Evidence of a bad act is not admissible to prove a person’s character in order to show that the person acted in accordance with that character on a particular occasion. Here, the man’s conduct is being admitted to prove that the man acted in accordance with his criminal propensity to falsely imprison women. Therefore, the evidence is inadmissible.

Answer choice A is incorrect because character evidence cannot be admitted to show a defendant’s criminal propensity in order to prove that he committed the crime for which he is charged.

Answer choice B is incorrect. Although a defendant’s crimes or other wrongful acts are not admissible to show his criminal propensity in order to prove that he committed the crime for which he is charged, such bad acts are admissible for another purpose, such as proving motive, opportunity, intent, preparation, plan, knowledge, identity, absence of mistake, or lack of accident. Here, the circumstances of the man’s prior bad act are so unrelated to the current charge of false imprisonment that they do not tend to prove preparation or common plan. The only common denominator in both bad acts is that the man falsely imprisoned women.

Answer choice D is incorrect. When a character witness is cross-examined, the court may allow a party to inquire into specific acts committed by the person about whom the witness is testifying. However, even on cross-examination, specific acts cannot be admitted to prove that, because the defendant had a propensity to commit crimes, the defendant committed the charged crime. Therefore, this evidence would also have been inadmissible during cross-examination.

340
Q

A car struck a truck at an intersection. The driver of the truck sued the driver of the car, claiming that the car driver ran a red light. In the investigating officer’s report, the only witness is quoted saying, “I saw the whole thing. The car had the green light.” At trial, the witness testified that he clearly remembered that the car’s traffic light had been red, and that the car ran the light. The defendant did not cross-examine the witness and the witness was dismissed and left the jurisdiction. After the plaintiff had presented his case, the defendant moved to introduce the witness’s statement from the investigating officer’s report solely to impeach the witness’s testimony. The plaintiff objected.

How should the court rule?

A. Overrule the objection, and admit the statement as substantive evidence that the car driver did not run the red light.

B. Overrule the objection, and admit the statement as impeachment evidence only.

C. Sustain the objection, because the statement is inadmissible hearsay.

D. Sustain the objection, because extrinsic evidence may not be used to impeach a witness under the circumstances.

A

Answer choice D is correct. A witness’s prior statement that is inconsistent with a material part of the witness’s testimony may be used to impeach the witness. However, extrinsic evidence of a witness’s prior inconsistent statement may be introduced only if the witness is given the opportunity to explain or deny the statement and the opposing party is given the opportunity to examine the witness about it. In this case, because the witness was not given an opportunity to explain or deny the statement, the evidence should be excluded.

Answer choice A is incorrect because, although the statement is relevant evidence on the substantive issue of whether the car driver did run the red light, the defendant has sought to use the statement only for impeachment purposes (i.e., to call into question the veracity of the witness’s testimony at trial).

Answer choice B is incorrect because extrinsic evidence is not admissible for impeachment purposes unless the witness is given the opportunity to explain or deny it.

Answer choice C is incorrect because, while the statement would be inadmissible hearsay if it were introduced for its truth, the statement is being introduced for impeachment purposes only.

341
Q

An artist entered into a written agreement to sell a patron a partially finished painting once it was complete. The patron later learned that the artist planned to sell the painting to a third party who offered to pay the artist more than the contract price. The patron filed suit to compel the artist to sell the painting to her in accord with the terms of their agreement, while the artist denied that the painting the artist planned to sell to the third party was the subject of the agreement with the patron. At trial, the patron did not introduce the written agreement or explain its absence. Rather, the patron sought to testify that, when she signed the agreement, the artist had pointed to the painting in question and stated that it was the patron’s painting. The artist’s attorney objected to the testimony that the artist identified the painting as belonging to the patron.

How should the court rule?

A. Sustain the objection, because the artist’s statement was hearsay.

B. Sustain the objection, because the patron failed to produce the written agreement or explain its absence.

C. Overrule the objection, because the statement is not hearsay.

D. Overrule the objection, because the statement was relevant.

A

Answer choice B is correct. Under the best evidence rule, the original document or a reliable duplicate must be used to prove the contents of a writing unless its absence is satisfactorily explained. Here, the plaintiff intends to introduce the statement to prove that the written agreement refers to a specific painting without introducing the written agreement itself. Consequently, the patron may not introduce that statement to establish that the painting in question was the painting that was the subject of the agreement.

Answer choice A is incorrect because, although the artist’s statement was being introduced for its truth, it is non-hearsay as a statement of a party to the action.

Answer choice C is incorrect because, although the artist’s oral statement was non-hearsay, the best evidence rule prevents its admission into evidence.

Answer choice D is incorrect because, while the statement meets the test for relevancy, in that it is both probative and material, it is inadmissible on other grounds.

342
Q

The holder of a patent for hybrid corn sued both an unlicensed wholesaler of that product and a retailer who purchased the corn from the wholesaler for alleged patent infringement. The patent holder reached an agreement with the retailer in which the holder released the retailer from liability in exchange for the payment of a nominal amount. After properly authenticating the agreement, the wholesaler sought to introduce it into evidence solely for the purpose of determining damages. The patent holder objected to the introduction of the agreement. Should the court admit the agreement into evidence?

A. No, because the agreement constituted the acceptance of a settlement offer.

B. No, because the danger of unfair prejudice outweighs the probative value of the agreement.

C. Yes, because the agreement is being introduced solely for the purpose of determining the amount of damages.

D. Yes, because the wholesaler was not a party to the agreement.

A

Answer choice A is correct. Evidence of a settlement offer, including evidence of the acceptance of such an offer, is not admissible for the purpose of establishing the validity of a claim or the amount of damages. For this reason, answer choice C is incorrect.

Answer choice B is incorrect because it fails to correctly state the law. In order for evidence to be inadmissible under Federal Rule 403, the dangers of unfair prejudice must substantially outweigh its probative value.

Answer choice D is incorrect because when there are more than two parties, a settlement agreement entered into by a party with an adverse party cannot be used by a remaining adverse party to prove or disprove the amount of an unsettled claim.

343
Q

A truck was struck by a train while the truck was crossing railroad tracks. Although the crossing gate had malfunctioned, the train engineer had blown a warning whistle before entering the crossing. Among the legal actions precipitated by this accident, the trucking company sued the railroad for damages stemming from the destruction of its truck and the contents of the truck. The railroad sought to introduce a written statement given to police by a passenger in the truck, who alternated driving duties with the person driving the truck at the time of the accident, that the driver had left his hearing aids at a rest stop shortly before the accident. Both the passenger and the driver of the truck, who are employees of the trucking company, are available to testify.

Is this statement admissible against the trucking company?

A. Yes, because the statement was made by the trucking company’s employee.

B. Yes, under the business records hearsay exception.

C. No, because the passenger was not employed as a spokesperson for the trucking company.

D. No, because both the passenger and the driver of the truck are available to testify.

A

Answer choice A is correct. A statement made by a party’s agent or employee constitutes an opposing party’s statement if it was made concerning a matter within the scope of and during the course of the relationship. Here, the statement was made by a passenger in the truck who was an employee of the trucking company and who alternated driving the truck with the person who was driving the truck at the time of the accident. The statement concerns the driver’s negligence that, according to the railroad company, was at least a contributing cause of the accident. Thus, the statement was within the scope and during the course of the passenger’s employment relationship with the trucking company. Consequently, the statement is admissible as a vicarious statement of an opposing party.

Answer choice B is incorrect because the passenger’s statement to police after the accident does not fall within the business records exception to hearsay because the passenger was not under a business duty to make the statement of the police.

Answer choice C is incorrect. Although one way in which a statement made by an employee is imputed to the employer is when the employee is a spokesperson for the employer, this is not the only way for a vicarious admission to occur. Here, the statement was made within the scope of and during the course of the passenger’s employment relationship with the trucking company—the statement was about the driver’s negligence in driving the truck without his hearing aids while both the speaker and the truck driver were employed by the trucking company to drive the truck.

Answer choice D is incorrect because a statement that is admissible as an opposing party’s statement does not require the declarant or the person about whom the statement is made to be available to testify.

344
Q

A defendant, his cousin, and a friend planned to rob a convenience store. The friend watched the defendant and his cousin put on ski masks and enter the convenience store. Moments later, the friend heard a gunshot, and the defendant fled. The cousin stumbled out of the convenience store, pointed at his bleeding foot, and told the accomplice, “He shot me! The moron dropped his gun and shot me, I’m going to kill that idiot!” The cousin ran after the defendant, and the friend called the police. The police found the cousin dead from a bullet wound to the chest and charged the defendant with murder. At trial, the defendant wants to introduce the cousin’s statement to the friend as evidence that the cousin was the initial aggressor. The prosecution objects.

What is the defendant’s best argument for finding that the statement is admissible?

A. The declarant is unavailable as a witness.

B. The statement indicates the cousin’s present intent.

C. The statement was a dying declaration.

D. The statement was made by a co-conspirator.

A

Answer choice B is correct. A statement of present intent, motive, or plan can be used to prove conduct in conformity with that state of mind. Here, the defendant is trying to offer the statement to show that his cousin intended to attack him to help establish his defense. Because a statement of the declarant’s then-existing state of mind falls within a hearsay exception, this is the defendant’s best argument.

Answer choice A is incorrect because a declarant is not deemed unavailable if the unavailability is due to the procurement or wrongdoing of the proponent of the statement in order to prevent the declarant from testifying at or attending the trial. Because of the possibility that the defendant killed his cousin in order to silence him at trial, this is not the defendant’s strongest argument.

Answer choice C is incorrect because it would be very difficult to argue that the cousin believed his death was imminent from a gunshot wound to his foot at the time he made the statement. This is evidenced by the fact that he ran after the defendant and made no statement indicating that he believed his death was imminent.

Answer choice D is incorrect because, although a statement made by a co-conspirator during and in furtherance of a conspiracy is admissible as an opposing party’s statement against other co-conspirators, the cousin’s outburst was not made in furtherance of the already failed robbery.

345
Q

A defendant is tried for armed robbery of a bank. On cross-examination
of the defendant, the prosecutor asks him whether he was
convicted of tax fraud the previous year.
The question is
A: proper, to show that the defendant is inclined to lie
B: proper, to show that the defendant is inclined to steal money
C: improper, because the conviction has insufficient similarity to
the crime charged
D: improper, because the probative value of evidence is outweighed
by the danger of unfair prejudice

A

A: proper, to show that the defendant is inclined to lie

346
Q

A plaintiff sued a defendant for battery. At trial, the plaintiff’s
witness testified that the defendant had made an unprovoked
attack on the plaintiff. On cross-examination, the defendant asks
the witness about a false claim that the witness had once filed on
an insurance policy.
The question is
A: proper, because the conduct involved untruthfulness.
B: improper, because the conduct had not resulted in conviction
of the witness.
C: improper, because the impeachment involved a specific instance
of misconduct.
D: improper, because the claim form would be the best evidence.

A

A: proper, because the conduct involved untruthfulness.

347
Q

A defendant was prosecuted for sexually abusing his 13-year-old
stepdaughter. The stepdaughter testified to the defendant’s conduct.
On cross-examination, defense counsel asks the stepdaughter,
“Isn’t it true that shortly before you complained that your
stepfather abused you, he punished you for maliciously ruining
some of his phonograph records?”
The question is:
A: proper, because it relates to a possible motive for the stepdaughter
to falsely accuse the defendant.
B: proper, because the stepdaughter’s misconduct is relevant to
her character for veracity.
C: improper, because the incident had nothing to do with the
stepdaughter’s truthfulness
D: improper, because it falls outside the scope of direct examination.

A

A: proper, because it relates to a possible motive for the stepdaughter
to falsely accuse the defendant.

348
Q

In an automobile negligence action by a plaintiff against a defendant,
a bystander testified for the plaintiff. The defendant later
called a witness, who testified that the bystander’s reputation for
truthfulness was bad. On cross-examination of the witness, the
plaintiff’s counsel asks, “Isn’t it a fact that when you bought your
new car last year, you made a false affidavit to escape paying the
sales tax?”
This question is:
A: proper, because it will indicate the witness’s standard of judgment
as to reputation for truthfulness.
B: proper, because it bears on the witness’s credibility.
C: improper, because character cannot be proved by specific
instances of conduct.
D: improper, because one cannot impeach an impeaching witness.

A

B: proper, because it bears on the witness’s credibility.

349
Q

A defendant is charged with mail fraud. At trial, the defendant has
not taken the witness stand, but he has called a witness who has
testified that the defendant has a reputation for honesty. On cross,
the prosecutor seeks to ask the witness, “Didn’t you hear that two
years ago the defendant was arrested for embezzlement?”
Should the court permit the question?
A: No, because the defendant has not testified and therefore has
not put his character at issue.
B: No, because the incident was an arrest, not a conviction.
C: Yes, because it seeks to impeach the credibility of the witness.
D: Yes, because the earlier arrest for a crime of dishonesty makes
the defendant’s guilt of the mail fraud more likely.

A

C: Yes, because it seeks to impeach the credibility of the witness.

350
Q

A defendant is tried for armed robbery of a bank. The prosecution,
in its case in chief, offers evidence that when the defendant was
arrested one day after the crime, he had a quantity of heroin and
a hypodermic needle in his possession.
This evidence should be
A: admitted to prove the defendant’s motive to commit the crime.
B: admitted to prove the defendant’s propensity to commit crimes.
C: excluded, because its probative value is substantially outweighed
by the danger of unfair prejudice.
D: excluded, because such evidence may be offered only to rebut
evidence of good character offered by the defendant.

A

C. excluded, because its probative value is substantially outweighed
by the danger of unfair prejudice.

351
Q

A defendant was prosecuted for armed robbery. At trial, the defendant
testified on his own behalf, denying that he had committed
the robbery. On cross-examination, the prosecutor intends to ask
the defendant whether he had been convicted of burglary six years
earlier. Assume that the court has already determined that the
probative value of the answer outweighs the prejudicial effect to
the defendant.
The question concerning the burglary conviction is
A: proper, because the probative value for impeachment outweighs
the prejudice to the defendant.
B: proper, because the prosecutor is entitled to make this inquiry
as a matter of right.
C: improper, because burglary does not involve dishonesty or false
statement.
D: improper, because the conviction must be proved by court
record, not by question on cross-examination.

A

A: proper, because the probative value for impeachment outweighs
the prejudice to the defendant.

352
Q

A defendant, who was charged with the crime of assaulting the
victim, admitted striking the victim but claimed to have acted
in self-defense when he was attacked by the victim, who was
drunk and belligerent after a football game. The defendant offered
testimony of a former employer that he had known and
who had employed the defendant for twelve years and knew the
defendant’s reputation among the people with whom he lived and
worked to be that of a peaceful, law-abiding, nonviolent person.
How should the trial judge rule on this testimony?
A: Admissible, because it is relevant to show the improbability of
the defendant’s having committed an unprovoked assault.
B: Admissible, because it is relevant to a determination of the
extent of punishment if the defendant is convicted
.C: Not admissible, because whether the defendant is normally a
person of good character is irrelevant to the specific charge.
D: Not admissible, because it is irrelevant without a showing that
the former employer was one of the persons among whom the
defendant lived and worked.

A

A: Admissible, because it is relevant to show the improbability of
the defendant’s having committed an unprovoked assault.

353
Q

A defendant was charged with battery for allegedly attacking
a man after the two of them left a local bar together. No one
else witnessed the incident. At trial, each testified that he had
acted only in self-defense. The defendant has called his next-door
neighbor as a witness to testify to the defendant’s reputation
both for truthfulness and for peacefulness. The government has
objected to the testimony in its entirety.
How should the court proceed?
A: Admit the evidence in its entirety.
B: Admit the evidence regarding the defendant’s reputation for
peacefulness, but exclude the evidence regarding his truthfulness.
C: Exclude the evidence regarding the defendant’s reputation for
peacefulness, but admit the evidence regarding his truthfulness.
D: Exclude the evidence in its entirety.

A

B: Admit the evidence regarding the defendant’s reputation for
peacefulness, but exclude the evidence regarding his truthfulness.

354
Q

At Defendant’s murder trial, Defendant calls Witness as his first
witness to testify that Defendant has a reputation in their community
as a peaceable and truthful person. The prosecutor objects
on the ground that Witness’s testimony would constitute improper
character evidence.
The court should:
(A) admit the testimony as to peaceableness, but exclude the
testimony as to truthfulness.
(B) admit the testimony as to truthfulness, but exclude the testimony
as to peaceableness.
(C) admit the testimony as to both character traits.
(D) exclude the testimony as to both character traits.

A

(A) admit the testimony as to peaceableness, but exclude the
testimony as to truthfulness.

355
Q

Defendant was charged with attempted murder of Victor in a
sniping incident in which Defendant allegedly shot at Victor from
ambush as Victor drove his car along an expressway. The prosecutor
offers evidence that seven years earlier Defendant had fired
a shotgun into a woman’s home and that Defendant had once
pointed a handgun at another driver while driving on the street.
This evidence should be:
(A) excluded, because such evidence can be elicited only during
cross-examination
(B) excluded, because it is improper character evidence.
(C) admitted as evidence of Defendant’s propensity toward violence
(D) admitted as relevant evidence of Defendant’s identity, plan, or
motive

A

(B) excluded, because it is improper character evidence.

356
Q

The plaintiff sues a bar for injuries suffered in an automobile
accident caused by a patron of the bar. The plaintiff claims that
the patron was permitted to drink too much liquor at the bar before
the accident.
A witness, who was present at the bar on the night of the incident,
testified that on the night of the accident the patron was drunk.
The witness then proposed to testify that he remarked to his
companion, “The patron is so drunk he can’t even stand up.”
The witness’s remark to his companion is:
A: admissible as an excited utterance.
B: admissible as a prior consistent statement.
C: admissible as a statement by the witness regarding a condition
he observed, made while he was observing it.
D: inadmissible if there was no evidence that the witness had
expertise in determining intoxication.

A

C: admissible as a statement by the witness regarding a condition
he observed, made while he was observing it.

357
Q

The plaintiff sues a bar for injuries suffered in an automobile
accident caused by a patron of the bar. The plaintiff claims that
the patron was permitted to drink too much liquor at the bar before
the accident.
The plaintiff offered evidence that, after the accident, the manager
of the bar established house rules limiting all customers to two
drinks per hour, with a maximum limit of four drinks per night.
This evidence is:
A. admissible to show that the prior conduct of the bar negligent
B. admissible to show that the bar was aware of the need for taking
precautionary measures
C. inadmissible because subsequent measures by an employee
are not binding on the bar
D. inadmissible because its admission would discourage the taking
of such remedial measures

A

D. inadmissible because its admission would discourage the taking
of such remedial measures

358
Q

A plaintiff sued a defendant for injuries suffered by the plaintiff
when their automobiles collided. At trial the plaintiff offers into
evidence a properly authenticated letter from the defendant that
says, “Your claim seems too high, but, because I might have been
a little negligent, I’m prepared to offer you half of what you ask.”
The letter is:
A. admissible as an admission by a party-opponent
B. admissible as a statement against pecuniary interest
C. inadmissible, because the defendant’s statement is lay opinion
on a legal issue
D. inadmissible, because the defendant’s statement was made in
an effort to settle the claim

A

D. inadmissible, because the defendant’s statement was made in
an effort to settle the claim

359
Q

A drug dealer is being tried in federal court for criminal conspiracy
with a friend to violate a federal narcotics law. At trial, the prosecutor
calls the drug dealer’s new wife and asks her to testify about a
meeting between the drug dealer and the friend that she observed
before she married the drug dealer.
Which of the following is the most accurate statement of the
applicable rule concerning whether the wife may testify?
A. the choice belongs to the defendant’s wife
B. the choice is the defendant’s
C. the defendant’s wife is permitted to testify only if both the wife
and defendant agree
D. the defendant’s wife is compelled to testify even if both the wife
and defendant object

A

A. the choice belongs to the defendant’s wife
**because they are currently married + case is criminal = W can
refuse to testify based on “spousal immunity”

360
Q

A defendant is tried for armed robbery of a bank. At the request
of police, the teller who was robbed prepared a sketch bearing
a strong likeness to the defendant, but the teller died in an automobile
accident before the defendant was arrested. At trial, the
prosecution offers the sketch.
The sketch is
A: admissible, as an identification of a person after perceiving him.
B: admissible, as past recollection recorded.
C: inadmissible, as hearsay not within any exception.
D: inadmissible, as an opinion of the teller

A

C: inadmissible, as hearsay not within any exception
**since the teller is unavailable to testify, the sketch can not come
in

361
Q

A defendant is tried for armed robbery of a bank. The defendant
testified on direct examination that he had never been in the bank
that had been robbed. His counsel asks, “What, if anything, did
you tell the police when you were arrested?” If his answer would
be, “I told them I had never been in the bank,” this answer would
be:
A: admissible, to prove that the defendant had never been in the
bank.
B: admissible, as a prior consistent statement.
C: inadmissible, as hearsay not within any exception.
D: inadmissible, because it was a self-serving statement by a
person with a substantial motive to fabricate.

A

C. inadmissible, as hearsay not within any exception.
**no exception applies because D’s statement is an out-of-court
statement offered for the truth of the matter asserted

362
Q

A bus passenger sued the transit company for injuries to his
back from an accident caused by the company’s negligence. The
company denies that the passenger received any injury in the
accident. The company calls an observer to testify that right after
the accident, the passenger told him that he had recently suffered
a recurrence of an old back injury.
The judge should rule the observer’s testimony:
A: admissible as an admission of a party opponent.
B: admissible as a spontaneous declaration.
C: inadmissible, because it is irrelevant.
D: inadmissible, because it is hearsay, not within any exception.

A

A. admissible as an admission of a party opponent.

363
Q

The driver of a car and three passengers were injured when their
car was struck by a truck owned by a corporation. The truck was
being driven by a corporate employee, a truck driver. A second
corporate employee, a supervisor, was also riding in the truck. The
issues in the case of the driver of the car versus the corporation
include the negligence of the truck driver in driving too fast and
failing to wear glasses, and the negligence of the driver of the car
in failing to yield the right of way.
The corporation’s counsel seeks to have a sheriff testify that while
he was investigating the accident he was told by the driver of the
car, “This was probably our fault.”
The judge should rule the proffered evidence
A: admissible, as an admission of a party.
B: admissible, because it is a statement made to a police officer
in the course of an official investigation.
C: inadmissible, because it is a mixed conclusion of law and fact.
D: inadmissible, because it is hearsay, not within any exception.

A

A: admissible, as an admission of a party.

364
Q

In a civil action for personal injury, the plaintiff alleges that he was
beaten up by the defendant during an altercation in a crowded
bar. The defendant’s defense is that he was not the person who
hit the plaintiff. To corroborate his testimony about the cause of
his injuries, the plaintiff seeks to introduce, through the hospital
records custodian, a notation in a regular medical record made
by an emergency room doctor at the hospital where the plaintiff
was treated for his injuries. The notation is: “Patient says he was
attacked by [the defendant].”
The notation is:
A: inadmissible, because the doctor who made the record is not
available for cross-examination.
B: inadmissible as hearsay, not within any exception.
C: admissible as hearsay, within the exception for records of
regularly conducted activity.
D: admissible as a statement made for the purpose of medical
diagnosis or treatment.

A

B. inadmissible as hearsay, not within any exception.
**P’s statement to Doc about who attacked him is unrelated to
medial diagnosis/treatment so it is inadmissible as a “medical
record exception”

365
Q

The defendant was on trial for murder. Following recess, the
defendant was in the crowed hallway outside the courtroom talking
with his attorney. During their convo, the defendant said “So what
if I killed him, big deal.” The defendant’s statement was overheard
by a witness as he was walking down the corridor. The witness
then informed the prosecution of the comment he heard. After the
trial is reconvened, the prosecutor calls the witness to testify to
what he heard the defendant tell the attorney.
The defendant’s attorney objects, arguing that this violates attorney-
client privilege. May the witness be called to testify?
A. no because the defendant’s statement was a confidential communication
and protected under attorney-client privilege
B. no because the defendant’s statement is inadmissible hearsay
not within an exception
C. yes because the witness has a duty to disclose incriminating
evidence
D. yes because the defendant’s statement was not a confidential
communication

A

D. yes because the defendant’s statement was not a confidential
communication

366
Q

Perez sued Dawson for damages arising out of an automobile collision.
At trial, Perez called Minter, an eyewitness to the collision.
Perez expected Minter to testify that she had observed Dawson’s
automobile for five seconds prior to the collision and estimated
Dawson’s speed at the time of the collision to have been 50 miles
per hour. Instead, Minter testified that she estimated Dawson’s
speed to have been 25 miles per hour. Without finally excusing
Minter as a witness, Perez then called Wallingford, a police officer,
to testify that Minter had told him during his investigation at the
accident scene that Dawson “was doing at least 50.”
Wallingford’s testimony is
A. admissible as a present sense impression.
B. admissible to impeach Minter.
C. inadmissible, because Perez may not impeach his own witness.
D. inadmissible, because it is hearsay not within any exception.

A

B. admissible to impeach Minter.

367
Q

Decker, charged with armed robbery of a store, denied that he
was the person who had robbed the store.
In presenting the state’s case, the prosecutor seeks to introduce
evidence that Decker had robbed two other stores in the past year.
This evidence is:
(A) admissible, to prove a pertinent trait of Decker’s character and
Decker’s action in conformity therewith
(B) admissible, to prove Decker’s intent and identity
(C) inadmissible, because the prosecutor may rely only on reputation
or opinion evidence as this point in the trial to prove the
defendant’s character
(D) inadmissible, because its probative value is substantially outweighed
by the danger of unfair prejudice.

A

(D) inadmissible, because its probative value is substantially outweighed
by the danger of unfair prejudice.

368
Q

The plaintiff is suing a legal journal for libel based on an article
that implied he had cheated the IRS by not paying his taxes. At
trial, plaintiff denies any wrongdoing. An accountant is called by
the defense to testify that 5 years ago the plaintiff had improperly
distorted the value of certain investments in an effort to lower his
taxable income.
Should the accountant’s testimony be admitted?
A. yes to challenge the plaintiff’s credibility
B. yes as substantive evidence of the plaintiff’s character
C. no because it is improper character evidence
D. no because the testimony must be corroborated

A

B. yes as substantive evidence of the plaintiff’s character
**party is NOT on the stand then it goes to his “character
**part is on the stand then it goes to his “credibility”
**credibility = impeaching a witness- plaintiff is NOT on the stand

369
Q

A defendant was charged with aggravated assault, At trial, the
victim testified that the defendant beat her savagely, but she was
not asked about anything said during the incident. The prosecutor
then called a witness to testify that when the beating stopped, the
victim screamed: “I’m dying—don’t let [the defendant] get away
with it!”
Is the testimony of the witness concerning the victim’s statement
admissible?
(A) No, because it is hearsay not within any exception.
(B) No, because the victim was not asked about the statement.
(C) Yes, as a statement under belief of impending death, even
though the victim did not die.
(D) Yes, as an excited utterance.

A

(D) Yes, as an excited utterance.

370
Q

A defendant is on trial for robbing a bank in State A. She testified
that she was in State B at the time of the robbery. The defendant
calls her friend, a witness, to testify that two days before the
robbery the defendant told him that she was going to spend the
next three days in State B.
The witness’s testimony is
A: admissible, because the statement falls within the present
sense impression exception to the hearsay rule.
B: admissible, because a statement of plans falls within the
hearsay exception for then-existing state of mind.
C: inadmissible, because it is offered to establish an alibi by the
defendant’s own statement.
D: inadmissible, because it is hearsay not within any exception.

A

B: admissible, because a statement of plans falls within the
hearsay exception for then-existing state of mind.

371
Q

Pike sued Day City Community Church for damages he suffered
when Pike crashed his motorcycle inan attempt to avoid a cow
that had escaped from its corral. The cow and corral belonged to
a farm that had recently been left by will to the church. At trial,
Pike seeks to ask Defendant’s witness, Winters, whether she is a
member of that church.
The question is:
(A) improper, because evidence of a witness’s religious beliefs is
not admissible to impeachcredibility.
(B) improper, because it violates First Amendment and privacy
rights.
(C) proper, for the purpose of ascertaining partiality or bias.
(D) proper, for the purpose of showing capacity to appreciate the
nature and obligation of an oath.

A

(C) proper, for the purpose of ascertaining partiality or bias.

372
Q

A victim was found dead, apparently from a gunshot wound.
A friend of the murder victim was charged with first-degree
murder. At trial, the suspect offered an alibi-defense during his
case-in-chief. On cross-examination, the prosecutor asked the
suspect if it was true that he had been convicted of a misdemeanor
for check fraud 2 years earlier. The suspect’s lawyer objected.
How should the judge rule?
A. overrule the objection because the suspect’s alleged conviction
is admissible to show his tenancy to make false statements
B. overrule the objection because the probative value of the evidence
outweighs the possibility of unfair prejudice against the
suspect
C. sustain the objection because the suspect’s conviction was for
a misdemeanor, not a felony
D. sustain the objection because the conviction was for a non-violent
crime

A

A. overrule the objection because the suspect’s alleged conviction
is admissible to show his tenancy to make false statements
**Defendant is on the stand as “witness” so Atty can impeach him
(credibility)

373
Q

Plaintiff sued Defendant for breach of a commercial contract in
which Defendant agreed to sell Plaintiff all of Plaintiff’s requirement
for widgets. Plaintiff called Expert Witness to testify as to
damages. Defendant seeks to show Expert Witness had provided
false testimony as aw witness in his own divorce proceedings.
This evidence should be:
A. admitted only if elicited from Expert Witness on cross-examination
B. admitted only if the false testimony is established by clear and
convincing evidence
C. exclude because it is impeachment on a collateral issue
D. exclude because it is improper character evidence

A

A. admitted only if elicited from Expert Witness on cross-examination
**“character evidence” are things about one of the PARTIES to the
case
**Atty can ALWAYS ask witness on the stand about things related
to lying or truthfulness

374
Q

A man was arrested and charged with involuntary manslaughter.
the victim died after a head-on collision with the man’s car. The
prosecution claimed that the man fell asleep while operating his
car and drove into oncoming traffic, killing the victim. At trial, the
prosecution called a witness to testify that she was visiting the
man on the morning of the accident and he told her, “i did not get
any sleep last night because of a terrible headache.” This man’s
attorney objects to the witness’s testimony.
How should the court rule on the testimony?
A. it is admissible as non-hearsay
B. it is admissible as a statement of declarant’s mental or physical
condition
C. it is inadmissible as a declaration against interest
D. it is inadmissible as hearsay not within any exception

A

A. it is admissible as non-hearsay
**a statement made by 1 of the parties can come in as “admission”
which is a non-hearsay exception

375
Q

A man brings a breach of contract action against the seller of a
vehicle that he was hoping to buy. The defense attorney calls his
client to testify as to the color of the car that the defendant was
offering for sale. The defendant testifies that it was blue.
On cross-examination, the plaintiff’s attorney asks the defendant if
it was true that he had unlawfully turned the odometer backwards
on the car that he sold to another buyer. The defense attorney
objects.
Is this testimony admissible?
A. no because the testimony is irrelevant
B. no because the question was beyond the scope of direct
examination
C. yes because the defendant put his character in issue by taking
the stand
D. yes because the defendant put his credibility in issue by taking
the stand

A

D. yes because the defendant put his credibility in issue by taking
the stand

376
Q

Charles is on trial for reckless driving. He is guilty of this crime if he knowingly drove fifteen miles an hour or more over the speed limit; he is accused of driving sixty-one miles an hour in a forty-mile an hour zone. His defense is that his speedometer was broken, and that therefore he did not know how fast he was going. He testifies in his own defense and tells the court that he conducted an experiment the day after arrest to determine whether the speedometer was working. He told his brother to get in his car and said: “Drive next to me, and as soon as your speedometer hits sixty-one miles an hour, stick your hand out the window.” Charles then drove next to his brother on the highway, and when his brother stuck his hand out the window, Charles looked down at his own speedometer and saw that it read fifty miles per hour.

Which of the following statements is true?

A. Charles’ statement to his brother telling him to drive at a certain rate and then stick his hand out the window at a certain time was hearsay.

B. Charles’ brother sticking his hand out the window was hearsay.

C. The readout on his brother’s speedometer was hearsay

D. Nothing; there are no hearsay statements in this scenario.

A

B. Charles’ brother sticking his hand out the window was hearsay.

When Charles stuck his hand out of the window, he was making a factual assertion; it was the equivalent of him yelling: “I am now driving at sixty-one miles an hour!” Therefore, his statement is an out-of-court statement offered to prove the truth of the matter asserted. The statement would almost certainly be admissible as a present sense impression, but the question asked about whether the statement was hearsay, not whether it would be admissible.

377
Q

Police had probable cause to believe that drugs were being stored in apartment 5D on the fifth floor of a certain building. They obtained a search warrant and searched the apartment, successfully finding a large quantity of heroin. However, there were no people in the apartment at the time, and the police were unable to determine who owned or lived in the apartment. The police then stationed themselves outside apartment 5D and waited. A teenager and his mother who lived in the building walked by and saw the police officers standing outside the apartment. The teenager turned to his mother and laughed, saying: “Those cops are morons. Big Jim isn’t coming back to his place if there are a bunch of police waiting outside.” A few minutes later an older woman walked by the apartment and the officer asked her: “Do you know where Big Jim lives?” and she pointed at the door of apartment 5D and then walked away.

Police later arrested James Fanning for possession of heroin. At trial, the prosecutor established that in the neighborhood, Fanning is known as “Big Jim.” The prosecutor then called the police officer to the stand, and asked him to testify about what the teenager said and what the older woman said. The evidence is being offered to prove that a person named “Big Jim” lived in apartment 5D. The defendant’s attorney objected to this testimony as hearsay.

How should the court rule?

A. Admit both the statement by the teenager and the pointing by the older woman.

B. Admit the statement by the teenager, but preclude the pointing by the older woman.

C. Preclude the statement by the teenager, but admit the pointing by the older woman.

D. Preclude both the statement by the teenager and the pointing by the older woman.

A

B. Admit the statement by the teenager, but preclude the pointing by the older woman.

The statement by the teenager was non-assertive conduct; that is, the information he meant to convey was that Big Jim would not return home, not that Big Jim did in fact live in the apartment. Thus, that statement is being offered not to prove the truth of the matter being asserted (that Big Jim would not return home), but to prove a fact that can be inferred from the fact that the declarant made the statement (that Big Jim lived in the apartment). The pointing by the older woman, although not an oral statement, was still a “statement” in the sense that it was intended to communicate a fact (that Big Jim lived in that apartment), and the prosecutor wants to admit this statement to prove the fact being asserted—thus, it is a hearsay statement.

378
Q

Officer Jackson arrested William after William sold heroin to an undercover officer. Officer Jackson searched William pursuant to the arrest, and found a cell phone in William’s pocket. The cell phone had a recent text message in plain view on the front: “How much stuff did you sell tonight?” Assume there are no Fourth Amendment problems with Officer Jackson reading the text. Officer Jackson now wants to testify at William’s trial and tell the jury about the text message he read off the cell phone.

Is Officer Jackson’s testimony about the text message admissible?

A. Yes, but only if the prosecutor locates the individual who sent the text message and s/he also testifies.

B. Yes, it is admissible as evidence that William was selling things that night.

C. Yes, it is admissible, but only to prove that William had knowledge of the text.

D. No, the text is not admissible.

A

B. Yes, it is admissible as evidence that William was selling things that night.

The statement is not hearsay, because the declarant was not trying to communicate any fact to anyone. Instead, we are inferring a fact about William (that he was selling something that night) from the fact that a third party expected him to be selling something that night.

379
Q

Agents from the Drug Enforcement Agency got an anonymous tip that someone was selling drugs out of a black car parked outside Happy Times bar. When they arrived at the Happy Times bar, they saw a black car in the parking lot. As they were waiting outside the bar, a woman came out of the bar and opened up the trunk of the car. As soon as she opened the trunk, one of the agents approached her and showed her his badge. When the woman saw the badge, she immediately took off running back into the bar. One of the agents caught her, and another one looked inside the open trunk and found a paper bag with ten small baggies of cocaine inside.

The woman was arrested and identified as Phoebe Gates. She was charged with knowingly possessing cocaine. At trial, the DEA agent wanted to testify about the fact that Phoebe ran as soon as she saw the law enforcement badge. Phoebe’s attorney objected.

Is the fact that Phoebe ran admissible?

A. The fact that Phoebe ran is admissible because it tends to prove she knew there was cocaine in the trunk.

B. The fact that Phoebe ran is only admissible, but only if Phoebe testifies.

C. The fact that Phoebe ran is inadmissible because it is irrelevant.

D. The fact that Phoebe ran is inadmissible because it is assertive conduct and therefore barred by the hearsay rule.

A

A. The fact that Phoebe ran is admissible because it tends to prove she knew there was cocaine in the trunk.

The prosecutor has to prove the Phoebe had knowledge of the cocaine in order to prove possession, and the running tends to prove knowledge. Phoebe was not trying to assert anything by running, so her running does not constitute assertive conduct.

380
Q

On the morning of January 24th, Larry and his wife Ingrid got into an argument. Larry slapped Ingrid in the face and threatened to break her nose if she talked back to him again. Ingrid was very upset and began crying, but she did not call the police. Instead, she went to her job as a nurse and worked a full day at the hospital. At the end of her shift, another nurse noticed that she had a mark on her face and asked her what happened. She shook her head and said: “Larry hit me again. Next time he said he would break my nose. I really don’t know what to do.” Ingrid’s co-worker was shocked at the news and convinced Ingrid to call the police. Larry was arrested and charged with assault and menacing.

At trial, the prosecutor called Ingrid to testify about what happened. Ingrid took the stand, but then testified that Larry never hit her or threatened her. The prosecutor then called Ingrid’s co-worker to testify about what Ingrid told her at the end of the day. The defense attorney objected to this testimony.

Is the testimony of Ingrid’s co-worker admissible?

A. The co-worker’s testimony is admissible to prove the truth of the matter asserted because it is a statement made for medical diagnosis or treatment.

B. The co-worker’s testimony is admissible to prove the truth of the matter asserted because it is an excited utterance.

C. The co-worker’s testimony is inadmissible to prove the truth of the matter asserted, but it is admissible to impeach Ingrid.

D. The co-worker’s testimony is inadmissible for any purpose.

A

C. The co-worker’s testimony is inadmissible to prove the truth of the matter asserted, but it is admissible to impeach Ingrid.

The statement is hearsay, and there is no exception to admit it. However, it contradicts Ingrid’s testimony, so it is admissible to impeach her under Rule 613 as a prior inconsistent statement. Since it is not a collateral statement (it is relevant for a purpose other than to impeach Ingrid), the prosecutor is allowed to use extrinsic evidence (such as the co-worker’s testimony) to prove the statement was made.

381
Q

Ronald is a truck driver who works as an independent contractor, hauling freight for small businesses across the country. A recycling center in California contacted Ronald and told him it would pay him a set fee if he transported ten truckloads of aluminum cans from their warehouse near San Francisco to the processing plant near Los Angeles. Ronald transported all ten truckloads, and the recycling center paid him $10,000. Ronald claims that the recycling center had promised to pay him $15,000, and he sued them for the difference.

At trial, the president of the recycling center takes the stand and testifies: “When we first contacted Ronald, I told him we would pay him $10,000, to haul ten loads.” Ronald objects this testimony as hearsay.

Is the president’s testimony admissible?

A. The testimony should be precluded as hearsay.

B. The testimony is hearsay, but it is admissible because the declarant (the president of the center) is now on the stand testifying.

C. The testimony is hearsay, but it is admissible because the witness is a party in the case.

D. The testimony is not hearsay, because it is not offered for the truth of the matter asserted, but instead to prove that an offer with specific terms was made.

A

D. The testimony is not hearsay, because it is not offered for the truth of the matter asserted, but instead to prove that an offer with specific terms was made.

The statement is being offered to prove that the statement was made (since the statement itself is a legally operative fact), not for the truth of the matter asserted. Terms of contracts are not considered hearsay, since legally all that matters is that the declarant made the statement. The president’s testimony can always be challenged on cross-examination; that is, just because it is admissible does not mean the jury will accept it as fact.

382
Q

Peter is suing Dr. Denard for malpractice. Peter had suffered from appendicitis, and Dr. Denard was the doctor who removed his appendix. Unfortunately, Dr. Denard left a surgical sponge inside of Peter’s body, which resulted in severe complications.

At trial, Dr. Denard claims that, consistent with standard medical practice, he relied on his nurse during the operation to keep track of how many sponges he had used. He testifies that as the operation was ending, he asked his nurse how many sponges he had put into Peter’s body, and she had said “four.” In fact, there had been five. Dr. Denard removed four sponges and closed the wound, inadvertently leaving a sponge inside of him.

Peter objects to the part of Dr. Denard’s testimony in which he testifies that the nurse answered “four” in response to his question. Peter argues this statement should be barred by the hearsay rule. The nurse has not testified in this case.

Is Dr. Denard’s statement admissible?

A. The statement is admissible, because it is not being offered for the truth of the matter asserted.

B. The statement is admissible, because it is a statement made for medical treatment or diagnosis.

C. The statement is admissible, because it is a business record.

D. The statement is barred by the hearsay rule.

A

A. The statement is admissible, because it is not being offered for the truth of the matter asserted.

The defendant is offering this statement not to prove the truth of the matter asserted (the truth of the matter asserted is that there were four sponges in Peter’s body, but both parties concede that this was not true—thus, the defendant is clearly not trying to prove the truth of the matter asserted). Instead, the statement is relevant because it proves knowledge (or lack of knowledge) on the part of the defendant doctor—it is being offered merely to prove that the nurse said the word “four” and thus the doctor reasonably believed there were only four sponges left in the defendant’s body.

383
Q

Nick is on trial for killing Stanley. Nick admits that he killed Stanley, but he argues that the crime was involuntary manslaughter and not murder. Murder is defined as knowingly killing a human being. Manslaughter is defined as knowingly killing a human being under extreme emotional distress brought on by a provocation that would cause a reasonable person to feel such distress. Nick testifies at trial. He says that thirty minutes before the killing, a friend of his named Nancy approached him while he was at a restaurant and told him that she had just come from his house. She further explained that she, Stanley, and Nick’s wife Nadine had been together at the house using cocaine, that Nadine and Stanley had gotten into an argument, that Stanley had started beating Nadine with a chair leg, and that Nadine was beaten so badly that she was taken to the hospital. Nick immediately left the restaurant and ran to his house, where he saw Stanley talking to a police officer outside his house. Nick then took out a switchblade and stabbed Stanley in the chest, killing him.The prosecutor objects to Nick’s testimony. She agrees that Nick can testify that Nancy came into the restaurant and said something, and he can testify that he left the restaurant and went to his house and stabbed Stanley. However, she argues that anything that Nancy told him is hearsay and should be precluded.

Are Nancy’s statements admissible?

A. Nancy’s statements to Nick are admissible, but only if she was under the stress of the excitement of the event when she said them.

B. Nancy’s statements to Nick are admissible, but only if she is unavailable for trial.

C. Nancy’s statements to Nick are admissible because they are not being offered to prove the truth of the matter asserted.

D. Nancy’s statements to Nick are barred by the hearsay rule.

A

C. Nancy’s statements to Nick are admissible because they are not being offered to prove the truth of the matter asserted.

The hearsay rule does not bar these statements, since they are not being offered for the truth of the matter asserted. The jury does not really need to know whether Stanley actually did beat Nick’s wife with a chair leg, only whether Nick believed that Stanley did so. Thus, the statements are relevant for a non-hearsay purpose: to demonstrate that Nick’s state of mind was sufficient to deserve the mitigation to manslaughter.

384
Q

One day Debra, a four-year-old girl, disappeared from her front yard where she had been playing. A few hours later her family got a ransom demand from her kidnapper. After three days of negotiations, Debra was released in a nearby park in exchange for $10,000.

Police officers investigating the crime developed probable cause to believe that Greg was the kidnapper. They arrested Greg and he was tried for kidnapping. Greg’s defense was mistaken identity; he claimed he was not the kidnapper.

The prosecutor did not want to put Debra on the stand because of her young age. However, Debra described to her father the location where she had been held for three days. She described a basement with a fish tank with large red fish, a Superman poster on the wall, and bright blue carpeting. The prosecutor wants to call the father to the stand to testify that Debra described the basement this way. The prosecutor will then call a police officer who will testify that he searched Greg’s home, and that Greg has a basement with a fish tank with large red fish, a Superman poster on the wall, and bright blue carpeting.

Should the judge allow the father to testify about what Debra told him?

A. Yes, because Debra is unavailable.

B. Yes, because Debra’s statements to the father are not hearsay.

C. Yes, because as the victim Debra is a party to this case.

D. No, Debra’s statements to her father are inadmissible.

A

B. Yes, because Debra’s statements to the father are not hearsay.

Debra’s statements are being offered to prove that Debra has knowledge of what Greg’s basement looked like. They are not being offered to prove the truth of the matter asserted—that is, they are not being offered to prove what Greg’s basement looks like. The officer’s testimony will establish that fact. Instead, they are being offered to prove that Debra has knowledge that would only be available to her if she had been kept in the room during her kidnapping.

385
Q

Jackson was pulled over after he was weaving across lanes of traffic. After observing him for a few minutes, the officer who pulled him over came to the conclusion that Jackson was drunk. Jackson refused to take a Breathalyzer test. The officer arrested Jackson for driving while intoxicated.After the arrest, the officer searched the car and found an empty bottle of wine and an empty paper bag. Inside the bag was an automated receipt from a liquor store that recorded a purchase of a bottle of wine with a credit card that matched Jackson’s credit card number. The time of purchase listed on the receipt was 9:30 PM on that night. Jackson had been pulled over at 10:30 PM.

At trial, the prosecutor wants to admit the receipt from the store to prove that Jackson bought the wine at 9:30 PM.

Is the receipt admissible?

A. No, the receipt is hearsay and no exception applies.

B. Yes, the receipt is admissible, but the prosecutor must first demonstrate that the machine that generated the receipt is reliable.

C Yes, the receipt is admissible.

D Yes, the receipt is admissible. It is hearsay, but a hearsay exception applies.

A

B. Yes, the receipt is admissible, but the prosecutor must first demonstrate that the machine that generated the receipt is reliable.

The receipt is not hearsay, because any information generated solely by a machine is not hearsay. However, before information that is generated by a machine can be admitted, the party must establish that the machine is reliable.

386
Q

Hank asked his friend Liam to invest in Hank’s restaurant business. Liam agreed, and gave Hank $100,000. Six months later the restaurant failed, and the money was gone. Liam asked Hank for some of his money back, and Hank said the money was all lost in the failed business venture. Liam then sued Hank, seeking $50,000. At trial, Liam testified on direct that he and Hank had made an oral contract—Hank told him that if Liam gave him $100,000 and the restaurant failed, Hank would give Liam half of his money bank. On cross-examination, Hank’s attorney asked Liam: “Isn’t it true that after the restaurant failed, you simply made up this story about an oral contract in which you get half your money back?” Liam denied that he made the story up. On re-direct, Liam’s attorney offers a letter written by Liam to his wife, dated on the day after Liam gave his money to Hank, in which Liam wrote: “I know this seems like a risky endeavor, but Hank has promised to give me half the money back if the restaurant falls through.”

Are there any hearsay problems with any of the evidence described in this scenario?

A. Yes—Liam’s testimony about the alleged oral contract between himself and Hank is an out-of-court statement offered to prove the truth of the matter asserted, and no hearsay exception applies.

B. Yes—the letter Liam wrote to his wife about the alleged oral contract is an out-of-court statement offered to prove the truth of the matter asserted, and no hearsay exception applies.

C. No, all of Liam’s testimony is admissible

D. Yes, both the testimony about the alleged oral contract and the letter Hank wrote to his wife are inadmissible hearsay.

A

C. No, all of Liam’s testimony is admissible.

Testimony about a contract is never hearsay because the statements that make up the contract are legally operative statements; there is no “truth” being asserted. Also, Liam’s letter to his wife is an out of court statement being offered to prove the truth of the matter asserted (that a contract was made), but Rule 801(d)(1)(B)(i) applies as an exception to the hearsay rule. This is a consistent prior statement by the witness which is being used to rebut an allegation of recent fabrication (the allegation made on cross-examination that Liam made up the story about the oral contract once the restaurant failed).

387
Q

Gary is suing Wendell’s Hardware store, claiming that he purchased an industrial-strength power saw on January 24th, and that the saw broke down after only five minutes of operation. Wendell’s Hardware claims that it never sold Gary the saw. In its case-in-chief, Wendell’s calls its store manager, who will testify that the company regularly keeps records of every item that it sells and those records are kept in the regular course of business. He will then seek to admit the company’s sales records for the entire month of January, in which there is no sale of an industrial-strength power saw. Gary objects to this evidence.

How should the court rule?

A. Sustain the objection, because the evidence is irrelevant.

B Sustain the objection, because the source of the information indicates a lack of trustworthiness.

C Sustain the objection, because evidence that the company did not make the record is a hearsay statement, and no exception applies.

D. Overrule the objection and allow the evidence.

A

D. Overrule the objection and allow the evidence.

The fact that someone does not make a statement is usually not considered to be assertive conduct, since the “declarant” was not trying to communicate something with his non-statement. Even if it were considered to be assertive conduct, Rule 803(7) allows absence of business records to be admitted.

388
Q

Dr. Gordon was the anesthesiologist during a routine tonsillectomy when the patient suddenly went into cardiac arrest and died. As it turns out, the patient suffered from an unusual heart condition, which made him susceptible to cardiac arrest if a certain kind of anesthetic was used. Dr.Gordon claimed that she had no way of knowing about this unusual heart condition, since the patient had not told her about it and it did not appear anywhere in the patient’s chart.

The patient’s family sued Dr. Gordon, claiming that the patient had in fact informed Gordon of the heart problem, and that the information had been on the chart, but that Dr. Gordon had gone back and altered the chart after the patient’s death in order to protect herself.

During the trial, Dr. Gordon offered to admit that she is fully insured for medical malpractice, and that if she was indeed responsible for the death, the insurance company will pay for any and all damages. What is the proper ruling?

A. The evidence is probably barred by Rule 411.

B. The evidence is probably not barred by Rule 411 because it shows that Dr. Gordon had less of an incentive to alter the chart after the death.

C. The evidence probably not barred by Rule 411 because it is being offered to show ownership or control.

D. The evidence is probably not barred by Rule 411 because it is relevant to show bias on the part of Dr. Gordon.

A

B. The evidence is probably not barred by Rule 411 because it shows that Dr. Gordon had less of an incentive to alter the chart after the death.

389
Q

Sandy is arrested by the police for robbery. In which of the following situation is Sandy’s confession LEAST likely to be admissible if offered by the prosecutor against Sandy? (Assume all Miranda rules have been complied with; i.e., there are no constitutional problems with any of the statements).

A. A few hours after the arrest, a police officer tells Sandy that she should “come clean,” and if she tells the truth now, the prosecutor “might cut you a deal.” Sandy confesses to the robbery.

B. A few hours after the arrest, Sandy meets with the prosecutor. The prosecutor says: “You’re in a lot of trouble. I may be able to help you out, but I need to know your side of the story.” Sandy confesses to the robbery.

C. A few hours after the arrest, a police officer tells Sandy: “I just got off the phone with the prosecutor. She told me that she would be willing to charge you with a misdemeanor if you plead guilty at the arraignment and tell us right now the name of the person who committed this robbery with you.” The police officer is lying—he has not spoken to the prosecutor—but there is no way for Sandy to know this. Sandy confesses to the robbery.

D. A few hours after the arrest, Sandy meets with the prosecutor. The prosecutor says: “We have your partner in the next room, and he is telling us everything about what happened. He’s hanging you out to dry. If I were you, I would start cooperating too—otherwise you are going to be left out in the cold.” The prosecutor is lying—the partner has not been apprehended—but there is no way for Sandy to know this. Sandy confesses to the robbery.

A

C. A few hours after the arrest, a police officer tells Sandy: “I just got off the phone with the prosecutor. She told me that she would be willing to charge you with a misdemeanor if you plead guilty at the arraignment and tell us right now the name of the person who committed this robbery with you.” The police officer is lying—he has not spoken to the prosecutor—but there is no way for Sandy to know this. Sandy confesses to the robbery.

390
Q

Frank “The Snitch” Dunaway was arrested by an undercover officer when he tried to sell the officer five kilos of cocaine. As he was being arrested, Dunaway said: “You don’t want me! You want my boss Victor! He gave me all these drugs to sell! I can tell you everything you need to know! Can we make a deal?” After Dunaway was arraigned, the officer took him to the prosecutor, where the prosecutor agreed to recommend probation if he pled guilty and testified against Victor. Dunaway then admitted to the crime and incriminated Victor in his statements. Later, pursuant to the agreement, Dunaway testified in front of the grand jury and admitted selling the drugs and incriminated Victor.

When it came time to enter his plea, Dunaway refused to plead guilty, stating that he wanted to take the case to trial. At trial, the prosecutor sought to admit the following statements:

(i) Dunaway’s statements to the police officer.
(ii) Dunaway’s statements to the prosecutor.
(iii) Dunaway’s testimony in the grand jury.

Should the court admit or preclude these statements?

A. Preclude all three statements as hearsay.

B. Preclude all three statements under Rule 410.

C. Admit (ii) and (iii), but preclude (i).

D. Admit (i) and (iii), but preclude (ii).

A

D. These statements are made by a party opponent, so they are exempted from the hearsay rule by Rule 801(d)(2). Rule 410 only precludes statements made during a plea bargaining session. When the defendant makes statements to the arresting police officer, he is not engaging in plea bargaining, since the officer has no authority to bargain. And when the defendant is testifying in front of the grand jury, he is not making statements during a plea bargaining session. However, the statements made to the prosecutor are inadmissible, since they were made during a plea bargaining session.

391
Q

Gary and Thomas were arrested for conspiring to kidnap a six-year-old child in their neighborhood. A few hours after the arrest, Gary was taken to the prosecutor’s office and the prosecutor read him his Miranda rights. The prosecutor talked with Gary for over an hour, and Gary ultimately agreed to plead guilty and testify against Thomas in exchange for a reduction in the charges against him.

Gary later changed his mind and decided not to plead guilty. Thomas and Gary were tried separately for the conspiracy. Gary testified in his own trial, and Thomas also called him as a witness in Thomas’ trial. When, if ever, can the prosecutor use the statements that Gary made to her after his arrest?

A. In the trial against Gary, in order to impeach Gary if his testimony is inconsistent with his statements.

B. In the trial against Gary, as substantive evidence against Gary.

C. In the trial against Thomas, in order to impeach Gary if his testimony is inconsistent with his statements.

D. The prosecutor cannot use Gary’s statements.

A

C. The statements made by Gary in the prosecutor’s office were statements made during plea discussions, and therefore the prosecutor would be barred under Rule 410 from using them against Gary for any purpose. However, Rule 410 does not bar the prosecutor from using them against another defendant, only against “the defendant who made the plea or participated in the plea discussions.”

392
Q

Sergeant Joe Friday arrests Dutch under suspicion of committing an armed robbery. Friday reads Dutch his Miranda rights and Dutch states that he understands his rights and waives them. Dutch then gives Friday an alibi for the crime.

Friday responds: ‘Look, both you and I know that story isn’t true. You really should be honest with me. Things will go better for you if you cooperate.” Dutch then gives Friday a full confession.

The prosecution later tries to admit the confession at Dutch’s trial, and the defense attorney objects. The confession is:

A. Irrelevant and therefore inadmissible.

B. Inadmissible under Rule 410.

C. Admissible only if Dutch takes the stand and testifies to facts inconsistent with the statement.

D. Admissible.

A

D. Admissible

393
Q

Harry Sherman had too much to drink one night at a college party, and in a drunken rage, he threw a chair out of a dormitory window. The police were called and Sherman was arrested for criminal damaging. At arraignments, Sherman pled guilty to the charge.The next week, the college sued Sherman for the damage done to the window and the chair. At trial, the attorney for the college wants to admit Sherman’s guilty plea for the incident. Sherman objects. What is the proper ruling?

A. The prior plea is barred by Rule 410.

B. The prior plea is barred by Rule 403, since it is of very little probative value and would unfairly prejudice the jury.

C. The prior plea is not barred by Rule 410, because this is a civil case, and Rule 410 only applies to criminal cases.

D. The prior plea is not barred by Rule 410, because it is a completed plea.

A

D. Rule 410 only applies to pleas of guilty which were later withdrawn, or to pleas of nolo contendere (otherwise known as “no contest” pleas).

394
Q

Westin Coal owns and operates dozens of coal mines in the state. One day there was a cave-in on one of their mines, and three miners died. The miners’ families are now suing Westin, arguing that the mines were negligently maintained. In their case-in-chief, the plaintiffs called a former Westin employee, who testified that no safety inspection had been conducted on the mine for over two years.

During the defendant’s case-in-chief, Westin called an employee from Prescott Insurance, a large national insurance company. The Prescott employee will testify that Westin has an insurance policy with Prescott, and that as part of that policy, Prescott employees conduct independent safety inspections on all of the Westin mines every three months. The Prescott employee will then authenticate and admit a business record from Prescott’s files which confirms that Prescott conducted regular safety inspections up until the time of the cave-in.

The plaintiffs object to the testimony of the Prescott employee. This evidence is:

A. Inadmissible as evidence of insurance under Rule 411.

B. Inadmissible as a subsequent remedial measure under Rule 407.

C. Inadmissible under Rule 402 because it is irrelevant.

D. Admissible.

A

D. Although Rule 411 prohibits evidence that a party was or was not insured, it only prohibits the evidence if it is used to prove that the party acted negligently or otherwise wrongfully. Here, Westin is offering the evidence of insurance as proof that the safety inspections were in fact conducted.

395
Q

Jane Young, a six-year-old girl, was given antibiotics for a minor infection. Unfortunately, she was allergic to the antibiotics and she became very ill, spending over two weeks in the hospital. After Jane got ill, the doctor met with his two nurses and told them that they needed to conduct a basic allergy test on every new patient with an infection before the doctor could determine which antibiotics should be prescribed.

Jane’s parents sued the doctor who prescribed the antibiotics, alleging that he should have known that Jane was allergic to this specific strain of antibiotics and thus should not have prescribed them. The Young family had health insurance, so they did not have to pay any expenses aside from a $100 deductible for the hospital stay. However, they sued the doctor seeking medical expenses for the hospital stay (which would ultimately go to reimburse the insurance company, except for the $100 deductible) and for mental suffering (which the Young family would keep for themselves).

At trial, Daniel Young, Jane’s father, testified about his mental suffering, saying that there were many days when he did not know whether his daughter would live or die, and that he had to seek therapy in order to recover from the trauma. Daniel also testified that he suffered further mental anguish because “Hospitals are so expensive, and I would lie awake at night wondering how we were going to pay for her care and whether we would be able to afford the treatment she needed.”

Which of the following evidence is admissible?

A. In their case-in-chief, the Young family seeks to admit evidence that the doctor told his nurses to conduct allergy tests on all new patients with infections.

B. In his case-in-chief, the doctor defendant seeks to admit evidence that the Young family was fully insured for the hospital stay and only paid a $100 deductible.

C. Neither (A) nor (B) are admissible.

D. Both (A) and (B) are admissible.

A

B. Although Rule 411 precludes evidence of insurance to prove negligence or wrongful conduct, the evidence of insurance in this context is being offered to prove that the Young family did not suffer additional mental trauma because they were worried that they would be unable to pay the medical expenses. In other words, Daniel Young “opened the door” to the evidence of the insurance being admissible when she claimed that she was worried about paying the medical expenses.

396
Q

Which of the following pieces of evidence is irrelevant under Rule 401?

a. During a political rally protesting police treatment of racial minorities, Donald allegedly broke a window of the police headquarters. Donald is now being prosecuted for criminal damaging. The elements of criminal damaging are: recklessly or knowingly causing damage to state property without permission or authority. Donald is Black. In Donald’s defense, he wishes to present statistical evidence that the police use excessive force when arresting Black suspects at a rate three times more often than when they arrest white suspects.

b. Stacy allegedly called Joshua on the telephone, used an anti-Semitic slur against him, and threatened to kill him and his family. She is charged with aggravated telephone harassment. The elements of aggravated telephone harassment are: knowingly communicating with another by telephone and knowingly causing fear of injury by reason of the victim’s race, ethnicity, or religion. The prosecutor wishes to present evidence that Joshua, the victim, is Jewish.

c. Bernard is charged with embezzling hundreds of thousands of dollars from his company. He was arrested after a three-week investigation, and during that time, he and his lawyer went to the prosecutor’s office twice to attempt to explain his conduct. The prosecutor wishes to introduce evidence that the day before Bernard was arrested, he went to his office on a Sunday and shredded every document in his file cabinet.

d. Debra is charged with driving a vehicle while under the influence of alcohol after she ran her car off the road and crashed into a tree. At the scene of the accident, she refused to take a Breathalyzer test. However, she was taken to the hospital and blood was drawn from her without her consent while she was being treated for her injuries. The prosecutor wishes to introduce evidence that the blood—which was drawn from her body approximately one hour after the accident—showed a blood-alcohol level of .15, well above the legal limit of .08.

A

a. is correct. The only relevant issues in the case are whether Donald caused the damage, whether he did so knowingly or recklessly, and whether he had permission. Whether he believed his actions were justified in order to make a political point is irrelevant to determining his guilt.

b. is incorrect. Ordinarily an individual’s religious beliefs would be irrelevant to a case—and they are inadmissible under Rule 610 if offered to impeach a witness—but in this case the prosecutor must prove that the defendant actually committed this crime with a discriminatory purpose. This fact is more likely to be true if the victim is in fact a member of the religion that the defendant is discriminating against.
c. is incorrect. Bernard was obviously aware that the prosecutor was considering charges against him, and thus his shredding of the documents is relevant to prove he had consciousness of guilt—that is, that he believed that the documents would somehow incriminate him. From his consciousness of guilt, a reasonable jury could infer that Bernard was, in fact, guilty.
d. is incorrect. The evidence that Debra’s blood alcohol level was so high only an hour after the crash is directly relevant to whether she was drunk when she crashed the car, because it makes the fact that she was drunk at the time of the accident more likely than it would be without the evidence. The fact that she did not consent to the blood test might mean the blood is excluded on constitutional grounds, but it does not affect its relevance under the rules of evidence.

397
Q

Garrett’s dog bit Kyle in the leg. Kyle sued Garrett, seeking monetary damages. Under the applicable law, every dog owner is strictly liable for any injuries caused by his dog. At trial, Kyle offers evidence that Garrett’s dog bit three other people in the year before biting Kyle. In his defense, Garrett offers evidence that Kyle kicked the dog just before the dog bit Kyle. Is either piece of evidence admissible?

a. No, because both pieces of evidence are irrelevant.
b. The evidence that Garrett’s dog bit three other people is inadmissible as propensity evidence, but the evidence that Kyle kicked the dog is admissible.
c. The evidence that Kyle kicked the dog is inadmissible because it is unfairly prejudicial, but the evidence that Garrett’s dog bit three other people is admissible because it shows that Garrett had knowledge that his dog was dangerous.
d. Both pieces of evidence are admissible.

A

a. is correct. Since the law is strict liability, it is legally irrelevant whether the dog was known to be dangerous or whether Kyle provoked the dog.

b. is incorrect. The evidence that Garrett’s dog bit three other people is not propensity evidence because it also proves that Garrett knew the dog was dangerous, but it is still inadmissible on relevance grounds. The evidence that Kyle kicked the dog is inadmissible because the applicable law is strict liability, so it is irrelevant whether Kyle provoked the dog.
c. is incorrect. The fact that Garrett’s dog bit three other people is not admissible because Garrett is liable for the dog bite whether or not the dog is known to be dangerous.
d. is incorrect. The two pieces of evidence do not prove anything that the jury needs to know in order to decide the case.

398
Q

A police officer pulled over Sam’s car after seeing it swerve numerous times into the oncoming lane of traffic. The officer ordered Sam out of the car and told him to take a Breathalyzer test. Sam refused. Based on Sam’s erratic driving and Sam’s refusal to take the test, the officer arrested Sam for driving while under the influence of alcohol. A person is guilty of this crime if his ability to drive a car is impaired because of consumption of alcohol.
At trial, the prosecutor wants the officer to testify that Sam refused to take the Breathalyzer test. Is this evidence relevant?

a. Yes, because his refusal to take the test makes it more probable that he was in fact intoxicated.
b. Yes, because his refusal to take the test is sufficient to prove that he was in fact intoxicated.
c. No, because he may have had many legitimate reasons for not wanting to take the test.
d. No, because there is no other evidence that he was intoxicated except for his swerving, and his swerving may be unrelated to intoxication.

A

a. is correct. Although there are other reasons why he may have refused to take the test, the legal standard for relevance is very low—the proffered evidence only needs to have some tendency to make a fact in consequence more or less probable. Since refusing to take the Breathalyzer may be probative of consciousness of guilt, the evidence is relevant.

b. is incorrect. This is not the correct standard for relevance—the party offering the evidence need not argue that the evidence is sufficient to prove a fact in consequence, only that the evidence makes it more likely that a fact in consequence is true
c. is incorrect. Just because there are other innocent reasons for his conduct does not mean that the conduct does not have some probative value in determining whether he is guilty. As long as his refusal to take the test has some tendency to make a fact in consequence more likely, it is relevant.
d. is incorrect. There is no requirement that evidence be corroborated in order to be relevant. As long as the evidence makes a fact in consequence more likely, it is relevant.

399
Q

Damon is on trial for vehicular manslaughter after he allegedly killed a ten-year-old child while driving. The prosecutor has three witnesses. The father of the victim will testify that he saw Damon swerve off the road and up onto the lawn, where he struck the child. The police officer who responded to the scene will testify that Damon was crying and distraught a few minutes after the incident, and said that he only looked away from the road for a second to check his cell phone. A coroner will also testify that he examined the child’s body, and that the cause of death was massive trauma to the chest, likely caused by a car.
Damon objects to the father’s testimony as unfairly prejudicial. He notes that the father will no doubt be very emotional when he testifies about his son’s death, and that this extreme amount of emotion will unfairly influence the jury. Damon is willing to agree to admit that he swerved off the road, up onto the lawn, and struck the child. Damon says that once he has admitted this fact, the father’s testimony has little extra probative value and should be precluded. The prosecutor still wants the father to testify. What should the court do?

a. Preclude the father from testifying even if Damon does not admit to these facts, because the unfair prejudice of his testimony in any form substantially outweighs its probative value.
b. Once Damon has admitted to these facts, the father should be precluded from testifying.
c. Allow the father to testify, but give the jury a limiting instruction that the father’s testimony may not be reliable because of his close connection to the victim.
d. Allow the father to testify with no limiting instruction.

A

d. is correct. Although there will be some level of unfair prejudice when the jury sees the father react emotionally on the stand, his testimony is extremely probative because he is an eyewitness to the event. Courts routinely allow victims and others with close relationships to the case testify even though they may display emotion; the unfair prejudice does not substantially outweigh the probative value of this testimony. Furthermore, in Old Chief v. United States (U.S. 1997), the Supreme Court stated that a party should only forced to accept an admission instead of offering live testimony if the probative value of the live testimony is extremely low.

a. is incorrect. The father’s testimony is extremely probative, since he is an eyewitness to the incident, and although his testimony will carry some danger of unfair prejudice, the danger does not substantially outweigh the probative value.
b. is incorrect. In Old Chief v. United States (U.S. 1997), the Supreme Court stated that a party should only forced to accept an admission instead of offering live testimony if the probative value of the live testimony is extremely low. In this case, the probative value of the father’s testimony is extremely high, since he is an eyewitness to the incident, and the prosecutor has the right to present his case with persuasive evidence.
c. is incorrect. There is no reason to believe that the father’s testimony will be unreliable because of his relationship to the victim, only that it will be emotional. And judges are precluded from commenting on the credibility of witnesses under Rule 605. Any bias that the father might have can be explored on cross-examination.

400
Q

While shopping in a busy grocery store, a five-year-old girl accidentally got separated from her mother. After a few minutes, a woman approached the girl and asked her if she was lost. When the girl replied that she was, the woman said, “Your mommy was in an accident and is in the hospital. She told me to take you there.” The girl went with the woman in her car. After driving for a few minutes, the woman saw police cars approaching. The woman immediately pulled over and forced the girl to get out of her car before speeding away.

Based upon eyewitness statements, the woman was apprehended and prosecuted for kidnapping the girl. At trial, the prosecution called the girl to testify about the kidnapping.

Which of the following is a true statement with regard to the admissibility of the girl’s testimony?

A. The testimony is not admissible, because the testimony of a five-year-old has little probative value.

B. The testimony is not admissible, because a five-year-old is too young to understand the importance of telling the truth.

C. The testimony is admissible if the prosecution offers evidence to convince the judge that a five-year-old is competent to testify.

D. The testimony is admissible if the court determines that the girl has personal knowledge and understands the difference between the truth and falsehood.

A

D. The testimony is admissible if the court determines that the girl has personal knowledge and understands the difference between the truth and falsehood.

401
Q

A father and son are charged with burglary. Prior to the trial, the prosecutor approached the son and asked him if he would be willing to testify against his father in exchange for a reduced sentence. After discussing the son’s role in the burglary and some negotiation, the prosecutor and the son’s defense attorney reached a settlement agreement. The son pleaded guilty, and was called to testify against his father. On cross-examination of the son, the father’s attorney brought up the fact that the son was also originally charged with the burglary, and asked whether it was true that he received a lesser sentence for agreeing to testify against his father. The prosecutor objected to this line of questioning.

How should the court rule on the prosecutor’s objection?

A. The objection should be sustained, because the response calls for hearsay.

B. The objection should be sustained, because it is against public policy to introduce evidence of a plea agreement.

C. The objection should be overruled, because the question concerns bias.

D. The objection should be overruled, because the son’s sentence is irrelevant to the father’s guilt.

A

C. The objection should be overruled, because the question concerns bias.

402
Q

Louis is on trial for robbing a convenience store. The prosecutor seeks to call his girlfriend as a witness to testify that Louis was a regular user of heroin, and spent over $50 on heroin every day. She will also testify that Louis owed his heroin dealer over $500. Louis objects to this testimony as inadmissible character evidence. The judge should:

A. Admit the testimony about Louis’ heroin use and his debt to the dealer to prove motive.

B. Admit the testimony about Louis’ heroin use and his debt to prove that he has a propensity to commit crimes.

C. Admit the testimony about Louis’ heroin use and debt only after Louis calls a character witness to testify that he is law-abiding and honest.

D. Preclude the testimony about Louis’ heroin use and debt.

A

A. Admit the testimony about Louis’ heroin use and his debt to the dealer to prove motive.

Correct. The evidence has significant probative value in proving that Louis had a motive to commit the crime, and it is admissible for that purpose under Rule 404(b). If Louis requests it, the judge will give a limiting instruction telling the jury not to consider the evidence for character purposes.

403
Q

Stanley is accused of shooting and killing his wife. Stanley admits that he shot his wife, but claimed that it was an accident—that he had picked up the gun to see whether it was loaded and it went off. The prosecutor wishes to call Greg as one of her witnesses. Greg is one of Stanley’s friends from the local bar, and Greg will testify that two weeks before Stanley shot his wife, Stanley offered Greg $1,000 if Greg would break into Stanley’s house and kill his wife. Greg refused the offer, but never went to the police with the information until after Stanley’s wife was killed. Stanley has not been charged with a crime for his offer to Greg. Can Greg testify about Stanley’s offer?

A. Yes, to prove intent and lack of accident.

B. Yes, to prove identity.

C. No, because character evidence is inadmissible.

D. No, because it is unfairly prejudicial.

A

A. Yes, to prove intent and lack of accident.

Correct. Greg’s testimony might have been admissible under Rule 404(b) even if Stanley were not claiming that the shooting was an accident, in order to prove that Stanley intended to kill his wife. Since Stanley is claiming that the shooting was an accident, it increases the probative value of Greg’s testimony significantly in order to rebut Stanley’s claim.

404
Q

Harry is on trial for robbery. He calls Diane, a character witness, who testifies that she has known Harry for fifteen years and in her opinion he was an honest person who would never steal anything from anyone. On cross-examination, the prosecutor asks Diane: “Did you hear about the time that Harry was arrested for stealing a car two years ago?” Harry objects to the question. How should the judge rule on Harry’s objection?

A. Overrule the objection and force Diane to answer.

B. Overrule the objection and force Diane to answer. If the defense attorney requests, give a limiting instruction telling the jury that this is not meant to be evidence that Harry actually committed a prior theft, only a method of testing Diane’s credibility and knowledge of Harry’s character.

C. Sustain the objection, because the question is about an arrest, not a conviction.

D. Sustain the objection unless Harry has testified in the case.

A

B. Overrule the objection and force Diane to answer. If the defense attorney requests, give a limiting instruction telling the jury that this is not meant to be evidence that Harry actually committed a prior theft, only a method of testing Diane’s credibility and knowledge of Harry’s character.

Correct. This evidence is relevant to test Diane’s credibility and knowledge of Harry’s character. If she has not heard about the arrest, she does not know him very well. But if she knows about the arrest, her opinion about Harry having an honest character loses credibility. However, the limiting instruction is important to ensure the jury does not conclude that Harry actually did steal a car two years ago, since it is not admissible for that purpose.

405
Q

Martha is charged with defrauding a government contractor. The prosecutor has a character witness who will testify that he has worked with Martha for seven years and knows her reputation. He is willing to testify that Martha’s reputation at work is that she is a very dishonest person. Under what conditions will the prosecutor’s witness be allowed to testify?

A. Only after Martha testifies in her own defense.

B. Only after Martha calls her own character witness to testify that Martha is an honest person.

C. Only after Martha testifies in her own defense OR calls her own character witness to testify that Martha is an honest person.

D. Never.

A

C. Only after Martha testifies in her own defense OR calls her own character witness to testify that Martha is an honest person.

Correct. The prosecutor’s character witness can be used to attack Martha’s credibility, or to rebut Martha’s character witness. Thus, either Martha testifying or calling a character witness will open the door to allowing the prosecutor’s character witness to testify.

406
Q

Stuart is charged with stabbing a person in an alleyway outside a bar. He is pleading self-defense. During his defense case, his attorney has four character witnesses he wants to call. Which of them is most likely to be allowed to testify?

A. Stuart’s wife will testify that she knows Stuart very well and in her opinion he is a very honest person.

B. Stuart’s boss at work will testify that he has known Stuart for fifteen years and Stuart is a model employee who is never late and has an excellent work ethic.

C. Stuart’s best friend will testify that he once saw a person threaten Stuart with a knife, and Stuart never lost his temper, remained calm, and convinced the person to put the knife down.

D. Stuart’s father will testify that he has known Stuart his entire life and that Stuart has always been peaceful and never loses his temper.

A

D. Stuart’s father will testify that he has known Stuart his entire life and that Stuart has always been peaceful and never loses his temper.

Correct. Stuart’s peaceful character is a pertinent trait, and Rule 404(a) allows the defendant to elicit evidence that supports a pertinent character trait if given in the form of opinion or reputation evidence.

407
Q

Carl is on trial for murder. He is pleading self-defense. The prosecutor has a character witness who will testify that in his opinion Carl is a vicious individual who routinely uses violence to get his way. Can the prosecutor’s character witness testify at Carl’s trial?

A. No, not under any conditions.

B. Yes, the character witness can testify in the prosecutor’s case-in-chief.

C. Yes, the character witness can testify, but only after the defendant testifies.

D. Yes, the character witness can testify, but only after the defendant calls his own character witness.

A

D. Yes, the character witness can testify, but only after the defendant calls his own character witness.

Correct. Under Rule 404(a), the defendant “opens the door” to an attack on his character if he calls a positive character witness.

408
Q

Sam is on trial for raping Rosemary. Sam wishes to call Frank as one of his witnesses. Frank knows Rosemary well and used to date her. He will testify that he is familiar with Rosemary’s reputation and that she is known as a woman who frequently engages in casual sexual intercourse. He will also testify that when he dated her, she was “sexually aggressive” and frequently initiated sex with him. Is Frank’s testimony admissible?

A. Yes, Rule 404(a) allows the defendant to offer propensity evidence about any of the victim’s pertinent traits.

B. Only partially. Frank can testify about Rosemary’s reputation, but he cannot testify about his specific experiences with her.

C. No. Rule 412 precludes the defendant in a sexual assault case from offering any evidence about a victim’s sexual predisposition.

D. Yes, but only to impeach Rosemary if she testifies.

A

C. No. Rule 412 precludes the defendant in a sexual assault case from offering any evidence about a victim’s sexual predisposition.

Correct. This type of evidence is covered by Rule 412, the rape shield law, and none of the exceptions to Rule 412 apply, so the defendant is precluded from offering this type of evidence.

409
Q

Oswald is accused of sexually assaulting his 17-year-old stepdaughter. The prosecutor calls a doctor to the stand to testify that she examined the stepdaughter one week after the alleged assault and found abrasions to the girl’s genitals which are consistent with forcible intercourse. The defendant seeks to admit evidence that a few days before this incident, the stepdaughter had been sexually assaulted by a 16-year-old who was living with her in a foster home at the time. Is this evidence admissible?

A. Yes, but only if the abrasions could have been caused by the earlier incident.

B. Yes, but only if the victim testifies in this case.

C. No, the evidence is barred by the rape shield law under Rule 412.

D. No, the evidence is barred as improper character evidence by Rule 404.

A

A. Yes, but only if the abrasions could have been caused by the earlier incident.

Correct. Rule 412 generally bars any evidence of prior sexual conduct on the part of the victim, but Rule 412(b)(1)(A) allows for such evidence if offered to prove that someone other than the defendant was the source of physical evidence.

410
Q

Vicky is suing her employer, Mershon Bank, for sexual harassment. She alleges that her immediate supervisor created a hostile work environment by making sexually explicit jokes in her presence and asking about her intimate relationships. In its defense, Mershon seeks to admit evidence that Vicky frequently told sexually explicit jokes at work and that she had a reputation as being sexually promiscuous with her co-workers. Is this evidence admissible?

A. Both pieces of evidence are admissible, because they are relevant and this is a civil case.

B. Neither piece of the evidence is admissible, because they are evidence of sexual behavior and predisposition.

C. The evidence about the sexually explicit jokes is probably admissible, but the evidence about her sexual reputation is not.

D. The evidence about her sexual reputation is admissible, but the evidence about the sexually explicit jokes is not.

A

C. The evidence about the sexually explicit jokes is probably admissible, but the evidence about her sexual reputation is not.

Correct. Under Rule 412(b)(2) , the evidence that Vicky told sexually explicit jokes is probably admissible because its probative value substantially outweighs the danger of harm to any victim or of unfair prejudice to any party, since it tends to show that Vicky would not find that behavior at work offensive. However, evidence of the victim’s reputation is inadmissible unless the victim herself places it in controversy, which she did not in this case.

411
Q

Melinda is suing her former employer, Dollar Bank, claiming that her supervisor Reed sexually harassed her for months while she was working there. In its defense, Dollar Bank will call Reed to the stand. He will testify that he has supervised twenty different women over the course of the past three years, and that none of them have ever accused him of any inappropriate sexual behavior. This evidence is:

A. Inadmissible because of the hearsay rule.

B. Inadmissible because of the bar on character evidence.

C. Inadmissible because it is irrelevant.

D. Admissible.

A

B. Inadmissible because of the bar on character evidence.

Correct. In a civil case, neither side can admit character evidence to prove propensity under Rule 404(a). The only relevance of this evidence is that it tends to show that Reed has a propensity to not harass his female employees, and therefore he is less likely to have harassed Melinda. As such, this testimony is character evidence and barred under Rule 404(a).

412
Q

An airline passenger nearly killed in a crash is suing the airline for personal injuries. To prove the extent of his injuries, the passenger offers a videotape taken by a local news station immediately after the crash that shows serious burns covering much of the passenger’s face. The airline moves to exclude the videotape on grounds that its probative value is substantially outweighed by the danger of unfair prejudice.
In making his ruling, which of the following is NOT appropriate for the judge to consider?
A) The videotape will make it more likely that the passenger will win the suit.
B) There are other methods of proving the passenger’s damages.
C) The videotape can be restricted to its proper purpose by instructing the jury to disregard any possible emotional appeal.
D) The videotape will encourage the jury to decide the suit on an emotional basis.

A

A. The judge may not consider whether the videotape will make it more likely that the passenger will win. This question involves the use of the probative value/prejudicial impact balancing test contained in Federal Rule of Evidence 403. Under this test, relevant evidence is admissible unless its prejudicial impact substantially outweighs its probative value. Since the question asks which argument is least appropriate for the trial judge to consider, turn the question around and ask which of the choices are appropriate to consider when balancing the probative value and prejudicial impact of an item of evidence. The key to answering this question is to recognize that the only kind of prejudice that can properly be balanced under this test is unfair prejudice. The fact that evidence, if admitted, will help one party and hurt another party could be considered prejudicial in the most general sense, but it does not make the evidence unfairly prejudicial within the meaning of the balancing test. Thus, (A) is the correct answer. In determining whether the videotape, which shows serious burns over the passenger’s face, should be excluded, it would be inappropriate for the trial judge to consider the argument that the videotape will make it more likely that the passenger will win the suit. That would not render the evidence unfairly prejudicial.

413
Q

A defendant is being prosecuted in federal court for illegally transporting persons across state lines for immoral purposes. The prosecutor alleges that her route was from New York to Tampa. The court takes judicial notice of the fact that it is impossible to get from New York to Tampa without crossing a state line.
What is the effect of the court’s action?
A) The fact judicially noticed is conclusively established.
B) The burden of persuasion is shifted to the defendant.
C) The burden of proof is shifted to the defendant.
D) The prosecutor’s burden of producing evidence on this point is satisfied.

A

The court’s action satisfies the prosecutor’s burden of producing evidence on this point. This question involves the effect of taking judicial notice of a fact. Judicial notice allows a party to “prove” a fact by the court’s recognizing that the fact is a matter of common knowledge within the jurisdiction or is able to be quickly determined by resorting to sources whose accuracy cannot reasonably be questioned. [Fed. R. Evid. 20l(b)] Since judicial notice functions are a substitute for more formal evidence (i.e., testimonial evidence), it has the same effect as more formal evidence. If the prosecutor had presented competent testimonial evidence to establish that it is impossible to get from New York to Tampa without crossing a state line, the prosecutor’s burden of producing evidence on that point would have been satisfied. Thus, the prosecutor’s burden of production is likewise satisfied if the point is established through judicial notice. (A) is incorrect because this is a criminal case. (A) would be correct if this were a civil case because, in civil cases, a fact judicially noticed is conclusively established, and thus binding on the jury. However, in criminal cases, a judicially noticed fact is not binding on the jury; the jury is permitted to find facts that have been judicially noticed, but never is required to do so. [Fed. R. Evid. 20l(g)] If a jury were required to find specific facts in a criminal case, even facts that had been judicially noticed, the defendant’s right to trial by jury would be undermined.

414
Q

In a property dispute, a granddaughter claims that her grandfather gave her a deed to his home just before he died. The grandfather’s son claims that the property is rightfully his by a previously executed will. At issue is the authenticity and content of the deed. The granddaughter begins to testify as to the content of the deed, but the son’s attorney objects.
Should the court sustain the objection?
A) Yes, because only the original deed itself is admissible to prove its contents.
B) Yes, unless the court is satisfied that the granddaughter is accurately testifying as to its contents.
C) No, if the judge is satisfied that the deed could not be found after a reasonable search.
D) No, if the deed is recorded, because the court can take judicial notice of the recorded deed.

A

The court should rule that the testimony is admissible if the deed could not be found after a reasonable search. When a proponent is attempting to prove the contents of a document, the best evidence rule applies and the original must be accounted for in order to introduce secondary evidence as to its contents. A proper foundation for the admissibility of secondary evidence may be laid by a showing that the original has been lost and cannot be found despite diligent search. Hence, (A) is incorrect. (D) is incorrect because the court may take judicial notice of court records, but the proponent of a recorded document must produce the document or record she desires introduced into evidence. A court has no power to take judicial notice of public records. (B) makes no sense because the court has no way of making such a determination.

415
Q

At the defendant’s trial for a brutal assault, in which the victim identified the defendant as her assailant, the defense calls a witness who will testify that he has worked with the defendant for 20 years and that all of his business associates regard the defendant as an honest person.
If the prosecution objects to the witness’s testimony, for which party should the court rule?
A) The defendant, because the testimony is relevant to show the defendant’s character for truth and veracity.
B) The defendant, because the testimony demonstrates the defendant’s good character, which is inconsistent with the particular crime charged.
C) The prosecution, because the testimony presents no evidence of any relevant character trait.
D) The prosecution, because the testimony is hearsay not within any exception.

A

The court should rule for the prosecution on relevancy grounds. This question involves the admissibility of “propensity” evidence-evidence that a person has a propensity to act in a certain way because of a character trait the person possesses. Although this type of character evidence is generally inadmissible, an exception to the rule is that a criminal defendant is permitted to introduce evidence that he possesses a character that is inconsistent with the crime charged. [Fed. R. Evid. 404(a)(1)] For example, a criminal defendant accused of murder is allowed to present evidence that he is a nonviolent person; a criminal defendant accused of fraud is allowed to present evidence that he is an honest person. The defendant is charged with a crime of violence (assault). The defendant’s witness is prepared to testify to the defendant’s reputation for honesty. However, possession of the character trait of honesty is not inconsistent with the commission of a violent crime. Honest people can and do commit violent crimes. Thus, the witness’s testimony does not pertain to a relevant character trait, and so it is inadmissible. For this reason, (C) is correct and (A) and (B) are incorrect. Although phrased slightly differently, both (A) and (B) assert that the character trait of honesty is inconsistent with the commission of a violent crime, and is thus relevant, but as stated, that assertion is incorrect. Note that (A) would be correct if the defendant had testified and his character as to veracity had been attacked. In that case, the witness’s testimony would be admissible to rehabilitate a witness (the defendant) whose testimony had been impeached. [Fed. R. Evid. 608(a)] But there is no indication that the defendant has testified as a witness or that, if he has, his character for truth and veracity has been attacked. Thus, (A)

416
Q

The defendant, a used car seller, is on trial for criminal fraud, charged with selling used cars with major mechanical problems while representing to buyers that the cars were mechanically sound. The defendant claims that she had no knowledge the cars were not fit for sale. At trial, the prosecution offers evidence to show that, eight months prior, the defendant was fired from a different used car lot for knowingly selling defective automobiles with major mechanical problems.
What is the best basis for admitting this evidence?
A) As evidence tending to show the defendant’s criminal character.
B) As evidence of the defendant’s criminal intent.
C) To impeach the defendant’s credibility.
D) As evidence that the defendant is not a competent used car seller.

A

The best basis for admitting this evidence is as evidence of intent. Evidence of past crimes or misconduct may not be admitted to show the accused’s criminal character or her disposition to commit the present crime. However, such evidence is admissible to show the accused’s knowledge or to show lack of mistake. This evidence, then, may be admitted to show the defendant’s intent to commit criminal fraud. (A) is wrong because, as stated, the evidence cannot be admitted to show her criminal character. (C) is wrong because, even if the defendant had testified as to her lack of knowledge, she cannot be impeached by extrinsic evidence of prior bad acts. (D) is wrong because the prosecution is not alleging that the defendant is an incompetent used car seller but rather a dishonest one. Hence, the prosecution is offering the evidence to show criminal intent.

417
Q

A disgruntled student was charged with murder for poisoning his professor by allegedly sending her a box of chocolates laced with rat poison immediately after he received a failing grade in the professor’s class. At trial, he denied sending her anything. The prosecutor seeks to have a witness testify that, four years earlier, the student poisoned his girlfriend on Valentine’s Day by lacing a box of chocolates with rat poison after he discovered she was unfaithful.
Is this evidence likely to be admitted?
A) No, because it is improper character evidence.
B) Yes, but only to impeach the student’s credibility.
C) Yes, as relevant evidence of the student’s identity, plan, or motive.
D) Yes, as evidence of defendant’s propensity towards violence.

A

The evidence of the prior act of poisoning his girlfriend four years earlier was sufficiently similar to be relevant to show defendant’s identity, plan, or motive. Although, the basic rule is that when a person is charged with one crime, extrinsic evidence of other crimes or misconduct is inadmissible if such evidence is offered solely to establish a criminal disposition, evidence of other crimes or misconduct may be admissible for other purposes that are independently relevant to some other issue, such as to show identity, plan, or motive whenever these issues are relevant in either a criminal or a civil case. [Fed. R. Evid. 404(b)] Here, the defendant’s prior act of poisoning his girlfriend was done in the exact same manner (e.g., poisoned chocolates) and for a similar reason. Thus, under these circumstances, the prior act would be admissible, not to show criminal disposition, but to show the student’s identity, plan, or motive. (A) is wrong because it is not being used to show criminal disposition, which would make it improper character evidence. Instead, there is an independent relevant purpose of using it to show identity, plan, or motive. (B) is wrong because evidence of prior bad acts cannot be used for impeachment unless probative of truthfulness. While Federal Rule 608(b) permits a witness to be interrogated upon cross-examination with respect to specific instances of misconduct that may affect his character, it is limited to misconduct that is probative of truthfulness (i.e., is an act of deceit or lying). [Fed. R. Evid. 608(b)] Here, poisoning someone is not probative of truthfulness, and therefore it could not be inquired about even on cross-examination of the defendant. In addition, although under certain circumstances, a witness may be impeached by proof of a conviction of certain types of crime, an actua

418
Q

A defendant charged with criminal battery testified on his own behalf at trial and asserted that he acted in self-defense.
Which of the following offered for impeaching the defendant’s credibility is most likely to be barred, if objected to by the defendant?
A) A public record showing that the defendant had been convicted of the felony of aggravated battery two years ago.
B) A public record showing that the defendant had been convicted of the misdemeanor of filing a false police report eight years ago.
C) Testimony from a competent witness that the defendant regularly cheats at cards.
D) Testimony from a competent witness that, in the opinion of the witness, the defendant is a habitual liar.

A

The defendant’s cheating at card games is most likely to be barred. This question involves the use of three different forms of evidence (reputation or opinion testimony, evidence of specific conduct, and prior criminal record) to impeach the testimony of a witness by showing that the witness has a propensity to be dishonest. Because the question asks which item of evidence is least likely to be admissible to impeach the defendant’s testimony, turn the question around and ask which of the items of evidence would be admissible to impeach the defendant’s testimony. The evidence in (B) is a public record of the defendant’s misdemeanor conviction for filing a false police report eight years ago. Although only a misdemeanor, this conviction is admissible under Federal Rule 609(a)(2) because it is for a crime involving dishonesty. It does not matter that the conviction is eight years old, because Rule 609(b)’s limitation on the admissibility of criminal convictions applies only if 10 years have passed since the conviction or the end of the convict’s incarceration. Nor does it matter that the prejudicial impact of the criminal offense involving dishonesty is offered to impeach a witness’s testimony. Thus, the evidence in (B) would be admissible and so (B) is wrong. The evidence in (D) is testimony from a competent witness that, in the opinion of the witness, the defendant is a habitual liar. Federal Rule 608(a) allows such testimony, as long as the witness is competent (i.e., able to base the opinion on personal knowledge). The testimony clearly relates to the defendant’s character for truthfulness or lack of truthfulness. Thus, the evidence in (D) is wrong. The evidence in (A), a public record of the defendant’s felony conviction for aggravated battery two years ago, is possibly admissible. Federal Rule 609(a)(1) permits evid

419
Q

A defendant charged with auto theft under a theory of accomplice liability testified at trial on his own behalf that although he was in the car when the police apprehended the driver, he was unaware that the car was stolen.
On cross-examination, the prosecutor asks the defendant whether he lied on an employment application three years ago when he falsely claimed to hold a college degree.
If the judge allows the question, what is the most likely reason?
A) The evidence may tend to establish that the defendant is a dishonest person and therefore may have committed the crime charged.
B) The prosecutor has a right to inquire about prior bad acts during cross-examination.
C) The evidence is relevant to the issue of the credibility of the witness and the court determines that its value is not outweighed by other considerations.
D) The evidence may tend to establish the intent of the defendant to commit auto theft.

A

The judge will allow the evidence if it is relevant and its value is not outweighed by other considerations. Under Federal Rule 608, the trial court is given the discretion to allow counsel to inquire, during cross-examination, about specific instances of bad conduct on the part of the witness which show a lack of credibility. Therefore, (C) is a more accurate answer than (B). (A) is wrong; specific instances of previous bad conduct on the part of the accused are not admissible to prove the accused is the type of person who may have committed the crime. (D) is wrong; it is true that the prosecutor can prove previous bad acts to establish intent, but the prior behavior must show more about the defendant’s intent at the time of the crime than this behavior does, because not all prior bad acts can be inquired about during cross-examination.

420
Q

In a wrongful death action after a car accident involving a drunk driver, the family of the decedent sued the driver’s employer for allowing its employee to drink too much at a company party.
When the company’s attorney asked the driver-employee on the stand how many drinks she had, she testified that she had four drinks. The company’s attorney now wants to question her about her deposition testimony, in which she said that she had two drinks.
Is this permissible?
A) No, because it is hearsay not within any exception.
B) No, because the company cannot impeach its own witness.
C) Yes, but only if it is being offered to help the driver refresh her memory.
D) Yes, because it can be offered as substantive evidence, even if it results in impeaching the driver’s testimony at trial.

A

The deposition testimony is admissible. This question raises important issues concerning hearsay evidence and impeachment evidence. The driver has made an out-of-court statement (at her deposition) that she had had two drinks on the night of the accident. At trial, she testifies that she had had four drinks that night. Is her prior inconsistent statement admissible? (A) says no, because it is hearsay not within any exception. This answer is incorrect for two reasons: (i) If the deposition testimony is being offered merely to impeach the driver’s credibility, then, by definition, it is not hearsay evidence. To constitute hearsay, an out-of-court statement must be offered into evidence to prove the truth of the matter asserted in the statement. [Fed. R. Evid. 801(c)] When a prior inconsistent statement is used to impeach, the truth or falsity of the statement is immaterial; what is important is that the witness has spoken inconsistently. (ii) Even if the deposition testimony is being offered for the purpose of establishing its truth, it still would not constitute hearsay evidence. Rule 801(d)(1)(A) excludes certain prior inconsistent statements altogether from the definition of hearsay evidence. If the declarant (the driver) testifies and is subject to cross-examination concerning her prior inconsistent statement, the prior statement is considered nonhearsay if it was given under penalty of perjury at a deposition. The driver’s statement was made under oath at a deposition, subject to the penalty of perjury, and thus qualifies as a nonhearsay statement. (B) incorrectly states that the driver’s deposition testimony cannot be used because the company (the driver’s employer) cannot impeach its own witness. Rule 607 explicitly allows any party to attack the credibility of any witness, even if the witness has been called by t

421
Q

A defendant is on trial for a murder that occurred during a robbery at the victim’s home. A witness helped the police artist compose an accurate depiction of the defendant. The witness was unavailable at the time of trial and the prosecutor offers the sketch into evidence.
Is the sketch admissible?
A) No, under the best evidence rule.
B) No, as hearsay not within any exception.
C) Yes, as a record by a public employee.
D) Yes, as prior identification.

A

The sketch is inadmissible on hearsay grounds. Under Rule 801 of the Federal Rules, a prior identification can be admissible nonhearsay, and the sketch could be deemed a prior identification by the witness. However, the hearsay exclusion for prior identifications only applies when the declarant is testifying at trial and subject to cross-examination. Here the witness is unavailable; hence, this exclusion does not apply. (D) is therefore incorrect. (A) applies to documentary evidence and has no relevance to this question. (C) is likewise not applicable, because this exception applies only to information within the personal knowledge of the public employee. In this case, the public employee gained the knowledge from the hearsay statements of an absent witness.

422
Q

The issue at trial is whether the sun was shining in a particular city on a particular day at 11 a.m. A witness testified that he was lying beside his wife on the beach in that city at 11 a.m. that day and she said to him, “Now that the sun is out, I’ll be able to get my great tan!”
Is the statement admissible?
A) No, as hearsay not within any exception.
B) No, because the witness does not have first-hand knowledge.
C) Yes, but only if the witness’s wife is unavailable.
D) Yes, whether or not the witness’s wife is unavailable.

A

The statement is admissible whether or not the witness’s wife is unavailable. The statement by the witness’s wife is hearsay, an out-of-court statement offered to prove the truth of the matter stated, that the sun was out at a specific time. However, it is admissible as a present sense impression under Federal Rule 803(1) because she was observing the condition of the sun at the time that she made the statement. Unavailability is not required for this exception. Therefore, (A) and (C) are incorrect. (B) is incorrect on the law and the facts. The present sense impression exception to the hearsay rule, which is applicable in this case, only requires that the witness actually hear the declarant make the statement at the time that the declarant is observing the event. It does not require that the witness also observe the event. Moreover, as a factual matter, although it is not legally relevant, the witness would have first-hand knowledge of the event described because he was on the beach, too.

423
Q

A pedestrian is suing a driver after being run over by the driver. At trial, a copy of a hospital record is offered into evidence. The record indicates that hospital personnel took the pedestrian’s statement 30 minutes after the accident, and includes the following statement: “The pedestrian’s leg was run over by a car.” The driver’s counsel objects to the admission of the pedestrian’s statement in the hospital record.
Should the court admit the pedestrian’s statement in the record?
A) Yes, even though hearsay within hearsay, because there is an applicable exception to each level of hearsay.
B) Yes, as a statement by a party.
C) No, as hearsay not within any exception.
D) No, because of the physician-patient privilege.

A

The statement in the record is admissible. Hearsay within hearsay is admissible if each level of the hearsay-here (i) the pedestrian’s statement to the admitting officer, and (ii) the admitting officer’s statement in the record-comes within a hearsay exception. The business records exception to the hearsay rule, under Federal Rule 803(6), admits into evidence those records kept in the course of the regular conduct of any business, organization, occupation, or calling. Therefore, the statement about the pedestrian’s medical condition would be admissible. The record itself (level (ii)) is kept in the ordinary course of the hospital’s business of treating patients, and the pedestrian’s statement about his medical condition (level (i)), while not made by someone with a duty to keep a record, is admissible both as a statement of present physical condition under Federal Rule 803(3), and as a statement made for purposes of medical treatment under Federal Rule 803(4). Thus, (C) is incorrect. (B) is incorrect. Statements by an opposing party are not hearsay under the Federal Rules; this hearsay exclusion does not apply when a party is introducing his own statement. (D) is incorrect because the physician-patient privilege is waived by the patient when he introduces his medical record at trial.

424
Q

At a trial in which a pedestrian is suing a driver, a hospital record was admitted into evidence that included the following statement: “The pedestrian’s leg was run over by a car driven by a driver who blew through a red light while the pedestrian was crossing in a crosswalk.”
The driver’s attorney now wishes to admit the other portion of the hospital record, which says, “The pedestrian stepped off the curb without first looking both ways for traffic.”
How should the court rule?
A) Admit the statement on fairness grounds because the plaintiff has the other portion of the record.
B) Admit the statement as a past recollection recorded.
C) Exclude it because it is hearsay not within any exception.
D) Exclude it because it is self-serving.

A

The court should admit the statement. Federal Rule 106 provides that, when a statement or part of a statement is introduced, the adverse party may introduce any other statement or part of the statement which ought, in fairness, to be considered at the same time. (B) is incorrect because the statement in the hospital record bears none of the required indicia for a past recollection recorded. There is no indication that the pedestrian’s memory has failed or that the report was made under his direction to accurately reflect his memory of the incident. Moreover, the statement is admissible under Federal Rule 106 because it ought, in fairness, be considered to counterbalance the evidence already improperly admitted, even if it is hearsay. (C) is incorrect because Federal Rule 106 applies even if both statements are inadmissible hearsay. (D) is incorrect. The fact that evidence is self-serving is often just another way of saying that it is hearsay. However, the rule stated in Rule 106 applies even when the remaining part of the document contains inadmissible hearsay.

425
Q

A music fan sued a well-known groupie of a rock band, alleging that she was assaulted by the groupie during a melee at a concert. The fan’s attorney calls the custodian of records for the hospital that treated her. He wishes to introduce a portion of the record by the emergency room physician, who is now deceased, reporting that the fan said she was assaulted by the groupie.
Assuming the custodian testifies that the record is an original, kept in the ordinary course of hospital business, is that portion of the hospital record admissible?
A) Yes, as hearsay within the exception for records of regularly conducted activity.
B) Yes, as a statement made for the purpose of medical diagnosis or treatment.
C) No, because the physician who made the record is not available for cross-examination.
D) No, as hearsay not within any exception.

A

That portion of the record is inadmissible. The statement is an assertive out-of-court statement by the music fan which does not qualify under any exception to the hearsay rule. For a business record to be admissible as an exception to the hearsay rule, the declarant must either have personal knowledge of the fact stated or must have received the information from someone with personal knowledge who transmitted it in the ordinary course of business. In this case, the music fan’s statement does not qualify for the business record exception because the emergency room physician had no personal knowledge of the attack and the fan did not transmit it in the ordinary course of business. It likewise does not qualify as a statement made for the purpose of medical diagnosis or treatment. The statement is not offered to describe the injuries which the music fan suffered so that she could be treated for them, but rather to identify the assailant. The exception for statements made for the purpose of medical treatment under Federal Rule 803(4) is inapplicable here. Therefore, (A) and (B) are incorrect. (C) is incorrect because the presence of the physician is irrelevant under the business record exception. That exception does not require the presence of the author of the record, only a custodian of the record.

426
Q

A town’s public works director was being sued by a resident for directing the dumping of hazardous waste in a vacant parcel of land owned by the town that was adjacent to the resident’s property. The resident is prepared to testify that she encountered a public works employee in a town dump truck dumping some debris on the parcel of land. When she asked him what he was doing, he said that he was just following his boss’s orders but that she should not go anywhere near the debris.
Should the court admit this testimony?
A) Yes, as a statement of a co-conspirator.
B) Yes, as a statement of an employee of the party-opponent.
C) No, because there is no showing that the employee was authorized to speak for the party.
D) No, because it is hearsay not within any exception.

A

The court should rule that the statement is admissible as a statement attributable to the party-opponent (i.e., a vicarious statement of an opposing party). Statements by an agent concerning any matter within the scope of his agency, made during the existence of the employment relationship, are admissible against the principal. Here, the employee’s statement was made during his employment and pertained to what he was doing on the property. Therefore, the statement can be admitted against the employee’s boss as a vicarious opposing party statement. (A) is incorrect. An admission of one conspirator is admissible against co-conspirators if made while participating in the conspiracy and in furtherance of a conspiracy to commit a crime or civil wrong. Here, there is no showing that the employee was part of a conspiracy to commit illegal dumping, and even if so, his statement was not made in furtherance of the conspiracy. (C) is incorrect. Even though his boss may not have authorized him to say anything about what he was dumping, the statement is admissible on account of the employment relationship. (D) is incorrect because, as discussed above, the statement is not hearsay but rather a vicarious statement of an opposing party.

427
Q

A wife is on trial for murdering her husband. At trial, the prosecution entered into evidence the fact that the wife fired the gun which killed her husband. The wife testified in her defense that her husband was threatening her with a knife when she picked up the gun and shot him.
In rebuttal, the prosecution calls one of the officers who responded to the wife’s 911 call right after the shooting. The officer will testify that the wife said, “I accidentally dropped my gun on the floor and it went off, killing my husband.”
Is the officer’s testimony admissible?
A) Yes, as an excited utterance.
B) Yes, to impeach the wife and as evidence that she did not act in self-defense.
C) No, because of the wife’s privilege against self-incrimination.
D) No, for the purpose of impeaching the wife, because the prosecutor did not call her attention to her statement to the officer on cross-examination.

A

The officer’s testimony is admissible. Because the wife is a party, any statement she makes can be offered against her as a statement by an opposing party (commonly called an admission). Statements by an opposing party are not hearsay and therefore need not qualify for an exception to the hearsay rule. The statement is admissible substantively because it contradicts, and therefore tends to disprove, the self-defense theory. It is likewise a prior inconsistent statement which can be used to impeach the wife’s credibility as a witness. (A) is incorrect because the statement made in this case was made to a police officer when he arrived at the house some time after the shooting. Moreover, the form of the statement shows it to be the product of reflection, rather than an unthinking response to an exciting event. (C) is incorrect because as a general rule the privilege against self-incrimination only permits an individual to refuse to give an answer to a question because it might tend to incriminate her; it does not require that an answer already given be held inadmissible later when offered in evidence. The exception is when an individual is held in custody and interrogated by the police without being given Miranda warnings and without waiving Miranda rights. However, Miranda warnings are not required when a statement is volunteered and not the product of interrogation, as here. (D) is incorrect. Extrinsic evidence of a prior inconsistent statement ordinarily may not be introduced unless the witness is given the opportunity to explain or deny the inconsistent statement. However, this foundational requirement does not apply to statements by an opposing party. Even if the wife were not a party, the extrinsic evidence would be proper because she is available to be recalled. While the opportunity to explain or deny an inconsis

428
Q

A passenger injured in a car accident is suing the owner of a bar for allegedly allowing the driver to become intoxicated. The passenger wants to show that, after the accident, the bar owner visited him in the hospital and offered to pay all of the passenger’s medical expenses, stating, “That’s the least I can do after letting your driver leave the bar so drunk last night.”
Is the statement that the driver was drunk when he left the bar on the night of the accident admissible?
A) Yes, as an admission by the bar owner that the driver was drunk when he left the bar.
B) Yes, as a factual admission made in connection with an offer of compromise.
C) No, as hearsay not within any exception.
D) No, as a statement made in connection with an offer to pay medical expenses.

A

The statement is admissible as an opposing party’s statement (commonly called an admission). This question involves an offer to pay medical expenses [Federal Rule 409], not an offer to settle a dispute [Federal Rule 408]. Although the line between these two types of offers is sometimes fuzzy, the facts in this case clearly indicate that no settlement offer was made. The defendant/bar owner offered to pay all of the passenger’s medical expenses, but there is no indication of an offer to settle the entire claim that the passenger ultimately brought against him. The timing of the offer (the day after the accident, before any demand has been communicated to the defendant) and its informality (being made orally by a nonattorney to a hospitalized victim) bolster the conclusion that this case involves an offer to pay medical expenses, not an offer to settle the case. Thus, (B), which states that an offer of compromise has been made, is wrong. Moreover, even if this case were misinterpreted as involving a settlement offer, (B) would still be wrong because statements made in connection with settlement offers are inadmissible for reasons of public policy. (C) is wrong because the defendant’s statement is an out-of-court statement by a party being offered against that party. Statements by an opposing party are not hearsay and do not need to fall within any hearsay exception. [Fed. R. Evid. 801(d)(2)] (D) is wrong because statements made in connection with an offer to pay medical expenses, unlike statements made in connection with a settlement offer, are admissible. An offer to pay medical expenses is essentially a unilateral and unconditional humanitarian gesture. Thus, negotiation and discussion are tangential to the offer itself. Consequently, there is little reason to protect from disclosure such statements as the defendant’s

429
Q
A